Sei sulla pagina 1di 609

Aug 12 – 1

The hypothesis that people ascribe more value to things merely because they own them is at the
heart of a highly influential theory in behavioral economics, known as the endowment
effect.Economists ascribing to the theory have tried to illustrate this effect with the help of several
experiments that seem to show that people value the things they own more than they value
identical products they do not own and that they prefer to hold on to those endowed items rather
than trade or sell them.Endowment theory says that such observable events, whatever their other
possible causes, are at least partially explained by the general phenomenon of loss aversion, which
holds that people give possible losses more weight than potential gains of the same
magnitude.Endowment theory is an application of this phenomenon, adding the hypothesis that
ownership determines whether one experiences a change as a gain or as a loss.Endowment theory
posits that ownership sets one’s reference point, the movement from which triggers either a
perceived gain or loss, and that people perceive the transfer or sale of endowments as losses.

Although endowment theory continues to enjoy considerable influence, recent experimental data
have cast doubt on the hypothesis that ownership sets the reference point and that loss aversion
generates the reluctance to trade.The empirical support for endowment theory was never perfect.In
recent years, experimentalists have published data suggesting that the results of earlier laboratory
experiments were not caused by loss aversion but by other factors.By making a few changes to the
experimental design—such as better training subjects in the auction mechanisms used in the
experiments, changing the way subjects were given the items, and modifying the procedures for
eliciting choices—to rule out alternative explanations, experimentalist were able to make
“endowment effects” that had been observed in the laboratory disappear.Many other researchers
have since replicated these results.The new data suggest that ownership alone is not enough to
change people’s expressed preferences.These results have led experimental economists and
cognitive psychologists to develop alternatives to endowment theory.

1.The economists ascribing to the theory of endowment effect would consider which
one of the following as an example of this effect?

A. A person who receives a gift of a hundred dollar bottle of wine would not normally
spend the same amount buying it and choses to drink the wine rather than sell it.
B. A person, who buys a bottle of his favorite wine for ten dollars, refuses to pay the same amount
for a bottle of wine that is very similar in every aspect to the other wine and that his friend is willing
to sell.
C. A person, who receives a bottle of wine as a gift from a friend, buys the same wine again from
the market at a more expensive price than the one paid by his friend.
D. A person sells a bottle of wine, which was gifted to him, at a price that is not lower than the
market price of the wine.
E. A person refuses to sell a bottle of wine that was gifted to him because market survey shows that
the price of that wine is likely to rise even further in the future.
2.The author is primarily concerned with
A. describing a theory in detail while analyzing the influence it enjoys in the field of economics
B. explaining a theory and its underlying basis to review the individuals experiments done in its
support
C. discussing a theory in detail and bringing out its deficiencies that were deliberately ignored by its
proponents
D. critically evaluating a theory and its underlying basis
E. describing a theory and suggesting that it should be discarded since it has no empirical soundness

3.Which of the following is most supported by the information given in the passage?
A. The value of the endowed item increases with the passage of time.
B. The way a response is extracted out of a participant in a study could have an effect in
the ultimate outcome of that study.
C. The endowment theory lacks any sort of factual support.
D. As per the endowment theory, the sole reason that people are reluctant to transfer or sell their
endowed item is because of the workings of the loss aversion phenomenon.
E. The real value of an endowed product is normally considerably less than the value ascribed to it
by people.

4.According to the passage, which of the following is NOT true?


A. The endowment theory enjoys more than limited influence.
B. The ownership of an endowed product has an important part to play in whether a particular
change is perceived as a gain or loss.
C. There are holes in the empirical data that support results not in favor of the
endowment theory.
D. Loss aversion is possibly not the only factor that the theory believes causes the endowment
effect.
E. Some experimental economists have been influenced by the recent data that does not favor the
endowment theory.

Ascribe (v) : miêu tả

Endowment (n) : tài trợ

Behavioral economic (n) : hành vi thương mại

Aversion (n) : ác cảm

Magnitude (n) : độ lớn, tính trọng đại

Posit (v) : đặt ra

Reluctance (n) : miễn cưỡng

Empirical (adj) : kinh nghiệm

Elicite (v) : khơi gợi

Extract (v) : trích xuất

Underlying (adj/v) : cơ bản


Aug 12 – 2

In contrast to traditional analyses of minority business, the sociological analysis contends that
minority business ownership is a group-level phenomenon, in that it is largely dependent upon
social-group resources for its development. Specifically, this analysis indicates that support networks
play a critical role in starting and maintaining minority business enterprises by providing owners with
a range of assistance, from the informal encouragement of family members and friends to
dependable sources of labor and clientele from the owner’s ethnic group. Such self-help networks,
which encourage and support ethnic minority entrepreneurs, consist of “primary” institutions, those
closest to the individual in shaping his or her behavior and beliefs. They are characterized by the
face-to-face association and cooperation of persons united by ties of mutual concern. They form an
intermediate social level between the individual and larger “secondary” institutions based on
impersonal relationships. Primary institutions comprising the support network include kinship, peer,
and neighborhood or community subgroups.

A major function of self-help networks is financial support. Most scholars agree that minority
business owners have depended primarily on family funds and ethnic community resources for
investment capital. Personal savings have been accumulated, often through frugal living habits that
require sacrifices by the entire family and are thus a product of long-term family financial behavior.
Additional loans and gifts from relatives, forthcoming because of group obligation rather than
narrow investment calculation, have supplemented personal savings. Individual entrepreneurs do
not necessarily rely on their kin because they cannot obtain financial backing from commercial
resources. They may actually avoid banks because they assume that commercial institutions either
cannot comprehend the special needs of minority enterprise or charge unreasonably high interest
rates.

Within the larger ethnic community, rotating credit associations have been used to raise capital.
These associations are informal clubs of friends and other trusted members of the ethnic group who
make regular contributions to a fund that is given to each contributor in rotation. One author
estimates that 40 percent of New York Chinatown firms established during 1900-1950 utilized such
associations as their initial source of capital. However, recent immigrants and third or fourth
generations of older groups now employ rotating credit associations only occasionally to raise
investment funds. Some groups, like Black Americans, found other means of financial support for
their entrepreneurial efforts. The first Black-operated banks were created in the late nineteenth
century as depositories for dues collected from fraternal or lodge groups, which themselves had
sprung from Black churches. Black banks made limited investments in other Black enterprises. Irish
immigrants in American cities organized many building and loan associations to provide capital for
home construction and purchase. They, in turn, provided work for many Irish home-building
contractor firms. Other ethnic and minority groups followed similar practices in founding ethnic-
directed financial institutions.
1. Based on the information in the passage, it would be LEAST likely for which of the
following persons to be part of a self-help network?

(A) The entrepreneur’s childhood friend


(B) The entrepreneur’s aunt
(C) The entrepreneur’s religious leader
(D) The entrepreneur’s neighbor
(E) The entrepreneur’s banker

2. Which of the following illustrates the working of a self-help support network, as such
networks are described in the passage?

(A) A public high school offers courses in book-keeping and accounting as part of its open-
enrollment adult education program.
(B) The local government in a small city sets up a program that helps teen-agers find summer jobs.
(C) A major commercial bank offers low-interest loans to experienced individuals who hope to
establish their own businesses.
(D) A neighborhood-based fraternal organization develops a program of on-the-job
training for its members and their friends.
(E) A community college offers country residents training programs that can lead to certification in a
variety of technical trades.

3. Which of the following can be inferred from the passage about rotating credit
associations?

(A) They were developed exclusively by Chinese immigrants.


(B) They accounted for a significant portion of the investment capital used by Chinese
immigrants in New York in the early twentieth century.
(C) Third-generation members of an immigrant group who started businesses in the 1920’s would
have been unlikely to rely on them.
(D) They were frequently joint endeavors by members of two or three different ethnic groups.
(E) Recent immigrants still frequently turn to rotating credit associations instead of banks for
investment capital.

4. The passage best supports which of the following statements?


(A) A minority entrepreneur who had no assistance from family members would not be able to start
a business.
(B) Self-help networks have been effective in helping entrepreneurs primarily in the last 50 years.
(C) Minority groups have developed a range of alternatives to standard financing of
business ventures.
(D) The financial institutions founded by various ethnic groups owe their success to their unique
formal organization.
(E) Successful minority-owned businesses succeed primarily because of the personal strengths of
their founders.

5. Which of the following best describes the organization of the second paragraph?
(A) An argument is delineated, followed by a counter-argument.
(B) An assertion is made and several examples are provided to illustrate it.
(C) A situation is described and its historical background is then outlined.
(D) An example of a phenomenon is given and is then used as a basis for general conclusions.
(E) A group of parallel incidents is described and the distinctions among the incidents are then
clarified.

6. According to the passage, once a minority-owned business is established, self-help


networks contribute which of the following to that business?
(A) Information regarding possible expansion of the business into nearby communities
(B) Encouragement of a business climate that is nearly free of direct competition
(C) Opportunities for the business owner to reinvest profits in other minority-owned businesses
(D) Contact with people who are likely to be customers of the new business
(E) Contact with minority entrepreneurs who are members of other ethnic groups

7. It can be inferred from the passage that traditional analyses of minority business
would be LEAST likely to do which of the following?
(A) Examine businesses primarily in their social contexts
(B) Focus on current, rather than historical, examples of business enterprises
(C) Stress common experiences of individual entrepreneurs in starting businesses
(D) Focus on the maintenance of businesses, rather than means of starting them
(E) Focus on the role of individual entrepreneurs in starting a business

In contrast to traditional analyses of minority business, the sociological analysis contends that
minority business ownership is a group-level phenomenon, in that it is largely dependent upon
social-group resources for its development

 Traditional analyses depends upon the opposition of social-group resource

8. Which of the following can be inferred from the passage about the Irish building and
loan associations mentioned in the last paragraph?
(A) They were started by third- or fourth-generation immigrants.
(B) They originated as offshoots of church-related groups.
(C) They frequently helped Irish entrepreneurs to finance business not connected with construction.
(D) They contributed to the employment of many Irish construction workers.
(E) They provided assistance for construction businesses owned by members of other ethnic groups.

Sociological (adj) : xã hội học

Primary (adj) : sơ cấp

Comprise (v) : bao gồm

Kinship (n) : quan hệ họ hàng

Frugal (adj) : giản dị, thanh đạm

Despository (n) : lưu ký

Fraternal (adj) : như anh em


Aug 13 – 1

Eating fish and other foods rich in omega-3 fatty acids could help people maintain healthy brains as
they age as well as protect their hearts, new research suggests. Per this research, participants with
low levels of omega-3 fatty acids in their blood had slightly smaller brains and scored lower on
memory and cognitive tests than people with higher blood levels of omega-3s.

The researchers then looked at and ranked the level of omega-3 fatty acids in the participants'
blood. People who scored in the bottom 25% in omega-3 fatty acid levels were compared with the
rest of the study participants. Researchers found that those who had the lowest level of omega-3
fatty acid levels in their blood had lower brain gray matter volume compared with those with higher
levels.

Gray matter volume is crucial to brain health. When it remains higher, brain health is being
maintained. Decrease in gray matter volume indicates that brain cells are shrinking. The findings
showed that consumption of baked or broiled fish on a weekly basis was positively associated with
retention of gray matter volume in several areas of the brain. In fact the greater hippocampal,
posterior cingulate, and orbital frontal cortex volumes obtained by optimal fish consumption led to
reduced risk of contracting MCI or Alzheimer’s by almost five-fold.

The most recent U.S. dietary guidelines—released last year—recommend at least two servings of
seafood a week. Some doctors and diet experts recommend that patients consume fish three times
a week or take fish-oil supplements so they get enough omega-3 fatty acids to obtain health
benefits.

1) According to the passage, people who eat foods rich in Omega 3 acids experience
which of the following benefits

A) Improve memory and retention

The passage talks about retention but it specifies retention of gray matter volume
B) Increase gray matter volume that leads to improved analytical skills
C) Do not suffer from diseases such as Alzheimer's
D) Are on an average at a much lower risk of contracting Alzheimer's when compared to the general
population, regardless of their family history
E) Are able to maintain a healthy brain for longer as they age

Eating fish and other foods rich in omega-3 fatty acids could help people maintain healthy brains as
they age as well as protect their hearts…

2) The experiment discussed in paragraph 2 suggests

A) People who scored in the top quartile took Omega 3 supplements while those who scored lower
did not.
B) For people in any age group, there is a direct correlation between Omega 3 and the amount of
gray matter.
C) There is a high probability that people with extremely low levels of Omega 3 in blood
will also have low gray matter volume
D) The reason why kids don’t do as well in Maths and science is due to the low intake of Omega 3.
E) Taking Omega 3 beyond a point has little impact on the amount of grey matter volume build up.

3) The author’s main purpose in writing the passage is

A) Describing a research and presenting its findings


B) Comparing the benefits provided by Omega 3 in regards to preventing MCI or Alzheimer’s.
C) Discussing the utility of various varieties of fish as a source of Omega 3.
D) Comparing how the risk of contracting MCI or Alzheimer’s reduces with the corresponding
increased intake of fish rich in Omega 3.
E) Justifying the reason that U.S. dietary guidelines recommend at least two servings of seafood a
week.

4) The passage suggests the following about diet and gray matter volume

A) Regular consumption of foods rich in Omega 3 help maintains gray matter volume for
longer periods.
B) People who eat foods richer in Omega 3 tend to have higher than average gray matter volume.
C) It is important to maintain gray matter volume above a certain level to avoid risk of MCI or
Alzheimer’s.
D) Any irregularity in consuming foods rich in Omega 3 would result in reduction in brain gray
matter volume.
E) Once the gray matter volume reduces, it is not possible to increase the same by consuming foods
rich in Omega 3.

Paragraph 1 states omega-3 fatty acids could help people maintain healthy brains as they age.
Paragraph 3 shows the relation between gray matter volume and brain health. From these, we can
infer this option

Hippocampal (n) : hồi hải mã

Posterior (adj) : sau

Cingulate cortex (n) : vỏ não

Frontal (n) : phía trước

Five-fold (adj) : gấp năm lần


Aug 13 – 2

The storms most studied by climatologists have been those that are most easily understood by
taking atmospheric measurements. Hurricanes and tornadoes, for example, are spatially confined,
the forces that drive them are highly concentrated, and they have distinctive forms and readily
quantifiable characteristics. Consequently, data about them are abundant, and their behavior is
relatively well understood, although still difficult to predict.

Hurricanes and tornadoes are also studied because they are highly destructive storms, and
knowledge about their behavior can help minimize injury to people and property. But other equally
destructive storms have not been so thoroughly researched, perhaps because they are more difficult
to study. A primary example is the northeaster, a type of coastal storm that causes significant
damage along the eastern coast of North America. Northeasters, whose diffuse nature makes them
difficult to categorize, are relatively weak low-pressure systems with winds that rarely acquire the
strength of even the smallest hurricane. Although northeasters are perceived to be less destructive
than other storms, the high waves associated with strong northeasters can cause damage
comparable to that of a hurricane, because they can affect stretches of coast more than 1,500
kilometers long, whereas hurricanes typically threaten a relatively small ribbon of coastline—roughly
100 to 150 kilometers.

1. The primary purpose of the passage is to

(A) evaluate the relative amounts of damage caused by different storm types
(B) describe the difficulties of classifying destructive storms by type
(C) examine the relationship between wave height and the destructive potential of storms
(D) discuss a theory that explains the origins of violent storms
(E) discuss reasons why certain types of storms receive more study than others

2. According to the passage, which of the following is true of northeasters?

(A) They have only recently been identified as a distinct storm type.
(B) They are more destructive than tornadoes.
(C) They are low-pressure systems.
(D) They affect a relatively small segment of the eastern coast of North America.
(E) Their winds are typically as strong as those of small hurricanes.

3. Which of the following can be inferred from the passage about storms that lend
themselves to atmospheric measurements?

(A) They are more likely than other storms to be studied by climatologists.
(B) They are likely to be less highly concentrated than are other storms.
(C) They are likely to be more difficult to predict than are other storms.
(D) They occur less frequently along the eastern coast of North America than in other areas.
(E) They tend to affect larger areas than do other storms.

1 Para: "The storms most studied by climatologists have been those that are most easily understood
by taking atmospheric measurements. Hurricanes and tornadoes,"
2 Para: "But other equally destructive storms have not been so thoroughly researched, perhaps
because they are more difficult to study. "
Aug 14 – 1

In the year 1898, the United States made its earliest plunge into imperialism. For the first time, the
nation secured overseas lands, dominions too thickly populated to be adapted for the purposes of
colonization. By earlier conquests and purchases such as those of Louisiana, Florida, Texas,
California, and New Mexico, the United States had secured relatively empty territories, which a flow
of emigrants from the Eastern States of the US could rapidly Americanize. But in Puerto Rico, the
Philippines and Hawaii, there was never a serious attempt to colonize on the part of the American
citizens. The reasons behind these conquests were similar to the ones that led to the European
partition of Africa in the 19th century. The year 1898 indeed represented a strong shift in the
foreign policy.

Even though in the years after the Civil War (1861-1865), Denmark offered to sell St. Thomas and
St. Johns cheaply, the United States refused to purchase them, and declined to annex San Domingo
or to entertain Sweden's proposal to purchase its West Indian territory. In 1893, instead of annexing
Hawaii, the US attempted to bolster up the sovereignty of the native Queen. Then suddenly Puerto
Rico, the Philippines and Guam were annexed; Hawaii was incorporated and Samoa was divided
with Germany. America finally abandoned isolationism for good.

In part, this change in foreign policy was due to military considerations. The possession of Hawaii,
Panama and Guantanamo in Cuba was obviously necessary for the defense of America’s coasts. Just
as the Monroe Doctrine was intended to protect the country from the approach of great military
powers, so these new acquisitions were desired to pre-empt close-lying bases, from which fleets in
enemy possession could assail the country’s trade or cut off its communication. Such strategic
considerations, however, do not explain the whole of the new imperialistic policy. Economic motives
played a greater part. As a result of the Industrial Revolution, merchants had begun to think in
terms of foreign markets and the US financiers in terms of foreign investments. The growing
population had made increasing demands upon food products, leaving less to be exported, and at
the same time exports of manufactures had increased. Many citizens believed that the United States
could not afford to adhere to a policy of isolation while other nations were reaching out for the
command of new markets.

1. Which of the following statements best explains America’s sudden shift from
isolationism to imperialism?

A) America was becoming a manufacturing nation, requiring markets for the disposal of surplus
products.
B) The United States had an over-exaggerated opinion on the progress made towards
industrialization.
C) Markets and investment fields of the world were being divided up among the United States' rivals
in Europe.
D) America required overseas territorial possessions in order to encroach on the lands of its
enemies.
E) The United States changed its foreign policy because the country was undergoing a
commercial and industrial revolution.

The passage states that, "at the same time exports of manufactures had increased" but it's too far
to suggest that American had not been a manufacturing nation prior to this policy shift.
2. Which of the following best describes the purpose of the third paragraph of the
passage?

A) to explain how military considerations were responsible for the rise of imperialism.
B) to describe the foundations of the Industrial Revolution and its effect on America.
C) to outline the territories affected by America's foreign policy decisions in the post-Civil War
years.
D) to use the Monroe Doctrine as a metaphor for America's growing imperialism.
E) to provide the major and minor reasons behind the United States' policy shift in 1898.

3. Which of the following inferences is best supported by the passage?

A) The United States did not actively pursue new territories until 1898.
B) America declined Sweden's proposal to purchase its West Indian territory for largely economic
reasons.
C) The United States was threatened by the economic success of countries such as Panama and
Cuba.
D) Prior to 1898, the United States political leadership had largely pursued an
isolationist strategy.
E) The motivations of American financiers were influenced at least in part by the military.

4. Which of the following is the author's overall goal in composing this passage?

A) To discuss the various changes in the US foreign policy at the end of the 19th century.
B) To present the reasons responsible for a new foreign policy development in a
historical context.
C) To ascribe a shift in the US foreign policy to economic as well as militaristic considerations.
D) To contrast two different causes for a major adjustment in US foreign policy
E) To highlight the unfair annexation of three countries by the US during the late nineteenth
century.

Plunge (n) : sự nhảy, sự phóng, sự lao mình xuống

Imperialism (n) : chủ nghĩa đế quốc

Secure (v) : bảo đảm

Dominion (n) : sự thống trị, quyền lực, chủ quyền, ảnh hưởng

Colonization (n) : chiếm làm thuộc địa

Partition (n) : vách ngăn, sự ngăn phòng

Annex (v) : thôn tính, sát nhập

Bolster (v) : tăng cường

Sovereignty (n) : chủ quyền

Isolationism (n) : chủ nghĩa cô lập

Assail (v) : tấn công


Imperialistic (adj) : chính sách đế quốc

Disposal (n) : xử lý

Undergoing (v) : trải qua


Aug 14 – 2

Until recently, zoologists believed that all species of phocids (true seals), a pinniped family, use a
different maternal strategy than do otariids (fur seals and sea lions), another pinniped family.
Mother otariids use a foraging strategy. They acquire moderate energy stores in the form of blubber
before arriving at breeding sites and then fast for 5 to 11 days after birth. Throughout the rest of
the lactation (milk production) period, which lasts from 4 months to 3 years depending on the
species, mother otariids alternately forage at sea, where they replenish their fat stores, and nurse
their young at breeding sites. Zoologists had assumed that females of all phocids species, by
contrast, use a fasting strategy in which mother phocids, having accumulated large energy stores
before they arrive at breeding sites, fast throughout the entire lactation period, which lasts from 4 to
50 days depending on the species. However, recent studies on harbor seals, a phocids species,
found that lactating females commenced foraging approximately 6 days after giving birth and on
average made 7 foraging trips during the remainder of their 24-day lactation period.

The maternal strategy evolved by harbor seals may have to do with their small size and the large
proportion of their fat stores depleted in lactation. Harbor seals are small compared with other
phocids species such as grey seals, all of which are known to fast for the entire lactation period.
Studies show that mother seals of these species use respectively 84 percent, 58 percent, and 33
percent of their fat stores during lactation. By comparison, harbor seals use 80 percent of their fat
stores in just the first 19 days of lactation, even though they occasionally feed during this period.
Since such a large proportion of their fat stores is exhausted despite feeding, mother harbor seals
clearly cannot support all of lactation using only energy stored before giving birth. Though smaller
than many other phocids, harbor seals are similar in size to most otariids. In addition, there is
already some evidence suggesting that the ringed seal, a phocids species that is similar in size to the
harbor seal, may also use a maternal foraging strategy.

1. It can be inferred from the passage that the females of all phocids species differ from
the females of all otariid species in that the female phocids

A. have shorter lactation periods


B. consume more food during lactation
C. consume a higher proportion of fat stores
D. forage for food occasionally during their lactation periods
E. deplete a smaller percentage of their fat stores during their lactation periods

2.The primary purpose of the passage is to


A. present evidence that several phocids species use the maternal fasting strategy
B. explain why the maternal strategy typically used by phocids is different from the maternal
strategy used by otariids
C. argue that zoologists 'current understanding of harbor seals' maternal strategy is incorrect

The author never argues that zoologists' understanding of harbors seals' strategy is incorrect.
D. describe an unexpected behavior observed in harbor seals and propose an
explanation that may account for the behavior
E. describe evidence concerning the maternal strategy of the harbor seal and suggest that the
harbor seal belongs to the otariid rather than to the phocids family
Paragraph 1
There are two kinds of pinnipeds: phocids (let's call them Group P) and otariids (let's call them
Group O).

Group O uses a foraging strategy, while Group P uses a fasting strategy.


However, recent studies show that harbor seals (which belong to Group P) do forage as well.

What's the purpose of Paragraph 1? To identify how the maternal strategies of Group O and
Group P differ, then present the harbor seal as an exception to the strategy of Group P.

Paragraph 2
Harbor seals from Group P might forage because of their small size and rate of fat store depletion.
There is evidence to support this explanation.
The behavior of ringed seals (also in Group P) may also support this explanation

What's the purpose of Paragraph 2? To explain why the harbor seal is an exception (their
smaller size and high fat depletion rate), and provide evidence to support this explanation.

3.According to the passage, until recently zoologists believed which of the following
about all phocids mothers?
A. Their fasting periods after giving birth were typically shorter than those of otariids.
B. Their lactation periods were generally as long as those of comparably sized otariids.
C. They acquired only moderate energy stores in the form of blubber before arriving at breeding
sites.
D. They depleted less than a third of their stored body fat during lactation.
E. The replenished their fat stores only after their lactation period ended.

4.The author of the passage mentions ringed seals most probably in order to
A. provide an example of a phocid species that fasts throughout its entire lactation period
B. provide an example of a phocid species whose maternal strategy is typical of phocid species
C. provide an example of a phocid species that may deplete an even higher proportion of its fat
stores during lactation than harbor seals do
D. support the assertion that harbor seals are smaller than many other phocids
E. support the assertion that harbor seals' maternal strategy may be related to their
small size.

Zoologist (n) : nhà động vật học

Maternal (adj) : thuộc về mẹ

Foraging (n) : tìm kiếm thức ăn

Blubber (n) : chất béo của động vật biển có vú

Fast (n) : nhịn ăn

Lactation (n) : cho con bú

Replenish (v) : bổ sung

Commence (v) : bắt đầu


Deplete (v) : cạn kiệt

Aug 15 – 1

The promise of finding long-term technological solutions to the problem of world food shortages
seems difficult to fulfill. Many innovations that were once heavily supported and publicized, such as
fish-protein concentrate and protein from algae grown on petroleum substrates, have since fallen by
the wayside. The proposals themselves were technically feasible, but they proved to be economically
unviable and to yield food products culturally unacceptable to their consumers. Recent innovations
such as opaque-2 maize, Antarctic krill, and the wheat-rye hybrid triticale seem more promising, but
it is too early to predict their ultimate fate.

One characteristic common to unsuccessful food innovations has been that, even with extensive
government support, they often have not been technologically adapted or culturally acceptable to
the people for whom they had been developed. A successful new technology, therefore, must fit the
entire sociocultural system in which it is to find a place. Security of crop yield, practicality of storage,
palatability, and costs are much more significant than had previously been realized by the advocates
of new technologies. For example, the better protein quality in tortillas made from opaque-2 maize
will be of only limited benefit to a family on the margin of subsistence if the new maize is not
culturally acceptable or is more vulnerable to insects.

The adoption of new food technologies depends on more than these technical and cultural
considerations; economic factors and governmental policies also strongly influence the ultimate
success of any innovation. Economists in the Anglo-American tradition have taken the lead in
investigating the economics of technological innovation. Although they exaggerate in claiming that
profitability is the key factor guiding technical change—they completely disregard the substantial
effects of culture—they are correct in stressing the importance of profits. Most technological
innovations in agriculture can be fully used only by large landowners and are only adopted if these
profit-oriented business people believe that the innovation will increase their incomes. Thus,
innovations that carry high rewards for big agribusiness groups will be adopted even if they harm
segments of the population and reduce the availability of food in a country. Further, should a new
technology promise to alter substantially the profits and losses associated with any production
system, those with economic power will strive to maintain and improve their own positions. Since
large segments of the populations of many developing countries are close to the subsistence margin
and essentially powerless, they tend to be the losers in this system unless they are aided by a
government policy that takes into account the needs of all sectors of the economy. Therefore,
although technical advances in food production and processing will perhaps be needed to ensure
food availability, meeting food needs will depend much more on equalizing economic power among
the various segments of the populations within the developing countries themselves.
1. Which of the following best describes the organization of the first paragraph?

(A) A suggestion is made and arguments in its favor are provided.


(B) A criticism is levied and an alternative proposal is suggested.
(C) A generalization is advanced and supporting evidence is provided.
(D) An example is analyzed and general conclusions are derived from it.
(E) A position is stated and evidence qualifying it is provided.

2. It can be inferred from the passage that the author was unable to assess the truth of
which of the following statements about opaque-2 maize?

(A) It is a more recent innovation than the use of fish-protein concentrate.


(B) It can be stored as easily as other varieties of maize.
(C) It is more popular than the wheat-rye hybrid triticale.
(D) It produces tortillas of greater protein content than do other varieties of maize.
(E) It is more susceptible to insects than are other varieties of maize.

3. The passage mentions all of the following as factors important to the success of a
new food crop EXCEPT the

(A) practicality of storage of the crop


(B) security of the crop yield
(C) quality of the crop’s protein
(D) cultural acceptability of the crop
(E) costs of production of the crop

4. According to the passage, the use of Antarctic krill as a food is an innovation whose
future is

(A) basically gloomy but still uncertain


(B) somewhat promising but very tentative
(C) generally bright and virtually assured
(D) tied to the success of opaque-2 maize
(E) endangered by certain technical problems

5. The author suggests that, in most developing countries, extensive government


intervention accompanying the introduction of a food innovation will

(A) usually be sufficient to guarantee the financial success of the innovation


(B) be necessary to ensure that the benefits of the innovation will be spread throughout
the society
(C) provide the incentive necessary to convince landowners to try the innovation
(D) generally cost the country more than will be earned by the innovation
(E) normally occur only when the innovation favors large landowners

6. It can be inferred from the passage that the author would be most likely to agree
with which of the following statements concerning the solution to food shortages in
developing countries?

(A) The introduction of technological innovations to reap profits might alleviate food
shortages to some degree, but any permanent solution can come only from effective
governmental intervention in the socioeconomic system.
(B) Innovations in agricultural technology will be of little help, and perhaps even harmful, in
combating food shortages, no matter how well designed they are to suit local circumstances.
(C) Long-lasting solutions will not be found until large landowners adopt improvements that will
make production more efficient and thus more profitable.
(D) In order to achieve a meaningful solution to the problem of food shortages, the tastes of the
general population must be educated to accept the new food products of modern agricultural
technology.
(E) Although a short-term solution to food shortages can be achieved by importing food from other
countries, a long-term solution requires a restructuring of the countries’ socioeconomic system.

7. The first paragraph of the passage best supports which of the following statements?

(A) Too much publicity can harm the chances for the success of a new food innovation.
(B) Innovations that produce culturally acceptable crops will generally be successful.
(C) A food-product innovation can be technically feasible and still not be economically
viable.
(D) It is difficult to decide whether a food-product innovation has actually been a success.
(E) Triticale will not be a success as a food source for most developing countries.

8. The author provides a sustained argument to support which of the following


assertions?

(A) Profitability is neither necessary nor sufficient for a new technology to be adopted.
(B) Profitability is the key factor guiding technological change.
(C) Economic factors and governmental policies strongly influence the ultimate success
of any innovation.
(D) Opaque-2 maize is of limited benefit to poor families in developing countries.
(E) Innovations carrying high rewards for big agribusiness groups harm the poor.

9. The primary purpose of the passage is to discuss the

(A) means of assessing the extent of the world food shortage


(B) difficulties of applying technological solutions to the problem of food shortages
(C) costs of introducing a new food technology into a developing country
(D) Anglo-American bias of those trying to alleviate world food problems
(E) nature of the new technological innovations in the area of food production

Advance (v) : đề xướng, nâng cao

Maize (n) : ngô

Tortilla (n) : bánh

Substrate (n) : chất nền


Rye (n) : lúa mạch đen

Margin of Subsistence (n) : tỉ lệ sinh hoạt phí

Aug 15 – 2

In the year 1885, the Eiffel firm, which was named after the French engineer and architect Gustave
Eiffel and which had extensive experience in structural engineering, undertook a series of
investigations of tall metallic piers based upon its recent experiences with several railway viaducts
and bridges. The most spectacular of these was the famous Garabit Viaduct, which carries a railroad
some 400 feet above the valley of the Truyere in southern France. The design of this structure was
the inspiration for the design of a 395-foot pier, which, although never incorporated into a bridge, is
said to have been the direct basis for the Eiffel Tower. Preliminary studies for a 300-meter tower
were made with the intention of showcasing it in the 1889 fair called Exposition Universelle. With an
assurance born of positive knowledge, Eiffel in June of 1886 approached the Exposition
commissioners with the project. There can be no doubt that only the singular respect with which
Eiffel was regarded not only by his profession but by the entire nation motivated the Commission to
approve a plan which, in the hands of a figure of less stature, would have been considered grossly
impractical.

Between this time and the commencement of the Tower’s construction at the end of January 1887,
there arose one of the most persistently annoying of the numerous difficulties, both structural and
social, which confronted Eiffel as the project advanced. In the wake of the initial enthusiasm—on the
part of the fair’s Commission that was inspired by the desire to create a monument to highlight
French technological achievement, and on the part of the majority of French people by the stirring
of their imagination at the magnitude of the structure—there grew a rising movement of disfavor. At
the center of this movement was, not surprisingly, the intelligentsia, but objections were made by
prominent French people from all walks of life.

The most interesting point to be noted in a retrospection of this often violent opposition is that,
although every aspect of the Tower was attacked, there was remarkably little criticism of its
structural feasibility, either by the engineering profession or, as seems traditionally to be the case
with bold and unprecedented undertakings, by large numbers of the technically uninformed
population. True, there was an undercurrent of what might be characterized as unease by many
property owners in the structure’s shadow, but the most obstinate element of resistance was that
which deplored the Tower as a mechanistic intrusion upon the architectural and natural beauties of
Paris. This resistance voiced its fury in a flood of special newspaper editions, petitions, and
manifestos signed by such lights of the fine and literary arts as De Maupassant, Gounod, Dumas fils,
and others.

1. Based on the discussion of public opinion regarding the Eiffel Tower's construction it
can be inferred that

A. the poorer Parisians took to the Tower more readily than the educated classes
B. early in its planning, the Tower was generally embraced by most of the French people
C. those who disagreed with the plans for the Tower were mostly members of the intelligentsia
D. there was no disagreement about whether a 300-meter tower could be constructed
E. those living closest to the tower's site were the most supportive of it
2. Which faction does the author refer to when he mentions “undercurrent” in the last
paragraph?

A. those who lived around the Eiffel Tower and were concerned about the mechanical
stability of Eiffel Tower
B. those who lived around the Eiffel Tower and were technically uninformed
C. those who voiced their concerns regarding Eiffel Tower through manifestos and petitions
D. those who deplored the Eiffel Tower as a mechanistic intrusion upon the architectural and natural
beauties of Paris
E. those belonging to engineering profession who were concerned about the structural feasibility of
the Eiffel Tower

3. De Maupassant, Gounod, Dumas fils are mentioned by the passage in order to

A. overview the leadership structure of the fine and literary arts


B. list the names of the Tower's most steadfast opponents
C. explain who in French society was most interested in French architecture at that time
D. reveal how the creative and scientific communities were at odds over the Tower
E. clarify that distinguished members of the artistic community considered the Tower as
a possible eye-sore

4. Which of the following is the author’s primary purpose behind this passage?

A. To discuss the feasibility of the Eiffel Firm's plan for the Tower.
B. To teach the history of the Eiffel Tower's precursors
C. To explain how the Tower would not have been approved without Eiffel's reputation and
knowledge of structural engineering.
D. To detail how the public turned against the Tower in the months after the Exposition.
E. To give a general history of the Tower's construction and the public reaction to it.

Undertake (v) : đảm nhiệm

Metallic (adj) : kim loại

Pier (n) : bến tàu, cầu tàu, đập

Viaduct (n) : cầu cạn

Spectacular (adj) : đẹp mắt

Preliminary (adj) : sơ bộ

Showcasing (n) : trưng bày

Exposition (n) : triển lãm

Universelle (n) : đại học

Commissioner (n) : đặc ủy viên, nhân viên


Grossly (adv) : rõ ràng

Impractical (adj) : không thực tế

Monument (n) : tượng đài

Stirring (n) : chuyển động, dấu hiệu ban đầu của hành động

Magnitude (n) : độ lớn

Intelligentsia (n) : đội ngũ trí thức

Retrospection (n) : hồi tưởng

Feasibility (n) : tính khả thi

Unprecedented (adj) : chưa từng có

Uniformed (adj) : không hiểu

Undercurrent (n) : dòng chảy ngầm, bí mật, ảnh hưởng ngầm

Unease (adj) : khó chịu

Obstinate (adj) : cố chấp

Deplore (v) : không tán thành mạnh mẽ

Intrusion (n) : sự xen vào, sự thỏa thuận

Fury (n) : cơn giận, cuồng bạo

Manifesto (n) : tuyên ngôn

Steadfast (adj) : kiên định


Aug 16 – 1

The progress of astronomy has always been closely associated with the development and application
of mechanical processes and skill. Before the seventeenth century, the size of the planets could not
be measured, the satellites of the planets except the Earth’s moon were unknown, the phases of
Mercury and Venus were merely conjectured, and accurate positions of the sun, moon, and planets
among the stars, and of the stars among themselves, were impossible all because there were no
telescopes. More than a half century elapsed after the invention of the telescope before Picard
combined it with a graduated circle in such a way that the measurement of angles was greatly
improved. Then arose the necessity for accurate time. Although Galileo had learned the principles
governing the pendulum, astronomy had to wait for the mechanical genius of Huygens before a
satisfactory clock was invented around 1657.

Nearly all the large reflecting telescopes ever built were constructed by astronomers who possessed
great facility in practical mechanics. The rapid and significant advances in nearly all departments of
astronomy in the past century would not have been possible except through the skill and patience of
glassmakers, opticians, and engineers. The principles of spectrum analysis as formulated by Kirchho
allowed for the discovery of the elements composing every heavenly body. The deftness of
Wollaston showed that light could not be analyzed unless it first passed through a very narrow slit.
Even in our modern day the power of the telescope and spectroscope has been vastly extended by
the optical skill and mechanical dexterity of individuals such as Rowland, Hastings and Brashear, all
Americans.

An observatory site should have a fairly unobstructed horizon, as much freedom from cloud as
possible, good foundations for the instruments, and a very steady atmosphere. To understand the
necessity of a steady atmosphere, look at some distant outdoor object through a window on a hot
summer day. The object appears blurry and wavering. Similarly, currents of warm air continually rise
from the earth to upper regions of the atmosphere, and colder air comes down and rushes
underneath. Although these atmospheric movements are often invisible to the eye, their effect is
plainly visible in the telescope as a blurry distortion. In order for an ideally-designed telescope to
perform perfectly, it must be located in a perfect atmosphere. Otherwise its full power cannot be
employed. All hindrances of atmosphere are most advantageously avoided in arid or desert regions
of the globe, at elevations of 3000 to 10,000 feet above sea level. Higher mountains have as yet
been only partially investigated, and it is not known whether difficulties of occupying them
permanently would more than counterbalance the gain which greater elevation would afford.

1. According to the passage, which of the following is NOT true about the telescope?

A. Knowledge about the satellites of the planets was limited to a large extent because of the lack of
the knowledge gained from the use of telescopes.
B. A telescope at 10,000 feet in the mountains could work better than a telescope at 5,000 feet in
the desert.
C. A climate with frequent variations in the weather would make a telescope potentially less
accurate than a climate with monotonous weather.
D. Picard incorporated the graduated circle into the telescope around 1640.
E. A desert such as the Sahara would be an ideal location for a telescope due to its lack of extreme
atmospheric movement.

2. The primary purpose of this passage is to

A. detail the advancement of a specific scientific field


B. recontextualize the reader’s understanding of what it takes to develop a successful telescope
C. explore the early contributions made in the field of astronomy, and some of the mechanical
processes that contributed to that development
D. advocate for more research into telescope placement and development
E. describe inventions that helped astronomy and the ideal use of the telescope.

3. Which of the following is the function of the first paragraph?

A. To describe the most progressive phase in the history of astronomy.


B. To overview how astronomy has developed in the past 400 years.
C. To emphasize the value of specific inventions in the advancement of a certain
scientific field.
D. To discuss some early contributors to the field of astronomy.
E. To explain how far astronomy has come in a relatively short period of time.

4. It can be inferred from the passage that


A. without telescopes, accurate readings of the positions of celestial bodies are currently impossible.

Although the passage does state that accurate positions of the sun, moon, and planets among the
stars, and of the stars among themselves, were impossible all because there were no telescopes, it
gives us no information to state that presently there are no other devices that are capable of the
same.
B. even the most superlatively designed telescope possibly functions below its full
ability at 2000 feet above sea level.
C. almost all the great telescopes were built by astronomers who possessed excellent knowledge of
practical mechanics.
D. the atmospheric movements that cause blurry distortion in a telescope can never be detected by
the naked eye.
E. a telescope requires an observatory site in order to function.

Observatory (n) : đài quan sát

Horizon (n) : đường chân trời

Unobstructed (adj) : không bị cản trở

Distortion (n) : làm sai lệch, méo mó

Hindrance (n) : cản trở

Arid (adj) : khô khan

Optician (n) : bác sĩ nhãn khoa

Conjecture (v) : phỏng đoán


Pendulum (n) : con lắc

Dexterity (n) : khéo léo

Aug 16 – 2

In an unfinished but highly suggestive series of essays, the late Sarah Eisenstein has focused
attention on the evolution of working women’s values from the turn of the century to the First World
War. Eisenstein argues that turn-of-the-century women neither wholly accepted nor rejected what
she calls the dominant “ideology of domesticity,” but rather took this and other available
ideologies—feminism, socialism, trade unionism—and modified or adapted them in light of their own
experiences and needs. In thus maintaining that wage-work helped to produce a new
“consciousness” among women, Eisenstein to some extent challenges the recent, controversial
proposal by Leslie Tentler that for women the work experience only served to reinforce the
attractiveness of the dominant ideology. According to the Tentler, the degrading conditions under
which many female wage earners worked made them view the family as a source of power and
esteem available nowhere else in their social world. In contrast, Eisenstein’s study insists that wage-
work had other implications for women’s identities and consciousness. Most importantly, her work
aims to demonstrate that wage-work enabled women to become aware of themselves as a distinct
social group capable of defining their collective circumstance. Eisenstein insists that as a group
working-class women were not able to come to collective consciousness of their situation until they
began entering the labor force, because domestic work tended to isolate them from one another.

Unfortunately, Eisenstein’s unfinished study does not develop these ideas in sufficient depth or
detail, offering tantalizing hints rather than an exhaustive analysis. Whatever Eisenstein’s overall
plan may have been, in its current form her study suffers from the limited nature of the sources she
depended on. She uses the speeches and writings of reformers and labor organizers, who she
acknowledges were far from representative, as the voice of the typical woman worker. And there is
less than adequate attention given to the differing values of immigrant groups that made up a
significant proportion of the population under investigation. While raising important questions,
Eisenstein’s essays do not provide definitive answer, and it remains for others to take up the
challenges they offer.

1. The primary purpose of the passage is to

(A) criticize a scholar’s assumptions and methodology


(B) evaluate an approach to women’s study
(C) compare two sociological theories
(D) correct a misconception about feminist theory
(E) defend an unpopular ideology

2. It can be inferred from the passage that, in Eisenstein’s view, working women at the
turn of the century had which of the following attitudes toward the dominant ideology
of their time?

(A) They resented the dominant ideology as degrading.


(B) They preferred the dominant ideology to other available ideologies.
(C) They began to view the dominant ideology more favorably as a result of their experiences in the
labor force.
(D) They accepted some but not all aspects of the dominant ideology.
(E) They believed that the dominant ideology isolated them from one another.

3. Which of the following best describes the organization of the first paragraph of the
passage?

(A) A chronological account of a historical development is presented, and then future developments
are predicted.
(B) A term is defined according to several different schools of thought, and then a new definition is
formulated.
(C) A theory is presented, an alternative viewpoint is introduced, and then the reasoning
behind the initial theory is summarized.
(D) A tentative proposal is made, reasons for and against it are weighed, and then a modified
version of the proposal is offered.
(E) A controversy is described, its historical implications are assessed, and then a compromise is
suggested.

4. Which of the following would the author of the passage be most likely to approve as a
continuation of Eisenstein’s study?

(A) An oral history of prominent women labor organizers


(B) An analysis of letters and diaries written by typical female wage earners at the turn
of the century
(C) An assessment of what different social and political groups defined as the dominant ideology in
the early twentieth century
(D) A theoretical study of how socialism and feminism influenced one another at the turn of the
century
(E) A documentary account of labor’s role in the introduction of women into the labor force
Aug 17 – 1

Often it was believed that super intelligence leads to financial success, but a study by Malcolm
Gadwall identified determination as the most important predictor of success. It certainly helps to be
smart, but there are plenty of people as smart as Bill Gates who have achieved next to nothing. The
study further concluded that there are aspects called components of determination that are more
relevant than others, and more importantly, some of these components can be cultivated while
others are innate. One of these components – sheer willfulness, the desire to get something when
you want it, no matter what – is most important. Unfortunately, the study concluded that a good
deal of willfulness must be inborn because it is common to see families in which one sibling has
much more of it than the other sibling. Circumstances can alter it, but at the high end of the scale,
nature seems to be more important than nurture. But a strong willed person needs to be disciplined
and not self-indulgent, making discipline the second component of determination. Hence,
determination implies your willfulness is balanced by discipline. This is because the stronger your
will, the less anyone will be able to argue with you except yourself, and someone should argue with
you because everyone has base impulses, but if you have more will than discipline you'll just give
into these impulses and as a result end up on a local maximum such as drug addiction. Another
important thing that the study concludes is that discipline can be cultivated, and in fact does tend to
vary quite a lot in the course of an individual's life, and since determination is the product of will and
discipline, you can become more determined by being more disciplined. The last component that the
study discovered – another malleable one – is ambition. If willfulness and discipline are what get
you to your destination, ambition is how you choose it and the fact that ambition is malleable means
there is a lot you can do to increase it. Hence to summarize, determination consists of willfulness
balanced with discipline and aimed by ambition - fortunately at least two of these three qualities can
be cultivated.

1. The author is primarily concerned with

A. comparing and contrasting the roles played by intelligence and determination as a predictor of
financial success.
B. determining the most important components of determination that lead to financial success.
C. discussing the three components of determination that have proven to be the
strongest predictors of financial success.
D. proving that anyone who possesses sheer willfulness can be financially successful.
E. disproving the notion that one needs innate intelligence to be successful.

2. Why does the author cite the example of Bill Gates in the passage?

A. To prove that one does not need to be smart to be financially successful.


B. To cite an instance in which smart people are successful.
C. To emphasize his point that there are people smarter than Bill Gates.
D. To emphasize his point that determination trumps smartness as a predictor of
financial success.
E. To prove that Bill Gates was one of the most determined people in the world.
3. The author of the passage will most likely agree with which one of the personalities
to not have the potential to be financially successful?

A. Someone who loves money, is ambitious to the core, finishes something when he/she
decides and indulges in expensive gifts regardless of whether he/she can afford it.
B. Someone who is extremely strong headed yet disciplined but possesses little ambition.
C. Someone who is extremely malleable and disciplined and intends to be a CEO.
D. Someone who is not very determined because of their tendencies of giving into life’s desires yet
is so obstinate that he finishes something once he decides to regardless of the
circumstances.
E. Someone who has great ambition, is stubborn and headstrong and is willing to give up
everything to achieve his/her goals.

Discipline can be cultivated but willfulness must be inborn

Malleable (adj) : dễ uốn, dễ bảo, dễ sai khiến

Sibling (n) : anh chị em

Cultivate (v) : trau dồi

Obstinate (adj) : cố chấp, bướng bỉnh


Aug 17 – 2

Grassland songbirds often nest in the same grassland-wetland complexes as waterfowl, particularly
in a certain part of those complexes, namely, upland habitats surrounding wetlands. Although some
wildlife management procedures directed at waterfowl, such as habitat enhancement or restoration,
may also benefit songbirds, the impact of others, especially the control of waterfowl predators,
remains difficult to predict. For example, most predators of waterfowl nests prey opportunistically on
songbird nests, and removing these predators could directly increase songbird nesting success.
Alternatively, small mammals such as mice and ground squirrels are important in the diet of many
waterfowl-nest predators and can themselves be important predators of songbird nest. Thus,
removing waterfowl-nest predators could affect songbird nesting success through subsequent
increases in small-mammal populations.

In 1995 and 1996, researchers trapped and removed certain waterfowl nest predators, primary
raccoons and striped skunks, then observed subsequent survival rates for songbird nests.
Surprisingly. They observed no significant effect on songbird nesting success. This may be due to
several factors. Neither raccoons nor striped skunks consume ground squirrels, which are important
predators of songbird nests. Thus, their removal may not have led to significant increases in
populations of smaller predators. Additionally, both raccoons and striped skunks prefer wetlands and
spend little time in upland habitats; removing these species may not have increased the nesting
success of songbirds in the uplands enough to allow detection.

1. According to the passage, which of the following is true about the role played by
ground squirrels in the ecology of grassland-wetland complexes?

(A) While not important in the diet of raccoons or striped skunks, ground squirrels are a
significant source of food for other waterfowl-nest predators.
(B) Whereas ground squirrels are typically important as predators of songbird nests, their
opportunistic predation on waterfowl nests also has an observable effect on waterfowl nesting
success.
(C) Although most waterfowl-nest predators prey on small mammals such as mice and ground
squirrels, populations of ground squirrels tend to increase quickly enough to compensate for this
level of predation.
(D) Although ground squirrels have been known to prey on songbird nests, a larger portion of their
diets is usually provided by predation on waterfowl nests.
(E) Since larger predators tend to prefer small mammals to songbird eggs as a food source, a large
population of ground squirrels plays an important role in controlling opportunistic predation on
songbird nests.

2. Which of the following best describes the function of the sentence “Neither
raccoons…songbird nests” in the context of the passage as a whole?

(A) It raises questions about the validity of a theory described in the first paragraph.
(B) It points out an oversimplification that is inherent in the argument presented in the first
paragraph.
(C) It introduces information that may help explain the results of the experiment that
are presented earlier in the paragraph.
(D) It provides a specific example of the type of data collected in the experiment described earlier in
the paragraph.
(E) It anticipates a potential objection to the conclusions drawn by the researchers involved in the
experiment described earlier in the paragraph.

3. The primary purpose of the passage is to

(A) describe some procedures used for wildlife management and consider some problems associated
with the execution of those procedures
(B) outline a problem related to a wildlife management procedure and offer potential
explanations for the results of an experiment bearing on that problem
(C) present experimental results that illustrate the need for certain wildlife management procedures
and point out some inconsistencies in those results
(D) argue that a certain procedure used for wildlife management should be modified because of its
unintended consequences
(E) propose that further experiments be performed to assess the long-term effects of certain wildlife
management procedures

4. The passage suggests that removing waterfowl-nest predators could possibly have a
negative effect on songbird populations because

(negative effect = ảnh hưởng tiêu cực, không phải là ảnh hưởng âm )

(A) songbird populations could then grow to unsustainable numbers


(B) small-mammal population could then move out of the uplands into wetland areas
(C) competition among remaining waterfowl-nest predators could decrease significantly
(D) a resulting increase in waterfowl populations could crowd out songbird populations
(E) a resulting increase in small-mammal populations could increase small-mammal
predation on songbirds

5. It can be inferred that the habitat preferences of raccoons and striped skunks
affected the results of the experiment described in the passage for which of the
following reasons?

(A) Songbird nests in the wetlands are usually located in places that most waterfowl-nest predators
cannot reach.
(B) Raccoons and striped skunks are not usually found in areas where songbird nests
tend to be located.
(C) Mice and ground squirrels tend to avoid predation by raccoons and striped skunks by remaining
exclusively in the uplands.
(D) The populations of small mammals in the wetlands are usually controlled by larger waterfowl-
nest predators such as raccoons and striped skunks.
(E) The waterfowl on which raccoons and striped skunks prey in the wetlands compete with
songbirds for food.

Waterfowl (n) : chim nước


Detection (n) : phát hiện, khám phá

Aug 18 – 1

Origami is capable of turning a simple sheet of paper into a pretty paper crane, but the principles
behind the paper-folding art can also be applied to making a microfluidic device for a blood test, or
for storing a satellite's solar panel in a rocket's cargo bay. A team of researchers is turning kirigami,
a related art form that allows the paper to be cut, into a technique that can be applied equally to
structures on those vastly divergent length scales. The researchers lay out the rules for folding and
cutting a hexagonal lattice, a structure made from strips of material that cross over each other with
spaces between, into a wide variety of useful three-dimensional shapes.

A hexagonal lattice may seem like an odd choice for a starting point, but the researchers think that
the pattern has advantages over a seemingly simpler tessellation, such as one made from squares;
for instance, it is easier to fill a space with a hexagonal lattice and move from 2-D to 3-D. Starting
from a flat hexagonal grid on a sheet of paper, the researchers outlined the fundamental cuts and
folds that allow the resulting shape to keep the same proportions of the initial lattice, even if some
of the material is removed. This is a critical quality for making the transition from paper to materials
that might be used in real-world applications.

Having a set of rules that draws on fundamental mathematical principles means that the kirigami
approach can be applied equally across length scales, and with almost any material that can be
selected on the basis of its relevance to the ultimate application, whether it is in nanotechnology,
architecture, or aerospace.The rules also guarantee that "modules," basic shapes such as channels
that can direct the flow of fluids, can be combined into more complex ones. Kirigami is particularly
attractive for nanoscale applications, where the simplest, most space-efficient shapes are necessary,
and self-folding materials would circumvent some of the fabrication challenges inherent in working
with other materials at such small scales.

1. Which of the following most aptly describes the function of the second paragraph?

A. To introduce a discussion on the most critical aspect of the technique that dictates the
transformation of origami from paper to potentially real structures
B. To highlight the scope of kirigami across various applications
C. To suggest how maintaining a quality of the technique of kirigami is key to its application to
various materials
D. To evaluate some key features of the technique of kirigami
E. To analyze what makes kirigami a successful technique for fabricating materials

The author does talk about this quality but this mention forms part of a bigger discussion of the
features of the technique; for instance, the author also talks about the benefit of the hexagonal
lattice vi-a-vis some other shapes.
2. Which of the following statements would the author most likely agree with?

A. Origami as an art form is less flexible than kirigami, making it less relevant for building
nanostructures.
B. In nanoscale projects, certain building materials can be tricky to work with as they
are relatively rigid for the purpose of shaping.
C. It is very difficult to fill in spaces in structures formed of patterns other than the hexagonal
lattice.
D. An important quality for transforming paper designs to materials that could be used in real-world
structures is that the shape of the hexagonal grid should not change.
E. The type of material to be used changes drastically with the change in the ultimate application.

3. The author is primarily concerned with

A. discussing the advantages and disadvantages of a technique


B. explaining how a technique is more than just a superficial art form
C. highlighting the relevance of a particular pattern in the application of a technique to building
structures
D. discussing revolutionary techniques for building 3-D structures
E. discussing the potential of a technique for building varied physical structures

Circumvent (v) : phá vỡ

Fabrication (n) : sự cấu tạo

Inherent (adj) : vốn có

Lattice (n) : lưới

Hexagonal (adj) : lục giác

Odd (adj) : kỳ lạ
Aug 18 – 2

The majority of successful senior managers do not closely follow the classical rational model of first
clarifying goals, assessing the problem, formulating options, estimating likelihoods of success,
making a decision, and only then taking action to implement the decision. Rather, in their day-by-
day tactical maneuvers, these senior executives rely on what is vaguely termed “intuition” to
manage a network of interrelated problems that require them to deal with ambiguity, inconsistency,
novelty, and surprise; and to integrate action into the process of thinking.

Generations of writers on management have recognized that some practicing managers rely heavily
on intuition. In general, however, such writers display a poor grasp of what intuition is. Some see it
as the opposite of rationality; others view it as an excuse for capriciousness.

Isenberg’s recent research on the cognitive processes of senior managers reveals that managers’
intuition is neither of these. Rather, senior managers use intuition in at least five distinct ways. First,
they intuitively sense when a problem exists. Second, managers rely on intuition to perform well-
learned behavior patterns rapidly. This intuition is not arbitrary or irrational, but is based on years of
painstaking practice and hands-on experience that build skills. A third function of intuition is to
synthesize isolated bits of data and practice into an integrated picture, often in an “Aha!”
experience. Fourth, some managers use intuition as a check on the results of more rational analysis.
Most senior executives are familiar with the formal decision analysis models and tools, and those
who use such systematic methods for reaching decisions are occasionally leery of solutions
suggested by these methods which run counter to their sense of the correct course of action.
Finally, managers can use intuition to bypass in-depth analysis and move rapidly to engender a
plausible solution. Used in this way, intuition is an almost instantaneous cognitive process in which a
manager recognizes familiar patterns. One of the implications of the intuitive style of executive
management is that “thinking” is inseparable from acting. Since managers often “know” what is
right before they can analyze and explain it, they frequently act first and explain later. Analysis is
inextricably tied to action in thinking/acting cycles, in which managers develop thoughts about their
companies and organizations not by analyzing a problematic situation and then acting, but by acting
and analyzing in close concert.

Given the great uncertainty of many of the management issues that they face, senior managers
often instigate a course of action simply to learn more about an issue. They then use the results of
the action to develop a more complete understanding of the issue. One implication
of thinking/acting cycles is that action is often part of defining the problem, not just of implementing
the solution.

1. The passage provides support for which of the following statements?

(A) Managers who rely on intuition are more successful than those who rely on formal decision
analysis.
(B) Managers cannot justify their intuitive decisions.
(C) Managers’ intuition works contrary to their rational and analytical skills.
(D) Logical analysis of a problem increases the number of possible solutions.
(E) Intuition enables managers to employ their practical experience more efficiently.

2. According to the passage, the classical model of decision analysis includes all of the
following EXCEPT

(A) evaluation of a problem


(B) creation of possible solutions to a problem
(C) establishment of clear goals to be reached by the decision
(D) action undertaken in order to discover more information about a problem
(E) comparison of the probable effects of different solutions to a problem

3. It can be inferred from the passage that which of the following would most probably
be one major difference in behavior between Manager X, who uses intuition to reach
decisions, and Manager Y, who uses only formal decision analysis?

(A) Manager X analyzes first and then acts; Manager Y does not.
(B) Manager X checks possible solutions to a problem by systematic analysis; Manager Y does not.
(C) Manager X takes action in order to arrive at the solution to a problem; Manager Y
does not.
(D) Manager Y draws on years of hands-on experience in creating a solution to a problem; Manager
X does not.
(E) Manger Y depends on day-to-day tactical maneuvering; manager X does not.

4. According to the passage, senior managers use intuition in all of the following ways
EXCEPT to

(A) speed up of the creation of a solution to a problem


(B) identify a problem
(C) bring together disparate facts
(D) stipulate clear goals
(E) evaluate possible solutions to a problem

5. The passage suggests which of the following about the “writers on


management” mentioned in line 12?

(A) They have criticized managers for not following the classical rational model of decision analysis.
(B) They have not based their analyses on a sufficiently large sample of actual managers.
(C) They have relied in drawing their conclusions on what managers say rather than on what
managers do.
(D) They have misunderstood how managers use intuition in making business decisions.
(E) They have not acknowledged the role of intuition in managerial practice.

6. Which of the following best exemplifies “an ‘Aha!’ experience” (line 28) as it is
presented in the passage?

(A) A manager risks taking an action whose outcome is unpredictable to discover whether the action
changes the problem at hand.
(B) A manager performs well-learned and familiar behavior patterns in creative and uncharacteristic
ways to solve a problem.
(C) A manager suddenly connects seemingly unrelated facts and experiences to create a
pattern relevant to the problem at hand.
(D) A manager rapidly identifies the methodology used to compile data yielded by systematic
analysis.
(E) A manager swiftly decides which of several sets of tactics to implement in order to deal with the
contingencies suggested by a problem.

7. It can be inferred from the passage that “thinking/acting cycles” (line 45) in
managerial practice would be likely to result in which of the following?

I. A manager analyzes a network of problems and then acts on the basis of that analysis.
II. A manager gathers data by acting and observing the effects of action.
III. A manager takes action without being able to articulate reasons for that particular action.

(A) I only
(B) II only
(C) I and II only
(D) II and III only
(E) I, II, and III

8. Which of the following best describes the organization of the first paragraph of the
passage?

(A) An assertion is made and a specific supporting example is given.


(B) A conventional model is dismissed and an alternative introduced.
(C) The results of recent research are introduced and summarized.
(D) Two opposing points of view are presented and evaluated.
(E) A widely accepted definition is presented and qualified.

Maneuver (n) : diễn tập

Intuition (n) : trực giác

Novelty (n) : mới lạ

Grasp (n) : sự hiểu biết

Rationality (n) : sự hợp lý

Capriciousness (n) : sự bất thường

Behavior pattern (n) : mô hình hành vi

Arbitrary (adj) : tùy ý, độc đoán, tự tiện

Irrational (adj) : phi lý

Painstacking (adj) : siêng năng

Synthesize (v) : tổng hợp

Plausible (adj) : có lý
Instantaneous (adv) : tức thời

Inseparable (adj) : không thể tách rời

Inextricably (adv) : gắn bó chặt chẽ

Instigate (v) : khởi xướng, xúi giục


Aug 19 – 1

Typically managed by small groups such as husband-and-wife teams and characterized by


insignificant growth opportunities, lifestyle businesses are far less attractive to outside investors
than growth businesses are. Lifestyle businesses mostly expand their capital through debt financing
as equity financing is much more difficult for them to attract. Even when equity financing is an
option, it is mostly because of the investments made by family and friends in the business.

However, outside investors’ lack of relative interest does not indicate that such businesses are not
profitable. All it means is that these companies do not necessarily have aspirations of rapid growth
in revenue and as such their goals are indicative of their prime aim of sustaining a particular level of
income that provides a basis to the founders of such companies to enjoy a particular lifestyle -
hence the name, lifestyle business. In fact, in that sense, a lifestyle business doesn’t have to be
small at all, either in revenue or employees.

Although lifestyle businesses do share some features with other types of businesses, their success is
not dependent on the scalability of their business models, which is an important factor for many
investor funded businesses including start-up companies typically founded by close family and
friends. Also, the focus of the founders in investor funded businesses is on building the equity value
of the company so that they can harvest their investment and labor through the sale of the
company in the short term. Whereas, even when the equity value of a lifestyle company is modest,
it can continue to provide very high salaries to the founders of the company who can choose to keep
it running as long as they want to.

1. The author implies that which of the following statements about lifestyle businesses
is true?

A. Mostly only these businesses are managed by small groups such as life partners.
B. The revenues of such businesses do not necessarily represent the aspirations of the founders of
such businesses.
C. Debt financing and equity financing by insiders are the only options available to such businesses
looking for capital.
D. In the short run, factors guiding the efforts of the founders of investor funded
companies could differ from those of life-style businesses.
E. It is very rare for the founders of such businesses to sell them in the short term.

2. The author is primarily concerned with

A. describing how lifestyle businesses provide less hectic lifestyles for their founders.
B. explaining how difficult it is for lifestyle businesses to obtain capital from outside investors
C. suggesting how lifestyle businesses are concerned with matters not entirely dissimilar from other
business types
D. advocating how lifestyle businesses are equally profitable as other businesses despite the limited
interest by outside investors
E. explaining various features of lifestyle businesses while differentiating them from
other business types
3. Which of the following is the function of the final paragraph in the passage?

A. To contrast lifestyle businesses from other business types.


B. To describe the similarities and dissimilarities between investor funded businesses and life-style
businesses.
C. To indicate how most of the factors determining the success of a life-style business are internal
and not external.
D. To discuss a feature of lifestyle businesses in detail.
E. To suggest how the efforts of lifestyle businesses differ from those of investor funded businesses
in the short-run.

4. Which of the following would the author most likely agree with?

A. The potential for fast growth in revenues is not synonymous with overall profitability
of a business.
B. Although lifestyle businesses are fundamentally different from investor funded businesses, most
of their features are common with those of other types of businesses.
C. Lifestyle businesses are generally limited in their scale because of their founders' modest
aspirations.
D. The fields entered by lifestyle businesses are generally not found profitable by other types of
businesses such as those funded by investors.
E. For companies typically founded by friends and family, equity value is not a prime concern.

Characterize (v) : đặc trưng, biểu thị đặc biệt

Insignificant (adj) : tầm thường

Growth business (n) : doanh nghiệp tăng trưởng

Debt financing (n) : nợ tài chính

Equity financing (n) : vốn chủ sở hữu

Aspiration (n) : nguyện vọng

Indicative (adj) : biểu thị

Scalability (n) : khả năng mở rộng

Equity value (n) : giá trị vốn cổ phần

Modest (adj) : khiêm tốn

Synonymous (adj) : đồng nghĩa


Aug 19 – 2

Despite the tremendous technological advances of the modern era, scholars of military strategy still
study the tactics of a general whose armies at one time dominated all of continental Europe:
Napoleon. In general, Napoleon’s tactics were based on innovation and the element of surprise. His
main objective was the complete destruction of the enemy’s main body. He achieved this feat in
different ways according to the situation at hand, but the general principles behind his tactics were
the same: keep the enemy at a disadvantage and maintain the initiative.

In battle, Napoleon’s tactics often diverged from the accepted battle plans of the day, one of the
reasons for his immense success. Where the situation allowed, he often severed enemy supply lines
and communications, creating confusion and putting his enemy at a significant disadvantage before
the battle even began. When Napoleon had the luxury of superior numbers, one of his maneuvers
was to create a diversion with some of his troops while secretly moving most of his forces around
the enemy, enveloping the opposing army. Whenever his forces were outnumbered, he relied on
other tactics, one of which was to wedge his army between two concentrations of enemy troops. His
forces could then converge on one group at a time, in each case enjoying superior numbers.

Napoleon’s victories in battle did not result only from the tactical surprises he inflicted on his enemy
on the battlefield; Napoleon also made notable changes in the manner in which his armies
functioned off the battlefield. Most notably, his soldiers learned to live off the land instead of
dragging around mile-long baggage trains, as was the custom at the time. This mobility allowed his
armies to march faster than the other armies of the day, giving him the ability to make surprising
strategic moves that were so crucial for his victories in battle.

1) Which of the following, if true, would most weaken the author's statement that
Napoleon's tactics were based on innovation and the element of surprise?

A) Napoleon's enemies often used his attack patterns when fighting against him after their allies had
been defeated by Napoleon using those same tactics.
B) Napoleon was eventually defeated by the unexpected attack of several armies of countries he
had previously occupied.
C) Further studies of Napoleon's battle plans show that he always planned every attack carefully and
never strayed from those plans during battle.
D) Memoirs of Napoleon's enemies often reported that Napoleon's troops were unpredictable in
battle and performed new and unexpected feats.
E) Battle plans of a British general were found showing a planned attack on Napoleon's
troops using a strategy previously thought to be one of Napoleon's design.

2) What may be inferred about other armies during Napoleon's time from the tactics
Napoleon used to fight against them?

A) They lacked resourceful and creative generals.


B) Their weaponry was not as effective as that of Napoleon.
C) They never defeated Napoleon.
D) They were not self sufficient.
E) They were no different in structure from Napoleon's armies.
3) Which of the following is stated in the passage as Napoleon's goal in warfare?

A) obtaining wealth and power


B) surprising the enemy
C) destroying most of the enemy's troops
D) dominating all of continental Europe
E) maintaining the initiative

4) The author mentions the general principles behind Napoleon's tactics to

A) show that, despite the variety of methods he used, Napoleon had a system
underlying his decisions
B) show that Napoleon always operated on the basis of well defined plans
C) compare Napoleon's actions with the accepted battle plans of the day
D) point out that Napoleon was not as flexible in his choice tactics as is commonly presumed
E) argue that Napoleon's tactics were the result of previous planning and not reactions to the
activity of his enemies

Initiative (n) : chủ động

Diverge (v) : phân ra, chia ra, chuyển hướng

Severe (v) : cắt đứt

Maneuver (n) : diễn tập

Diversion (n) : nghi binh

Envelop (v) : bao bọc

Converge (v) : hội tụ

Baggage (n) : hành lý


Aug 20 – 1

Recent studies on Native American languages point towards an alarming problem in the United
States today. Language loss, a global phenomenon, is accelerating among indigenous groups in the
United States. A large majority of Native American vernaculars are spoken only by elders and the
remainder are fast approaching that status, as growing numbers of children speak only English.

To many, the precedence of one “common” language seems like an achievement of globalization
and hence they argue that it would be wiser to spend resources on improving the English speaking
skills of Native Americans rather than resuscitating fading tongues. However, no language is just a
collection of words and, therefore, languages are not so simply substitutable for each other. Each
language is a unique tool for analyzing and synthesizing the world and to lose such a tool is to
forget a way of constructing reality, blotting out a perspective evolved over many generations.
Native American languages express ideas on which Native American cultures are anchored; a native
language does not just reflect a culture - in a functional sense it is the culture. These languages are
based on entirely different histories, scientific and natural-world understandings, spiritual beliefs,
and political and legal ideas. They capture concepts that do not exist in English. In essence, they are
based on different realities.

Realising the magnitude of this language-loss, most indigenous tribes today are making some type
of effort toward a language-comeback. These efforts include everything from instituting
apprenticeship programs, which pair a fluent elder with a student, to using, what may seem like an
unusual tool because of the inherent reservations in Native American communities to being
photographed or recorded in any form, technology such as YouTube videos of native speakers or
Google Hangout video chats for live, long-distance conversations. The idea is to engage the younger
members of the tribe who, in their effort to fit into the more popular culture, are quickly losing ties
with their unique heritage.

1. Which of the following statements would the author most likely agree with?

A. Because some Native American communities are inherently averse to being photographed or
recorded in any form, their languages are at an even greater risk of reaching extinction.
B. Same events can be understood differently across cultures, cultures which do not
necessarily share the same contexts.
C. Globalization has had many unintended consequences such as the dominance of English as a
language over some ethnic minorities.
D. Although language loss is a global phenomenon, the scale at which it has affected Native
American languages is probably the most severe.
E. The younger generation of Native Americans is not as aware of the near extinct state of Native
American languages as the older generation is.

Each language is a unique tool… do not exist in English.

The author then goes on to say that these Native American languages capture concepts that do
not exist in English. From this analysis, we can understand that the author would agree that
perspectives are context driven and accordingly, across cultures same events could be
viewed/interpreted differently.

2. The author is primarily concerned with


A. To explain the nature of a problem and certain remedial measures being taken for it
B. To present two contradictory perspectives on a problem
C. To explain a problem while proposing a solution to it
D. To corroborate the findings of certain recent studies
E. To appreciate the efforts made to rectify a situation that is not normally seen as a problem

3. Which of the following can be inferred from the statement below?

...a native language does not just reflect a culture - in a functional sense it is the culture.

A. A native language does not by itself stand for a culture.


B. Native American cultures are rooted in ideas that do not exist in some of the other popular
languages.
C. Even when a native language does not portray a culture, it could be the culture itself.
D. The culture a native language reflects is more than the ideas expressed in it.
E. A native language is synonymous with the culture it is spoken in.

4. Which of the following is NOT mentioned in the passage?

A. Majority of the Native American tribes are aware of the scale of the problem of language-loss in
their culture.
B. There are more chances of engaging Native American youth with their culture
through the use of technology than through other means.
C. Most of the Native American languages today are spoken only by the elders of the indigenous
communities.
D. Use of audio-visual mediums by Native Americans to resuscitate their languages could seem like
an odd choice.
E. More than a few people think that language loss in Native American cultures is not really a
problem.

Realising the magnitude of this language-loss, most indigenous tribes today are making
some type of effort toward a language-comeback.
Aug 20 – 2

Two modes of argumentation have been used on behalf of women’s emancipation in Western
societies. Arguments in what could be called the “relational” feminist tradition maintain the doctrine
of “equality in difference,” or equity as distinct from equality. They posit that biological distinctions
between the sexes result in a necessary sexual division of labor in the family and throughout society
and that women’s procreative labor is currently undervalued by society, to the disadvantage of
women. By contrast, the individualist feminist tradition emphasizes individual human rights and
celebrates women’s quest for personal autonomy, while downplaying the importance of gender roles
and minimizing discussion of childbearing and its attendant responsibilities.

Before the late nineteenth century, these views coexisted within the feminist movement, often
within the writings of the same individual. Between 1890 and 1920, however, relational feminism,
which had been the dominant strain in feminist thought, and which still predominates among
European and non-Western feminists, lost ground in England and the United States. Because the
concept of individual rights was already well established in the Anglo-Saxon legal and political
tradition, individualist feminism came to predominate in English-speaking countries. At the same
time, the goals of the two approaches began to seem increasingly irreconcilable. Individualist
feminists began to advocate a totally gender-blind system with equal rights for all. Relational
feminists, while agreeing that equal educational and economic opportunities outside the home
should be available for all women, continued to emphasize women’s special contributions to society
as homemakers and mothers; they demanded special treatment for women, including protective
legislation for women workers, state-sponsored maternity benefits, and paid compensation for
housework.

Relational arguments have a major pitfall: because they underline women’s physiological and
psychological distinctiveness, they are often appropriated by political adversaries and used to
endorse male privilege. But the individualist approach, by attacking gender roles, denying the
significance of physiological difference, and condemning existing familial institutions as hopelessly
patriarchal, has often simply treated as irrelevant the family roles important to many women. If the
individualist framework, with its claim for women’s autonomy, could be harmonized with the family-
oriented concerns of relational feminists, a more fruitful model for contemporary feminist politics
could emerge.

1. The author of the passage alludes to the well-established nature of the concept of
individual rights in the Anglo-Saxon legal and political tradition in order to

(A) illustrate the influence of individualist feminist thought on more general intellectual trends in
English history
(B) argue that feminism was already a part of the larger Anglo-Saxon intellectual tradition, even
though this has often gone unnoticed by critics of women’s emancipation
(C) explain the decline in individualist thinking among feminists in non-English-speaking countries
(D) help account for an increasing shift toward individualist feminism among feminists
in English-speaking countries
(E) account for the philosophical differences between individualist and relational feminists in English-
speaking countries
2. The passage suggests that the author of the passage believes which of the following?

(A) The predominance of individualist feminism in English-speaking countries is a historical


phenomenon, the causes of which have not yet been investigated.
(B) The individualist and relational feminist views are irreconcilable, given their theoretical
differences concerning the foundations of society.
(C) A consensus concerning the direction of future feminist politics will probably soon emerge, given
the awareness among feminists of the need for cooperation among women.
(D) Political adversaries of feminism often misuse arguments predicated on differences
between the sexes to argue that the existing social system should be maintained.
(E) Relational feminism provides the best theoretical framework for contemporary feminist politics,
but individualist feminism could contribute much toward refining and strengthening modern feminist
thought.

3. It can be inferred from the passage that the individualist feminist tradition denies the
validity of which of the following causal statements?

(A) A division of labor in a social group can result in increased efficiency with regard to the
performance of group tasks.
(B) A division of labor in a social group causes inequities in the distribution of opportunities and
benefits among group members.
(C) A division of labor on the basis of gender in a social group is necessitated by the
existence of sex-linked biological differences between male and female members of the
group.
(D) Culturally determined distinctions based on gender in a social group foster the existence of
differing attitudes and opinions among group members.
(E) Educational programs aimed at reducing inequalities based on gender among members of a
social group can result in a sense of greater well-being for all members of the group.

4. According to the passage, relational feminists and individualist feminists agree that

(A) individual human rights take precedence over most other social claims
(B) the gender-based division of labor in society should be eliminated
(C) laws guaranteeing equal treatment for all citizens regardless of gender should be passed
(D) a greater degree of social awareness concerning the importance of motherhood would be benefi
cial to society
(E) the same educational and economic opportunities should be available to both sexes

5. According to the author, which of the following was true of feminist thought in
Western societies before 1890?
(A) Individualist feminist arguments were not found in the thought or writing of non-English-
speaking feminists.
(B) Individualist feminism was a strain in feminist thought, but another strain,
relational feminism, predominated.
(C) Relational and individualist approaches were equally prevalent in feminist thought and writing.
(D) The predominant view among feminists held that the welfare of women was ultimately less
important than the welfare of children.
(E) The predominant view among feminists held that the sexes should receive equal treatment
under the law.

6. The author implies that which of the following was true of most feminist thinkers in
England and the United States after 1920?

(A) They were less concerned politics than with intellectual issues.
(B) They began to reach a broader audience and their programs began to be adopted by
mainstream political parties.
(C) They called repeatedly for international cooperation among women’s groups to achieve their
goals.
(D) They moderated their initial criticism of the economic systems that characterized their societies.
(E) They did not attempt to unite the two different feminist approaches in their thought

Emancipation (n) : giải phóng

Doctrine (n) : giáo lý, đạo giáo, học thuyết

Equity (n) : công bằng

Equality (n) : bình đẳng

Division of labor (n) : phân công lao động

Procreative (adj) : sinh sản

Autonomy (n) : quyền tự trị, tự chủ

Chilbearing (n) : sinh con, có mang

Irreconcilable (adj) : không thể hòa giải

Pitfall (n) : cạm bẫy

Adversary (n) : đối thủ

Endorse (v) : chứng thực

Privilege (n) : đặc quyền

Condemn (v) : lên án

Harmonize (v) : hài hòa

Fruitful (adj) : hiệu quả

Strain (n) : khuynh hướng


Aug 21 – 1

Although some progress has been made in the last two decades on the treatment of Alzheimer’s
disease, the causes of the disease are still largely unknown. A recent study conducted by the
National Institute of Aging (NIA) appears to have found new evidence concerning the origins of
Alzheimer’s. Scientists who carried out the study at NIA focused on the correlation between a
patient’s genetic history, which scientists usually agree is the basis of Alzheimer’s, and the onset of
Alzheimer’s. The evidence cited by the NIA team includes the identification of the apolipoprotein E
(APOE) as the specific gene that may be responsible for the onset of Alzheimer’s. The APOE gene
can be sub-divided into many different forms, but the NIA research team has identified one specific
form of APOE, the APOE ?4 gene, as a very likely cause of Alzheimer’s. The presence of APOE ?4 is
known to make an individual particularly vulnerable to both cognitive and vascular dysfunction,
which are among the chief symptoms of Alzheimer’s.

However, a review of the study and its results by the Center for Disease Control and Prevention
(CDC) maintains that the evidence discovered by the NIA may not necessarily indicate that the
APOE ?4 gene is responsible for Alzheimer’s. CDC experts have pointed out that APOE ?4 is known
to cause conditions such as cardiac disease and diabetes in patients who have no genetic history of
Alzheimer’s and are not considered at risk for the disease. Further, the CDC review team has
observed that APOE ?4 has been found in individuals who show no symptoms of Alzheimer’s. In
response to the CDC’s review, the NIA team has pointed out that the APOE ?4 gene may be dormant
in some individuals, suggesting that the absence of Alzheimer’s in individuals who have the APOE ?4
gene cannot be used as evidence to rule out this gene as the cause of Alzheimer’s disease.

1. The primary purpose of the passage is to

A. present information that goes against a particular finding about a disease


B. propose further investigation to come to a conclusive result in a dispute about the cause of a
disease
C. resolve the controversy about the treatment of a disease
D. evaluate a dispute about whether genetic factors could be the basis of a disease
E. present the findings of a new study and debate over the same

2. The author makes which of the following statements about the relevance of genetic
history as the basis of Alzheimer’s disease?

A. The NIA is the first to claim it.


B. It is disputed by the CDC’s expert review team.
C. It is not generally disputed by scientists.
D. It has been challenged by the researchers at NIA.
E. It makes the NIA’s finding unlikely to be correct.
3. Which of the following claims does the NIA support by providing the information that
the APOE ?4 gene may be dormant in some individuals?

A. The presence of the APOE ?4 gene and the absence of Alzheimer’s in the same
individual does not weaken the claim that the APOE ?4 likely causes Alzheimer’s.
B. Individuals in whom the APOE ?4 gene is dormant are at high risk for Alzheimer’s disease.
C. The fact that individuals with dormant APOE ?4 gene show no sign of Alzheimer’s does not
indicate that genetic history is not the basis of the disease.
D. The absence of Alzheimer’s in individuals with the APOE ?4 gene makes it unlikely that APOE ?4 is
the cause of Alzheimer’s.
E. The dormancy of the APOE ?4 gene is an indication that this gene may cause cardiac disease and
diabetes even when it doesn’t cause Alzheimer’s.

4. Which of the following best describes the function of the first sentence of the second
paragraph?

A. It provides evidence to support the claim that the APOE ?4 gene is the likely cause of Alzheimer’s
disease.
B. It offers new evidence that contradicts the previously accepted belief that a person’s genetic
history is the basis of Alzheimer’s disease.
C. It states a position that does not favor the findings of a study conducted at the
National Institute of Aging.
D. It provides an opinion that, if true, could lead to more effective methods of treatment for
conditions such as cardiac disease and diabetes.
E. It suggests that methods of data collection used in the study conducted by the National Institute
of Aging were flawed.

Dormant (v) : ngủ đông

On set (n) : bệnh mới phát, khởi phát

Vascular (adj) : huyết quản, mạch máu

Dysfunction (n) : rối loạn chức năng


Aug 21 – 2

In an effort to explain why business acquisitions often fail, scholars have begun to focus on the role
of top executives of acquired companies. Acquired companies that retain their top executives tend to
have more successful outcomes than those that do not. Furthermore, existing research suggests
that retaining the highest-level top executives, such as the CEO (chief executive officer) and COO
(chief operating officer), is related more positively to postacquisition success than retaining lower-
ranked top executives. However, this explanation, while insightful, suffers from two limitations. First,
the focus on positional rank does not recognize the variation in length of service that may exist in
top executive posts across companies, nor does it address which particular top executives (with
respect to length of service) should be retained to achieve a successful acquisition outcome. Second,
the relationship between retained top executives and acquisition outcomes offered by existing
research is subject to opposing theoretical explanations related to length of service. The resource-
based view (RBV) suggests that keeping acquired company top executives with longer organizational
tenure would lead to more successful outcomes, as those executives have idiosyncratic and
nontransferable knowledge of the acquired company that would be valuable for the effective
implementation of the acquisition. The opposing position, offered by the upper echelons perspective
(UEP), suggests that retaining top executives having short organizational tenure would lead to more
successful outcomes, as they would have the adaptability to manage most effectively during the
uncertainty of the acquisition process.

Responding to these limitations, Bergh conducted a study of executive retention and acquisition
outcome that focused on the organizational tenure of retained company top executives in 104
acquisitions, followed over 5 years. Bergh considered the acquisition successful if the acquired
company was retained and unsuccessful if it was divested. Bergh’s findings support the RBV
position. Apparently, the benefits of long organizational tenure lead to more successful outcomes
than the benefits of short organizational tenure. While longer tenured top executives may have
trouble adapting to change, it appears that their perspectives and knowledge bases offer unique
value after the acquisition. Although from the UEP position it seems sensible to retain less tenured
executives and allow more tenured ones to leave, such a strategy appears to lower the probability of
acquisition success.

1.According to the passage, the research mentioned in line 6 suggests which of the
following about lower-ranked top executives and postacquisition success?

(A) Given that these executives are unlikely to contribute to postacquisition success, little effort
should be spent trying to retain them.
(B) The shorter their length of service, the less likely it is that these executives will play a significant
role in postacquisition success.
(C) These executives are less important to postacquisition success than are more highly
ranked top executives.
(D) If they have long tenures, these executives may prove to be as important to postacquisition
success as are more highly ranked top executives.
(E) Postacquisition success is unlikely if these executives are retained.
2.The resource-based view, as described in the passage, is based on which of the
following ideas?

(A) The managerial skills of top executives become strongest after the first five years of their tenure.
(B) Company-specific knowledge is an important factor in the success of an acquisition
process.
(C) The amount of nontransferable knowledge possessed by long-tenured top executives tends to be
underestimated.
(D) Effective implementation of an acquisition depends primarily on the ability of executives to adapt
to change.
(E) Short-tenured executives are likely to impede the implementation of a successful acquisition
strategy.

3.The passage suggests that Bergh and a proponent of the upper echelons perspective
would be most likely to disagree over which of the following?

(A) Whether there is a positive correlation between short organizational tenure and managerial
adaptability
(B) Whether there is a positive correlation between long organizational tenure and the acquisition of
idiosyncratic and nontransferable knowledge
(C) Whether adaptability is a useful trait for an executive who is managing an acquisition process
(D) Whether retaining less-tenured top executives of an acquired company is an optimal
strategy for achieving postacquisition success
(E) Whether retaining highest-level top executives of acquired companies is more important than
retaining lower-ranked top executives

We are not looking for a statement that BOTH Bergh and UEP would disagree with. Remember, each
answer choice starts with "whether" and thus represents multiple perspectives. Each answer choice
essentially presents a question.

For example, consider choice (D): "Whether retaining less-tenured top executives of an acquired
company is an optimal strategy for achieving post-acquisition success." In other words, is retaining
less-tenured top executives of an acquired company is an optimal strategy for achieving post-
acquisition success? Bergh/RBV-supporters would say, "No." UEP-supporters would say yes. Thus,
Bergh and UEP supporters would disagree over choice (D), making (D) our answer.

4.According to the passage, prior to Bergh’s study, research on the role of top
executives of acquired companies in business acquisition success was limited in which
of the following ways?

(A) It did not address how the organizational tenure of top executives affects
postacquisition success.
(B) It did not address why some companies have longer-tenured CEOs than others.
(C) It did not consider strategies for retaining long-tenured top executives of acquired companies.
(D) It failed to differentiate between the contribution of highest-level top executives to
postacquisition success and that of lower-ranked top executives.
(E) It underestimated the potential contribution that lower-level top executives can make to
postacquisition success.
Tenure (n) : nhiệm kỳ

Idiosyncratic (adj) : bình dị, tính riêng biệt

Nontransferable (adj) : không thể chuyển nhượng

Echelon (n) : cấp độ, cấp bậc trong một tổ chức, một nghề nghiệp hoặc xã hội
Aug 22 – 1

Characterized by change of ideals, Sanskritization traditionally denoted the process by which castes
placed lower in the caste hierarchy sought upward mobility by emulating the rituals and practices of
the upper castes. It became an inevitable feature of those castes that experienced a vast gap
between their ritual and politico-economic positions. Such castes enjoyed a high level of political and
economic power. Accordingly, economic elevation was not a sought- after goal through
Sanskritization. Infact, sometimes a group could start by acquiring political power, which led to
economic betterment and Sanskritization.

Although there is no unanimity on whether it has been a major process of cultural change in Indian
history, it has occurred almost in every part of the Indian sub-continent. However, the nature of the
process is by no means uniform as the cultural norms or customs being imitated may vary from
Sanskrit or Hindu traditional forms to tribal and even the Islamic patterns. Also, with the passage of
time, the boundaries of the definition have blurred and today Sanskritization is no longer limited to
the changes in the customs, rituals and ideology and way of life of a caste placed lower in the caste
hierarchy; it has now begun to stand for cross imitation of customs and way of life among different
social groups. In essence, Sanskritization is only an illustration of the operation of the ‘reference
group’ process. A reference group is a group that is used as a standard to evaluate one’s rituals,
attitudes, customs etc. As long as the caste has prestige in the social groups, it serves as a model.
Accordingly, a ruling group or caste will also serve as a model if it commands prestige in the society.

1. The author is primarily concerned with

A. To trace the development of a process across various sects of a religion


B. To mitigate potential criticism for a process by presenting a liberal perspective on it
C. To define a process by tracing its development over time
D. To define a process of social, cultural and political change by discussing its features
E. To enlist the basic features of a process

2. Which of the following statements would the author most likely agree with?

A. A group that has prestige and dominance in the society but ranks lower than the highest group in
the hierarchical order will quite likely not be a reference group.
B. Although upward social mobility could result out of Sanskiritization, economic elevation could not.
C. An affluent caste that enjoyed limited ritual position was extremely likely to
experience Sanskritization.
D. Even though Sanskritization could involve emulation of tribal or Islamic patterns, it now normally
involves adopting the ideologies of the dominant Hindu caste, which is not necessarily the highest
ranking caste.
E. There is no one opinion on the ability of Sanskritization to act as a major process of social
change.

3. Which of the following most aptly describes the function of the first paragraph?
A. To describe a rigid concept that has become more accepted over time
B. To introduce a process within a specific context
C. To define a process that was more politically motivated than economically
D. To introduce a phenomenon by discussing its limitations
E. To present a social scenario that accentuated hierarchical differences
4. In the passage, which of the following is NOT mentioned?
A. Sanskirtization was an unavoidable feature of castes that faced a huge gap between their ritual
and politico-economic positions.
B. The nature of Sanskritization is not the same everywhere.
C. Sanskritization now represents emulation of customs etc. across social groups.
D. Sanskritization has occurred in most of the Indian sub-continent.
E. A ruling group is synonymous with a reference group.
Synonymous (adj) : đồng nghĩa

Upward mobility (n) : sự tiến lên

Affluent (adj) : giàu có

Emulate (v) : bắt chước

Unanimity (n) : nhất trí

Prestige (n) : uy tín


Aug 22 – 2

No very satisfactory account of the mechanism that caused the formation of the ocean basins has
yet been given. The traditional view supposes that the upper mantle of the earth behaves as a liquid
when it is subjected to small forces for long periods and that differences in temperature under
oceans and continents are sufficient to produce convection in the mantle of the earth with rising
convection currents under the mid-ocean ridges and sinking currents under the continents.
Theoretically, this convection would carry the continental plates along as though they were on
a conveyor belt and would provide the forces needed to produce the split that occurs along the
ridge. This view may be correct: it has the advantage that the currents are driven by temperature
differences that themselves depend on the position of the continents. Such a back-coupling, in
which the position of the moving plate has an impact on the forces that move it,could produce
complicated and varying motions.

On the other hand, the theory is implausible because convection does not normally occur along
lines. and it certainly does not occur along lines broken by frequent offsets or changes in direction,
as the ridge is. Also it is difficult to see how the theory applies to the plate between the Mid-Atlantic
Ridge and the ridge in the Indian Ocean. This plate is growing on both sides, and since there is no
intermediate trench, the two ridges must be moving apart. It would be odd if the rising convection
currents kept exact pace with them. An alternative theory is that the sinking part of the plate, which
is denser than the hotter surrounding mantle, pulls the rest of the plate after it. Again it is difficult to
see how this applies to the ridge in the South Atlantic, where neither the African nor the American
plate has a sinking part.

Another possibility is that the sinking plate cools the neighboring mantle and produces convection
currents that move the plates. This last theory is attractive because it gives some hope of explaining
the enclosed seas, such as the Sea of Japan. These seas have a typical oceanic floor, except that
the floor is overlaid by several kilometers of sediment. Their floors have probably been sinking for
long periods. It seems possible that a sinking current of cooled mantle material on the upper side of
the plate might be the cause of such deep basins. The enclosed seas are an important feature of the
earth's surface, and seriously require explanation in because, addition to the enclosed seas that are
developing at present behind island arcs, there are a number of older ones of possibly similar origin,
such as the Gulf of Mexico, the Black Sea, and perhaps the North Sea.

1. According to the traditional view of the origin of the ocean basins, which of the
following is sufficient to move the continental plates?

(A) Increases in sedimentation on ocean floors


(B) Spreading of ocean trenches
(C) Movement of mid-ocean ridges
(D) Sinking of ocean basins
(E) Differences in temperature under oceans and continents.
2. It can be inferred from the passage that, of the following, the deepest sediments
would be found in the

(A) Indian Ocean


(B) Black Sea
(C) Mid-Atlantic
(D) South Atlantic
(E) Pacific

3. The author refers to a "conveyor belt" (highlighted text) in order to

(A) illustrate the effects of convection in the mantle.


(B) show how temperature differences depend on the positions of the continents.
(C) demonstrate the linear nature of the Mid-Atlantic Ridge.
(D) describe the complicated motions made possible by back-coupling.
(E) account for the rising currents under certain mid-ocean ridges.

4. The author regards the traditional view of the origin of the oceans with

(A) slight apprehension


(B) absolute indifference
(C) indignant anger
(D) complete disbelief
(E) guarded skepticism

5. According to the passage, which of the following are separated by a plate that is
growing on both sides?

(A) The Pacific Ocean and the Sea of Japan


(B) The South Atlantic Ridge and the North Sea Ridge
(C) The Gulf of Mexico and the South Atlantic Ridge
(D) The Mid-Atlantic Ridge and the Indian Ocean Ridge
(E) The Black Sea and the Sea of Japan

6. Which of the following, if it could be demonstrated, would most support the


traditional view of ocean formation?

(A) Convection usually occurs along lines.


(B) The upper mantle behaves as a dense solid.
(C) Sedimentation occurs at a constant rate.
(D) Sinking plates cool the mantle.
(E) Island arcs surround enclosed seas.
7. According to the passage, the floor of the Black Sea can best be compared to a

(A) rapidly moving conveyor belt


(B) slowly settling foundation
(C) rapidly expanding balloon
(D) violently erupting volcano
(E) slowly eroding mountain

8. Which of the following titles would best describe the content of the passage?

(A) A Description of the Oceans of the World


(B) Several Theories of Ocean Basin Formation
(C) The Traditional View of the Oceans
(D) Convection and Ocean Currents
(E) Temperature Differences Among the Oceans of the World

Basin (n) : lưu vực, lòng chảo

Mantle (n) : lớp phủ

Convection (n) : đối lưu

Ridge (n) : chõm núi

Implausible (adj) : không thể tin được

Erupt (v) : phun trào

Erode (v) : xói mòn

Apprehension (n) : e ngại

Indifference (n) : thờ ơ

Indignant (adj) : phẫn nộ, tức giận


Aug 23 – 1

Irrespective of the general state of the economy, paying less for the same thing appeals to most
consumers, who seem to have an intuitive dislike for inflation. But much as we may like to pay less,
a sustained decline in prices, deflation, may not be in our best interests. Deflation, in fact, is
considered to be a bigger evil than inflation, and evokes strong action by policymakers who would
try everything possible to prop up prices. Deflation often results from a slowdown in which reduction
in demand vis-à-vis supply causes prices to dip. With a sharp decline in prices, consumers tend to
postpone purchases in the belief that prices will head further lower. This adds to the pressure on
businesses, which in addition to a fall in prices also see an accumulation of inventory. Production
cuts are hence resorted to, resulting in factory closures and consequent layoffs or salary cuts. With
unemployment increasing, income levels in the economy fall, leading to further cuts in consumer
spending and more pressure on prices. A vicious cycle emerges; the cascade effect is felt across
sectors, and the economy goes into a defeatist mode.

To prevent deflations and to tackle the downward spiral caused by them, governments resort to
large-scale spending, undertaking massive projects to increase employment, incomes, and prices
and pumping in huge sums of money to propel demand. For instance, in the aftermath of the
financial market crash in 2008, the US government undertook big-ticket stimulus measures and QE
(quantitative easing) to revive the economy.

However, not all dips in prices are bad. A fall in prices of goods and services due to technological
innovations and increased competition could actually benefit both the consumers and the producers.
Such a situation is very different from deflation, which affects prices across the economy on a
sustained basis (mainly due to decreasing demand or liquidity problems) and which should indeed
be considered a red flag.

1.Which of the following describes the function of the final paragraph?

A. To raise a consideration to add to the understanding of a phenomenon


B.To differentiate between different kinds of situations that are mistaken for one another
C.To introduce a discussion on an aspect related to a phenomenon explained earlier in detail
D.To give information that is not conflicting with the definition of a phenomenon discussed earlier
E.To warn against a common generalization

2. Which of the following statements would the author most likely agree with?

A.Although inflation can be beneficial for the economy deflation cannot be.
B.The decision to buy a product later in the hope of benefitting from a further price
reduction is part of the cyclical chain of events that breeds deflation.
C.Deflation has little in common with dips in prices that are not substantially caused by decreasing
demand.
D.Production cuts undertaken due to accumulated inventory help producers deal with an excess
supply, leading to a momentary increase in prices.
E.Whenever supply exceeds current demand, deflation occurs.
3.As per the passage, which of the following would most likely be considered a dip in
prices beneficial to both the consumer and the producer?

A.Because of a breakthrough in diamond mining, the price of the diamond dips, with it becoming
affordable for more people but losing its unique selling appeal as a luxury item.
B.Because of the cancellation of an order, the producer reduces the price of the goods in stock
considerably and sells them to another buyer.
C.Because of a change in policy, producers can now procure raw materials for producing garments
at cheaper prices from nearby countries and hence save on their cost of production.
D.Because of an increasing competition from cheaply produced and imported goods, the local
producers reduce the prices of their goods to sell off their existing stock at initiate production cuts.
E.Because of a new technology, the manufacturers of personal computers are able to
reduce their cost of production and pass these savings to the consumers as well by
lowering the prices.

4.Which of the following is mentioned in the passage?

A.Most people have a natural dislike for deflation.


B.Those people who lose their jobs in deflation keep postponing their purchasing decisions in the
hope that prices will decrease further.
C.Large-scale spending by the government is one of the most effective ways to tackle deflation.
D.The financial market crash of 2008 was caused by deflation in the economy.
E.The effect of sustained price rise is considered less harmful than that of sustained
price fall.

Irrespective (adj) : không phân biệt

Deflation (n) : giảm phát

Stimulus (n) : kích thích kinh tế

Easing (v) : nới lỏng

Substantially (adv) : đáng kể


Aug 23 – 2

In the eighteenth century, Japan's feudal overlords, from the shogun to the humblest samurai,
found themselves under financial stress. In part, this stress can be attributed to (5) the overlords'
failure to adjust to a rapidly ex-panding economy, but the stress was also due to factors beyond the
overlords' control. Concentration of the samurai in castle-towns had acted as a stimulus to trade.
Commercial efficiency, in(10) turn, had put temptations in the way of buyers. Since most samurai
had been reduced to idleness by years of peace, encouraged to engage in scholarship and martial
exercises or to perform administrative tasks that took little time, it is (15) not surprising that their
tastes and habits grew expensive. Overlords' income, despite the increase in rice production among
their tenant farmers, failed to keep pace with their expenses. Although shortfalls in overlords'
income resulted (20) almost as much from laxity among their tax collectors (the nearly inevitable
outcome of hereditary officeholding) as from their higher standards of living, a misfortune like a fire
or flood, bringing an increase in expenses or a drop (25) in revenue, could put a domain in debt to
the city rice-brokers who handled its finances. Once in debt, neither the individual samurai nor
the shogun himself found it easy to recover. It was difficult for individual samurai overlords (30) to
increase their income because the amount of rice that farmers could be made to pay in taxes was
not unlimited, and since the income of Japan's central government consisted in part of taxes
collected by the shogun from his (35)huge domain, the government too was constrained. Therefore,
the Tokugawa shoguns began to look to other sources for revenue. Cash profits from government
owned mines were already on the decline because the most (40) easily worked deposits of silver
and gold had been exhausted, although debasement of the coinage had compensated for the loss.
Opening up new farmland was a possibility, but most of what was suitable had already been
exploited (45) and further reclamation was technically unfeasible. Direct taxation of the samurai
themselves would be politically dangerous. This left the shoguns only commerce as a potential
source of government income.

(50) Most of the country's wealth, or so it seemed, was finding its way into the hands of city
merchants. It appeared reasonable that they should contribute part of that revenue to ease
the shogun's burden of financing the state. A means (55) of obtaining such revenue was soon found
by levying forced ioans, known as goyo-kin; although these were not taxes in the strict sense, since
they were irregular in timing and arbitrary in amount, they were high in yield. Unfortunately, (60)
they pushed up prices. Thus, regrettably, the Tokugawa shoguns' search for solvency for
the government made it increasingly difficult for individual Japanese who lived on fixed stipends to
make ends meet.

1.The passage is most probably an excerpt from

(A) an economic history of Japan


(B) the memoirs of a samurai warrior
(C) a modern novel about eighteenth-century Japan
(D) an essay contrasting Japanese feudalism with its Western counterpart
(E) an introduction to a collection of Japanese folktales
2. Which of the following financial situations is most analogous to the financial situation
in which Japan’s Tokugawa shoguns found themselves in the eighteenth century?

(A) A small business borrows heavily to invest in new equipment, but is able to pay off its debt early
when it is awarded a lucrative government contract.
(B) Fire destroys a small business, but insurance covers the cost of rebuilding.
(C) A small business is turned down for a loan at a local bank because the owners have no credit
history.
(D) A small business has to struggle to meet operating expenses when its profits
decrease.
(E) A small business is able to cut back sharply on spending through greater commercial efficiency
and thereby compensate for a loss of revenue.

3. Which of the following best describes the attitude of the author toward the samurai
discussed in lines 11-16?

(A) Warmly approving


(B) Mildly sympathetic
(C) Bitterly disappointed
(D) Harshly disdainful
(E) Profoundly shocked

4. According to the passage, the major reason for the financial problems experienced by
Japan’s feudal overlords in the eighteenth century was that

(A) spending had outdistanced income


(B) trade had fallen off
(C) profits from mining had declined
(D) the coinage had been sharply debased
(E) the samurai had concentrated in castle-towns

Overlords' income, despite the increase in rice production among their tenant farmers, failed to keep
pace with their expenses

5. The passage implies that individual samurai did not find it easy to recover from debt
for which of the following reasons?

(A) Agricultural production had increased.


(B) Taxes were irregular in timing and arbitrary in amount.
(C) The Japanese government had failed to adjust to the needs of a changing economy.
(D) The domains of samurai overlords were becoming smaller and poorer as government revenues
increased.
(E) There was a limit to the amount in taxes that farmers could be made to pay.
6. The passage suggests that, in eighteenth-century Japan, the office of tax collector

(A) was a source of personal profit to the officeholder


(B) was regarded with derision by many Japanese
(C) remained within families
(D) existed only in castle-towns
(E) took up most of the officeholder’s time

7. Which of the following could best be substituted for the word “This” in line 47
without changing the meaning of the passage?

(A) The search of Japan’s Tokugawa shoguns for solvency


(B) The importance of commerce in feudal Japan
(C) The unfairness of the tax structure in eighteenth-century Japan
(D) The difficulty of increasing government income by other means
(E) The difficulty experienced by both individual samurai and the shogun himself in extricating
themselves from debt

8. The passage implies that which of the following was the primary reason why the
Tokugawa shoguns turned to city merchants for help in financing the state?

(A) A series of costly wars had depleted the national treasury.


(B) Most of the country's wealth appeared to be in city merchants' hands.
(C) Japan had suffered a series of economic reversals due to natural disasters such as floods.
(D) The merchants were already heavily indebted to the shoguns.
(E) Further reclamation of land would not have been economically advantageous.

9. According to the passage, the actions of the Tokugawa shoguns in their search for
solvency for the government were regrettable because those actions

(A) raised the cost of living by pushing up prices


(B) resulted in the exhaustion of the most easily worked deposits of silver and gold
(C) were far lower in yield than had originally been anticipated
(D) did not succeed in reducing government spending
(E) acted as a deterrent to trade

Feudal (adj) : chế độ

Overlord (n) : chế độ

Stimulus (n) : kích thích kinh tế

Temptation (n) : cám dỗ

Idleness (n) : nhàn rỗi

Tenant (adj) : người thuê

Laxity (n) : sự lỏng lẻo


Debasement (n) : giảm giá trị

Reclamation (n) : khai hoang

Unfeasible (adj) : không khả thi

Levy (v) : đánh thuế

Arbitrary (adj) : tùy ý

Solvency (n) : khả năng thanh toán

Stipend (n) : quy định, lương bổng

Disdainful (adj) : coi thường


Aug 24 – 1
A team of researchers has been able to successfully study the highly complex molecular structure of
mitoribosomes, which are the ribosomes of mitochondria. Ribosomes are found in the cells of all
living organisms, and they serve as a primary location for biological protein synthesis, but certain
organisms such as fungi, plants, animals, and humans contain much more complex ribosomes than
bacteria do. In organisms with complex cells, ribosomes can also be divided into two types: those in
the cytosol -- which comprises the majority of the cell -- and those found in the mitochondria or
"power houses" of cells. Mitochondria are found only in eukaryotes. Every ribosome consists of two
subunits. The smaller subunit uses transfer ribonucleic acids to decode the genetic code, which is
stored in the DNA, it receives in the form of messenger ribonucleic acids, while the larger subunit
joins the amino acids delivered by the transfer ribonucleic acids together like a string of pearls.

Since they are found only in small amounts and are difficult to isolate, mitochondrial ribosomes or
mitoribosomes are particularly difficult to study. But because of the recent technical advances in
cryo-electron microscopy and the development of direct electron detection cameras that can correct
for specimen motion during the exposure, it recently became possible to capture images of
biomolecules at a resolution high enough to capture the details, especially those of the peptidyl
transferase centre (PTC).

This research is of special importance to producing the right kind of antibiotics for humans. PTC is
where the amino acid building blocks are combined, leading to protein synthesis. As per the
researchers, this process of synthesizing proteins is medically relevant as the tunnel through which
the proteins pass, after being synthesized, is a target for specific antibiotics. The antibiotic blocks
the tunnel, preventing the proteins that have just been synthesized from leaving the tunnel.
However, for an antibiotic to be used in humans, it must not attack human ribosomes and should
inhibit protein synthesis only in the ribosomes of bacteria. The problem arises since mitochondrial
ribosomes resemble those of bacteria, which is why certain antibiotics also interfere with
mitoribosomes, possibly leading to serious side effects. The findings of the research will make it
possible in the future to design antibiotics that inhibit only bacterial and not mitochondrial
ribosomes, the one basic requirement for using them in clinical applications.

1. The author is primarily concerned with

A. To encapsulate the significance of a key medical research


B. To evaluate the achievements and failure of a study undertaken to understand the workings of
antibiotics on humans
C. To educate the reader about the functions of certain parts and how certain antibiotics can be
harmful to these parts
D. To discuss the relevance of a research while setting the context for it
E. To describe a key medical finding while discussing some of the key problems faced by the
pharmaceutical companies
2. Which of the following can be inferred from the passage?

A. The same antibiotic will have different reactions on humans and animals.
B. The pharmaceutical companies that have been producing antibiotics that inadvertently harm
people by blocking the release of the proteins will gain significant monetary benefit from producing
antibiotics that do not attack the human body in such a way.
C. The quantity of a specific kind of ribosomes was partly responsible for the limited
success in studying them.
D. Without the latest technical advances in cryo-electron microscopy, it would not have been
possible to study mitoribosomes at all.
E. Mitoribosomes are the only kind of ribosomes that are adversely affected by antibiotics.

Since they are found only in small amounts and are difficult to isolate, mitochondrial
ribosomes or mitoribosomes are particularly difficult to study

3. Which of the following is mentioned in the passage?

A. The attack on human ribosomes by certain antibiotics could prove fatal.


B. Ribosomes are solely responsible for producing protein in the body.
C. Ribosomes are defenseless against the attack launched by certain antibiotics on them.
D. All living organisms have ribosomes that can be divided into two types.
E. Ribosomes in humans are not as simple as the ones found in bacteria.

4. Which of the following most aptly describes the function of the first paragraph?

A. To discuss the overall functioning of the human body


B. To explain the scope of relevance of the mentioned research
C. To give details about two unrelated concepts
D. To mention an achievement while setting the context for it
E. To provide information about ribosomes particularly about mitoribosomes

The key between (D) and (E) is in the wording. (E) says "provide information about ribosomes" -
which is true, although it doesn't actually give a whole lot of information about ribosomes, which
makes it immediately suspect. The second half - "particularly about mitoribosomes" - is
demonstrably untrue. While mitoribosomes are mentioned in the first sentence, no other information
is given about them for the rest of the paragraph. For something to be a main purpose of a
paragraph, it needs to be mentioned more than just in passing.

Molecular (n) : phân tử

Mitochondria (n) : ty thể

Eukaryote (n) : sinh vật nhân chuẩn

Subunit (n) : tiểu đơn vị

Decode (v) : giải mã

String of pearl (n) : chuỗi ngọc trai

Specimen (n) : mẫu vật


Aug 24 – 2

Virtually everything astronomers known about objects outside the solar system is based on the
detection of photons—quanta of electromagnetic radiation. Yet there is another form of radiation
that permeates the universe: neutrinos. With (as its name implies) no electric charge, and negligible
mass, the neutrino interacts with other particles so rarely that a neutrino can cross the entire
universe, even traversing substantial aggregations of matter, without being absorbed or even
deflected. Neutrinos can thus escape from regions of space where light and other kinds of
electromagnetic radiation are blocked by matter. Furthermore, neutrinos carry with them
information about the site and circumstances of their production: therefore, the detection of cosmic
neutrinos could provide new information about a wide variety of cosmic phenomena and about the
history of the universe.
But how can scientists detect a particle that interacts so infrequently with other matter? Twenty-five
years passed between Pauli’s hypothesis that the neutrino existed and its actual detection: since
then virtually all research with neutrinos has been with neutrinos created artificially in large particle
accelerators and studied under neutrino microscopes. But a neutrino telescope, capable of detecting
cosmic neutrinos, is difficult to construct. No apparatus can detect neutrinos unless it is extremely
massive, because great mass is synonymous with huge numbers of nucleons (neutrons and
protons), and the more massive the detector, the greater the probability of one of its nucleon’s
reacting with a neutrino. In addition, the apparatus must be sufficiently shielded from the interfering
effects of other particles.

Fortunately, a group of astrophysicists has proposed a means of detecting cosmic neutrinos by


harnessing the mass of the ocean. Named DUMAND, for Deep Underwater Muon and Neutrino
Detector, the project calls for placing an array of light sensors at a depth of five kilometers under
the ocean surface. The detecting medium is the seawater itself: when a neutrino interacts with a
particle in an atom of seawater, the result is a cascade of electrically charged particles and a flash of
light that can be detected by the sensors. The five kilometers of seawater above the sensors will
shield them from the interfering effects of other high-energy particles raining down through the
atmosphere.

The strongest motivation for the DUMAND project is that it will exploit an important source of
information about the universe. The extension of astronomy from visible light to radio waves to x-
rays and gamma rays never failed to lead to the discovery of unusual objects such as radio galaxies,
quasars, and pulsars. Each of these discoveries came as a surprise. Neutrino astronomy will
doubtless bring its own share of surprises.

1. Which of the following titles best summarizes the passage as a whole?

(A) At the Threshold of Neutrino Astronomy


(B) Neutrinos and the History of the Universe
(C) The Creation and Study of Neutrinos
(D) The DUMAND System and How It Works
(E) The Properties of the Neutrino
2. With which of the following statements regarding neutrino astronomy would the
author be most likely to agree?

(A) Neutrino astronomy will supersede all present forms of astronomy.


(B) Neutrino astronomy will be abandoned if the DUMAND project fails.
(C) Neutrino astronomy can be expected to lead to major breakthroughs in astronomy.
(D) Neutrino astronomy will disclose phenomena that will be more surprising than past discoveries.
(E) Neutrino astronomy will always be characterized by a large time lag between hypothesis and
experimental confirmation.

3. In the last paragraph, the author describes the development of astronomy in order to

(A) suggest that the potential findings of neutrino astronomy can be seen as part of a
series of astronomical successes
(B) illustrate the role of surprise in scientific discovery
(C) demonstrate the effectiveness of the DUMAND apparatus in detecting neutrinos
(D) name some cosmic phenomena that neutrino astronomy will illuminate
(E) contrast the motivation of earlier astronomers with that of the astrophysicists working on the
DUMAND project

4. According to the passage, one advantage that neutrinos have for studies in
astronomy is that they

(A) have been detected for the last twenty-five years


(B) possess a variable electric charge
(C) are usually extremely massive
(D) carry information about their history with them
(E) are very similar to other electromagnetic particles

5. According to the passage, the primary use of the apparatus mentioned in lines 24-32
would be to

(A) increase the mass of a neutrino


(B) interpret the information neutrinos carry with them
(C) study the internal structure of a neutrino
(D) see neutrinos in distant regions of space
(E) detect the presence of cosmic neutrinos

6. The passage states that interactions between neutrinos and other matter are

(A) rare
(B) artificial
(C) undetectable
(D) unpredictable
(E) hazardous
7. The passage mentions which of the following as a reason that neutrinos are hard to
detect?

(A) Their pervasiveness in the universe


(B) Their ability to escape from different regions of space
(C) Their inability to penetrate dense matter
(D) The similarity of their structure to that of nucleons
(E) The infrequency of their interaction with other matter

8. According to the passage, the interaction of a neutrino with other matter can produce

(A) particles that are neutral and massive


(B) a form of radiation that permeates the universe
(C) inaccurate information about the site and circumstances of the neutrino’s production
(D) charged particles and light
(E) a situation in which light and other forms of electromagnetic radiation are blocked

9. According to the passage, one of the methods used to establish the properties of
neutrinos was

(A) detection of photons


(B) observation of the interaction of neutrinos with gamma rays
(C) observation of neutrinos that were artificially created
(D) measurement of neutrinos that interacted with particles of seawater
(E) experiments with electromagnetic radiation

Threshold (n) : ngưỡng cửa

Permeate (v) : thấm qua

Negligible (adj) : không đáng kể

Traverse (v) : đi qua

Substantial (adj) : đáng kể

Aggregation (n) : tập hợp

Deflect (v) : làm chệch hướng

Cosmic (adj) : vũ trụ

Accelarator (n) : máy gia tốc

Apparatus (n) : máy móc, thiết bị, dụng cụ

Astrophysicist (n) : nhà vật lý thiên văn

Harness (v) : khai thác

Atom (n) : nguyên tử

Cascade (n) : thác nước


Aug 25 – 1

Scholars studying diversity in students pursuing higher education stress the need for diversity
among the faculty and staff of institutions offering such education.It is seen as a way of creating a
more welcoming and supportive environment for minorities and of enhancing education for all
students.Accordingly, they argue that hiring more Native American faculty and staff is a potential
way to help others in the university to become more aware of and responsive to the challenges
faced by some Native American students, thereby leading to more effective and culturally competent
programs.However, this argument ignores a crucial ground reality.Even though many proponents of
the idea discuss variations in cultural orientations among indigenous people, there seems to be an
assumption that Native American professors and students share common values and hence teachers
of Native American ethnicity are thought to have more insight than their counterparts from other
ethnicities into Native American students.But the reality is that Native Americans form an extremely
heterogeneous population.Students and teachers of Native American origin can differ in a variety of
ways, such as tribal background and level of connection to their culture, and therefore making
blanket assumptions about the cultural identity of Native American people who become faculty is
just as inappropriate as it is to make stereotypical assumptions about Native American people in
general.Accordingly, before any key changes are implemented, it would be reasonable to probe how
Native American people who have achieved an extensive background in higher education differ
culturally from their tribal communities of origin.

1.Which of the following statements would the author most likely agree with?

A.Scholars arguing for hiring more Native American staff and faculty underestimate the ways in
which diversity can be achieved in institutions offering higher education.
B.Had the Native Americans formed a homogenous population, the suggestion proposed by the
scholars would have been successful in achieving its purpose.
C.Because Native American teachers have an extensive background in education, they are not a
good fit for teaching Native American students with deep ties to their ethnic origin.
D. The fact that various Native Americans belong to different tribal origins needs to be
accounted for while addressing the issue of diversity through hiring more Native
American staff and faculty.
E.Hiring more Native American faculty and staff can be a way to eliminate prejudices against Native
American students.

2. The author is primarily concerned with

A.advocating a course of action that is contrary to a proposed argument


B.explaining how an idea, though not fully sound, could lead to unintended consequences
C.explaining why an idea needs to be explored more before it is actually adopted
D.describing the merits and demerits of a proposed argument
E.questioning the underlying thought behind a suggestion while offering a simpler plan to achieve
the stated purpose
3.What can be inferred from the following statement made in the passage?

…making blanket assumptions about the cultural identity of Native American people who become
faculty is just as inappropriate as it is to make stereotypical assumptions about Native American
people in general...

A. Finding parallels between the two groups is unwarranted as they differ extensively with respect to
their academic accomplishments.
B.The prejudice that Native American faculty face is a result of the stereotypical assumptions made
about Native American people in general.
C.Irrespective of their achievements in the field of education, Native American faculty and staff are
often mistreated because of their ethnic origin.
D. Drawing opinions based purely on ethnic origins is equally unseemly in both the
situations.
E.Assumptions that are not rooted in ethnic affiliations lead to progressive attitude towards Native
Americans.

Competent (adj) : có tài năng, đủ sức

Indegenous (adj) : bản địa

Insight (adj) : sâu sắc

Heterogeneous (adj) : không đồng nhất

Probe (v) : thăm dò

Unseemly (adj) : không phù hợp

Affiliation (n) : sự liên kết

Progressive (adj) : tiến bộ

Irrespective (adj) : không phân biệt

Mistreat (v) : ngược đãi

Prejudice (n) : định kiến


Aug 25 – 2

In 1977 the prestigious Ewha Women’s University in Seoul, Korea, announced the opening of the
first women’s studies program in Asia. Few academic programs have ever received such public
attention. In broadcast debates, critics dismissed the program as a betrayal of national identity, an
imitation of Western ideas, and a distraction from the real task of national unification and economic
development. Even supporters underestimated the program; they thought it would be merely
another of the many Western ideas that had already proved useful in Asian culture, akin to airlines,
electricity, and the assembly line. The founders of the program, however, realized that neither view
was correct. They had some reservations about the applicability of Western feminist theories to the
role of women in Asia and felt that such theories should be closely examined. Their approach has
thus far yielded important critiques of Western theory, informed by the special experience of Asian
women.

For instance, like the Western feminist critique of the Freudian model of the human psyche, the
Korean critique finds Freudian theory culture-bound, but in ways different from those cited by
Western theorists. The Korean theorists claim that Freudian theory assumes the universality of the
Western nuclear, male-headed family and focuses on the personality formation of the individual,
independent of society. An analysis based on such assumptions could be valid for a highly
competitive, individualistic society. In the Freudian family drama, family members are assumed to be
engaged in a Darwinian struggle against each other—father against son and sibling against sibling.
Such a concept projects the competitive model of Western society onto human personalities. But in
the Asian concept of personality there is no ideal attached to individualism or to the independent
self. The Western model of personality development does not explain major characteristics of the
Korean personality, which is social and group-centered. The “self” is a social being defined by and
acting in a group, and the well-being of both men and women is determined by the equilibrium of
the group, not by individual self-assertion. The ideal is one of interdependency.

In such a context, what is recognized as “dependency” in Western psychiatric terms is not, in


Korean terms, an admission of weakness or failure. All this bears directly on the Asian perception of
men’s and women’s psychology because men are also “dependent.” In Korean culture, men cry and
otherwise easily show their emotions, something that might be considered a betrayal of masculinity
in Western culture. In the kinship-based society of Korea, four generations may live in the same
house, which means that people can be sons and daughters all their lives, whereas in Western
culture, the roles of husband and son, wife and daughter, are often incompatible.\

1. Which of the following best summarizes the content of the passage?

(A) A critique of a particular women’s studies program


(B) A report of work in social theory done by a particular women’s studies program
(C) An assessment of the strengths and weaknesses of a particular women’s studies program
(D) An analysis of the philosophy underlying women’s studies programs
(E) An abbreviated history of Korean women’s studies programs
2. It can be inferred from the passage that Korean scholars in the field of women’s
studies undertook an analysis of Freudian theory as a response to which of the
following?

(A) Attacks by critics of the Ewha women’s studies program


(B) The superficiality of earlier critiques of Freudian theory
(C) The popularity of Freud in Korean psychiatric circles
(D) Their desire to encourage Korean scholars to adopt the Freudian model
(E) Their assessment of the relevance and limitations of Western feminist theory with
respect to Korean culture

They had some reservations about the applicability of Western feminist theories to the role of
women in Asia and felt that such theories should be closely examined. Their approach has thus far
yielded important critiques of Western theory, informed by the special experience of Asian women.

3. Which of the following conclusions about the introduction of Western ideas to Korean
society can be supported by information contained in the passage?

(A) Except for technological innovations, few Western ideas have been successfully transplanted into
Korean society.
(B) The introduction of Western ideas to Korean society is viewed by some Koreans as a
challenge to Korean identity.
(C) The development of the Korean economy depends heavily on the development of new academic
programs modeled after Western programs.
(D) The extent to which Western ideas must be adapted for acceptance by Korean society is
minimal.
(E) The introduction of Western ideas to Korean society accelerated after 1977.

4. It can be inferred from the passage that the broadcast media in Korea considered the
establishment of the Ewha women’s studies program ( not only the program )
(A) praiseworthy
(B) insignificant
(C) newsworthy
(D) imitative
(E) incomprehensible

answer C is newsworthy because the passage says that few academic courses received such public
attention as this one in the past. If you read closely, the question is -"It can be inferred from the
passage that the broadcast media in Korea considered the establishment of the Ewha women’s
studies program - ", the establishment was not imitative, it was rather newsworthy, by being one of
the few catching attention.
5. It can be inferred from the passage that the position taken by some of the supporters
of the Ewha women’s studies program was problematic to the founders of the program
because those supporters

(A) assumed that the program would be based on the uncritical adoption of Western
theory
(B) failed to show concern for the issues of national unification and economic development
(C) were unfamiliar with Western feminist theory
(D) were not themselves scholars in the field of women’s studies
(E) accepted the universality of Freudian theory

6. Which of the following statements is most consistent with the view of personality
development held by the Ewha women’s studies group?

(A) Personality development occurs in identifiable stages, beginning with dependency in childhood
and ending with independence in adulthood.
(B) Any theory of personality development, in order to be valid, must be universal.
(C) Personality development is influenced by the characteristics of the society in which
a person lives.
(D) Personality development is hindered if a person is not permitted to be independent.
(E) No theory of personality development can account for the differences between Korean and
Western culture.

7. Which of the following statements about the Western feminist critique of Freudian
theory can be supported by information contained in the passage?

(A) It recognizes the influence of Western culture on Freudian theory.


(B) It was written after 1977.
(C) It acknowledges the universality of the nuclear, male-headed family.
(D) It challenges Freud’s analysis of the role of daughters in Western society.
(E) It fails to address the issue of competitiveness in Western society.

"Like Western feminist critique, the Korean critique finds Freudian theory culture-bound" implies that
Western feminist critique is culture-bound and another way of saying option A

8. According to the passage, critics of the Ewha women’s studies program cited the
program as a threat to which of the following?
I. National identity
II. National unification
III. Economic development
IV. Family integrity

(A) I only
(B) I and II only
(C) I, II, and III only
(D) II, III, and IV only
(E) I, II, III, and IV
Prestigious (adj) : uy tín

Dismiss (v) : giải tán, miễn nhiệm

Identity (n) : danh tính, sự nhận biết

Imitation (n) : sự bắt chước

Unification (n) : thống nhất

Akin (adj) : giống như

Applicability (n) : khả năng ứng dụng

Yield (v) : mang lại

Psyche (n) : tâm lý

Culture-bond (n) : rằng buộc về văn hóa

Universality (n) : tính phổ quát

Individualistic (n) : chủ nghĩa cá nhân

Interdependency (n) : phụ thuộc lẫn nhau

Psychiatric term (adj/n) : thuật ngữ về tâm thần

Admission (n) : cho phép

Kinship (n) : họ hàng

Incompatible (adj) : không tương thích, mâu thuẫn

Integrity (n) : toàn vẹn, trạng thái nguyên lành

Assessment (n) : thẩm định, đánh giá

Newsworthy (adj) : đáng tin cậy

Insignificant (adj) : tầm thường

Praiseworthy (adj) : đáng khen

Incomprehensible (adj) : không thể hiểu được, khó giải thích

Superficiality (n) : sự nông cạn, hời hợt


Aug 26 – 1

Research has discovered that lucid dreamers, people who are aware that they are asleep when they
are dreaming, have better problem-solving abilities in the waking world than those who remain
unaware of their dreams until they wake up. Studies have also shown that since lucid dreamers are
aware of dreaming while dreaming, they can even play an active role in their dreams.
Neuroscientists hypothesize that the reason behind these special skills of lucid dreamers could be
their special brain structure.

On comparing the brain structures of frequent lucid dreamers and participants who never or only
occasionally have lucid dreams, neuroscientists have found that the anterior prefrontal cortex, i.e.,
the brain area controlling conscious cognitive processes and playing an important role in the
capability of self-reflection, is larger in lucid dreamers, suggesting a link between lucid dreaming and
metacognition, which is defined as cognition about cognition or knowing about knowing. The theory
has found more supporting evidence in the brain images of test subjects, which were taken while
these subjects were solving metacognitive tests in their awake state. The images show that the
brain activity in the prefrontal cortex was higher in lucid dreamers than in non-lucid dreamers,
indicating that lucid dreamers have a higher level of insight than non-lucid dreamers.Neuroscientists
say that it is this level of cognitive ability in lucid dreamers that enables them to spot incongruities
within their dream, signaling their brain that they are in a dream, since the events in the dream
would not make much sense otherwise, and making it possible for them to take an active part in
their dreams. The same insight also transcends in to the waking world when it comes to finding
solution to a problem by spotting hidden connections or inconsistencies.

1. Which of the following is the primary purpose of the passage?

A. To detail a study dealing with the reason behind the mentioned skills of lucid
dreamers
B. To distinguish between the metacognitive skills of lucid and non-lucid dreamers
C. To describe the skills of lucid dreamers in two specific areas
D. To evaluate a study that deals with finding the reasons behind the special skills of lucid dreamers
E. To establish a link between two unrelated skills that lucid dreamers possess

The author does not evaluate or analyze the merits/demerits of the study. In fact, the author never
gives any opinion in the passage.

2. Which of the following is supported by the analysis presented by the neuroscientists?

A. While solving a problem, non-lucid dreamers are not able to spot any connections because their
metacognitive abilities are not as developed as those of the lucid dreamers.
B. Not all happenings in dreams are considered coherent with what is regarded as
sensible in the awake world.
C. Because lucid dreamers are aware of their higher than average metacognition abilities, they know
that they are asleep when they experience a dream.
D. The ability to find connections or inconsistencies helps the development of metacognition skills.
E. By developing self-reflection skills, non-lucid dreamers can experience lucid dreaming.
3. Which of the following is mentioned in the passage?

A. Neuroscientists think that lucid dreamers have better problem-solving abilities than non-lucid
dreamers since they can take active parts in their dreams.
B. As per the neuroscientists, the metacognitive abilities of those who experience lucid dreaming
once and those who never experience it are not comparable.
C. As per the neuroscientists, the higher level of insight possessed by lucid dreamers, as
opposed to non-lucid dreamers, enables them to spot odd events both in dreams and
reality.
D. Studies have shown that the awareness of the fact that they are asleep while experiencing a
dream shows that lucid dreamers have a higher level of metacognition than non-lucid dreamers.
E. The neuroscientists found that the non-lucid dreamers did not have a high level of insight as the
brain activity in their prefrontal cortex was limited.

Lucid (adj) : sáng suốt, minh mẫn

Neuroscientist (n) : nhà thần kinh học

Anterior (adj) : trước

Prefrontal (adj) : trước trán

Cognitive (adj) : nhận thức

Conscious (adj) : có ý thức

Metacognition (n) : siêu nhận thức

Spot (v) : nhận ra

Incongruity (n) : sự không thống nhất

Transcend (v) : vượt qua, siêu việt

Inconsistency (n) : mâu thuẫn

Coherent (adj) : mạch lạc, dính liền

Sensible (adj) : hợp lý, có ý thức


Aug 26 – 2

A majority taken collectively may be regarded as a being whose opinions and, most frequently,
whose interests are opposed to those of another being, which is styled a minority. If it is admitted
that a man possessing absolute power may misuse that power by wronging his adversaries, why
should a majority not be liable to the same reproach? Men are not apt to change their characters by
agglomeration; nor does their patience in the presence of obstacles increase with the consciousness
of their strength. For these reasons we should not willingly invest any group of our fellows with that
unlimited authority which we should refuse to any individual.

One social power must always predominate over others, but liberty is endangered when this power
is checked by no obstacles which may retard its course and force it to moderate its own vehemence.
Unlimited power is in itself a bad and dangerous thing, and no power on earth is so worthy of honor
for itself or of reverential obedience to the rights which it represents that we should admit its
uncontrolled and all-predominant authority. When the right and means of absolute command are
conferred on a people or a king, on an aristocracy or a democracy, a monarchy or a republic, there
has been implanted the germ of tyranny.

The main evil of the present democratic institutions of the United States does not arise, as is often
asserted in Europe, from their weakness, but from their overpowering strength; the excessive liberty
which reigns in that country is not so alarming as is the very inadequate security which exists
against tyranny.

When an individual or a party is wronged in the United States, to whom can he apply for redress? If
to the public opinion, public opinion constitutes the majority; if to the legislature, it represents the
majority and implicitly obeys its injunctions; if to the executive power, it is appointed by the majority
and remains a passive tool in its hands; the public troops consist of the majority under arms; the
jury is the majority invested with the right of hearing judicial cases, and in certain states even the
judges are elected by the majority. However iniquitous or absurd the evil complained about, no sure
barrier is established to defend against it.

1. Which of the following would be the most appropriate title for the passage?

(A) The Tyranny of the Majority


(B) Democracy: Triumph of the People
(C) Abuses of Power
(D) The Failure of Democracy in the United States
(E) Minority Rights

this should have had different instances/examples of abuse of power


2. Which of the following best paraphrases the author’s statement in the highlighted
text ?

(A) Individuals do not change their behavior when they act in concert with others who
are likeminded, and, knowing they are acting as part of the group, they are not likely to
show greater restraint when opposed than they would if they were acting individually.
(B) Groups are not different from one another, they all show strong impatience when thwarted.
(C) The character of men is formed by the accumulation of their traits, and patience is not a
common trait among men of strength.
(D) The leopard does not change its spots no matter how long it lives, and it is, and remains, patient
in the presence of obstacles.
(E) Men change their behavior when they act in groups; they are more patient when they are in the
company of their fellows than they are when they are alone.

3. With which of the following statements would the author of the passage be most
likely to agree?

(A) Democracy is no greater defense against tyranny than is monarchy or aristocracy.


(B) Minority rule would probably be more responsive to the needs of all people than majority rule.
(C) No government should be trusted since all governments are equally tyrannical.
(D) Since one social power must always predominate over others, it is futile to provide checks and
balances in government.
(E) To render itself immune to the germ of tyranny, the United States should strengthen its political
institutions.

4. Which of the following, assuming that each is true, would most weaken the point that
the author is making in the last two paragraphs of the passage?

(A) The framers of the U.S. Constitution deliberately separated the three branches of the
government to prevent tyranny.
(B) There is not a single majority in the United States; there are many majorities, each
composed of a different collection of individuals and each acting as a restraint on the
others.
(C) The First Amendment to the U.S. Constitution specifically guarantees the right of each citizen to
petition the government for redress of grievances.
(D) Even though the United States is not a direct democracy, all U.S. citizens have an equal
opportunity to participate in political life and to hold public office.
(E) The framers of the U.S. Constitution had two primary concerns: to prevent the government from
exercising tyranny over the people and to prevent the majority from exercising tyranny over the
minority.

5. The author’s treatment of the topic of the passage can best be described as

(A) ironic
(B) neutral
(C) logical
(D) irreverent
(E) diffident
6. In the passage, the author is primarily concerned with

(A) challenging a commonly held belief


(B) contrasting two opposing views
(C) advocating a course of action
(D) reconciling an apparent conflict
(E) proposing a solution to an unrecognized problem

Agglomeration (n) : sự kết lại, họp thành số đông, tích tụ

Moderate (v) : tiết chế, ức chế

Vehemence (n) : sự kịch liệt, mãnh liệt, hăng hái

Reverential (adj) : tôn kính

Aristocracy (n) : quý tộc

Monarchy (n) : quân chủ

Germ (n) : mầm mống

Tyranny (n) : chuyên chế, quyền bạo quân

Redress (v) : khôi phục, phục hồi

Injunction (n) : lệnh

Jury (n) : ban bồi thẩm

Judical case (n) : vụ án

Inquitous (adj) : độc ác, bất công

Absurd (adj) : vô lý, không hợp lý

Petition (n) : kiến nghị

Diffident (adj) : khác thường, lúng túng, thiếu tin tưởng

Irreverent (adj) : khiếm nhã

Ironic (adj) : mỉa mai

Neutral (adj) : trung tính


Aug 27 – 1

Tit-for-Tat – a strategy that is a variation of an eye for an eye and a tooth for a tooth - is clear and
simple, never initiates cheating, and provocable that it is, it never allows cheating to go unpunished.
It is surprisingly successful in two-person prisoner’s dilemma games. In a tournament that pitched
150 game theorists from around the world and in which contestants were ranked by the sum of their
scores, the winner Anatol Rapoport successfully deployed this strategy. The result and the winner
remained the same when the same tournament was repeated with an expanded audience. One of
the impressive features about Tit-for-Tat is that it did so well overall, even though it did not (nor
could it) beat any one of its rivals in a head-on. At best, Tit-for-Tat ties its rivals. Hence, if the
competition was scored as a winner take-all contest, Anatol would not have won. The two
advantages of Tit-for-Tat are that firstly, it always comes close and secondly, it usually encourages
cooperation while avoiding exploitation.

In-spite of the above, Tit-for-Tat is a flawed strategy in certain situations. The slightest possibility of
misperception results in the complete breakdown in the success of Tit-for-Tat. For example, in 1987,
the United States responded to the Soviet spying and wiretapping of the US embassy in Moscow by
reducing the number of Soviet diplomats permitted in United States. The Soviets responded by
cutting the support staff at the US embassy in Moscow and reducing the number of US diplomats.
As a result, both countries found it difficult to carry on their diplomatic functions. The problem with
Tit-for-Tat is that any mistake echoes back and forth and sets up a chain reaction that has the
potential to cause grave damage.

1. The passage suggests that Anatol Rapoport won the two-person prisoner’s dilemma
tournaments because

A. Anatol deployed Tit for tat while his rivals did not.
B. Anatol was able to tie with each one of his rivals.
C. The tournaments were not scored as a winner take all contest but ranked contestants
by the sum of their scores.
D. The tournament was based on a classic prisoner’s dilemma principle, one that Anatol may have
been well versed with.
E. Anatol was able to ensure that all his rivals cooperate with him in each of the games he played.

2. What can be inferred about the two-person prisoner’s dilemma tournaments


discussed in the passage?

A. The tournaments were designed to prove the efficacy of the prisoner’s dilemma.
B. None of the participants were as smart as Anatol who won the tournament both times.
C. The winner might have been different the second time had the scoring been changed
to winner takes all method.
D. There were many candidates who may have been extremely close to Anatol in terms of total
number of points.
E. The first iteration included domestic participants while the second iteration included more global
participation.
3. The author of the passage is primarily concerned with

A. Evaluate the various strategies that can be effective against head on competition.
B. Discuss the pros and cons of the Tit-for-Tat as a strategy.
C. Prove that while Tit-for-Tat may be beneficial for business, it is not the right strategy when
diplomatic ties are involved.
D. Compare and contrast 2 scenarios, one in which Tit-for-Tat works and one in which it does not.
E. Demonstrate the inability of Tit- for-Tat to provide long term competitive advantage.

4. According to the passage, the reason why both US and Soviet embassies found it
difficult to carry on their diplomatic functions is

A. They aided cross country spying and wiretapping.


B.Both their parent countries employed Tit-for-tat strategies.
C.The embassies did not employ enough support staff to carry out operations.
D.Both of them lacked enough qualified diplomats to fulfill all their obligations once the Tit-for-tat
treaties were exercised.
E.Both did mistakes that they could not rectify in time.

Only soviet cut the support staff.

Provocable (v) : khiêu khích

Dilemma (n) : tình trạng khó xử

Pitch (v) : ném

Theorist (n) : nhà lý luận

Head-on (n) : đối đầu trực tiếp

Exploitation (n) : sự lợi dụng

Diplomat (n) : nhà ngoại giao

Grave (adj) : trọng yếu, nghiêm trọng


Aug 27 – 2

The new school of political history that emerged in the 1960’s and 1970’s sought to go beyond the
traditional focus of political historians on leaders and government institutions by examining directly
the political practices of ordinary citizens. Like the old approach, however, this new approach
excluded women. The very techniques these historians used to uncover mass political behavior in
the nineteenth-century United States—quantitative analyses of election returns, for example—were
useless in analyzing the political activities of women, who were denied the vote until 1920.

By redefining “political activity,” historian Paula Baker has developed a political history that includes
women. She concludes that among ordinary citizens, political activism by women in the nineteenth
century prefigured trends in twentieth-century politics. Defining “politics” as “any action taken to
affect the course of behavior of government or of the community,” Baker concludes that, while
voting and holding office were restricted to men, women in the nineteenth century organized
themselves into societies committed to social issues such as temperance and poverty. In other
words, Baker contends, women activists were early practitioners of nonpartisan, issue-oriented
politics and thus were more interested in enlisting lawmakers, regardless of their party affiliation, on
behalf of certain issues than in ensuring that one party or another won an election. In the twentieth
century, more men drew closer to women’s ideas about politics and took up modes of issue-oriented
politics that Baker sees women as having pioneered.

1. The primary purpose of the passage is to

(A) enumerate reason why both traditional scholarly methods and newer scholarly methods have
limitations
(B) identify a shortcoming in a scholarly approach and describe an alternative approach
(C) provide empirical data to support a long-held scholarly assumption
(D) compare two scholarly publications on the basis of their authors’ backgrounds
(E) attempt to provide a partial answer to a long-standing scholarly dilemma

2. The passage suggests which of the following concerning the techniques used by the
new political historians described in the first paragraph of the passage?

(A) They involved the extensive use of the biographies of political party leaders and political
theoreticians.
(B) They were conceived by political historians who were reacting against the political climates of
the 1960's and 1970's
(C) They were of more use in analyzing the positions of United States political parties in the
nineteenth century than in analyzing the positions of those in the twentieth century.
(D) They were of more use in analyzing the political behavior of nineteenth-century
voters than in analyzing the political activities of those who could not vote during that
period.
(E) They were devised as a means of tracing the influence of nineteenth-century political trends on
twentieth-century political trends.
3. It can be inferred that the author of the passage quotes Baker directly in the second
paragraph primarily in order to

(A) clarify a position before providing an alternative to that position


(B) differentiate between a novel definition and traditional definitions
(C) provide an example of a point agreed on by different generations of scholars
(D) provide an example of the prose style of an important historian
(E) amplify a definition given in the first pargraph

4. According to the passage, Paula Baker and the new political historians of the 1960’s
and 1970’s shared which of the following?

(A) A commitment to interest-group politics


(B) A disregard for political theory and ideology
(C) An interest in the ways in which nineteenth-century politics prefigured contemporary politics
(D) A reliance on such quantitative techniques as the analysis of election returns
(E) An emphasis on the political involvement of ordinary citizens

5. Which of the following best describes the structure of the first paragraph of the
passage?

(A) Two scholarly approaches are compared, and a shortcoming common to both is
identified.
(B) Two rival schools of thought are contrasted, and a third is alluded to.
(C) An outmoded scholarly approach is described, and a corrective approach is called for.
(D) An argument is outlined, and counter arguments are mentioned.
(E) A historical era is described in terms of its political trends.

6. The information in the passage suggests that a pre-1960’s political historian would
have been most likely to undertake which of the following studies?

(A) An analysis of voting trends among women voters of the 1920’s


(B) A study of male voters’ gradual ideological shift from party politics to issue-oriented politics
(C) A biography of an influential nineteenth-century minister of foreign affairs
(D) An analysis of narratives written by previously unrecognized women activists
(E) A study of voting trends among naturalized immigrant laborers in a nineteenth-century logging
camp

The new school of political history that emerged in the 1960’s and 1970’s sought to go beyond the
traditional focus of political historians on leaders and government institutions by examining
directly the political practices of ordinary citizens

Pre-1960 = traditional focus of political historians

Temperance (n) : kiềm chế, ôn hòa

Nonpartisan (n) : phi đảng phái

Enlist (v) : chiêu mộ, đầu quân

Lawmaker (n) : nhà lập pháp


Affiliation (n) : liên kết, sự nhận vào hội

Biography (n) : tiểu sử

Foreign affair (n) : đối ngoại

Novel (adj) : lạ thường, mới lạ


Aug 28 – 1

Though the truism about Inuits having a hundred words for snow is an exaggeration, languages
really are full of charming quirks that reveal the character of a culture. Dialects of Scottish Gaelic,
for instance, traditionally spoken in the Highlands and, later on, in fishing villages, have a great
many very specific words for seaweed, as well as names for each of the components of a rabbit
snare and a word for an egg that emerges from a hen sans shell. Unfortunately for those who find
these details fascinating, languages are going extinct at an incredible clip, - one dies every 14 days -
and linguists are rushing around with tape recorders and word lists, trying to record at least a
fragment of each before they go. The only way the old tongues will stick around is if populations
themselves decide that there is something of value in them, whether for reasons of patriotism,
cultural heritage, or just to lure in some language-curious tourists. But even when the general public
opinion is for preservation of their linguistic diversity, linguists are finding it increasingly difficult to
achieve such a task.

Mathematicians can help linguists out in this mission. To provide a test environment for programs
that encourage the learning of endangered local languages, Anne Kandler and her colleagues
decided to make a mathematical model of the speakers of Scottish Gaelic. This was an apposite
choice because the local population was already becoming increasingly conscious about the cultural
value of their language and statistics of the Gaelic speakers was readily available. The model the
mathematicians built not only uses statistics such as the number of people speaking the languages,
the number of polyglots and rate of change in these numbers but also figures which represent the
economic value of the language and the perceived cultural value amongst people. These numbers
were substituted in the differential equations of the model to find out the number of new Gaelic
speakers required annually to stop the dwindling of the Gaelic population. The estimate of the
number determined by Kandler’s research helped the national Gaelic Development Agency to
formulate an effective plan towards the preserving the language.

Many languages such as Quechua, Chinook and Istrian Vlashki can be saved using such
mathematical models. Results from mathematical equations can be useful in strategically planning
preservation strategies. Similarly, mathematical analysis of languages which have survived against
many odds can also provide useful insights which can be applied towards saving other endangered
languages.

1. The passage is primarily concerned with which of the following?

A. Outlining the various idiosyncrasies of some languages and their diversity.


B. Emphasizing the versatility of Mathematics.
C. Discussing the steps to be taken for saving languages.
D. Describing the importance of languages.
E. Describing the role of Mathematics in preserving languages.
2. Which of the following best describes the relation of the first paragraph to the
passage as a whole?

A. It makes a general observation to be exemplified.


B. It outlines a process to be analyzed.
C. It identifies a problem the solution of which is to be discussed.
D. It advances an argument to be disputed.
E. It introduces conflicting arguments to be reconciled.

3. The Author’s conclusion that ‘languages such as Quechua, Chinook, and Istrian
Vlashki can be saved using such mathematical models’ (beginning of the last para.) is
most weakened if which of the following is found to be true?

A. The speakers of these languages perceive them as being inferior to other non-native
languages
B. The mathematical models used for Gaelic language are simplistic and not applicable to these
languages
C. Statistics of speakers of these languages have not been properly documented.
D. Patterns in the numbers of the speakers can only be identified by mathematical research.
E. Linguists have shown very little interest in these languages.

Saving a language requires good data and will by the natives. If natives are not
motivated, then mathematical models will not help much.

Linguist (n) : nhà ngôn ngữ học

Truism (n) : sự thật

Quirk (n) : kỳ quặc

Dialect (n) : thổ ngữ

Seaweed (n) : rong biển

Snare (n) : bẫy

Hen sans (n) : gà mái

Patriotism (n) : tinh thần yêu nước

Apposite (adj) : thích hợp, ứng dụng

Polyglot (n) : đa âm
Aug 28 – 2

During the 1960's and 1970's, the primary economic development strategy of local governments in
the United States was to attract manufacturing industries. Unfortunately, this strategy was usually
implemented at another community's expense: many manufacturing facilities were lured away from
their moorings elsewhere through tax incentives and slick promotional efforts. Through the transfer
of jobs and related revenues that resulted from this practice, one town's triumph could become
another town's tragedy.

In the 1980's the strategy shifted from this zero-sum game to one called “high-technology
development,” in which local governments competed to attract newly formed high-technology
manufacturing firms. Although this approach was preferable to victimizing other geographical areas
by taking their jobs, it also had its shortcomings: high-tech manufacturing firms employ only a
specially trained fraction of the manufacturing workforce, and there simply are not enough high-tech
firms to satisfy all geographic areas.

Recently, local governments have increasingly come to recognize the advantages of yet a third
strategy: the promotion of homegrown small businesses. Small indigenous businesses are created
by a nearly ubiquitous resource, local entrepreneurs. With roots in their communities, these
individuals are less likely to be enticed away by incentives offered by another community.
Indigenous industry and talent are kept at home, creating an environment that both provides jobs
and fosters further entrepreneurship.

1. The primary purpose of the passage is to

(A) advocate more effective strategies for encouraging the development of high- technology
enterprises in the United States
(B) contrast the incentives for economic development offered by local governments with those
offered by the private sector
(C) acknowledge and counter adverse criticism of programs being used to stimulate local economic
development
(D) define and explore promotional efforts used by local governments to attract new industry
(E) review and evaluate strategies and programs that have been used to stimulate
economic development

2. The passage suggests which of the following about the majority of United States
manufacturing industries before the high-technology development era of the 1980's?

(A) They lost many of their most innovative personnel to small entrepreneurial enterprises.
(B) They experienced a major decline in profits during the 1960’s and 1970’s.
(C) They could provide real economic benefits to the areas in which they were located.
(D) They employed workers who had no specialized skills.
(E) They actively interfered with local entrepreneurial ventures.
3. The tone of the passage suggests that the author is most optimistic about the
economic development potential of which of the following groups?

(A) Local governments


(B) High-technology promoters
(C) Local entrepreneurs
(D) Manufacturing-industry managers
(E) Economic development strategists

4. The passage does NOT state which of the following about local entrepreneurs?

(A) They are found nearly everywhere.


(B) They encourage further entrepreneurship.
(C) They attract out-of-town investors.
(D) They employ local workers.
(E) They are established in their communities.

5. The author of the passage mentions which of the following as an advantage of high-
technology development?

(A) It encourages the modernization of existing manufacturing facilities.


(B) It promotes healthy competition between rival industries.
(C) It encourages the growth of related industries.
(D) It takes full advantage of the existing workforce.
(E) It does not advantage one local workforce at the expense of another.

Adverse (adj) : bất lợi

Criticism (n) : chỉ trích

Counter (v) : chống lại, phản đối


Aug 29 – 1

Proverbial wisdom states that “birds of a feather flock together.” Studies have shown that people of
similar geographical and educational backgrounds and functional experience are extremely likely to
found companies together. Not considering spousal teams in the dataset, it has been found that a
founding team is five times more likely to be all-male or all-female team. Also, it is more likely to
find founding teams that are remarkably homogeneous with regard to skills and functional
backgrounds.

Homogeneity has important benefits. For the founder struggling to meet the challenges of a growing
startup, selecting cofounders from among the people with whom he or she probably has important
things in common is often the quickest and easiest solution. Not only does it generally take less time
to find such people, but it also generally takes less time to develop effective working relationships
with such similar people. When founders share a background, they share a common language that
facilitates communication, ensuring that the team begins the work relationship with a mutual
understanding and hence can skip over part of the learning curve that would absorb the energies of
people with very different backgrounds. Increasing homogeneity may, therefore, be a particularly
alluring- and, in some ways, a particularly sensible - approach for novice founders heading into
unfamiliar territory. Certainly, studies have found that the greater the heterogeneity among
executive team members, the greater the risk of interpersonal conflict and the lower the group-level
integration. Even though it is very appealing to opt for the “comfortable” and “easy” decision to
found with similar cofounders, by doing so founders may be causing long-term problems. Teams
with a wide range of pertinent functional skills may be able to build more valuable and enduring
startups. Conversely, homogenous teams tend to have overlapping human capital, making it more
likely that the team will have redundant strengths and be missing critical skills.

1. From the passage, which of the following cannot be inferred as a benefit of


homogeneous teams?

A. Finding a suitable PR and advertising person to add skills that the founders lack.
B. Using Six Sigma tools that the founders are familiar with to exchange information.
C. Evolving from concept to product quickly due to flawless execution.
D. Quickly dividing a complex task into subtasks and assigning them to different teams for
execution.
E. Reduce interpersonal conflict while making key decisions.

2. Which of the following can be inferred about start-ups that comprise of homogeneous
teams?

A. They may take longer than average to make decisions in areas that the founding members are
not familiar about.
B. When they comprise of members who are remarkably homogeneous with regard to
skills and functional backgrounds, they may not be able to build more valuable ventures.
C. They usually do not head into unfamiliar territory.
D. They are at a far greater risk of interpersonal conflict than an average startup is.
E. They may have redundant strengths that go underutilized.

Teams with a wide range of pertinent functional skills may be able to build more valuable and
enduring startups.
3. The author’s main purpose of writing the passage is to:

A. evaluate the benefits and downsides of startups with a particular buildup of founding
teams.
B. disprove an accepted notion regarding the success of startups comprising of homogeneous
teams.
C. describe a thesis by presenting its upsides and downsides.
D. list the scenarios under which a particular buildup of founding teams may be successful.
E. submit contrasting benefits of various team structures in achieving a particular task

Proverbial (adj) : tục ngữ

Feather (n) : lông

Flock (v) : tụ tập

Spousal (adj) : vợ chồng

Homogeneous (adj) : đồng nhất

Alluring (adj) : lôi cuốn, say mê

Sensible (adj) : hợp lý

Particularly (adv) : đặc biệt

Novice (adj) : người mới

Heterogeneity (n) : không đồng nhất

Opt (v) : lựa chọn

Appealing (adj) : hấp dẫn

Pertinent (adj) : thích hợp, vừa vặn

Enduring (adj) : bền bỉ

Conversely (adv) : ngược lại


Aug 29 – 2

In a 1984 book, Claire C. Robertson argued that, before colonialism, age was a more important
indicator of status and authority than gender in Ghana and in Africa generally. British colonialism
imposed European-style male dominant notions upon more egalitarian local situations to the
detriment of women generally, and gender became a defining characteristic that weakened women’s
power and authority.

Subsequent research in Kenya convinced Robertson that she had overgeneralized about Africa.
Before colonialism, gender was more salient in central Kenya than it was in Ghana, although age
was still crucial in determining authority. In contrast with Ghana, where women had traded for
hundreds of years and achieved legal majority (not unrelated phenomena), the evidence regarding
central Kenya indicated that women were legal minors and were sometimes treated as male
property, as were European women at that time. Factors like strong patrilinearity and patrilocality,
as well as women’s inferior land rights and lesser involvement in trade, made women more
dependent on men than was generally the case in Ghana. However, since age apparently remained
the overriding principle of social organization in central Kenya, some senior women had much
authority. Thus, Robertson revised her hypothesis somewhat, arguing that in determining authority
in precolonial Africa age was a primary principle that superseded gender to varying degrees
depending on the situation.

1. The primary purpose of the passage is to

(A) present evidence undermining a certain hypothesis


(B) describe a particular position and its subsequent modification
(C) discuss two contrasting viewpoints regarding a particular issue
(D) describe how a social phenomenon varied by region
(E) evaluate an assumption widely held by scholars

2. The passage indicates that Robertson’s research in Kenya caused her to change her
mind regarding which of the following?

(A) Whether age was the prevailing principle of social organization in Kenya before colonialism
(B) Whether gender was the primary determinant of social authority in Africa generally before
colonialism
(C) Whether it was only after colonialism that gender became a significant determinant
of authority in Kenyan society
(D) Whether age was a crucial factor determining authority in Africa after colonialism
(E) Whether British colonialism imposed European-style male-dominant notions upon local situations
in Ghana
3. The passage suggests that after conducting the research mentioned in highlighted
text, but not before, Robertson would have agreed with which of the following about
women’s status and authority in Ghana?

(A) Greater land rights and greater involvement in trade made women in precolonial Ghana less
dependent on men than were European women at that time.
(B) Colonialism had a greater impact on the status and authority of Ghanaian women
than on Kenyan women.
(C) Colonialism had less of an impact on the status and authority of Ghanaian women that it had on
the status and authority of other African women.
(D) The relative independence of Ghanaian women prior to colonialism was unique in Africa.
(E) Before colonialism, the status and authority of Ghanaian women was similar to that of Kenyan
women.

4. The author of the passage mentions the status of age as a principle of social
organization in precolonial central Kenya in highlighted text most likely in order to

(A) indicate that women’s dependence on men in precolonial Kenya was not absolute
(B) contrast the situation of senior women to that of less senior women in precolonial Kenyan
society
(C) differentiate between the status and authority of precolonial Kenyan women and that of
precolonial Ghanaian women
(D) explain why age superseded gender to a greater extent in precolonial Kenya than it did
elsewhere in Africa
(E) identify a factor that led Robertson to revise her hypothesis about precolonial Africa

Colonialism (n) : chủ nghĩa thực dân

Authority (n) : quyền thế

Impose (v) : áp đặt

Egalitarian (adj) : bình đẳng

Detriment (n) : thiệt hại, hao tổn

Subsequent (adj) : sau đó, đến sau

Overgeneralize (v) : quá khái quát hóa

Salient (adj) : nổi bật, quan trọng

Patrilineality (n) : lòng nhân ái

Patrilinearity (n) : lòng yêu nước

Supersede (v) : thay thế


Aug 30 – 1

Researchers traditionally have attempted to track animals by tagging them. Seeking an alternative
tracking method, Brian W. Bowen, an evolutionary geneticist, turned to four green turtle breeding
sites in the Atlantic and the Caribbean. He and his co-workers reported that turtles from the four
breeding sites tended to have slightly different genetic markings.

Bowen analyzed mitochondrial DNA from eggs and hatchlings. The existence of variations in DNA
among geographically distinct groups has helped scientists evaluate the different theories of the
mating habits of the green turtle. Their findings, Bowen asserts, lend credence to the “natal homing”
theory. This theory holds that while reptiles born in different regions may share common feeding
grounds away from home, the animals part company at breeding time, each swimming hundreds or
thousands of miles to breed and nest at its own birthplace.

Bowen and his co-workers, however, offer several caveats to the interpretation of their findings.
They note that turtles from two of the nesting sites—Florida’s Hutchinson Island and Costa Rica’s
Tortuguero sanctuary—had indistinguishable mitochondrial DNA sequences. This finding indicates
that either some social mixing occurred between these two groups or that the DNA test was not
sensitive enough to detect extremely subtle differences in DNA. In addition, they report that one of
eight study turtles from Aves Island, off Venezuela, showed the same genetic pattern as the
Tortuguero and Hutchinson turtles in their sample. Nonetheless, Bowen’s study appears to have
bolstered the idea that most populations of green turtles are genetically distinct, and that the turtles
return to their birthplaces at nesting time.

1. It can be inferred from the passage that the turtles from the Aves Island study

A. had, with the exception of one, genetic patterns distinct from those in the Tortuguero
study
B. have shown that using DNA to track turtles is a flawed technique
C. all exhibited genetic patterns similar to those of the Hutchinson and Tortuguero study turtles
D. are part of an endangered species
E. had DNA that differs from those turtles that are from Venezuela

In addition, they report that one of eight study turtles from Aves Island, off Venezuela, showed the
same genetic pattern as the Tortuguero and Hutchinson turtles in their sample.

2. The primary purpose of this passage is to

A. disprove a previously existing theory


B. present a new idea along with some data
C. establish the infallibility of a new concept
D. confirm the validity of a novel hypothesis
E. track the development of a particular species
3. The information provided in the passage about Bowen's research can be used to
answer which of the following questions?

A. Does social mixing occur between turtles from Florida's Hutchinson Island and Costa Rica's
Tortuguero sanctuary?
B. Is there extensive interbreeding among green turtles from different areas?
C. To what extent can DNA testing be used to detect genetic differences?
D. Does tagging wild animals with metal disks or wires alter their mating habits?
E. By what mechanism do green turtles find their way back to their birthplace when they are ready
to breed?

Caveat (n) : báo trước, phản đối

Hatchling (n) : ấm trứng

Credence (n) : lòng tin

Natal (adj) : nơi sinh đẻ

Reptile (n) : bò sát

Sanctuary (n) : thánh địa, nhà thờ, trú ẩn

Indistinguishable (adj) : không thể phân biệt

Interbreed (v) : liên kết, giao hợp


Aug 30 – 2
When Jamaican-born social activist Marcus Garvey came to the United States in 1916, he arrived at
precisely the right historical moment. What made the moment right was the return of African
American soldiers from the First World War in 1918, which created an ideal constituency for
someone with Garvey's message of unity, pride, and improved conditions for African American
communities.

Hoping to participate in the traditional American ethos of individual success, many African American
people entered the armed forces with enthusiasm, only to find themselves segregated from white
troops and subjected to numerous indignities. They returned to a United States that was as
segregated as it had been before the war. Considering similar experiences, anthropologist Anthony
F. C. Wallace has argued that when a perceptible gap arises between a culture’s expectations and
the reality of that culture, the resulting tension can inspire a revitalization movement: an organized,
conscious effort to construct a culture that fulfills longstanding expectations.

Some scholars have argued that Garvey created the consciousness from which he built, in the
1920s, the largest revitalization movement in African American history. But such an argument only
tends to obscure the consciousness of identity, strength, and sense of history that already existed in
the African American community. Garvey did not create this consciousness; rather, he gave this
consciousness its political expression.

1. According to the passage, which of the following contributed to Marcus Garvey's


success?

(A) He introduced cultural and historical consciousness to the African American community.
(B) He believed enthusiastically in the traditional American success ethos.
(C) His audience had already formed a consciousness that made it receptive to his
message.
(D) His message appealed to critics of African American support for United States military
involvement in the First World War.
(E) He supported the movement to protest segregation that had emerged prior to his arrival in the
United States.

2. The passage suggests that many African American people responded to their
experiences in the armed forces in which of the following ways?

(A) They maintained as civilians their enthusiastic allegiance to the armed forces.
(B) They questioned United States involvement in the First World War.
(C) They joined political organizations to protest the segregation of African American troops and the
indignities they suffered in the military.
(D) They became aware of the gap between their expectations and the realities of
American culture.
(E) They repudiated Garvey's message of pride and unity.
3. It can be inferred from the passage that the “scholars” mentioned in the highlight
text believe which of the following to be true?

(A) Revitalization resulted from the political activism of returning African American soldiers following
the First World War.
(B) Marcus Garvey had to change a number of prevailing attitudes in order for his mass
movement to find a foothold in the United States.
(C) The prevailing sensibility of the African American community provided the foundation of Marcus
Garvey’s political appeal.
(D) Marcus Garvey hoped to revitalize consciousness of cultural and historical identity in the African
American community.
(E) The goal of the mass movement that Marcus Garvey helped bring into being was to build on the
pride and unity among African Americans.

4. According to the passage, many African American people joined the armed forces
during the First World War for which of the following reasons?

(A) They wished to escape worsening economic conditions in African American communities.
(B) They expected to fulfill ideals of personal attainment.
(C) They sought to express their loyalty to the United States.
(D) They hoped that joining the military would help advance the cause of desegregation.
(E) They saw military service as an opportunity to fulfill Marcus Garvey’s political vision.
Subject (v) : chịu đựng, là nạn nhân

Constituency (n) : khu vực bầu cử, các cử tri, người bỏ phiếu

Segregate (v) : tách biệt

Indignity (n) : phẫn nộ

Perceptible (adj) : cảm nhận được

Revitalization (n) : hồi sinh

Obscure (v) : làm mờ, tối nghĩa

Prevailing (adj) : thịnh hành

Foothold (n) : chỗ đứng


Aug 31 – 1

I submit that impact of solid bodies is the most fundamental of all interstellar processes that have
taken place on the terrestrial planets: without impact, Earth, Mars, Venus, and Mercury would not
exist.

Simply put, the collision of smaller objects is the process by which the terrestrial planets were born.
On the surface, that the geological record of the earliest history of impacts on the terrestrial planets
has been lost, is troubling. As the process is self-erasing, to a certain extent, the earliest record
would have been lost even if processes of melting and internal evolution of the planets had not
occurred. But much of the record of the last stages of accretion of the planets is preserved,
especially on the moon, Mercury, and Mars. In fact, the last stage of accretion is still going on, albeit
at a very slow rate.

This is fortunate, because we can study many aspects of the processes of planetary birth by
investigation of the nature of small bodies that still exist, the dynamics of their orbital evolution, and
the effects that they produce when they ultimately collide with a planet. If impact and accretion
were not still occurring, it would be hard to come to grips with a number of difficult problems of
planetary origin and early evolution.

1. The primary purpose of the passage is to

A. present evidence that argues against a common misconception in the formation of planets
B. undermine a claim regarding the role accretion plays in planetary evolution
C. argue for the importance of using existing planetary conditions to understand prior cosmic
occurrences
D. underscore the importance of an astronomical process and describe ways in which we
can understand this process
E. discuss how, unless immediate action is taken, astronomers will squander an opportunity to
better understand planetary formation

The tone is not arguing.

2. It can be most reasonably inferred that which of the following accounts for the lack of
a geological record concerning the history of impacts on the planets?

A. the violence of the initial impact


B. an outcome that is not self-erasing
C. a process of change in planets themselves
D. the absence of proof relating to a hypothetical collision
E. the ongoing process of accretion

3. The author suggests that at least some of “a number of difficult problems...”can be


understood by

A. extrapolating from observable phenomenon


B. anticipating the result of the collision of small bodies
C. studying the rate of accretion on planets
D. observing the internal process of planets
E. discounting the dynamics of how orbits change over time

What author means is we can observe the current phenomena to understand the past phenomena.
It is kind of extrapolating from the present data.

Underscore (v) : gạch dưới, nhấn mạnh

Grip (v) : nắm chặt

Extrapolate (v) : suy luận

Anticipate (v) : dự đoán

Discount (v) : không để ý đến


Aug 31 – 2

A firm’s default risk, the measurement of the chances of the event in which the company will be
unable to make the required payments on its debt obligations, reflects not only the likelihood that
the firm will have bad luck but also the risk that the firm’s managerial decisions will lead the firm to
default. Such management risk occurs because the impact of management on the firm’s value is
uncertain, and this uncertainty affects the market’s perception of a firm’s risk. Uncertainty about
management is likely to be the highest when there is a new management team and should decrease
over time as management’s ability becomes known more precisely. In particular, when the new CEO
is not considered an “heir apparent” prior to getting the position, or when he comes from outside of
the company, or when the new CEO is younger, the market is expected to perceive relatively high
uncertainty about the CEO’s ability or future actions. Accordingly, it comes as no surprise that the
CDS spread, a measure of a firm’s expected default risk, is about 35 basis points higher when a new
CEO takes office than three years into his tenure. The CEO, however, is not the only member of the
management team who is relevant for decision making in the firm. Chief Financial Officers (CFOs)
have a large role in financial decision-making, so uncertainty about new CFOs could also affect the
firm’s default risk and cost of borrowing.

Now, a central feature of financial markets is that the interest rate a firm pays on debt increases
with an increase in the market’s perception of the firm’s risk. This risk occurs because of factors that
affect the value of the firm’s underlying assets and because of uncertainty about how these assets
will be managed. The literature on debt pricing typically does not distinguish between these types of
underlying risks. However, all risks, including those generated by uncertainty about management,
affect the likelihood of default. Consequently, a rational market should incorporate managerial-
generated uncertainty into its assessment of a firm’s risk when pricing its securities. Also since
uncertainty about management affects firms’ costs of borrowing and consequently their financial
policies, the value of maintaining transparency in managerial policies and communicating them to
the marketplaceshould be realised.

1.Which of the following statements would the author most likely agree with?

A. Even though uncertainty about a new CEO’s ability has more impact on a firm’s default risk,
uncertainty about a new CFO could affect the rate at which the firm is lent money and its default
risk
B. The uncertainty about a new CEO is likely to be comparably lower when an expected
candidate takes over the position vis-à-vis an unexpected one.
C. Because the literature on debt pricing normally does not differentiate between the types of risks,
sometimes the default risk is not calculated thoroughly.
D. As the tenure of a new CEO progresses, the uncertainty regarding his ability decreases
considerably.
E. By maintaining transparency in managerial policies, a firm can successfully negotiate its terms in
the market.

In particular, when the new CEO is not considered an “heir apparent” prior to getting the position…
the market is expected to perceive relatively high uncertainty about the CEO’s ability or future
actions.
2.The author is primarily concerned with

A.highlighting the importance of a risk factor that is normally not easily understood in the business
world
B.discussing how a particular factor, though important, gets neglected most of the time
C. describing how a risk factor in the business world gets more importance in some situations than
in others
D. explaining how different risk factors need to be given importance as per their relative weightage
E. discussing the relevance of a risk factor that affects more than one aspect in the
business world

3. Which of the following CANNOT be inferred from the passage?

A. Market’s perception of firm’s risk is minimal when the new CEO is an “heir apparent”
or is from within the existing management team.
B. The uncertainty regarding a new CEO is likely to be more in the first years of his tenure than in
the fourth year.
C. The default risk of a firm represents more than one thing about the firm.
D. It is unusual for a piece of literature on debt pricing to differentiate between the risk generated
from the factors affecting a firm’s asset and one generated from how these assets will be managed.
E. Uncertainty about management affects a firm’s financial policies.

4. Which of the following is mentioned in the passage?

A.The default risk, as per the CDS spread, is highest when the new CEO of a firm is younger than an
average CEO.
B. Besides the CFO and the CEO, there are other members in a firm whose decisions could affect
default risk.
C. The literature on debt pricing normally ignores underlying risks.
D.As the perception of the market regarding a firm’s risk increases, the rate at which the
firm pays interest on debt also increases.
E. The uncertainty surrounding the perception of a firm’s risk leads to management risk.

Default (n) : vỡ nợ

Likelihood (n) : khả năng

Heir apparent (n) : người thừa kế

Debt pricing (n) : định giá nợ

Underlying (adj) : cơ bản

Assessment (n) : thẩm định, đánh giá


Sep 1 – 1

Recent studies of sediment in the North Atlantic’deep waters reveal possible cyclical patterns in the
history of Earth’s climate. The rock fragments in these sediments are too large to have been
transported there by ocean currents, they must have reached their present locations by traveling in
large icebergs that floated long distance from their point of origin before melting. Geologist Gerard
Bond noticed that some of the sediment grains were stained with iron oxide, evidence that they
originated in locales where glaciers had overrun outcrops of red sandstone. Bond’s detailed analysis
of deep-water sediment cores showed changes in the mix of sediment sources over time: the
proportion of these red-stained grains fluctuated back and forth from lows of 5 percent to highs of
about 17 percent, and these fluctuations occurred in a nearly regular 1,500-year cycle.

Bond hypothesized that the alternation cycles might be evidence of changes in ocean-water
circulation and therefore in Earth’s climate. He knew that the sources of the red-stained
grains were generally closer to the North Pole than were the places yielding a high
proportion of“clean” grains. At certain times, apparently, more icebergs from the Arctic Ocean in
the far north were traveling south well into the North Atlantic before melting and shedding their
sediment.

Ocean waters are constantly moving. And water temperature is both a cause and an effect of this
movement. As water cools, it becomes denser and sinks to the ocean’s bottom. During some
periods, the bottom layer of the world’s oceans comes from cold, dense water sinking in the far
North Atlantic. This causes the warm surface waters of the Gulf Stream to be pulled northward.
Bond realized that during such periods, the influx of these warm surface waters into northern
regions could cause a large proportion of the icebergs that bear red grains to melt before traveling
very far into the North Atlantic. But sometimes the ocean’s dynamic changes, and waters from the
Gulf stream do not travel northward in this way. During these periods, surface waters in the North
Atlantic would generally be colder, permitting icebergs bearing red-stained grains to travel farther
south in the North Atlantic before melting and depositing their sediment.

The onset of the so-called Little Ice Age(1300-1860), which followed the Medieval Warm Period of
the eighth through tenth centuries, may represent the most recent time that the ocean’s dynamic
changed in this way. If ongoing climate-history studies support Bond’s hypothesis of 1,500-year
cycles, scientists may establish a major natural rhythm in Earth’s temperatures that could then be
extrapolated into the future. Because the midpoint of the Medieval Warm Period was about AD.850,
an extension of Bond’s cycles would place the midpoint of the next warm interval in the twenty-
fourth century.

1. According to the passage, which of the following is true of the rock fragments
contained in the sediments studied by Bond?

A) the majority of them are composed of red sandstone.


B) they must have reached their present location over 1,500 years ago.
C) they were carried by icebergs to their present location.
D) Most of them were carried to their present location during a warm period in Earth’s climatic
history.
E) They are unlikely to have been carried to their present location during the Little Ice Age.
2. In the final paragraph of the passage, the author is concerned primarily with

A) answering a question about Earth’s climatic history


B) pointing out a potential flaw in Bond’s hypothesis
C) suggesting a new focus for the study of ocean sediments
D) tracing the general history of Earth’s climate
E) discussing possible implications of Bond’s hypothesis.

3. According to the passage, Bond hypothesized that which of the following


circumstances would allow red-stained sediment grains to reach more southerly
latitudes?

A) Warm waters being pulled northward from the Gulf Stream


B) Climatic conditions causing icebergs to melt relatively quickly
C) Icebergs containing a higher proportion of iron oxide than usual
D) The formation of more icebergs than usual in the far north
E) the presence of cold surface waters in the North Atlantic

4. It can be inferred from the passage that in sediment cores from the North Atlantic’s
deep waters, the portions that correspond to the Little Ice Age
A) differ very little in composition from the portions that correspond to the Medieval Warm Period.
B) fluctuate significantly in composition between the portions corresponding to the 1300s and the
portions corresponding to the 1700s
C) would be likely to contain a proportion of red-stained grains closer to 17 percent than
to 5 percent.
D) show a much higher proportion of red-stained grains in cores extracted from the far north of the
North Atlantic than in cores extracted from further south
E) were formed in part as a result of Gulf Stream waters having been pulled northward

Warm Medieval period was followed by little ice age, which means surface water was cold.
Therefore, ice bergs would have retained more of red sediments as they would not have
melted .And as mentioned that the red content in these sediments were form 5 to 17 ,further, was
brought only by water from north .Finally,in little ice age ice bergs would have carried the maximum
red pigment .

Glacier (n) : sông băng, đá băng

Overrun (v) : tràn ngập

Outcrop (v/n) : trồi lên/ sự trồi ngang mặt đất

Sandstone (n) : sa thạch

Stain (v) : làm dơ, nhuộm màu

Shed (v) : rụng

Influx (n) : dòng, sự chảy vào

Onset (n) : khởi phát

Extrapolate (v) : ngoại suy


Sep 1 – 2

Inspired by earlier artists but developed as a genre in its own right in the last few decades of the
twentieth century, installation art is now recognized by academic institutions.It has long been seen
as an example of ‘anti-establishment’ artwork, since it is typically created in public spaces and
involves the use of ‘everyday’ materials, unlike conventional artwork that, even though usually
meant for public consumption, is created in a private space.Installation art in contemporary times
also uses emerging forms of artistic representation such as videos, social networks and aspects of
virtual reality.This form of art, like traditional art, can be either realistic or abstract in its
representation of reality.

Straddling the boundaries between sculpture and painting, installation art gives more priority to the
intention of the artist rather than the physical form or structure of the art: its purpose is often to
relay a specific message through the medium chosen by the artist.Artists such as ‘Banksy’, known
only by his pseudonym and famous for his street art that is meant as a caricature of political
institutions, often use their art as a medium of socio-political activism.

From the audience’s perspective, installation art also gives an unprecedented amount of importance
to the viewer’s subjective perception of the artwork. The intention of the artist is therefore
supplemented by the way in which viewers perceive the artwork.This vision of art overturns the
traditional Platonic view of image theory, which gives the form and intention of the artwork far more
predominance than external perspectives on the work. There are no ‘ideal’ forms in the creation and
exhibition of installation art.This attribute is as it should be: art, after all, requires both an artist and
an audience in order to flourish, and nowhere is this creator-viewer relationship more prominent
than in the study and appreciation of installation art, which embraces the flawed and fragmented
nature of reality.

1. Which of the following CANNOT be inferred from the passage?

A. Installation art as a separate genre of art is not more than a century old.
B. Most artists from the genre of installation art use their art form as a medium of socio-
political activism.
C. Installation art contains elements of both sculpture and painting.
D. It is unlikely that in the past, installation art was studied in school by students of art.
E. Installation art is a less rigid form of art, laying out no guidelines for how art is to be created or
viewed.

2. The author’s reference to Banksy serves primarily to

A. give an example of contemporary artists who often use pseudonyms.


B. establish that contemporary art often criticizes political organizations.
C. prove that caricatures can also be categorized as legitimate art.
D. illustrate how street art forms a well-known part of installation art.
E. to exemplify the use of installation art to convey messages
3. According to the author, which of the following is a feature of installation art but not
of traditional art?

A. sticks to ideal forms in the creation of art


B. not created in a private space
C. follows the Platonic model of image theory
D. can be either realistic or abstract
E. is used for social activism

It has long been seen as an example of ‘anti-establishment’ artwork, since it is typically


created in public spaces and involves the use of ‘everyday’ materials, unlike
conventional artwork that, even though usually meant for public consumption, is
created in a private space

4. Which of the following is the primary purpose of the passage?

A. To argue that a certain art genre, though different from traditional art, should get the same
status as most other art forms
B. To analyze an art genre while differentiating it from conventional art
C. To discuss the features of an art form that is by far the most unconventional of all art genres
D. To trace the history of an art form that is a departure from traditional art
E. To describe the relevance of the perspectives of the viewers of art who are given special
importance by one art genre

Partial Scope. This choice partially covers the scope of only the third paragraph.

Straddling (v) : giằng co

Caricature (n) : biếm họa

Unprecedented (adj) : chưa từng có

Supplement (v) : bổ sung

Overturn (v) : xô ngã

Platonic (adj) : đơn phương

Embrace (v) : bao quát

Rigid (adj) : cứng rắn, nghiêm khắc


Sep 2 – 1

Defined as an aggregate measure of production equal to the sum of the gross values added of all
resident institutional units engaged in production, plus and taxes, and minus any subsidies, on
products not included in the value of their outputs, GDP is a basic standard to measure the standard
of living in a country and in turn the economic well-being of the population. However, critics are of
the opinion that national GDP figures do not reveal significant regional variations in output,
employment and incomes per head of population. They justly contend that within each region there
are areas of relative prosperity contrasting with unemployment black-spots and deep-rooted social
and economic deprivation and that GDP figures on their own do not represent the distribution of
income and the uneven spread of financial wealth.

The idea that rising national prosperity can still be accompanied by rising relative poverty is rightly
at the heart of the criticism meted out for using GDP as a measure of the general well-being of the
population of a region. Changes such as increase in pollution and other negative externalities can
accompany rising national output. Also, output figures reflect nothing but little on the quality of
goods and services produced. Not to mention that the quality of life for many could be severely
diminished if the increase in national output has been achieved at the expense of the leisure time of
the workers working longer than usual hours.

1.The author is primarily concerned with

A. defining a concept while presenting two sides of an argument


B. presenting the merits and demerits of a phenomenon
C. describing the criticism meted out to the use of a parameter for a specific purpose
D. criticizing a particular usage of a concept
E. evaluating a phenomenon on the basis of certain unconventional, non-economic parameters

2.Which of the following can be inferred from the passage?

A. In countries where there is little disparity in the distribution of wealth, GDP is the best measure of
standard of living.
B. Increase in pollution and other negative changes negate any increase in national output.
C. An increase in the overall goods produced by a nation does not on its own signify a
corresponding increase in the quality of these goods.
D. The quality of life suffers when workers work for long hours.
E. A country with a high GDP is very likely to have uneven spread of financial wealth.

3.Which of the following is mentioned in the passage?

A. Unemployment black spots and social deprivation are two of the biggest threats to any economy.
B. An increase in national prosperity is most likely accompanied by an increase in relative poverty.
C. Even when there is an equal distribution of wealth, GDP fails as a measure of the income of a
country.
D. There is an inverse relationship between an increase in the output of goods in a country and the
quality of this output.
E. The usage of GDP as a measure of standard of living is linked to measuring the
economic health of the population.

The idea that rising national prosperity can still be accompanied by rising relative poverty
4. Which of the following most aptly describes the function of the first paragraph?

A. To define and criticize the basis of an economic term


B. To criticize the effort to ignore the marginalized sections in a society by using a particular
measure
C. To evaluate a particular term that is little understood by most countries
D. To introduce criticism against a particular use of a concept
E. To discuss measures that can increase the relevance of a particular concept

Corresponding (adj) : tương ứng

Meted out (v) : gặp

Prosperity (n) : sự phồn thịnh


Sep 3 – 1

The generally agreed upon definition of hallucinations is that they are actually perceptions in the
absence of an external stimulus of the relevant sensory organ. These perceptions are accompanied
by a persuasive sense of their reality. Hallucinations are not to be confused with illusions, which are
misinterpretations of actual external stimuli. In other words, in the case of a hallucination, for a
person hallucinating a sound for instance, the sound actually exists, but in reality it exists nowhere
but in the person’s mind; however, in the case of an illusion the stimulus that caused the illusion
does exist in the real world. Although hallucinations are a key diagnostic feature of schizophrenia, a
mental disorder that distorts the way a person thinks, acts, expresses emotions, perceives reality,
and relates to others, they can occur in individuals devoid of any physical or mental disorder.

In order to understand why people hallucinate, some researchers have been studying people
suffering from what is called the phantom limb syndrome, a condition in which people who have
undergone amputation feel as though the missing body part were still very much present and
attached to the body; for instance, it is not uncommon for a person who has lost a leg to
unknowingly try to stand and walk without any external support after their surgery. One of the two
main hypotheses behind the phantom limb syndrome offers an explanation that is possibly
applicable to why people hallucinate in certain circumstances. As per this theory, the brain is
programmed for a body where every body part is intact and in the appropriate place. Accordingly,
when some signals go missing, because of the missing body part, the brain compensates for the
lack of sensory input by triggering spontaneous nerve cell activity. Although this theory has limited
applicability for understanding why patients such as those of schizophrenia hallucinate in conditions
not deemed out of the ordinary, it is a step forward in understanding why people experience visual
or auditory hallucinations when they are placed in solitary confinements; after all the different areas
of the brain that were used to receiving signals through the senses start to stimulate themselves in
to action.

1. Each of the following can be inferred from the passage EXCEPT

A. Solitary confinement is a condition that it is not necessarily deemed out of the


ordinary.
B. An illusion follows an actual stimulus whereas a hallucination does not.
C. On detecting that a patient is experiencing hallucinations, a psychiatrist is likely to check the
patient for schizophrenia.
D. When a body part is amputated, the brain stops receiving some of the signals it usually does.
E. The applicability of phantom limb syndrome is limited to some instances of hallucinations.

Although this theory has limited applicability for understanding why patients such as those of
schizophrenia hallucinate in conditions not deemed out of the ordinary, it is a step forward in
understanding why people experience visual or auditory hallucinations when they are placed in
solitary confinements;

Through this sentence the author presents a contrast. In the first bolded statement, he/she
indicates that the theory has limited applicability in conditions that are not deemed out of the
ordinary, i.e. the theory has limited applicability in ordinary conditions. The author then presents the
contrast – this theory is a step forward to understand hallucinations in solitary confinements. Thus,
by combing these two aspects of contract, one can infer that the author does not consider solitary
confinement as an ordinary condition.

2. The author is primarily concerned with


A. Differentiating hallucinations from other forms of disorders and discussing the relevance of a
theory.
B. Discussing the relevance of a proposed hypothesis while defining hallucination and
differentiating it from another phenomenon.
C. Explaining why a theory does not provide full insight into the circumstances leading up to
hallucinations.
D. Defining hallucination by citing a unanimously agreed upon definition and providing insight into
why hallucinations take place in circumstances not regarded out of the ordinary.
E. Elaborating on a theory that deals with a phenomenon that is triggered by circumstances not
dissimilar to hallucinations in certain conditions.

3. Which of the following is mentioned in the passage?


A. Hallucinations are caused by chemical imbalances in the brain, imbalances typical of people with
mental disorders.
B. Hallucinations cannot be experienced at the same time as illusions.
C. A person suffering from the phantom limb syndrome is likely to suffer from hallucination at some
point during the condition.
D. Auditory and visual hallucinations are the two most common types of hallucinations experienced
by people in solitary confinement.
E. An illusion happens after there has been a real external trigger for the same.

4. Which of the following most aptly describes the function of the first paragraph?
A. To elaborate a concept while differentiating it from other related concepts.
B. To define a phenomenon while discarding a common misunderstanding about its central feature.
C. To explain a key difference between two concepts.
D. To introduce a phenomenon while differentiating it from another.
E. To initiate a discussion about a concept that is normally mistaken for another phenomenon.

Ordinary (n) : thông thường

Amputate (v) : cắt cụt

Phantom limb syndrome (n) : hội chứng chân tay ảo

Hallucinate (v) : ảo giác

Illusion (n) : ảo tưởng, ảo ảnh

Schirophrenia (n) : tâm thần phân liệt

Deem (v) : coi la

Out of the ordinary (n) : khác thường

Misinterpretation (n) : giải thích sai

Stimulus/Stimuli (n) : kích thích

Trigger (v) : gây ra


Sep 4 – 1

John Maynard Keynes and Friedrich Hayek, two prominent public figures associated with the Great
Depression that started with the collapse of the stock market in 1929, both proposed different ways
to restore the economy. Keynes, an economist from Cambridge, believed that the government
should intervene in the situation, whereas Hayek, a professor from Austria, maintained that
government involvement would be futile.

Although Hayek almost singlehandedly established the distinction between microeconomics and
macroeconomics, engineering the transfer from classical economics to the more practical and
application-oriented methodology of mathematical economics, his solution for emerging from the
Great Depression was underestimated in the light of Keynes’ contribution to economic theory.

In his 2012 study Keynes Hayek: The Clash that Defined Modern Economics, famous theorist
Nicholas Wapshott traces the history of the applications of both men’s theories, giving
unprecedented importance to Hayek’s work.Describing how the Keynesian vision dominated Western
economic analyses until the 1970s, Wapshott surmises that the primary reason for the dominance of
Keynes’ theory during the Depression was that Hayek’s solutions to the issue of economic recession
were not politically viable.Hayek prescribed to the belief that, when left to its own devices, the
market would eventually recover from its downturn and resume its equilibrium; this position was,
not surprisingly, rejected by both the US and European political organizations, each seeking to prove
its worth in reestablishing the economy’s prosperity.

1. According to the passage, which of the following is true?

A. Hayek’s solution to the economic depression was not guided in the right direction.
B. Before Wapshott’s 2012 study, Hayek’s contribution to the field of economics was unknown.
C. Hayek’s contribution to economics began in the 1970s and has continued into the twenty-first
century.
D. Macroeconomics was singlehandedly created by Hayek.
E. A specific work gave more importance to Hayek’s work than ever before.

2. The primary purpose of the passage is to

A. contrast two solutions to the same issue.


B. establish the superiority of one hypothesis over another.
C.prove that one theorist's contribution to a particular field was more notable than another's.
D. outline different approaches to a historical problem.
E. contrast two different approaches to a historical problem and share the reason for the
limited appreciation for one

3. According to the passage, which of the following was true of political parties at the
time of the Great Depression?

A. They wanted to speed up the process of reestablishing the economy’s prosperity.


B. They believed it was futile for the government to get involved in market mechanisms.
C. They believed they had a responsibility to intervene in the situation.
D. They wanted to prove that they were capable of reinstating the economy’s well-
being.
E. They believed that Hayek's solution would undermine their future in politics.
The US and European political organizations, each seeking to prove its worth in
reestablishing the economy’s prosperity.

4. Which of the following would the author most likely agree with?

A. Hayek’s theory was dominated by Keynes’ because Keynes had a better long-term vision than
Hayek did.
B. Even though the reaction of the political organization could be anticipated, the
underwhelming response received by Hayek in general was not indicative of the
potential of his theory.
C. In addition to being politically viable, Hayek’s ideas were probably ahead of their time.
D. Had Keynes also argued for limited government intervention, his ideas too would have been
vehemently rejected by the political organizations.
E. In the light of the real reason that Hayek’s theory did not receive much appreciation, Keynes
probably did not contribute as much to economics as he is credited for.

Intervene (v) : can thiệp

Futile (adj) : vô ích

Methodology (n) : phương pháp luận

Surmise (v) : phỏng đoán

Viable (adj) : khả thi


Sep 4 – 2

The Montreal Protocol on Substances that Deplete the Ozone Layer, signed in 1987 by more than
150 nations, has attained its short-term goals: it has decreased the rate of increase in amounts of
most ozone-depleting chemicals reaching the atmosphere and has even reduced the atmospheric
levels of some of them. The projection that the ozone layer will substantially recover from ozone
depletion by 2050 is based on the assumption that the protocol's regulations will be strictly followed.
Yet there is considerable evidence of violations, particularly in the form of the release of ozone-
depleting chlorofluorocarbons (CFCs), which are commonly used in the refrigeration, heating, and
air-conditioning industries. These violations reflect industry attitudes; for example, in the United
States, 48% of respondents in a recent survey of subscribers to Air Conditioning, Heating, and
Refrigeration News, an industry trade journal, said that they did not believe that CFCs damage the
ozone layer. Moreover, some in the industry apparently do not want to pay for CFC substitutes,
which can run five times the cost of CFCs. Consequently, a black market in imported illicit CFCs has
grown. Estimates of the contraband CFC trade range from 10,000 to 22,000 tons a year, with most
of the CFCs originating in India and China, whose agreements under the Protocol still allow them to
produce CFCs. In fact, the United States Customs Service reports that CFC-12 is a contraband
problem second only to illicit drugs.

1. The passage suggests which of the following about the illicit trade in CFCs?

(A) It would cease if manufacturers in India and China stopped producing CFCs.
(B) Most people who participate in such trade do not believe that CFCs deplete the ozone layer.

The passage only mentions about the respondents, not all people who trade CFCs
(C) It will probably surpass illicit drugs as the largest contraband problem faced by the United States
Customs Service.
(D) It is fostered by people who do not want to pay the price of CFC substitutes.
(E) It has grown primarily because of the expansion of the refrigeration, heating, and air-
conditioning industries in foreign countries.

2. According to the passage, which of the following best describes most ozone-depleting
chemicals in 1996 as compared to those in 1987?

(A) The levels of such chemicals in the atmosphere had decreased.


(B) The number of such chemicals that reached the atmosphere had declined.
(C) The amounts of such chemicals released had increased but the amounts that reached the
atmosphere had decreased.
(D) The rate of increase in amounts of such chemicals reaching the atmosphere had
decreased.
(E) The rate at which such chemicals were being reduced in the atmosphere had slowed.

3. The author of the passage compares the smuggling of CFC’s to the illicit drug trade
most likely for which of the following reasons?

(A) To qualify a previous claim


(B) To emphasize the extent of a problem
(C) To provide an explanation for an earlier assertion
(D) To suggest that the illicit CFC trade, likely the illicit drug trade, will continue to increase
(E) To suggest that the consequences of a relatively little-known problem are as serious as those of
a well-known one

Contraband (n) : hàng lậu

Illicit (adj) : bất hợp pháp

Deplete (v) : suy giảm


Sep 5 – 1

A recent report from the International Energy Agency (IEA) on international oil supplies indicates
that oil prices could increase to over $100 a barrel because of the global energy shortage. The
International Energy Agency, an organization representing the 26 nations of the Organization for
Economic Cooperation and Development (OECD), is generally considered a reliable indicator of
international energy supplies. In a distinct change from its former reports, the IEA has suggested in
its recent report that world governments need to make urgent revisions on energy-related policies if
a global energy crisis is to be avoided.

According to the report, the reason for the IEA's warning is its anticipation that economic progress
will increase global energy demands by about 50% in the next fifteen years. The majority of this
demand is expected to come from China and India, which will be accountable for over 60 percent of
carbon emissions, the chief reason for global warming, by 2030. Another reason stated for the IEA’s
cautionary notice is that the number of oil suppliers is steadily decreasing. Since oil deposits in the
North Sea region will soon be exhausted, countries supplying oil will be limited to those in the Gulf
of Persia.

The government can address this imminent crisis through just one policy change. A team of Harvard
analysts has predicted that if oil production is considerably expanded, oil prices could fall in the next
decade. The team maintains that if previously under-utilized sources of oil are harnessed and more
funding is provided for the setting up of new local refineries, the impending crisis could be avoided.

1. According to the passage, the International Energy Agency agrees over which of the
following?

A. Members of the OECD will have primary access to oil supplies.


B. There is no other reliable indicator of international energy supplies.
C. Steps can be taken to avert an energy crisis on a worldwide level.
D. Some countries with increased demand for oil will escalate the issue of global warming by 60%.
E. Countries with dwindling oil supplies are likely to become much larger consumers of oil.

2. Which of the following can be derived on the basis of the information given in the
passage?

A. The IEA probably believes that it is unlikely that there will be increased funding for the setting up
of new local refineries.
B. The new report reflects a change in the IEA’s stated position on the global energy
crisis.
C. The increased oil consumption in India and China will reduce the amount of oil that is available
for other countries and hence retard their economic progress.
D. Oil prices will decrease if previously unused sources of oil are harnessed.
E. World governments are not taking any tangible steps to avoid the oil crisis.

In a distinct change from its former reports, the IEA has suggested in its recent report
that world governments need to make urgent revisions on energy-related policies if a
global energy crisis is to be avoided.
3. Which of the following statements would the author most likely agree with?

A. Both the IEA and the team of Harvard analysts agree that the oil crisis is avoidable.
B. Other reliable indicators of international energy supplies are needed.
C. If the impending energy crisis is avoided, global warming will decrease.
D. The Harvard analysts disagree that a global oil crisis is impending.
E. The opinion of the team of Harvard analysts has more credibility than the report issued by the
IEA.

4. Which of the following best describes the author’s main purpose behind sharing the
opinions of the team of Harvard analysts?

A. To suggest that the IEA might have wrongly preempted an impending crisis
B. To give information about the possible sources of oil supply
C. To share an opinion that could be superior to the one stated by the IEA about the supply of oil
D. To bring out an inherent contradiction in the IEA’s prediction of oil deposits
E. To suggest one way in which the oil supply crisis can be averted.

Avert (v) : ngăn chặn

Harness (v) : khai thác

Imminent (adj) : sắp xảy ra, cấp bách

Refinery (n) : nhà máy lọc dầu

Impending (adj) : đang đe dọa


Sep 5 – 2

It is now possible to hear a recording of Caruso's singing that is far superior to any made during his
lifetime. A decades-old wax-cylinder recording of this great operatic tenor has been digitized, and
the digitized signal has been processed by computer to remove the extraneous sound, or “noise,”
introduced by the now “ancient” wax-cylinder recording process.

Although this digital technique needs improvement, it represents a new and superior way of
recording and processing sound which overcomes many of the limitations of analog recording. In
analog recording systems, the original sound is represented as a continuous waveform created by
variations in the sound's amplitude over time. When analog playback systems reproduce this
waveform, however, they invariably introduce distortions. First, the waveform produced during
playback differs somewhat from the original waveform. Second, the medium that stores the analog
recording creates noise during playback which gets added to the recorded sounds.

Digital recordings, by contrast, reduce the original sound to a series of discrete numbers that
represent the sound's waveform. Because the digital playback system “reads” only numbers, any
noise and distortion that may accumulate during storage and manipulation of the digitized signal will
have little effect: as long as the numbers remain recognizable, the original waveform will be
reconstructed with little loss in quality. However, because the waveform is continuous, while its
digital representation is composed of discrete numbers, it is impossible for digital systems to avoid
some distortion. One kind of distortion, called “sampling error,” occurs if the sound is sampled (i.e.,
its amplitude is measured) too infrequently, so that the amplitude changes more than one quantum
(the smallest change in amplitude measured by the digital system) between samplings. In effect,
the sound is changing too quickly for the system to record it accurately. A second form of distortion
is “quantizing error,” which arises when the amplitude being measured is not a whole number of
quanta, forcing the digital recorder to round off. Over the long term, these errors are random, and
the noise produced (a background buzzing) is similar to analog noise except that it only occurs when
recorded sounds are being reproduced.

1. Which of the following best describes the relationship of the first paragraph to the passage as a
whole?

(A) The first paragraph introduces a general thesis that is elaborated on in detail elsewhere in the
passage.
(B) The first paragraph presents a concrete instance of a problem that is discussed elsewhere in the
passage.
(C) The first paragraph describes a traditional process that is contrasted unfavorably with a newer
process described elsewhere in the passage.
(D) The first paragraph presents a dramatic example of the potential of a process that is
described elsewhere in the passage.
(E) The first paragraph describes a historic incident that served as the catalyst for developments
described elsewhere in the passage.

2. According the passage, one of the ways in which analog recording systems differ from digital
recording systems is that analog systems

(A) can be used to reduce background noise in old recordings


(B) record the original sound as a continuous waveform
(C) distort the original sound somewhat
(D) can avoid introducing extraneous and nonmusical sounds
(E) can reconstruct the original waveform with little loss quality

3. Which of the following statements about the numbers by which sound is represented in a digital
system can be inferred from the passage?

(A) They describe the time interval between successive sounds in a passage of music.
(B) They model large changes in the amplitude of the initial sound with relatively poor precision.
(C) They are slightly altered each time they are read by the playback apparatus.
(D) They are not readily altered by distortion and noise accumulated as the digital signal
is stored and manipulated.
(E) They are stored in the recording medium in small groups that can be read simultaneously by the
playback apparatus.

4. Which of the following can be inferred from the passage about the digital approach to the
processing of sound?

(A) It was developed in competition with wax-cylinder recording technology.


(B) It has resulted in the first distortion-free playback system.
(C) It has been extensively applied to nonmusical sounds.
(D) It cannot yet process music originally recorded on analog equipment.
(E) It is not yet capable of reprocessing old recordings in a completely distortion-free
manner.

Elaborate (v) : xây dựng, nghiên cứu, sửa soạn

Amplitude (n) : biên độ

Cylinder (n) : hình trụ

Tenor (n) : kỳ hạn

Digitize (v) : số hóa

Extraneous (adj) : không liên quan

Invariably (adv) : luôn luôn

Distortion (n) : méo mó, biến dạng

Medium (n) : vật trung gian

Accumulate (v) : tích trữ

Manipulation (n) : thao tác

Infrequently (adv) : không thường xuyên

Discrete (adj) : rời rạc

Round off (v) : làm tròn


Sep 6 – 1

Globally, about a third of the food produced for human consumption goes to waste, implying that a
third of the water, land use, energy and financial resources that go into producing it are also
squandered. Yet people often think of food as environmentally benign because it is biodegradable,
while label food packaging as a wasteful use of resources leading to nothing but more pollution,
despite the reality that the energy that goes into packaging makes up a mere 10% of the total
energy that goes into producing, transporting, storing and preparing food. Needless to say, their
view ignores the negative impact of food production, supply, and consumption, and the benefits
possible from the right kind of food packaging.

Indeed the dislike for food packaging is not all baseless. There is a lot of bad and wasteful
packaging out there. But any assessment of its impact on the environment must take into account
the benefits one can derive from packaging in the shape of reduced food waste that can be realized
by protecting and dispensing food properly. For instance, two percent of the milk produced in the
US goes bad on supermarket shelves before it can be purchased. This dairy waste can be avoided
with packaging technology such as Tetra Pak that saves milk from spoiling, even without
refrigeration. However, environmentally aware consumers tend to dislike Tetra Pak material because
they think it cannot be recycled. The truth, however, is that it can be recycled, but the process is
rather complicated. Irrespective of the recycling aspect, Tetra Pak is a good environmental bet
because it can extend the shelf life of milk up to nine months, reducing the need for refrigeration —
and reducing the amount of milk that goes bad on retail shelves. Clearly, the environmental benefit
of the food-protection technology outweighs the negative impact of the packaging itself.

1. The author is primarily concerned with

A. presenting a more complete picture of a situation and suggesting a radical solution to the
problem
B. citing an example of a belief that is not entirely baseless
C. summarizing the negative impacts of an industry, effects of which people are not fully aware
D. arguing against a popular belief
E. attacking a mindset that has no empirical basis

2. Which of the following statement can be derived from the passage?

A. The popularity of Tetra Pak in the packaging industry would increase manifold if the
environmentally aware customers change their opinion about it.
B. The complexity involved in the process of recycling Tetra Pak is the reason behind the material’s
limited popularity with environmentally aware consumers.
C. It is likely that developed countries, which use a lot more food packaging material than
developing countries, have lower rates of food wastage than developing countries.
D. No biodegradable substance can be labelled as completely benign for the environment.
E. In some cases, the recyclability of a material is not the overriding factor in
determining its impact on the environment.
3. Which of the following is the function of the first paragraph in the passage?

A. To introduce a view that is responsible for a significant proportion of wastage in an industry


B. To raise a few considerations against a popular belief
C. To contrast two views on a highly debated topic
D. To state a situation that has severe damaging effects on the environment
E. To highlight that a popular belief, although credible sometimes, does not take in to account the
full situation

Manifold (n) : nhiều thứ khác

Biodegradable (adj) : phân hủy sinh học

A third (n) : một phần ba

Squander (v) : phung phí

Benign (adj) : nhẹ

Baseless (adj) : vô căn cứ

Dispense (v) : phân chia

Dairy (adj) : sữa

Irrespective (adj) : không phân biệt


Sep 6 – 2

Maps made by non-Native Americans to depict Native American land tenure, resources and
population distributions appeared almost as early as Europeans' first encounters with Native
Americans and took many form: missionaries' field sketches, explorers' drawings, and surveyors'
maps, as well as maps rendered in connection with treaties involving land transfers. Most existing
maps of Native American lands are reconstructions that are based largely on archaeology, oral
reports, and evidence gathered from observers' accounts in letter, diaries, and official reports;
accordingly, the accuracy of these maps is especially dependent on the mapmakers' own interpretive
abilities.

Many existing maps also reflect the 150-year role of the Bureau of Indian Affairs (BIA) in
administering tribal lands. Though these maps incorporate some information gleaned directly from
Native Americans, rarely has Native American cartography contributed to this official record, which
has been compiled, surveyed, and authenticated by non-Native Americans. Thus our current
cartographic record relating to Native American tribes and their migrations and cultural features, as
well as territoriality and contemporary trust lands, reflects the origins of the data, the mixed
purposes for which the maps have been prepared, and changes both in United States government
policy and in non-Native Americans' attitudes toward an understanding of Native Americans.

1. Which of the following best describes the content of the passage?

(A) A chronology of the development of different methods for mapping Native Americans
(B) A discussion of how the mapmaking techniques of Native Americans differed from those of
Europeans
(C) An argument concerning the present-day uses to which historical maps of Native American lands
are put
(D) An argument concerning the nature of information contained in maps of Native
American lands
(E) A proposal for improving the accuracy of maps of Native American lands

2. The passage mentions each of the following as a factor affecting current maps of Native American
lands EXCEPT

(A) United States government policy


(B) non-Native Americans’ perspective on Native Americans
(C) origins of the information utilized to produce the maps
(D) changes in ways that tribal lands are used
(E) the reason for producing the maps

3. The passage suggests which of the following about most existing maps of Native American lands?

(A) They do not record the migrations of Native American tribes.


(B) They have been preserved primarily because of their connection with treaties involving land
transfers.
(C) They tend to reflect archaeological evidence that has become outdated.
(D)They tend to be less accurate when they are based on oral reports than when they are based on
written documents.
(E) They are not based primarily on the mapmakers’ first-hand observations of Native
American lands.

4. All of the following are examples of the type of evidence used in creating “Most existing maps”
(line 7-8) EXCEPT

(A) a nineteenth-century government report on population distribution of a particular tribe


(B) taped conversations with people who lived on Native America tribal lands in the early twentieth
century
(C) aerial photographs of geological features of lands inhabited by Native Americans
(D) findings from a recently excavated site once inhabited by a certain Native American people
(E) a journal kept by a non-Native American explorer who traveled in Native American territory in
the early nineteenth century

The non-native american reconstruct maps of native american land tenure based on the
evidence of native americans

Aerial (adj) : trên không

Journal (n) : bài báo

Archaeology (n) : khảo cổ học

Outdate (v) : lỗi thời

Sketch (n) : bản phác thảo

Missionary (n) : nhà truyền giáo

Surveyor (n) : điều tra viên

Cartography (n) : bản đồ

Comply (v) : biên soạn

Authenicate (v) : chứng thực

Territoriality (adv) : lãnh thổ

Contemporary (adj) : đồng thời


Sep 7 – 1

Despite radical differences in what limbs do and what they look like, the underlying blueprint of all
limbs in land based animals, whether those limbs are wings in birds, flippers in penguins, or hands
in humans, is the same - one bone, the humerus in the arm or the femur in the leg, articulates with
two bones, which attach to a series of small blobs, which connect with the fingers or toes. Want to
make a bat wing? Make the fingers really long. Want to make a horse? Elongate the middle fingers
and toes and lose the outer ones. The differences between creatures lie in differences in the shapes
and sizes of the bones and the numbers of blobs, fingers, and toes.

In the 1950s and 1960s a number of biologists, including Edgar Zwilling and John Saunders, did
extraordinarily creative experiments on chicken eggs to understand how skeletal structure of limbs
forms and uncovered some of the key mechanisms that build limbs that have the same architecture
but are as different as bird wings, lizards webbings, and human hands . They discovered that two
little patches of tissue essentially control the development of the pattern of bones inside limbs.
These patches of tissue were named the zone of polarizing activity (ZPA). The cells in the ZPA made
special molecules that then spread across the limb to instruct cells to make femur, articulates, and
connecting toes. The concentration of these molecules was the important factor and decided the
length of femur, articulates, connecting toes, and even the length of individual toe fingers. Later
experiments with other animals such as the bat, frog, etc. proved that the mechanism to form limbs
remained the same - formation of limbs in every creature was controlled by the ZPA – just the
concentration of these special molecules varied corresponding to the desired structure.

The reason for this inherent commonality in architecture still remained a conundrum for decades.
Why did nature not develop architectures better optimized to the functional needs of various
organisms? The discovery of Tiktaalik, a transition between non-tetrapod vertebrates ("fish") to
early tetrapods solved this mystery, providing evidence that all land based creatures that have limbs,
hands, etc. share a common ancestor. The fact that our DNA and that of all land based animals can
be traced back to Tiktaalik further provided evidence that all appendages, whether they are hands
or limbs, are built by similar kinds of genes and that this great evolutionary transformation did not
involve the origin of new DNA: much of the shift likely involved using ancient genes, such as those
involved in development of Tiktaalik’s limbs, in new ways to make wings in birds or various sized
limbs with fingers and toes, explaining the diversity in shapes while maintaining the similarity in
architecture.

1. According to the passage the mechanism that builds limbs as different as bird wings, penguin
flippers, and human hands etc. is

A. controlled through the ZPA, uses the same DNA and the same genes in different ways
and only differs in the concentration of special molecules.
B. controlled through the ZPA and uses the same DNA and complementary genes suited to the
shape and size of the limb.
C. defined by the DNA coming from fish and the corresponding complement of genes activated as
per the needs of the shape, size and strength of the limb.
D. controlled through the ZPA in most tetrapods with some exceptions and uses DNA from
corresponding ancestral heritage of tetrapods with suitable genes turned on.
E. not controlled through ZPA but differs from one vertebrate to another.
2. In the context of the passage, why was the discovery of the Tiktaalik important?

A. It proved beyond doubt and without the need of experimentation the commonality of skeletal
architectures of all land based organisms.
B. It proved the reason why the architecture of limbs in various organisms was optimized for the
organism’s corresponding functional needs.
C. It provided the reason for commonality of skeletal architecture described earlier in
the passage.
D. It traced our ancestral heritage to the fish, providing the reason why we share majority of our
DNA with aquatic animals.
E. It showed how ancient genes could be used in new ways for structures as diverse as wings in
birds and fins in sharks.

3. From the passage, what can be inferred about the architecture of limbs?

A. All land-based creatures that have limbs with similar architecture are also structurally similar.
B. It is different in land-based animals due to the difference in expression of the genes.
C. It evolved since the time of Tiktaalik to suit the functional needs of the land-based animals.
D. It may have been possible to develop an architecture better optimized for the
functional needs of various organisms had nature not adhered to the common
architectural scheme.
E. It is defined by the concentration of special molecules present in the cells of ZPA.

Molecules were not present in cells but made by cells and it was an important factor .
saying it defined is too strong

4. The primary purpose of the passage is

A. To explain the reasons for the similarity in the architecture of the limbs of all land-
based organisms.
B. To evaluate the implications of the experiments of Edgar Zwilling and John Saunders.
C. To trace the importance of the discovery of Tiktaalik.
D. To describe the role a common DNA plays in the unified architecture of the limbs of all land-
based organisms.
E. To deduce how limbs of various sizes and shapes are built by the ZPA tissue.

Architecture (n) : sự kết cấu, mô hình

Radical (adj) : căn bản

Limb (n) : chân tay

Underlying (adj) : cơ bản

Blueprint (n) : bản vẽ thiết kế

Articulate (v) : khớp nối

Elongate (v) : kéo dài

Patch (n) : miếng vá

Polarize (v) : phân cực


Molecule (n) : phân tử

Conundrum (n) : câu hỏi hóc búa

Tetrapod (n) : tứ giác – động vật 4 chân

Vertebrate (n) : động vật có xương sống

Appendage (n) : vật thêm vào

Evolve (v) : tiến hóa, phát triển


Sep 7 – 2

In the 1980's, astronomer Bohdan Paczynski proposed a way of determining whether the enormous
dark halo constituting the outermost part of the Milky Way galaxy is composed of MACHO's (massive
compact halo objects), which are astronomical objects too dim to be visible. Paczynski reasoned that
if MACHO's make up this halo, a MACHO would occasionally drift in front of a star in the Large
Magellanic Cloud, a bright galaxy near the Milky Way. The gravity of a MACHO that had so drifted,
astronomers agree, would cause the star's light rays, which would otherwise diverge, to bend
together so that, as observed from Earth, the star would temporarily appear to brighten, a process
known as microlensing. Because many individual stars are of intrinsically variable brightness, some
astronomers have contended that the brightening of intrinsically variable stars can be mistaken for
microlensing. However, whereas the different colors of light emitted by an intrinsically variable star
are affected differently when the star brightens, all of a star's colors are equally affected by
microlensing. Thus, if a MACHO magnifies a star's red light tenfold, it will do the same to the star's
blue light and yellow light. Moreover, it is highly unlikely that a star in the Large Magellanic Cloud
will undergo microlensing more than once, because the chance that a second MACHO would pass in
front of exactly the same star is minuscule.

1. The passage is primarily concerned with


A. outlining reasons why a particular theory is no longer credited by some astronomers
B. presenting data collected by a researcher in response to some astronomers' criticism of a
particular line of reasoning
C. explaining why a researcher proposed a particular theory and illustrating how influential that
theory has been
D. showing how a researcher's theory has been used to settle a dispute between the researcher and
some astronomers
E. describing a line of reasoning put forth by a researcher and addressing a contention
concerning that line of reasoning

2. According to the passage, Paczynski's theory presumes that if MACHO's constituted the Milky
Way's dark halo, occasionally a MACHO would
A. drift so as to lie in a direct line between two stars in the outer Milky Way
B. affect the light rays of a star in the Large Magellanic Cloud with the result that the
star would seem for a time to brighten
C. become obscured as a result of the microlensing of a star in the Large Magellanic Cloud
D. temporarily increase the apparent brightness of a star in the Large Magellanic Cloud by increasing
the gravity of the star
E. magnify each color in the spectrum of a star in the Large Magellanic Cloud by a different amount

3. It can be inferred from the passage that which of the following would constitute the strongest
evidence of the microlensing of a star in the Large Magellanic Cloud?

A. The brightness of such a star is observed to vary at irregular intervals.


B. The brightening of such a star is observed to be of shorter duration than the brightening of
neighboring stars.
C. The red light of such a star is observed to be brighter than its yellow light and its blue light.
D. The red light, yellow light, and blue light of such a star are observed to be magnified
temporarily by the same factor.
E. The red light of such a star is observed to have increased tenfold.
Intrinsically (adv) : bản chất

Halo (n) : hào quang

Dim (adj) : lờ mờ

Drift (v) : trôi dạt

Diverge (v) : phân ra, chia ra

Microlensing (v) : vi lọc

Variable (adj) : bất định

Magnify (v) : phóng đại

Undergo (v) : trải qua

Minuscule (adj) : cực kỳ thấp

Vary (v) : khác nhau

Contention (n) : sự thảo luận, sự xác định

Dispute (v/n) : tranh chấp/xung đột, tranh luận, đấu khẩu

Temporarily (adv) : tạm thời


Sep 8 – 1

Disruptive innovators can hurt successful and immensely profitable incumbents that tend to ignore
the markets most susceptible to disruptive innovations. Disruptive innovators offer technologically
straightforward solutions consisting of off-the-shelf components put together in a product
architecture that is often simpler, initially lower performing, and cheaper than established
approaches. Considering disruptive technologies unprofitable, the executives at incumbents often
ignored them at their own and companies’ peril. In 1981, the old 8 inch drives used in mini
computers were "vastly superior" and much more profitable to the new 5.25 inch drives used in
desktop computers. However, 8 inch drives were not affordable for the new desktop machines.
Slowly, the makers of 5.25 inch drives improved the performance of the drives and moved the 8
inch drive companies that did not invest in the 5.25 inch technology out of the market as the latter
could not compete on price. Similarly, digital cameras, when introduced in 1997 performed
extremely poorly as compared to traditional film cameras. Consequently, many traditional film
companies such as Kodak ignored this market only to be bankrupted by the rise of digital cameras a
decade later.

Leaders and strategists should be cautious while rejecting a technology that does not seem to be as
high performing and hence not as profitable as their dominant technologies. A technology that
initially provides low performance can drastically improve over time and often exceed the
performance of the dominant technology at a much lower price-point, a scenario that could
potentially bankrupt the incumbents who ignored the technology at their peril.

1. The passage provides information in support of which of the following assertions?

A. The downsides of ignoring to invest in a new technology often outweigh the money savings
associated with the same.
B. A company that ignores a disruptive technology may find itself on the brink of
bankruptcy as the technology improves.
C. Most technologies that initially offer inferior solutions to an existing problem improve to an extent
that they exceed the performance required.
D. Disruptive innovators are always successful in driving the incumbents out of the market.
E. None of the companies who produced the 8 inch drives were able to survive once the
performance of the 5.25 inch drive improved considerably.

2. The author of the passage would make which of the following recommendations to the managers
of the incumbents.

A. Before rejecting a technology that performs poorly when compared to an existing


technology, the managers should carefully evaluate whether the technology can
improve to a point to be disruptive.
B. They should only invest in new technologies that have the potential to disrupt their business
model and throw them out of business.
C. They should take into account the long term shareholder returns before rejecting any new
technology.
D. They should always be wary of start-ups in their industry as they have a potential to disrupt the
incumbent’s operating business model.
E. Before investing huge sums of money in in-house Research and Development, they should
evaluate technologies that have the potential to disrupt the enhancements produced by the R&D
teams.

3. Which of the following exemplifies a technological disruption as described in the passage through
the 8 inch and 5.25 inch disc drive example?

A. Personal computers that initially are extremely unreliable and require frequent service become so
reliable that over 90% of the service people deployed to service the personal computers are let go.
B. Circuit city - a specialty electronics retail chain was bankrupted as general purpose retailers such
as Walmart and Target started carrying electronics.
C. Apple takes over the casual computing marketing previously dominated by the netbook by
introducing the iPad that provides a different level of experience over the general PC experience.
D. Mini mills, such as those operated by companies such as Nucor, that initially made
lowest grade steel improve their quality to the extent that they are able to compete
with large steel mills on quality while beating them on price.
E. DVDs and Blue Ray discs drive out the slightly cheaper VHS tapes as the tapes are not able to
match the discs in terms of quality and convenience.

Brink (n) : bờ vực, bờ sông

Disruptive (adj) : gây rối, sự phân chia

Innovator (n) : người đổi mới

Immensely (adv) : vô cùng

Incumbent (n) : đương nhiệm

Susceptible (adj) : mẫn cảm

Peril (n) : mỗi nguy hiểm, hoài nghi

Affordable (adj) : giá cả phải chăng


Sep 8 – 2

In 1675, Louis XIV established the Parisian seamstresses’ guild, the first independent all-female
guild created in over 200 years. Guild members could make and sell women’s and children’s
clothing, but were prohibited from producing men’s clothing or dresses for court women. Tailors
resented the ascension of seamstresses to guild status; seamstresses, meanwhile, were impatient
with the remaining restrictions on their right to clothe women.

The conflict between the guilds was not purely economic, however. A 1675 police report indicated
that since so many seamstresses were already working illegally, the tailors were unlikely to suffer
additional economic damage because of the seamstresses’ incorporation. Moreover, guild
membership held very different meanings for tailors and seamstresses. To the tailors, their status as
guild members overlapped with their role as heads of household, and entitled them to employ as
seamstresses female family members who did not marry outside the trade. The seamstresses,
however, viewed guild membership as a mark of independence from the patriarchal family. Their
guild was composed not of family units but of individual women who enjoyed unusual legal and
economic privileges. At the conflict’s center was the issue of whether tailors’ female relatives should
be identified as family members protected by the tailors’ guild or as individuals under the jurisdiction
of the seamstresses’ guild.

1. The primary purpose of the passage is to

A. outline a scholarly debate over the impact of the Parisian seamstresses’ guild
B. summarize sources of conflict between the newly created Parisian seamstresses’
guild and the tailors’ guild
C. describe opposing views concerning the origins of the Parisian seamstresses’ guild
D. explore the underlying reasons for establishing an exclusively female guild in seventeenth-century
Paris
E. correct a misconception about changes in seamstresses’ economic status that took place in Paris
in the late seventeenth century

A "scholarly debate" requires at least two competing ideas from some kind of academic or research-
based viewpoint. This passage does not contain that kind of debate -- instead, the author presents
only his or her own research findings about the seamstresses' guild. (A) is out.

2. According to the passage, one source of dissatisfaction for Parisian seamstresses


after the establishment of the seamstresses’ guild was that

A. seamstresses were not allowed to make and sell clothing for all women
B. tailors continued to have the exclusive legal right to clothe men
C. seamstresses who were relatives of tailors were prevented from becoming members of the
seamstresses’ guild
D. rivalry between individual seamstresses increased, thus hindering their ability to compete with
the tailors for business
E. seamstresses were not allowed to accept male tailors as members of the guild

3. It can be inferred from the passage that which of the following was true of
seamstresses employed by relatives who were members of the tailors’ guild?
A. They were instrumental in convincing Louis XIV to establish the seamstresses’ guild.
B. They were rarely allowed to assist master tailors in the production of men’s clothing.
C. They were considered by some tailors to be a threat to the tailors’ monopoly.
D. They did not enjoy the same economic and legal privileges that members of the
seamstresses’ guild enjoyed.
E. They felt their status as working women gave them a certain degree of independence from the
patriarchal family.

4. The author mentions the seamstresses’ view of guild membership as a “mark of


independence from the patriarchal family” (lines 40-41) primarily in order to

A. emphasize that the establishment of the seamstresses’ guild had implications that
were not solely economic
B. illustrate the conflict that existed between tailors and their female family members over
membership in the tailors’ guild
C. imply that the establishment of the seamstresses’ guild ushered in a period of increased economic
and social freedom for women in France
D. provide an explanation for the dramatic increase in the number of women working as
seamstresses after 1675
E. indicate that members of the seamstresses’ guild were financially more successful than were
tailors’ female relatives protected by the tailors’ guild

By saying that "women in France" enjoyed increased freedoms, this answer choice makes a much
broader claim than can be supported by the passage. It may be true that women in the
seamstresses' guild experienced increased economic and social freedoms, but we cannot expand
that to argue that French women in general enjoyed these freedoms. You can eliminate (C) for
question #4.

Seamstress (n) : thợ may nữ

Tailor (n) : thợ may

Ascension (n) : thăng thiên, lên cao

Privilege (n) : đặc quyền

Jurisdiction (n) : quyền hạn


Sep 9 – 1

The concept of difference feminism, proposed by psychologist Carol Gilligan and criticized by many,
holds that men and women have genuine differences that need not all be considered equal. What’s
most enraging, aptly so, to the critics of Gilligan is the idea proposed by her in her book In a
Different Voice that women have their own special version of morality rooted in relationships and
caring rather than the supposedly abstract notions of justice and equity.

But however regressive her ideas were, Gilligan’s views garnered popularity and admiration in the
writings of many others; lesser-known imitations of her book include Nell Noddings’ Caring, Sara
Ruddick’s Maternal Thinking, and Belenky, Clunchy, Golberger and Tarule’s Women’s way of
Knowing.In fact Women’s way of Knowing, a book based on the responses gathered from 135
women interviewees, goes so far as to claim that women are not comfortable with argumentation
and disagreement and that they have a distinct approach to knowledge, one that underlines
collaboration, consensus, and mutual understanding.

The findings of the book have rightly been challenged by critics who point out that the responses
elicited from the interviewees were not fully uncontaminated as these women were told the topic of
the study in advance. However, irrespective of the authenticity of the findings of studies that
confirm the ideas propagated in books such as In a Different Voice and Women’s way of Knowing,
the conclusions drawn by the authors of these books are unwarranted. If the idea that women
prefer to avoid disagreement and to promote understanding and acceptance over judgment and
assessment had been true, there would have been clear and loud demands from women to change
the basic nature of education provided to them as clearly the whole point of critical thinking, which
forms an integral part of the education system, is to know what to accept and what to reject by
assessment and judgment.

1. The author is primarily concerned with

A. To discuss the various merits and demerits of a particular view on women in general.
B. To evaluate the relevance of a proposed theory in the modern world.
C. To objectively describe a commonality among the works of certain authors.
D. To advocate a line of reasoning that is counter to a proposed old-fashioned idea.
E. To criticize a line of thought deemed evidence-less.

Out of Scope. There is no mention of a single proposed theory. The author does discuss, however,
how the views expressed don't reflect the mindset of the women, but there is no way to determine
whether these women are set in the modern world.

2. Which of the following statements can be derived from the passage?

A. Women have a way of acquiring knowledge that is unique to them.


B. In order to obtain unbiased responses, the participants of a study should not have
prior awareness of the subject of the study.
C. Critical thinking is the most central aspect of the education system.
D. Books such as Nodding's Caring are regressive because they are unoriginal in nature.
E. It has been easy for women to voice their demands and opinions when they have felt the need to
do so.
The author agrees with the critics of Women's way of Knowing who challenge that the findings are
not fully uncontaminated, i.e. they are contaminated to some extent since the interviewees were
told the topic of the study in advance. Hence, one can infer that the author would agree with the
statement given in this choice.

The findings of the book have rightly been challenged by critics who point out that the responses
elicited from the interviewees were not fully uncontaminated as these women were told the topic of
the study in advance.
3. Which of the following is the function of the second paragraph in the passage?

A. To raise a consideration that is in the opposite direction to the information given in the preceding
paragraph.
B. To criticize the regressive thinking expressed by Gilligan and her followers.
C. To suggest that regressive ideas about women's individuality become popular despite any merits
in them.
D.To mention the influence a particular author's views had on various other writers
while using one book as an example.
E. To discuss the writings of various authors inspired by a particular writer's views.

4. Which of the following is mentioned in the passage?

A. Many people who criticize Gilligan's concept of difference feminism are more enraged by her take
on women's idea of morality than by any idea in Women's Way of Knowing.
B. The findings of Women's Way of Knowing were deliberately contaminated by the writers of the
book.
C. The process of evaluating and forming an opinion forms the basis for thinking
critically.

This information is explicitly stated in the final sentence of the passage: ... the whole point of critical
thinking...is to know what to accept and what to reject by assessment and judgment.

D. Both In a Different Voice and Women's way of Knowing are based on studies that are somewhat
contaminated.
E. The 135 women interviewees who contributed to Women's Way of Knowing were aware of how
their opinions could be construed as regressive.

A. Out of Scope. Although the author does mention that what most infuriates the critics of Gilligan is
her view that women have their own special version of morality rooted in relationships and caring
rather than the supposedly abstract notions of justice and equity, there is no such comparison given
for critics' take on the views expressed in the two books.

Deliberately (adv) : có chủ đích

Contaminate (v) : tiêm nhiễm một tư tưởng, suy nghĩ

Authenticity (n) : tính xác thực


Sep 9 – 2

Dark matter and dark energy have both been postulated by astrophysicists to fill a gap in the fabric
of the cosmos which is known as the puzzle of the missing mass. In doing so, astrophysicists sought
to bring the characteristics of the universe better in line with Einstein’s Theory of Relativity, which
predicts that the density of the universe is about 25 times greater than has been heretofore
observed. Only 4% of the universe is thought to be composed of conventional matter and energy;
about 22% is thought to be composed of dark matter and the remaining 74% is thought to be dark
energy. Dark matter does not interact with conventional matter or energy by electromagnetic forces
and thus its presence cannot be detected directly. The presence of dark matter must be inferred
through its indirect manifestations: anomalous gravitational effects, the rotational velocity data of
stars in spiral galaxies and the unusual distribution of mass in a star system known as the Bullet
Cluster. Although dark matter was conclusively detected through its gravitational lensing effect in
August 2006, many aspects of dark matter remain speculative. It has been noted that the terms
“dark matter” and “dark energy”serve mainly as expressions of human ignorance, much like the
marking of early maps with “terra incognita.”

1. Which of the following best expresses the central idea of the passage?

(A) Dark matter and dark energy can be directly identified.


(B) Astrophysicists are ignorant of many important details of the universe’s composition.
(C) Although not well understood, dark matter and dark energy help reconcile theory
with observation.
(D) Despite a recent observation, the existence of dark matter and dark energy is still debated in the
scientific community.
(E) Dark matter and dark energy were suggested by Einstein’s Theory of Relativity.

Dark matter and dark energy were suggested by astrophysicists and not Einstein.

2. According to the passage, each of the following is true of dark matter or dark energy
EXCEPT:

(A) They explain why the universe may be 25 times denser than has been observed.
(B) They are thought to account for 96% of the universe.
(C) They are not well understood by physicists.
(D) Their existence can be deduced from observations such as unexpected mass distributions.
(E) They may solve the puzzle of the missing mass.

This is stated in the passage. "The presence of dark matter must be inferred through its
indirect manifestations: the unusual distribution of mass in a star system known as the
Bullet Cluster."

3. The passage suggests which of the following about anomalous gravitational effects?

(A) The Theory of Relativity does not adequately account for them.
(B) Their appearance is not understood by physicists.
(C) They may explain the puzzle of the missing mass.
(D) Without the existence of dark matter, they cannot be satisfactorily explained.
(E) They are related to the distribution of mass in the Bullet Cluster.
The passage doesn't specify any relation between Theory of Relativity and anomalous gravitational
effects. They are mentioned in the passage in different contexts

Postulate (v) : quy định

Astrophysicist (n) : nhà vật lý thiên văn

Fabric (n) : sự cấu tạo

Cosmos (n) : vũ trụ

Manifestation (n) : biểu hiện, sự biểu lộ

Anomalous (adj) : dị thường, vô quy tắc

Rotational (adj) : luân phiên

Spiral (adj) : xoắn ốc

Gravitational lensing (n) : thấu kính hấp dẫn

Ignorance (n) : thiếu hiểu biết


Sep 10 – 1

Although the journal Social Text was never at the forefront of publishing articles on feminism and
never debated whether capitalism was the source of women’s oppression in 1970s or whether male
supremacy was itself a systematic form of domination, it is not clear whether social feminist’s
classification of the journal as the one run by “boy’s club” could have been completely justified till
recently. There could have been many reasons that the journal’s mission statement as set out in its
first prospectus in 1979 did not take notice of the burning issues feminists were then
discussing. May be triumvirate of founding editors were too focused on Marxist high theory to
consider gender alongside economic class as an important mode of social organization and
oppression, or on the other hand they may have simply chosen on purpose to not include feminism
specifically in its charter.

The recent paper by Rosa Luxemburg suggests that the first prospectus contained the seeds of its
own feminist undoing. The founders demarcated fields of focus for the journal that could hardly be
explored without attention to gender, sexuality, and the historical experiences of women. They were
rather interested in “everyday life,” “mass culture,” and “consumer society”. Hence, the little
feminist work that appears in Social Text is in the realm of cultural analysis not revolutionary praxis
and is often buried in the back of the journal in “Unequal Developments,” the section that offers
reviews and experimental writing.

For example, in the second edition of the journal in the section Unequal Developments, Christine
Holmland performs a thorough feminist dissection of the then-current Disney film ‘The North Avenue
Irregulars’, showing how this comedy about a group of church ladies who take on the local mafia
superficially celebrates, but finally deflates the idea of women’s activism, and along the way
reinforces gendered roles at every level of social life.

1. What does the passage suggest about the kind of articles published by Social Text
journal in relation to feminism?

A. It actively published articles on the feminism debating the causes of oppression of women.
B. It did not publish any articles on feminism or made efforts to further women’s cause.
C. While it did publish some articles, they were not the more provocative or
controversial.
D. Its founders discounted the need to focus on women and women’s issues.
E. It refrained from focusing on any burning issues whether related to feminism or any other
societal ill.

2. What does the author imply in the highlighted text


A. Argue that the triumvirate of founding editors did not take that challenges the women face to be
serious enough to merit a place in the journal.
B. Suggest that omission of feminist issues might have been an innocent mistake rather
than a deliberate act.
C. Prove that the triumvirate of founding editors was more focused on male related sufferings rather
than female related ones.
D. State that the original purpose of Social Text excluded the focus on women.
E. Present one side of the argument blaming the deliberate action of editors for Social Text’s lack of
articles on feminism.
3. The passage suggested that while drafting the first prospectus, the editors

A. Excluded the journal from focusing on those fields that required specific attention to
gender, sexuality, and the historical experiences of women.
B. Chose only those fields of focus that were not concerned with the well-being of women.
C. More interested in the emancipation of common man than women.
D. Were too focused on society to focus on the status of women.
E. Only allowed articles related to women in the last sections of the journals.

4. Why does the author cite Christine Holmland’s example? To

A. Illustrate the non-revolutionary nature of work related to women published by the


Social Text.
B. Argue that while not on the forefront of feminism, the journal still did some important
investigative reports.
C. Prove that while the feminist work might not have hogged the limelight of the cover page, it was
nevertheless given enough space to be expressed completely.
D. Demonstrate that the prospectus provided enough freedom to the members of the journal to
publish articles on issues that they deemed important.
E. Compare and contrast the work done by Social Text with that done by other journals in that era.

Forefront (n) : đi đầu, khởi xướng

Article (n) : bài báo

Supremacy (n) : quyền tối cao

Systematic (adj) : có hệ thống

Set out (n) : buổi đầu, đặt ra

Triumvirate (n) : chiến thắng

Prospectus (n) : thông cáo

Undoing (n) : hoàn tác

Demarcate (v) : phân định ranh giới

Realm (n) : vương quốc, địa hạt

Praxis (n) : lời khen, phong tục, tập quán, thói quen

Unequal (adj) : không đồng đều

Dissection (n) : sự mổ xẻ, sự phân tích, khảo sát

Superficially (adv) : hời hợt

Deflat (v) : giảm bớt, xẹp xuống

Reinforce (v) : củng cố


Sep 10 – 2

Anthropologists studying the Hopi people of the southwestern United States often characterize Hopi
society between 1680 and 1880 as surprisingly stable, considering that it was a period of diminution
in population and pressure from contact with outside groups, factors that might be expected to
cause significant changes in Hopi social arrangements.

The Hopis’ retention of their distinctive socio-cultural system has been attributed to the Hopi
religious elite’s determined efforts to preserve their religion and way of life, and also to a
geographical isolation greater than that of many other Native American groups, an isolation that
limited both cultural contact and exposure to European diseases. But equally important to Hopi
cultural persistence may have been an inherent flexibility in their social system that may have
allowed preservation of traditions even as the Hopis accommodated themselves to change. For
example, the system of matrilineal clans was maintained throughout this period, even though some
clans merged to form larger groups while others divided into smaller descent groups. Furthermore,
although traditionally members of particular Hopi clans appear to have exclusively controlled
particular ceremonies, a clan’s control of a ceremony might shift to another clan if the first became
too small to manage the responsibility. Village leadership positions traditionally restricted to
members of one clan might be similarly extended to members of other clans, and women might
assume such positions under certain unusual conditions.

1. The author of the passage would be most likely to agree with which of the following
statements about the explanation outlined in highlighted text?

(A) It fails to take into account the effect of geographical circumstances on Hopi culture.
(B) It correctly emphasizes the role of the religious elite in maintaining the system of matrilineal
clans.
(C) It represents a misreading of Hopi culture because it fails to take into account the actual
differences that existed among the various Hopi clans.
(D) It underestimates the effect on Hopi cultural development of contact with other cultural groups.
(E) It is correct but may be insufficient in itself to explain Hopi sociocultural persistence.

2. Which of the following can be inferred from the passage about the Hopis' geographic
situation between 1680 and 1880?

(A) It prevented the Hopis from experiencing a diminution in population.


(B) It helped to promote flexibility within their social system.
(C) It limited but did not eliminate contact with other cultural groups.
(D) It reinforced the religious elite's determination to resist cultural change.
(E) It tended to limit contact between certain Hopi clans.

3. The passage is primarily concerned with

(A) reassessing a phenomenon in light of new findings


(B) assessing the relative importance of two factors underlying a phenomenon
(C) examining the assumptions underlying an interpretation of a phenomenon
(D) expanding on an explanation of a phenomenon
(E) contrasting two methods for evaluating a phenomenon
4. The author of the passage mentions the control of ceremonies by particular Hopi
clans most likely in order to

(A) provide an example of the way in which the religious elite protected their position within Hopi
society
(B) note an exception to a larger pattern evident in many Native American cultures
(C) explain the connection in Hopi culture between religious and political leadership
(D) illustrate how the Hopis combined flexibility with tradition
(E) cite evidence that counters a common misconception about the structure of Hopi society

5. The passage suggests that sociocultural change in Hopi society between 1680 and
1880 was

A. initiated primarily by contact with other cultural groups


B. greater than that experienced by many other Native American peoples during that period
C. less pronounced than might be expected, given the demographic conditions and
external pressures during that period
D. less pronounced than it had been previously because of diminution in the Hopi population
occurring after 1680
E. accelerated by the particular geographic circumstances of the Hopis

Pronounced (adj) : đáng chú ý, dễ thấy

Diminution (n) : giảm bớt, nhỏ dần

Arrangement (n) : sắp xếp

Retention (n) : giữ lại

Persistence (n) : kiên trì, cố chấp

Accommodate (v) : thích ứng

Matrilineal (adj) : mẫu hệ, thuộc về họ ngoại

Exclusively (adv) : duy nhất

Assume (v) : gánh vác, đảm đương


Sep 11 – 1

Neurodegenerative diseases were once considered disorders of the mind that were deeply rooted in
psychology. Now viruses rank among the environmental factors thought to trigger brain-ravaging
diseases such as multiple sclerosis (MS) and Alzeimer’s disease. Human herpesvirus-6 (HHV-6), in
particular, has been linked to MS in past studies. Neuroscientists have determined that the virus
makes its entry to the human brain through the olfactory pathway, right along with the odors
wafting into our nose. The researchers tested samples of brain cells from people with MS and
healthy control subjects and found evidence of the virus in the olfactory bulb in both
groups. Infection via the nasal passage is probably quite common, as is harboring a dormant
reservoir of HHV-6, but in people with MS, the virus is active. The virus appears to invade the brain
by infecting a type of glial cell called olfactory ensheathing cells (OECs), which nourish smell-sensing
neurons and guide the virus from the olfactory bulb to their targets in the nervous system. These
targets include the limbic system, a group of evolutionarily old structures deep in the brain, which is
where viruses like to reactivate. The olfactory neurons and their OECs are among the few brain cells
known to regenerate throughout our life. This neurogenesis may keep our sense of smell sharp, but
at the cost of providing the virus the opportunity to spread.

1. According to the passage, which of the following is true?

A. The limbic system and the olfactory ensheating cells are one of the many target areas where
viruses like to reactivate.
B. The regeneration of the OECs is linked with the spread of the HHV-6 virus.
C. The smell sensing neurons guide the HHV-6 virus to potential targets in the nervous system.
D. An inactive reservoir of HHV-6 in the body is indicative of the person’s lack of susceptibility to
neurodegenerative diseases.
E. Viruses like to flourish in deep, humidity ridden areas.

2. Why does the author say “Infection via the nasal passage is probably quite
common…”?

A. To indicate that it is extremely difficult to stop infection through the nasal passage.
B. To prove that there is a greater likelihood of multiple sclerosis in people who have active strains
of virus rather than those who do not.
C. To illustrate that HHV-6 infects in the same manner as most other viruses do.
D. To explain the finding that the samples of both the mentioned groups showed the
presence of HHV-6.
E. To indicate that there is a chance that the occurrence of multiple sclerosis may increase in the
future if the strains of HHV-6 are activated.

3. Which of the following can be inferred from the passage?


A. Those who possess higher concentrations of HHV-6 in the olfactory bulb are at a high risk of
developing multiple sclerosis.
B. Those who have a sharper sense of smell are at a greater risk of multiple sclerosis.
C. The absence of multiple sclerosis in a person does not necessarily indicate an absence
of the disease causing virus in that person
D. Multiple sclerosis, alongwith other neurodegenerative diseases, can have disease causing viruses
lying dormant in the human body without any external symptom of the same.
E. In the absence of the limbic system, it would be difficult for the HHV-6 virus to reactivate.
Sep 11 – 2

The term "episodic memory" was introduced by Tulving to refer to what he considered a uniquely
human capacity — the ability to recollect specific past events, to travel back into the past in one's
own mind — as distinct from the capacity simply to use information acquired through past
experiences. Subsequently, Clayton et al. developed criteria to test for episodic memory in animals.
According to these criteria, episodic memories are not of individual bits of information; they involve
multiple components of a single event "bound" together. Clayton sought to examine evidence of
scrub jays' accurate memory of "what," "where," and "when" information and their binding of this
information. In the wild, these birds store food for retrieval later during periods of food scarcity.
Clayton's experiment required jays to remember the type, location, and freshness of stored food
based on a unique learning event. Crickets were stored in one location and peanuts in another. Jays
prefer crickets, but crickets degrade more quickly. Clayton's birds switched their preference from
crickets to peanuts once the food had been stored for a certain length of time, showing that they
retain information about the what, the where, and the when. Such experiments cannot, however,
reveal whether the birds were reexperiencing the past when retrieving the information. Clayton
acknowledged this by using the term "episodic-like" memory.

1. The primary purpose of the passage is to

A. explain how the findings of a particular experiment have been interpreted and offer an alternative
interpretation
B. describe a particular experiment and point out one of its limitations
C. present similarities between human memory and animal memory
D. point out a flaw in the argument that a certain capacity is uniquely human
E. account for the unexpected behavior of animal subjects in a particular experiment

2. According to the passage, Clayton's experiment depended on the fact that scrub jays

A. recall "when" and "where" information more distinctly than "what" information
B. are not able to retain information about a single past event for an indefinitely long period of time
C. choose peanuts over crickets when the crickets have been stored for a long period of
time
D. choose crickets over peanuts whenever both are available
E. prefer peanuts that have been stored for a short period to crickets that have been stored for a
short period

3. The passage suggests that Clayton's experiment demonstrated scrub jays' ability to

A. choose different storage places for different kinds of food to minimize the rate at which a food
will degrade
B. unlearn a behavior they use in the wild in order to adapt to laboratory conditions
C. bind together information about different aspects of a single past event
D. re-experience a past event in memory and act accordingly
E. distinguish one learning event from a subsequent learning event
4. It can be inferred from the passage that both Tulving and Clayton would agree with
which of the following statements?

A. Animals' abilities to use information about a specific past event are not conclusive
evidence of episodic memory.
B. Animals do not share humans' abilities to reexperience the past through memory.
C. The accuracy of animals' memories is difficult to determine through direct experimentation.
D. Humans tend to recollect single bits of information more accurately than do animals.
E. The binding of different kinds of information is not a distinctive feature of episodic memory

Tulving says episodic memory is for humans only, and Clayton says that his work on Jays is
uncertain so calls it 'episodic like'. So neither CONCLUSIVELY argues for this.

Episodic memory (n) : nhớ phân đoạn

Recollect (v) : hồi tưởng

Subsequently (adv) : sau đó

Retrievel (n) : thu hồi, lấy lại

Cricket (n) : con dế

Degrade (v) : xuống cấp


Sep 12 – 1

The role of nurturing in determining one’s behavioral traits has been hotly contested. Historically,
geneticists believed that behavioral traits are inherited. After all, many properties of the brain are
genetically organized and don't depend on information coming in from the senses. Since active
genes are essentially inherited, most traditional geneticists believe that nurturing environment plays
little role in shaping one’s behavioral traits.

However, a new line of research indicated that methyl groups can activate dormant genes, bringing
about a slew of changes much later in a person’s life. The methyl group works like a placeholder in a
cookbook, attaching to the DNA within each cell to select only those recipes - er, genes - necessary
for that particular cell’s proteins, telling the DNA what kind of cells to form. The first such
observation was in which methyl groups activated by causes ranging from exposure to certain
chemicals to changes in diet set off a cascade of cellular changes resulting in cancer. Because
methyl groups are attached to the genes, residing beside but separate from the double-helix DNA
code, their study is dubbed epigenetics - “epi” referring to Greek for outer or above.

Behavioral geneticists, encouraged by this discovery proved that traumatic experiences such as child
neglect, drug abuse, or other severe stresses also set off epigenetic changes to the DNA inside the
neurons of a person’s brain, permanently altering behavior. Similarly, through multivariate analysis,
they proved that identical twins, in scenarios where one twin has gone through a life altering event,
can have vastly different reaction to a stressful situation.

1. The primary purpose of the passage is to

A. propose a new framework that incorporates the role of genetics and the role to life experiences in
determining an individual’s behavioral traits.

- looks close but when this situation pick words ask, is it this ? is it "new framework" ? Also is it
"determining" "behavioral traits" ? like C shaping one’s behavioral traits. "determining" is used when
more like you need to tell which one of these is something.

B. summarize the new research on behavioral traits and inheritance.


C. present findings indicating that, like inheritance, nurturing plays a role in shaping
one’s behavioral traits
D. argue through new findings that life experiences play a vital role in shaping an individual’s
behavioral traits.
E. establish a causal relationship between an individual’s life experiences and his reaction to stressful
situations.

2. Why does the author state highlighted text:

A. to provide a reason for geneticists belief that behavioral traits are inherited.
B. to prove that active genes and not dormant genes define key behavioral traits.
C. to describe the role played by genetic inheritance in determining one’s personality.
D. to indicate that there have been few experiments in the past linking behavior and life experience.
E. to lay the foundation for the development of epigenetics.
3. Which of the following may be inferred from the passage?

A. It is quite likely for twins to develop different personality traits.


B. Resetting the changes introduced by corresponding methyl groups can help cure cancer.
C. Most cancers are caused due to one’s life experiences rather than due to one’s genetic make-up.
D. Traumatic experiences that activate dormant genes can bring about behavioral
changes.
E. Identical twins who have led different lives are likely to have different behavioral traits.

4. In the context of this passage, what is the importance of the example illustrating how
cancer is caused?

A. It proved that life experiences can alter a person’s personality traits.


B. It provided definitive proof of personality change in a person much later in his life.
C. It showed that genes that may be dormant could be activated by external triggers
many years after birth.
D. It led to the coining of the term - epigenetics.
E. It led to a boom in the study of Behavioral genetics.

Framework (n) : an essential supporting structure of a building, vehicle, or object.


Sep 12 – 2

Many managers are influenced by dangerous myths about pay that lead to counterproductive
decisions abouthow their companies compensate employees. One such myth is that labor rates, the
rate per hour paid to workers, are identical with labor costs, the money spent on labor in relation to
the productivity of the labor force. This myth leads to the assumption that a company can simply
lower its labor osts by cutting wages. But labor costs and labor rates are not in fact the same: one
company could pay its workers considerably more than another and yet have lower labor costs if
that company's productivity were higher due to the talent of its workforce, the efficiency of its work
processes, or other factors. The confusion of costs with rates persists partly because labor rates are
a convenient target for managers who want to make an impact on their company's budgets.
Because labor rates are highly visible, managers can easily compare their company's rates with
those of competitors. Furthermore, labor rates often appear to be a company's most malleable
financial variable: cutting wages appears an easier way to control costs than such options as
reconfiguring work processes or altering product design.

The myth that labor rates and labor costs are equivalent is supported by business journalists, who
frequently confound the two. For example, prominent business journals often remark on the "high"
cost of German labor, citing as evidence the average amount paid to German workers. The myth is
also perpetuated by the compensation-consulting industry, which has its own incentives to keep
such myths alive. First, although some of these consulting firms have recently broadened their
practices beyond the area of compensation, their mainstay continues to be advising companies on
changing their compensation practices. Suggesting that a company's performance can be improved
in some other way than by altering its pay system may be empirically correct but contrary to the
consultants' interests. Furthermore, changes to the compensation system may appear to be simpler
to implement than changes to other aspects of an organization, so managers are more likely to find
such advice from consultants palatable. Finally, to the extant that changes in compensation create
new problems, the consultants will continue to have work solving the problems that result from their
advice.

1. The passage suggests that the “myth” mentioned in line 5 persists partly because

A. managers find it easier to compare their companies’ labor rates with those of
competitors than to compare labor costs
B. managers tend to assume that labor rates affect their companies’ budgets less than they actually
do
C. managers tend to believe that labor rates can have an impact on the efficiency of their
companies’ work processes
D. the average amount paid to workers differs significantly from one country to another
E. many companies fail to rely on compensation consultants when making decisions about labor
rates

2. The author of the passage mentions business journals (line 39) primarily in order to

A. demonstrate how a particular kind of evidence can be used to support two different conclusions
B. cast doubt on a particular view about the average amount paid to German workers
C. suggest that business journalists may have a vested interest in perpetuating a particular view
D. identify one source of support for a view common among business managers
E. indicate a way in which a particular myth could be dispelled

3. It can be inferred from the passage that the author would be most likely to agree
with which of the following statements about compensation?

A. A company’s labor costs are not affected by the efficiency of its work processes.
B. High labor rates are not necessarily inconsistent with the goals of companies that
want to reduce costs
C. It is more difficult for managers to compare their companies’ labor rates with those of
competitors than to compare labor costs.
D. A company whose labor rates are high is unlikely to have lower labor costs than other companies.
E. Managers often use information about competitors’ labor costs to calculate those companies’
labor rates.

4. The author of the passage suggests which of the following about the advice that the
consulting firms discussed in the passage customarily give to companies attempting to
control costs?

A. It often fails to bring about the intended changes in companies’ compensation


systems.
B. It has highly influenced views that predominate in prominent business journals.
C. It tends to result in decreased labor rates but increased labor costs.
D. It leads to changes in companies’ compensation practices that are less visible than changes to
work processes would be.
E. It might be different if the consulting firms were less narrowly specialized.

5. According to the passage, which of the following is true about changes to a


company's compensation system?

A. They are often implemented in conjunction with a company's efforts to reconfigure its work
processes.
B. They have been advocated by prominent business journals as the most direct way for a company
to bring about changes in its labor costs.
C. They are more likely to result in an increase in labor costs than they are to bring about
competitive advantages for the company.
D. They sometimes result in significant cost savings but are likely to create labor-relations problems
for the company.
E. They may seem to managers to be relatively easy to implement compared with other
kinds of changes managers might consider.

Furthermore, changes to the compensation system may appear to be simpler to implement than
changes to other aspects of an organization, so managers are more likely to find such advice
from consultants palatable.

6. The primary purpose of the passage is to

(A) describe a common practice used by managers to control labor costs


(B) examine the relation between labor costs and other costs incurred by businesses
(C) explain why labor rates are a more significant factor than labor costs for most businesses
(D) identify a common misperception held by managers and point out some of the
reasons for its persistence
(E) distinguish between a factor that companies can easily control and another that is more difficult
to change

Myth (n) : lối tư duy, niềm tin thường mang tính sai lầm

Malleable (adj) : dễ uốn, dễ bảo

Perpetuate (v) : làm cho lâu dài

Empirically (adv) : theo kinh nghiệm

Palatable (adj) : dẻo miệng, ngon miệng

Extant (adj) : phần còn lại

Consultant (n) : người tư vấn

Conjunction (n) : kết hợp


Sep 13 – 1

The basic theory of plate tectonics recognizes two ways continental margins can grow seaward.
Where two plates move away from a midocean rift that separates them, the continental margins on
those plates are said to be passive. Such continental margins grow slowly from the accumulation of
riverborne sediments and of the carbonate skeletons of marine organisms. Since most sequences of
such accretions, or miogeoclinal deposits, are unreformed, passive margins are not associated with
mountain building.

Along active margins continents tend to grow much faster. At an active margin an oceanic plate
plunges under a continental plate, fragments of which then adhere to the continental margin. The
process is met with extensive volcanism and mountain building. A classic example is the Andes of
the west coast of South America.

In the original plate-tectonic model western North America was described as being initially passive
and then active. It was assumed that the continent grew to a limited extent along this margin as
oceanic rocks accreted in places such as the Coast Ranges of California. The model was successful in
explaining such disparate features as the Franciscan rocks of the California Coast Ranges, created by
subduction, and the granite rocks of the Sierra Nevada that originated in volcanoes.

The basic plate-tectonic reconstruction of the geologic history of western North America remains
unchanged in the light of microplate tectonics, but the details are radically changed. It is now clear
that much more crust was added to North America in the Mesozoic era than can be accounted for by
volcanism and by the simple accretion of sediments. Further, some adjacent terranes are not
genetically related, as would be expected from simple plate tectonics, but have almost certainly
travelled great distances from entirely different parts of the world.

1. Which one of the following best expresses the main idea of the passage?

(A) The margin of the west coast of North America developed through a combination of active and
passive mechanisms.
(B) The growth of continental margins is only partially explained by the basic theory of
plate tectonics.
(C) Continental margins can grow seaward in two ways, through sedimentation or volcanism.
(D) The introduction of microplate tectonics poses a fundamental challenge to the existing theory of
how continental margins are formed.
(E) Continental margins grow more rapidly along active margins than along passive margins.

Since most sequences of such accretions, or miogeoclinal deposits, are unreformed, passive margins
are not associated with mountain building.

2. The passage supplies information for answering all of the following questions
regarding continental margins EXCEPT:

(A) How have marine organisms contributed to the formation of passive continental margins?
(B) What were some of the processes by which the continental margin of the west coast of North
America was formed?
(C) Are miogeoclinal deposits associated with mountain building along continental margins?
(D) How was the continental margin of the west coast of South America formed?
(E) How much crust added to North America in the Mesozoic era can be accounted for by
the accretion of sediments from the ocean floor?

3. The author mentions the Franciscan rocks of the California Coast Ranges in order to
make which one of the following points?

(A) The basic theory of plate tectonics accounts for a wide variety of geologic features.
(B) The original plate tectonic model falls short of explaining such features.
(C) Subduction processes are responsible for the majority of the geologic features found along the
west coast of North America.
(D) Passive margins can take on many geologic forms.
(E) The concept of microplate tectonics was first introduced to account for such phenomena.

4. Which one of the following does the author mention as evidence for the inadequacy of
the original plate tectonic model to describe the formation of continental margins?

(A) Nearly flat, unreformed crystal blocks have been found along some continental margins where
there are mountains further inland.
(B) Sediments and fragments from the depths of the ocean accumulate along continental margins.
(C) Large pieces of the Earth's crust that appear to be completely unrelated are found in
the same area today.
(D) Unreformed miogeoclinal deposits are usually not linked to mountain building.
(E) Oceanic plates drop beneath continental plates along active margins.

It is now clear that much more crust was added to North America in the Mesozoic era than can be
accounted for by volcanism and by the simple accretion of sediments

Plate tectonic (n) : kiến tạo địa tầng

Continental margin (n) : lề lục địa

Seaward (adv) : hướng biển

Midocean rift (n) : rạn nứt ở giữa

Riverbourne (n) : bờ sông

Accretion (n) : bồi đắp

Unreform (v) : không phù hợp

Plunge (v) : lao xuống

Subduction (n) : hút chìm

Crust (n) : vỏ trái đất

Terrane (n) : đất nung

Adjacent (adj) : liền kề


Sep 13 – 2

Thanks to her inclusion in Alexander Pope's mock epic poem "The Dunciad" as a vacuous figure
willing to write (and perhaps do) anything for financial recompense, Eliza Haywood was long
regarded as of marginal importance in the literary history of the "long"18th century. Many of her
best-known novels have been described as "amatory fiction", a genre not considered central to the
British novel tradition. Thus, until recently, even her supporters had little to say for her style, often
emphasizing the historical importance of her journalism.

A closer examination of Haywood's novels indicates that disregarding her fiction in such a manner
would be a mistake. Even if works such as the early "Love in Excess" and the late "The History of
Miss Betsy Thoughtless" did not influence 19th-century novelists to the extent that more famous
novels such as Richardson's "Pamela" and Fielding's "Tom Jones" did, they reveal a prose writer
presenting both structural innovations and a surprisingly forward-looking view of society. Her
shifting attitudes regarding class, courtship, and marriage are reflected in a noticeable change from
her early novels to her more mature works, suggesting that she ought to be considered as more
than a simple polemicist.

In fact, as Paula Backscheider has shown, Haywood's substantial corpus had more of an impact on
the British novel tradition than was previously suspected. Haywood's "Anti-Pamela", a satire of
Richardson's didactic novel, was as widely read as Fielding's parody of the same, and "Betsy
Thoughtless" can be seen as the beginning of a tradition of novels of marriage, which culminated in
Bronte's "Jane Eyre". Most importantly, the 18th-century novel was largely concerned with domestic
issues hinging on the role of women, and Haywood's dozens of works are among the best
representations we have of a female perspective on such topics.

1.The author's main point is that

(A) next to parodies such as "The Dunciad" and "Anti-Pamela", novels of marriage were much more
crucial to the development of the novel.
(B) it is a mistake to treat Haywood's polemical journalism as more than a historical curiosity.
(C) Haywood's novels are valuable in more ways than were thought by previous
generations of scholars.
(D) 18th-century novels by women such as Haywood deserve more attention that novels by men,
such as Richardson and Fielding, on women's topics.
(E) "amatory fiction" influenced many important novelists writing in the 19th century.

2. The primary purpose of the passage is to

(A) offer sociohistorical explanations for the prevalence of female novelists in the 18th century
(B) examine how one novelist has been viewed by scholars throughout history
(C) show why an underrated novelist should be taken more seriously
(D) propose a new ranking of prose writers in the 18th century to reflect more modern literary
preferences
(E) outline specific distinctions between well-known novelists that have long been considered equals

Vacuous figure (adj) : nhân vật không có số má

Recompense (n) : bù đắp


Amatory (adj) : liên quan đến tình yêu hoặc ham muốn tình dục

Disregard (v) : không quan tâm, bỏ qua, xem thường

Courtship (n) : tán tỉnh, trêu gẹo, sự tán gái

Polemicist (n) : nhà chính trị

Corpus (n) : văn thể, tập văn

Satire (n) : châm biếm, chế nhạo

Didactic (adj) : mô phạm, để học, giáo khoa

Culminate (v) : lên đến đỉnh điểm

Hing on (v) : dựa vào


Sep 14 – 1

The great French moralists presented many of their musings on human nature in the form of
maxims, pithy statements of general truths or principles. These maxims often contained a clever
twist, used by the moralists to draw attention to the complexity of the human psyche. Maxims
attempt to emphasize one particular part of human nature or human experience, one that is often
contrary to the received wisdom of the culture. The French moralists often stated their maxims as a
universal truth for maximum effect; the true point of the maxim, however, was to act as a
counterpoint to popular opinion.

A maxim and its contrary might both be true, or more precisely, sometimes true. Consider the
maxim, “we have not the strength to follow our reason all the way,” and its contrary, “we have not
the reason to follow our strength all the way.” Both statements are true enough to warrant a
spirited defense, depending, of course, on the perspectives of the reader. However, the moralists
were not interested in synthesizing these contrary positions into one law. Rather, they wished to
show the deficiencies of conventional wisdom.

1. The author suggests that the French moralists

A. wrote a number of books on morality


B. were more interested in being descriptive than prescriptive
C. did not believe that their maxims were true
D. were upset with the received wisdom of their culture
E. found the human psyche easy to understand

2. According to the information in the passage, the contrary of the maxim “one may
forgive an infidelity, but not forget it,” would most likely be

A. “one may forget an infidelity, but not forgive it”


B. “to forget is human, to forgive divine”
C. “all things may be forgiven except not being forgiven”
D. “fidelity means being true to everyone but oneself”
E. “there is no better feeling than to forgive oneself”

3. The author of the passage would most likely agree with which of the following
statements about maxims?

A. maxims are universal laws


B. the lesson of a maxim is subjective
C. the French moralists wrote the best maxims
D. maxims and their contraries are always true
E. maxims are not effective unless they critique cultural norms

Moralist (n) : nhà đạo đức

Musing (n) : suy nghĩ

Maxim (n) : châm ngôn

Pithy (adj) : đầy khí phách, mạnh mẽ


Twist (n) : khuynh hướng

Wisdom (n) : trí tuệ

Deficiency (n) : thiếu sót, khuyết điểm


Sep 15 – 1

It was in the mid-1970s that women’s studies courses began to make their appearance in Japanese
universities. Thus far, very few studies have actually looked closely at the types of courses offered;
the content and methodology of the courses; how they are perceived and evaluated by students,
faculty, and administrators; and the impact of these courses – both short-term and long-term – on
students as well as on the institutions themselves. An annual survey of women’s studies course
offerings in higher institutions nationwide conducted by National Women’s Education Center since
1983 is one of few such sources.

According to its 1994 report, in 1992, 268 institutions of higher education offered courses in
women’s studies out of a total of 1,101 institutions. There is no question that this represents a
significant increase compared to the late 1970s, when only a handful of institutions offered such
courses. Yet, looking again at the courses and the proportion of institutions offering them, it must
be pointed out that the figure is deceptive. Even in cases in which courses are offered, usually just
one or two courses are taught at each institution. Another questionable point concerns the extent to
which the various courses are taught from a feminist perspective. The NWEC survey gives no
definition for the term women’s studies. The questionnaire asks whether the respondent runs any
‘women’s studies-related course’ by which it basically means any course that is labeled ‘women’s’,
such as ‘Women’s Studies’, ‘Women’s Issues’, or ‘Women’s History’ as well as courses in any of the
various disciplines that deal with the subject of women or give special attention to women. Our
estimate is that courses with explicitly feminist perspectives make up about one-third of all courses
offered. The rest seem to consist of best of an ‘add women and stir’ type of approach, as it were.
Thus the instructor of a course on Korean history whose theme is ‘The History of Korean Family and
Women’ makes the comment: “The course is not from the perspective of women’s studies, but
rather to fill a gap in Japanese research on Korean history”.This is not surprising in the light that, in
the NWEC survey, only 26 out of 499 instructors teaching women’s studies-related courses – all of
whom were women – claimed their field of specialization to be that of women’s studies per se.

1. Which of the following best summarizes the content of the passage?

A. A critique of women's studies as an academic discipline


B. A report on the progress of women's studies programs in Japan
C. An assessment of the status of women's studies programs in the Japanese academia
D. An analysis of the philosophy of women's studies programs
E. An abbreviated history of women's studies programs in Japan in the 1970s

2. According to the author, the 1994 report from National Women's Education Center

A. is the only important annual survey of women's studies programs in higher education in Japan
since 1983
B. shows for the first time a significant increase in total number of women's studies courses offered
since the 1970s
C. highlights that the percentage of male students in women's studies programs is low
D. shows that the male-female ratio in faculties at Japanese universities is not conducive for
women's studies programs
E. indicates significant increase in total number of women's studies courses offered
since the 1970s, but is inadequate in portraying the correct status of women's studies
3. Which of the following can be said about the course ‘The History of Korean Family and
Women’?
A. It represents historic research more than an explicit feminist perspective.
B. It does not conform to National Women's Education Center's definition of "women's studies".
C. It is an old course revised by subtly adding a feminist perspective.
D. It is not likely taught by an instructor who claims women's studies as his/her field of
specialization.
E. It highlights concerns over the extent to which women's studies are taught from feminist
perspectives.

Institution (n) : tổ chức, học viện

Handful (adj) : số ít

Deceptive (adj) : sai sự thật, lừa dối, lừa đảo

Extent (n) : khoảng rộng, mức độ

Theme (n) : chủ đề

Claim (v) : tuyên bố

Field (n) : lĩnh vực

Specialization (n) : chuyên môn hóa

Subtly (adv) : tinh tế

Abbreviated (adj) : ngắn, rút gọn

Academia (n) : học viện


Sep 15 – 2

In American Genesis, which covers the century of technological innovation in the United States
beginning in 1876,Thomas Hughes assigns special prominence to Thomas Edison as archetype of
the independent nineteenth-century inventor. However, Hughes virtually ignores Edison's famous
contemporary and notorious adversary in the field of electric light and power, George Westinghouse.
This comparative neglect of Westinghouse is consistent with other recent historians' works, although
it marks an intriguing departure from the prevailing view during the inventors' lifetimes (and for
decades afterward) of Edison and Westinghouse as the two "pioneer innovators" of the electrical
industry.

My recent reevaluation of Westinghouse, facilitated by materials found in railroad archives, suggests


that while Westinghouse and Edison shared important traits as inventors, they differed markedly in
their approach to the business aspects of innovation. For Edison as an inventor, novelty was always
paramount: the overriding goal of the business of innovation was simply to generate funding for
new inventions. Edison therefore undertook just enough sales, product development, and
manufacturing to accomplish this. Westinghouse, however, shared the attitudes of the railroads and
other industries for whom he developed innovations: product development, standardization, system,
and order were top priorities. Westinghouse thus better exemplifies the systematic approach to
technological development that would become a hallmark of modern corporate research and
development.

1) The primary purpose of the passage is to

(A) reevaluate a controversial theory


(B) identify the flaws in a study
(C) propose a new method of historical research
(D) compare two contrasting analyses
(E) provide a fresh perspective

2) According to the passage, Edison’s chief concern as an inventor was the

(A) availability of a commercial market


(B) costs of developing a prototype
(C) originality of his inventions
(D) maintenance of high standards throughout production
(E) generation of enough profits to pay for continued marketing

3) The author of the passage implies that the shift away from the views of
Westinghouse’s contemporaries should be regarded as

(A) a natural outgrowth of the recent revival of interest in Edison


(B) a result of scholarship based on previously unknown documents
(C) reflective of modern neglect of the views of previous generations
(D) inevitable, given the changing trends in historical interpretations
(E) surprising, given the stature that Westinghouse once had

This comparative neglect of Westinghouse is consistent with other recent historians' works, although
it marks an intriguing departure from the prevailing view during the inventors' lifetimes (and for
decades afterward) of Edison and Westinghouse as the two "pioneer innovators" of the electrical
industry.

This tells us that Edison and Westinghouse had a similar reputation during their lifetime. Hence,
Westinhouse's contemporaries regarded him as one of the two pioneer innovators of their time.
Therefore, it is surprising that historians later on didn't pay attention to Westinhouse anymore (they
shifted away the existing view on Westinghouse).

Originality (n) : sự độc đáo, đặc biệt, kỳ dị

Assign (v) : thừa nhận

Prominence (n) : nổi bật

Archetype (n) : khuôn mẫu, nguyên hình, hình mẫu

Virtually (adv) : hầu như, thật ra

Contemporary (adj) : đồng thời, cùng lúc

Notorious (adj) : khét tiếng, ai cũng biết, rõ ràng

Adversary (n) : đối thủ

Neglect (n) : bỏ mặc

Consistent (adj) : thích hợp

Intriguing (adj) : hấp dẫn

Pioneer (v) : tiên phong

Archive (n) : tài liệu lưu trữ

Novelty (n) : mới lạ

Paramount (adj) : tối quan trọng

Overriding (adj) : quan trọng hơn tất cả

Hallmark (n) : dấu ấn

Corporate research (n) : nghiên cứu doanh nghiệp

Stature (n) : tầm vóc


Sep 16 – 1

Studies of the Weddell seal in the laboratory have described the physiological mechanisms that allow
the seal to cope with the extreme oxygen deprivation that occurs during its longest dives, which can
extend 500 meters below the ocean’s surface and last for over 70 minutes. Recent field studies,
however, suggest that during more typical dives in the wild, this seal’s physiological behavior is
different.

In the laboratory, when the seal dives below the surface of the water and stops breathing, its heart
beats more slowly, requiring less oxygen, and its arteries become constricted, ensuring that the
seal’s blood remains concentrated near those organs most crucial to its ability to navigate
underwater. The seal essentially shuts off the flow of blood to other organs, which either stop
functioning until the seal surfaces or switch to an anaerobic (oxygen-independent) metabolism. The
latter results in the production of large amounts of lactic acid which can adversely affect the pH of
the seal’s blood, but since the anaerobic metabolism occurs only in those tissues which have been
isolated from the seal’s blood supply, the lactic acid is released into the seal’s blood only after the
seal surfaces, when the lungs, liver, and other organs quickly clear the acid from the seal’s
bloodstream.

Recent field studies, however, reveal that on dives in the wild, the seal usually heads directly for its
prey and returns to the surface in less than twenty minutes. The absence of high levels of lactic acid
in the seal’s blood after such dives suggests that during them, the seal’s organs do not resort to the
anaerobic metabolism observed in the laboratory, but are supplied with oxygen from the blood. The
seal’s longer excursions underwater, during which it appears to be either exploring distant routes or
evading a predator, do evoke the diving response seen in the laboratory. But why do the seal’s
laboratory dives always evoke this response, regardless of their length or depth? Some biologists
speculate that because in laboratory dives the seal is forcibly submerged, it does not know how long
it will remain underwater and so prepares for the worst.

1. The passage provides information to support which of the following generalizations?

(A) Observations of animals’ physiological behavior in the wild are not reliable unless verified by
laboratory studies.
(B) It is generally less difficult to observe the physiological behavior of an animal in the wild than in
the laboratory.
(C) The level of lactic acid in an animal’s blood is likely to be higher when it is searching for prey
than when it is evading predators.
(D) The level of lactic acid in an animal’s blood is likely to be lowest during those periods in which it
experiences oxygen deprivation.
(E) The physiological behavior of animals in a laboratory setting is not always consistent
with their physiological behavior in the wild.

2. It can be inferred from the passage that by describing the Weddell seal as preparing
“for the worst” (line 41), biologists mean that it

(A) prepares to remain underwater for no longer than twenty minutes


(B) exhibits physiological behavior similar to that which characterizes dives in which it heads directly
for its prey
(C) exhibits physiological behavior similar to that which characterizes its longest dives
in the wild
(D) begins to exhibit predatory behavior
(E) clears the lactic acid from its blood before attempting to dive

Studies of the Weddell seal in the laboratory have described the physiological mechanisms that allow
the seal to cope with the extreme oxygen deprivation that occurs during its longest dives

3. The passage suggests that during laboratory dives, the pH of the Weddell seal’s blood
is not adversely affected by the production of lactic acid because

(A) only those organs that are essential to the seal’s ability to navigate underwater revert to an
anaerobic mechanism
(B) the seal typically reverts to an anaerobic metabolism only at the very end of the dive
(C) organs that revert to an anaerobic metabolism are temporarily isolated from the
seal’s bloodstream
(D) oxygen continues to be supplied to organs that clear lactic acid from the seal’s bloodstream
(E) the seal remains submerged for only short periods of time

4. Which of the following best summarizes the main point of the passage?

(A) Recent field studies have indicated that descriptions of the physiological behavior of
the Weddell seal during laboratory dives are not applicable to its most typical dives in
the wild.
(B) The Weddell seal has developed a number of unique mechanisms that enable it to remain
submerged at depths of up to 500 meters for up to 70 minutes.
(C) The results of recent field studies have made it necessary for biologists to revise previous
perceptions of how the Weddell seal behaves physiologically during its longest dives in the wild.
(D) Biologists speculate that laboratory studies of the physiological behavior of seals during dives
lasting more than twenty minutes would be more accurate if the seals were not forcibly submerged.
(E) How the Weddell seal responds to oxygen deprivation during its longest dives appears to depend
on whether the seal is searching for prey or avoiding predators during such dives.

5. According to the author, which of the following is true of the laboratory studies
mentioned in line 1?

(A) They fail to explain how the seal is able to tolerate the increased production of lactic acid by
organs that revert to an anaerobic metabolism during its longest dives in the wild.
(B) They present an oversimplified account of mechanisms that the Weddell seal relies on during its
longest dives in the wild.
(C) They provide evidence that undermines the view that the Weddell seal relies on an anaerobic
metabolism during its most typical dives in the wild.
(D) They are based on the assumption that Weddell seals rarely spend more than twenty minutes
underwater on a typical dive in the wild.
(E) They provide an accurate account of the physiological behavior of Weddell seals
during those dives in the wild in which they are either evading predators or exploring
distant routes.
6. The author cites which of the following as characteristic of the Weddell seal’s
physiological behavior during dives observed in the laboratory?

I. A decrease in the rate at which the seal’s heart beats


II. A constriction of the seal’s arteries
III. A decrease in the levels of lactic acid in the seal’s blood
IV. A temporary halt in the functioning of certain organs

(A) I and III only


(B) II and IV only
(C) II and III only
(D) I, II, and IV only
(E) I, III, and IV only

7. The passage suggests that because Weddell seals are forcibly submerged during
laboratory dives, they do which of the following?

(A) Exhibit the physiological responses that are characteristic of dives in the wild that last less than
twenty minutes.
(B) Exhibit the physiological responses that are characteristic of the longer dives they
undertake in the wild.
(C) Cope with oxygen deprivation less effectively than they do on typical dives in the wild.
(D) Produce smaller amounts of lactic acid than they do on typical dives in the wild.
(E) Navigate less effectively than they do on typical dives in the wild.

Deprivation (n) : thiếu hụt

Artery (n) : động mạch

Constrict (v) : teo lại, siết lại

Navigate (v) : điều hướng

Anaerobic (adj) : kỵ khí

Metabolism (n) : sự trao đổi chất

Resort (v) : dựa vào, trông cậy vào

Excursion (n) : du ngoạn

Evade (v) : trốn tránh

Evoke (v) : gợi lại

Forcibly (adv) : cưỡng bức, ép buộc

Submerge (v) : làm cho chìm, ngập nước


Sep 16 – 2

In the fourteenth and fifteenth centuries, many Western Pueblo settlements in what is now the
southwestern United Line States may have possessed distinctly hierarchical organizational
structures. These communities' agricultural systems-which were "intensive" in the use of labor rather
than "extensive" in area-may have given rise to political leadership that managed both labor and
food resources. That formal management of food resources was needed is suggested by the large
size of storage spaces located around some communal Great Kivas (underground ceremonial
chambers). Though no direct evidence exists that such spaces were used to store food, Western
Pueblo communities lacking sufficient arable land to support their populations could have preserved
the necessary extra food, including imported foodstuffs, in such apparently communal spaces.

Moreover, evidence of specialization in producing raw materials and in manufacturing ceramics and
textiles indicates differentiation of labor within and between communities. The organizational and
managerial demands of such specialization strengthen the possibility that a decision-making elite
existed, an elite whose control over labor, the use of community surpluses, and the acquisition of
imported goods would have led to a concentration of economic resources in their own hands.
Evidence for differential distribution of wealth is found in burials of the period: some include large
quantities of pottery, jewelry, and other artifacts, whereas others from the same sites lack any such
materials.

1. According to the passage, which of the following is probably true of the storage
spaces mentioned in line 14?

(A) They were used by the community elite for storage of their own food supplies.
(B) They served a ceremonial as well as a practical function.
(C) Their size is an indication of the wealth of the particular community to which they belonged.
(D) Their existence proves that the community to which they belonged imported large amounts of
food.
(E) They belonged to and were used by the community as a whole.

That formal management of food resources was needed is suggested by the large size of storage
spaces located around some communal Great Kivas

2. The primary purpose of the passage is to

(A) outline the methods by which resources were managed within a particular group of communities
(B) account for the distribution of wealth within a particular group of communities
(C) provide support for a hypothesis concerning the social structure of a particular
society
(D) explain how political leadership changed in a particular historical situation
(E) present new evidence that contradicts previous theories about a particular historical situation

3. Which of the following, if true, would most clearly undermine the author's statement
in the last sentence of the passage (lines 38-43) regarding the distribution of wealth in
Western Pueblo settlements?

(A) Only community members of exceptional wealth are likely to have been buried with their
personal possessions.
(B) Members of communities with extensive agricultural systems are usually buried without personal
possessions.
(C) Most artifacts found in burial sites were manufactured locally rather than imported from other
communities.
(D) Burial artifacts are often ritual objects associated with religious practices rather
than being the deceased's personal possessions.
(E) The quality of burial artifacts varies depending on the site with which they are associated.

Hierarchical (adj) : thứ bậc

Intensive (adj) : chuyên sâu

Extensive (adj) : sâu rộng, hiểu biết rộng

Communal (adj) : tài sản chung

Chamber (n) : buồng, phòng

Arable (adj) : trồng trọt

Ceramic (n) : đồ gốm

Deceased (adj) : người chết, người quá cố


Sep 18 – 1

According to a theory advanced by researcher Paul Martin, the wave of species extinctions that
occurred in North America about 11,000 years ago, at the end of the Pleistocene era, can be directly
attributed to the arrival of humans, i.e., the Paleoindians, who were ancestors of modern Native
Americans. However, anthropologist Shepard Krech points out that large animal species vanished
even in areas where there is no evidence to demonstrate that Paleoindians hunted them. Nor were
extinctions confined to large animals: small animals, plants, and insects disappeared, presumably
not all through human consumption. Krech also contradicts Martin's exclusion of climatic change as
an explanation by asserting that widespread climatic change did indeed occur at the end of the
Pleistocene. Still, Krech attributes secondary if not primary responsibility for the extinctions to the
Paleoindians, arguing that humans have produced local extinctions elsewhere. But, according to
historian Richard White, even the attribution of secondary responsibility may not be supported by
the evidence. White observes that Martin's thesis depends on coinciding dates for the arrival of
humans and the decline of large animal species, and Krech, though aware that the dates are
controversial, does not challenge them; yet recent archaeological discoveries are providing evidence
that the date of human arrival was much earlier than 11,000 years ago.

1: Which of the following is true about Martin’s theory, as that theory is described in the
passage?

(A) It assumes that the Paleoindians were primarily dependent on hunting for survival.
(B) It denies that the Pleistocene species extinctions were caused by climate change.
(C) It uses as evidence the fact that humans have produced local extinctions in other situations.
(D) It attempts to address the controversy over the date of human arrival in North America.
(E) It admits the possibility that factors other than the arrival of humans played a role in the
Pleistocene extinctions.

2: In the last sentence of the passage, the author refers to “recent archaeological
discoveries” (lines 36-37) most probably in order to

(A) refute White’s suggestion that neither Maritn nor Krech adequately account for Paleoindians’
contributions to the Pleistocene extinctions
(B) cast doubt on the possibility that a more definitive theory regarding the causes of the
Pleistocene extinctions may be forthcoming
(C) suggest that Martin’s, Krech’s, and White’s theories regarding the Pleistocene extinctions are all
open to question
(D) call attention to the most controversial aspect of all the current theories regarding the
Pleistocene extinctions
(E) provide support for White’s questioning of both Martin’s and Krech’s positions
regarding the role of Paleoindians in the Pleistocene extinctions

3: Which of the following, if true, would most weaken Krech’s objections to Martin’s
theory?

(A) Further studies showing that the climatic change that occurred at the end of the Pleistocene era
was even more severe and widespread than was previously believed
(B) New discoveries indicating that Paleoindians made use of the small animals, plants,
and insects that became extinct
(C) Additional evidence indicating that widespread climatic change occurred not only at the end of
the Pleistocene era but also in previous and subsequent eras
(D) Researchers’ discoveries that many more species became extinct in North America at the end of
the Pleistocene era than was previously believed
(E) New discoveries establishing that both the arrival of humans in North America and the wave of
Pleistocene extinctions took place much earlier than 11,000 years ago

4: The passage suggests that Krech would be most likely to agree with a theory of the
Pleistocene species extinctions that

(A) included climate change as one of the causes of the extinctions


(B) incorporated a revised date for human arrival in North America
(C) eliminated the Paleoindians as a factor in the extinctions
(D) identified a single cause for the extinctions
(E) emphasized the role of hunting in causing most species extinctions

5. The passage mentions the extinction of species other than large animals (see
highlighted text)[Nor were extinctions confined to large animals: small animals, plants,
and insects disappeared] most probably in order to

(A) suggest that the Paleoindians were responsible for more extinctions than Martin's theory
assumes
(B) provide support for the speculation that humans arrived in North America significantly earlier
than the end of the Pleistocene era
(C) point out the only area in which Martin, Krech, and White agree concerning the circumstances of
the Pleistocene extinctions
(D) cite additional evidence tending to support Krech's conclusions about the role of
humans in the Pleistocene extinctions
(E) raise a question about the logical consistency of Krech's view of Martin's theory

Vanish (v) : biến mất

Confine (v) : hạn chế

Presumably (adv) : có lẽ

Consumption (n) : sự tiêu diệt

Climatic (adj) : khí hậu

Coincide (v) : trùng hợp


Sep 18 – 2

After evidence was obtained in the 1920s that the universe is expanding, it became reasonable to
ask: Will the universe continue to expand indefinitely, or is there enough mass in it for the mutual
attraction of its constituents to bring this expansion to a halt? It can be calculated that the critical
density of matter needed to brake the expansion and “close” the universe is equivalent to three
hydrogen atoms per cubic meter. But the density of the observable universe—luminous matter in
the form of galaxies—comes to only a fraction of this. If the expansion of the universe is to stop,
there must be enough invisible matter in the universe to exceed the luminous matter in density by a
factor of roughly 70.

Our contribution to the search for this “missing matter” has been to study the rotational velocity of
galaxies at various distances from their center of rotation. It has been known for some time that
outside the bright nucleus of a typical spiral galaxy luminosity falls off rapidly with distance from the
center. If luminosity were a true indicator of mass, most of the mass would be concentrated toward
the center. Outside the nucleus the rotational velocity would decrease geometrically with distance
from the center, in conformity with Kepler’s law. Instead we have found that the rotational velocity
in spiral galaxies either remains constant with increasing distance from the center or increases
slightly. This unexpected result indicates that the falloff in luminous mass with distance from the
center is balanced by an increase in nonluminous mass.

Our findings suggest that as much as 90 percent of the mass of the universe is not radiating at any
wavelength with enough intensity to be detected on the Earth. Such dark matter could be in the
form of extremely dim stars of low mass, of large planets like Jupiter, or of black holes, either small
or massive. While it has not yet been determined whether this mass is sufficient to close the
universe, some physicists consider it significant that estimates are converging on the critical value.

1. The passage is primarily concerned with

(A) defending a controversial approach


(B) criticizing an accepted view
(C) summarizing research findings
(D) contrasting competing theories
(E) describing an innovative technique

2. The authors’ study indicates that, in comparison with the outermost regions of a
typical spiral galaxy, the region just outside the nucleus can be characterized as having

(A) higher rotational velocity and higher luminosity


(B) lower rotational velocity and higher luminosity
(C) lower rotational velocity and lower luminosity
(D) similar rotational velocity and higher luminosity
(E) similar rotational velocity and similar luminosity

It has been known for some time that outside the bright nucleus of a typical spiral galaxy
luminosity falls off rapidly with distance from the center.

Outside the nucleus the rotational velocity would decrease geometrically with distance from the
center, in conformity with Kepler’s law. Instead we have found that the rotational velocity in
spiral galaxies either remains constant with increasing distance from the center or increases
slightly.

3. The authors’ suggestion that “as much as 90 percent of the mass of the universe is
not radiating at any wavelength with enough intensity to be detected on the Earth”
(lines 34–37) would be most weakened if which of the following were discovered to be
true?

(A) Spiral galaxies are less common than types of galaxies that contain little
nonluminous matter.
(B) Luminous and nonluminous matter are composed of the same basic elements.
(C) The bright nucleus of a typical spiral galaxy also contains some nonluminous matter.
(D) The density of the observable universe is greater than most previous estimates have suggested.
(E) Some galaxies do not rotate or rotate too slowly for their rotational velocity to be measured.

Instead we have found that the rotational velocity in spiral galaxies either remains
constant with increasing distance from the center or increases slightly. This unexpected result
indicates that the falloff in luminous mass with distance from the center is balanced by an increase
in nonluminous mass.
4. It can be inferred from information presented in the passage that if the density of the
universe were equivalent to significantly less than three hydrogen atoms per cubic
meter, which of the following would be true as a consequence?

(A) Luminosity would be a true indicator of mass.


(B) Different regions in spiral galaxies would rotate at the same velocity.
(C) The universe would continue to expand indefinitely.
(D) The density of the invisible matter in the universe would have to be more than 70 times the
density of the luminous matter.
(E) More of the invisible matter in spiral galaxies would have to be located in their nuclei than in
their outer regions.

It can be calculated that the critical density of matter needed to brake the expansion and “close” the
universe is equivalent to three hydrogen atoms per cubic meter

5. The authors propose all of the following as possibly contributing to the “missing
matter” in spiral galaxies EXCEPT

(A) massive black holes


(B) small black holes
(C) small, dim stars
(D) massive stars
(E) large planets

Constituent (n) : thành phần

Halt (n) : tạm dừng lại

Luminous (adj) : dạ quang

Rotational velocity (n) : vận tốc quay


Rotation (n) : vòng xoay

Nucleus (n) : trung tâm

Spiral (n) : xoắn ốc

Luminosity (n) : độ sáng

Geometrically (adv) : hình học

Conformity (n) : sự phù hợp

Radiate (v) : tỏa ra, phóng xạ

Converge (v) : hội tụ


Sep 19 – 1

Individual studies and experiments the world over have shown that a strong correlation exists
between music and mood --- listening to a particular kind of music impacts the mood of the listener.
In fact studies have also shown that, in everyday life, music is primarily used for mood and emotion
regulation. Indeed, one of the reasons music is able to transcend the barriers of language and
garner universal appeal is the emotional response it evokes in its listeners. However, not all people
respond similarly to the same kind of music.

A new study shows that listening to the same piece of sad music can actually make some people
happy while others sad. Clearly, the difference in the response generated is due to the difference or
the lack of the same in the perceived and the induced emotions experienced by the participants.
Perceived emotion is defined as the act of sensing the emotional content of the stimuli whereas
induced emotion is the emotion felt by the receiver after being subjected to the stimuli.

In the study, the participants with a higher level of exposure to and knowledge of music were the
ones who reported being happy after listening to the music, while others reported as being sad.
These participants rated the piece of sad music as highly unpleasant on the scale of perceived
emotions; however, their induced emotion score was really low for the level of unpleasantness
experienced by them and hence did not match with their perceived emotions score. For this reason,
these participants felt that they could enjoy the piece of music without feeling sad and hence
reported their emotional state as closer to being happy.

However, the findings of the study do not necessarily suggest that the perceived emotion score by
the not so musically literate was low. It could just mean that either their perceived and induced
emotion scores coincided or that the difference between the two was insignificant.

1. The author is primarily concerned with

A. debunking a myth about the effect a piece of music has on people


B. suggesting how individual knowledge can shape the effect a piece of music has on different
people
C. discussing various studies that relate to the effect music has on people's mood
D. explaining why different personality types respond differently to the same piece of music
E. explaining an observed result of a study done on the link between music and mood

2. Which one of the following statements can be derived from the passage?

A. People who have a high knowledge of music in general are more likely to distance themselves
from the emotional content of a piece of music.
B. The emotional response generated in a listener to a piece of music can override the
blockages created by the unfamiliar content in it.
C. The participants who reported closer to being happy were not actually happy but instead felt no
sadness.
D. A sad piece of music can easily be used to evoke happiness in listeners.
E. The result of the study would surely not have been the same had all the participants had the
same background in music.
The author says that Indeed, one of the reasons music is able to transcend the barriers of language
and garner universal appeal is the emotional response it evokes in its listeners. This choice is a
restatement of the author's opinion.

3. Which of the following is the function of the final paragraph in the passage?

A. To raise a consideration that sheds lights on a possible implication of the finding of


the study
B. To leave the reader with a thought that is completely in the opposite direction to the preceding
discussion
C. To prescribe further investigation in to the reasoning behind the offered explanation
D. To suggest that the study's findings are ambiguous and hence not reliable
E. To rule out the myth that people with higher knowledge of music are also the ones with a better
understanding of music

4. Each of the following can be inferred from the passage EXCEPT

A. Participants were unaware of the difference between their perceived and induced
emotion score.
B. The main reason people listen to music in day to day life is to adjust their emotions.
C. It is possible that even when the perceived score was high, the participant felt sad after listening
to the piece of sad music
D. The emotional response to a piece of music with lyrics in an unknown language is partly
responsible for the possible enjoyment it offers to its listeners.
E. Various studies have shown a causal relationship between music and its effect on the listener's
mood.

Induce (v) : gây ra

Stimuli (n) : kích thích


Sep 19 – 2

Many economists believe that a high rate of business savings in the United States is a necessary
precursor to investment, because business savings, as opposed to personal savings, comprise
almost three-quarters of the national savings rate, and the national savings rate heavily influences
the overall rate of business investment.

These economists further postulate that real interest rates-the difference between the rates charged
by lenders and the inflation rates-will be low when national savings exceed business investment
(creating a savings surplus),and high when national savings fall below the level of business
investment (creating a savings deficit ).

However, during the 1960's real interest rates were often higher when the national savings surplus
was large. Counterintuitive behavior also occurred when real interest rates skyrocketed from 2
percent in 1980 to 7 percent in 1982, even though national savings and investments were roughly
equal throughout the period. Clearly, real interest rates respond to influences other than the
savings/investment nexus. Indeed, real interest rates may themselves influence swings in the
savings and investment rates. As real interest rates shot up after 1979, foreign investors poured
capital into the United States, the price of domestic goods increased prohibitively abroad, and the
price of foreign-made goods became lower in the United States. As a result, domestic economic
activity and the ability of businesses to save and invest were restrained.

1. The passage is primarily concerned with

(A) contrasting trends in two historical periods


(B) presenting evidence that calls into question certain beliefs
(C) explaining the reasons for a common phenomenon
(D) criticizing evidence offered in support of a well-respected belief
(E) comparing conflicting interpretations of a theory

2. According to the passage, which of the following resulted from foreign investment in
the United States after 1979?

(A) An increase in real interest rates


(B) A decrease in the savings rate of certain other nations
(C) An increase in American investment abroad
(D) An increase in the price of American goods abroad
(E) A decrease in the price of domestic goods sold at home

3. The author of the passage would be most likely to agree with which of the following
statements regarding the economists mentioned in line 1?

(A) Their beliefs are contradicted by certain economic phenomena that occurred in the
United States during the 1960's and the 1980's.
(B) Their theory fails to predict under what circumstances the prices of foreign and domestic goods
are likely to increase.
(C) They incorrectly identify the factors other than savings and investment rates that affect real
interest rates.
(D) Their belief is valid only for the United States economy and not necessarily for other national
economies.
(E) They overestimate the impact of the real interest rate on the national savings and investment
rates.

Prohibitively (adv) : cực kỳ

Precursor (n) : tiền thân

Comprise (v) : bao gồm

Postulate (v) : yêu cầu, định đề

Counterintuitive (adj) : phản trực giác

Nexus (n) : mối quan hệ


Sep 20 – 1

This much was well known that the purpose of a gene is to store the recipe for making proteins and
that we inherit from our parents a gigantic list of recipes for making proteins and protein-making
machines. What was not clear as late as the early 20th century until Gregor Mendel’s experiments
were published and understood was that when crossed, the expression of these genes skipped a
generation.

Mendel discovered that when white flower and purple flower plants are crossed, the result is not a
blend. Rather than being a mix of the two, the offspring was purple flowered. He then conceived the
idea of heredity units, which he called "factors", one of which is a recessive characteristic and the
other dominant. Mendel said that factors, later called genes, normally occur in pairs in ordinary body
cells, yet segregate during the formation of sex cells. The dominant gene, such as the purple flower
in Mendel's plants, will hide the recessive gene, the white flower. Mendel crossed over 29,000 plants
including inflated seed pods with wrinkled seed pods, green unripe pods with yellow unripe pods,
and axial flowers with terminal flowers. In every case the resulting hybrids were just like one parent
with the essence of other parent missing. However, as these hybrids were allowed to self-fertilize,
the essence of the recessed parent re-appeared in exactly one third of the cases. This re-
appearance is called the law of thirds. This law is not just obeyed in plants but also in every living
species as demonstrated by Nageli’s experiment on crossing angora cats with another breed. Nageli
observed that the angora coat disappeared in the next generation but re-appeared in the kittens in
the third.

Mendel’s law has been put to a variety of uses such as selecting the right plants to hybridize to get
the desired mix of dominant factors. A surprising use of the law has been to explain Alkaptonuria in
which the patients suffered from arthritis and their urine and the ear wax turned reddish black upon
exposure to air. Rare in the general population but frequent in children of first-cousin marriages, the
incidence of Alkaptonuria is attributed to expression of recessive factors.

1. According to the passage, all of the following about Mendel’s dominant factors are
true except

A. They were later called genes.

“Mendel said that factors, later called genes”.

B. They exist in all living organisms including human beings.


C. They belong to only one of the crossed parents.
D. They define some of the key characteristics of the first offspring.
E. They are responsible for the incidence of Alkaptonuria.

the incidence of Alkaptonuria is attributed to expression of recessive factors.”, recessive factors and
not dominant factors are responsible

2. The author mentions Nageli’s experiment to illustrate

A. How unique property of the Angora cat – its fur can be transferred to another cat species.
B. An example of the validity of law of thirds in other living organisms.
C. That it is possible to cross-fertilize extremely distinct species.
D. An instance in which the recessive factor dominates the first offspring.
E. Demonstrate how selective species when cross-hybridized could lead to an offspring with
extremely desirable characteristics.

3. The primary purpose of the passage is to

A. Explain Mendel's law and its utilities


B. Describe why a phenomena that was readily observed took so long to be formalized
C. Illustrate how Mendel discovered the law of thirds.
D. Show with appropriate examples, that crossbreeding can lead to offsprings that may show
characteristics that are present in neither of the parents.
E. To evaluate some of the benefits of Mendal’s law of thirds

4. According to the passage, which of the following is not true about genes?

A. Genes store a recipe for making proteins.


B. We inherit a number of genes from our parents.
C. They occur in pairs in every cell but those that govern our sex.
D. The dominant genes in crossed offspring may never allow the recessive gene to
express even in a future generations.
E. The recessive gene in a crossed offspring is not expressed in the first generation.

Gigantic (adj) : khổng lồ

Heredity (n) : di truyền

Recessive (adj) : lặn

Self-fertilize (v) : tự thụ tinh

Hybridize (v) : lai tạo

Arthritis (n) : viêm khớp

Urine (n) : nước tiểu

Incidence (n) : tỷ lệ mắc


Sep 20 – 2

Behavior science courses should be gaining prominence in business school curricula. Recent
theoretical work convincingly shows why behavioral factors such as organizational culture and
employee relations are among the few remaining sources of sustainable competitive advantage in
modern organizations. Furthermore, empirical evidence demonstrates clear linkages between human
resource (HR) practices based in the behavioral sciences and various aspects of a firm’s financial
success. Additionally, some of the world’s most successful organizations have made unique HR
practices a core element of their overall business strategies.

Yet the behavior sciences are struggling for credibility in many business schools. Surveys show that
business students often regard behavioral studies as peripheral to the mainstream business
curriculum. This perception can be explained by the fact that business students, hoping to increase
their attractiveness to prospective employers, are highly sensitive to business norms and practices,
and current business practices have generally been moving away from an emphasis on
understanding human behavior and toward more mechanistic organizational models. Furthermore,
the status of HR professionals within organizations tends to be lower than that of other executives.

Students’ perceptions would matter less if business schools were not increasingly dependent on
external funding—form legislatures, businesses, and private foundations—for survival. Concerned
with their institutions’ ability to attract funding, administrators are increasingly targeting low-
enrollment courses and degree programs for elimination.

1. The primary purpose of the passage is to

(A) propose a particular change to business school curricula


(B) characterize students’ perceptions of business school curricula
(C) predict the consequences of a particular change in business school curricula
(D) challenge one explanation for the failure to adopt a particular change in business school
curricula
(E) identify factors that have affected the prestige of a particular field in business school
curricula

2. The author of the passage mentions "empirical evidence" (line 2) primarily in order to

(A) question the value of certain commonly used HR practices


(B) illustrate a point about the methodology behind recent theoretical work in the behavioral
sciences
(C) support a claim about the importance that business schools should place on courses
in the behavioral sciences
(D) draw a distinction between two different factors that affect the financial success of a business
(E) explain how the behavioral sciences have shaped HR practices in some business organizations

3. The author of the passage suggests which of the following about HR professionals in
business organizations?

(A) They are generally skeptical about the value of mechanistic organizational models.
(B) Their work increasingly relies on an understanding of human behavior.
(C) Their work generally has little effect on the financial performance of those organizations.
(D) Their status relative to other business executives affects the attitude of business
school students toward the behavioral sciences.
(E) Their practices are unaffected by the relative prominence of the behavioral sciences within
business schools.

4. The author of the passage considers each of the following to be a factor that has
contributed to the prevailing attitude in business schools toward the behavioral sciences
EXCEPT

(A) business students' sensitivity to current business norms and practices


(B) the relative status of HR professionals among business executives
(C) business schools' reliance on legislatures, businesses, and private foundations for funding
(D) businesses' tendency to value mechanistic organizational models over an understanding of
human behavior
(E) theoretical work on the relationship between behavioral factors and a firm's
financial performance

Prestige (n) : uy tín

Prominence (n) : nổi bật

Curricula (n) : chương trình giảng dạy

Theoretical (adj) : lý thuyết

Credibility (n) : uy tín

Peripheral (adj) : ngoại vi

Curriculum (n) : chương trình giáo dục

Norm (n) : định mức, quy tắc


Sep 21 – 1

Premillennialism - a type of eschatology (the study of the last things) based on a literal and rigid
interpretation of the Bible - and the corresponding global events were the major reasons of the rise
of American fundamentalism in the early 1900s. It also played an important part in the fall of Hitler
and in the rise of America as a superpower. Premillennialists, most of them fundamentalists, then
generally believed that the teachings of the Bible, especially when overlaid with some of Jesus’ and
Paul’s New Testament statements, reveal a hidden “plan of the ages” according to which the current
age will climax with the restoration of the land of birth of Jesus, the emergence of powerful empires
in Rome, Russia, and the Far East, and then war. The plan further stated that out of this chaos, a
new leader will appear, promising peace and security, and most political and religious leaders,
unwilling or unable to recognize that this leader is actually the prophesied antichrist, will cede their
sovereignty and independence to him. But just before this tyrant is revealed for the threat that he is,
all true Christians will join the resurrected Jesus, battle the forces of evil at Armageddon, and
establish a millennial kingdom of peace and prosperity on earth.

This belief, malleable enough to fit the many crises and changing geopolitical contexts of the
twentieth century, especially World War II, proved stable enough to provide fundamentalists with a
secure sense of their place in this world. Along with the rebirth of the Roman Empire, the rise of
Hitler and the corresponding growth of anti-Semitism around the globe emerged as signs seized by
fundamentalists as evidence of the looming rupture and the “plan of ages” coming true. After
completing a 1933 tour of Germany, the Boston minister Harold Ockenga, a fundamentalist, called
Hitler, the Nazi leader, “an instrument in the hand of God for the restoring the birth place of Jesus.”
Many fundamentalists and other conservatives, already predisposed to see evidence of impending
doom and extremely motivated to end Hitler’s reign, were convinced by F.D. Roosevelt’s utopian
promises that he might be laying the foundations for a revolution against Hitler. They supported him
even though they had initially greeted Roosevelt’s campaign for president with skepticism. As a
result, upon being elected as president, Roosevelt appointed several fundamentalists on the cabinet.
This reciprocal action led to help spread the fundamentalist movement to new geographies almost
spanning the entire country.

1. In the light of the information provided in the passage, it may be inferred that the
fundamentalists considered themselves to play the role of

A. Forces of the new leader who will appear out of chaos promising peace and prosperity.
B. Citizens of the newly resurrected Roman or Russian empires.
C. Leaders of the world who cede control to the tyrant.
D. True Christians who battle the forces of the evil at Armageddon.
E. People who restore the place of birth of Jesus

2. It may be inferred from the author’s statement in the beginning of paragraph 2 “this
belief, malleable enough to fit the many crises, …” that.

A. The Premillennialism belief was so generic that it might have fit some other
geopolitical crisis had World War II not taken place.
B. The Premillennialism belief fit World War II perfectly, much better than it fit any other crisis.
C. The Premillennialism belief lacked any historical credibility or solid grounds for its existence.
D. Premillennialists had twisted the belief on purpose to the extent that it fit World War II perfectly.
E. The unique formulation of the belief not only allowed it to be specific enough to fit World War II
but also stable enough to last throughout the war.

3. The main purpose of the passage is to

A.Illustrate that Premillennialism and World War 2 were the reasons of Roosevelt’s election as
president.
B. Disprove the belief that fundamentalists had little relationship with world politics.
C. Renew the interest in eschatology by illustrating its contribution to World War 2.
D. Provide a correlation between beliefs of Premillennialists and rise of Hitler.
E. Illustrate how Premillennialism and World War 2 led to the rise of American
Fundamentalism

Pre-millennialism (n) : tiền niên đại

Eschatology (n) : thần học liên quan đến cái chết, sự phán xét, và số mệnh cuối cùng của tâm hồn
và loài người

Literal (adj) : nghĩa đen

Rigid (adj) : cứng rắn

Bible (n) : kinh thánh

Fundamentalism (n) : chủ nghĩa cơ bản

Premillennialist (n) : người tiền sử

Restoration (n) : phục hồi

Emergence (n) : sự xuất hiện

Anti-christ (n) : kẻ phản đối chúa

Prophesy (v) : tiên tri

Cede (v) : bỏ lại, để lại

Tyrant (n) : bạo chúa

Christian (n) : người công giáo

Resurrect (v) : hồi sinh

Millennial (adj) : ngàn năm

Prosperity (n) : sự phồn thịnh

Malleable (adj) : dễ uốn, dễ thay đổi

Anti-Semitism (n) : chủ nghĩa bài do thái

Seize (v) : nắm bắt

Looming (adj) : thấp thoáng, lờ mờ

Rupture (n) : sự bất hòa


Conservatives (n) : người bảo thủ

Predispose (v) : mở đường cho

Impending (adj) : đang đe dọa

Reign (n) : trị vì

Utopian (adj) : không tưởng

Appoint (v) : chỉ định

Cabinet (n) : nội các chính phủ

Reciprocal (adj) : đối ứng

Generic (adj) : chung chung


Sep 21 – 2

Even more than mountainside slides of mud or snow, naturally occurring forest fires promote the
survival of aspen trees. Aspens' need for fire may seem illogical since aspens are particularly
vulnerable to fires; whereas the bark of most trees consists of dead cells, the aspen's bark is a
living, functioning tissue that―along with the rest of the tree―succumbs quickly to fire.

The explanation is that each aspen, while appearing to exist separately as a single tree, is in fact
only the stem or shoot of a far larger organism. A group of thousands of aspens can actually
constitute a single organism, called a clone, that shares an interconnected root system and a unique
set of genes. Thus, when one aspen―a single stem―dies, the entire clone is affected. While alive, a
stem sends hormones into the root system to suppress formation of further stems. But when the
stem dies, its hormone signal also ceases. If a clone loses many stems simultaneously, the resulting
hormonal imbalance triggers a huge increase in new, rapidly growing shoots that can outnumber the
ones destroyed. An aspen grove needs to experience fire or some other disturbanceregularly, or it
will fail to regenerate and spread. Instead, coniferous trees will invade the aspen grove's borders
and increasingly block out sunlight needed by the aspens.

1. The primary purpose of the passage is to explain the

(A) qualities that make a particular organism unique


(B) evolutionary change undergone by a particular organism
(C) reasons that a phenomenon benefits a particular organism
(D) way in which two particular organisms compete for a resource
(E) means by which a particular organism has been able to survive in a barren region

2. It can be inferred from the passage that when aspen groves experience a
"disturbance", such a disturbance

(A) leads to a hormonal imbalance within an aspen clone


(B) provides soil conditions that are favorable for new shoots
(C) thins out aspen groves that have become overly dense
(D) suppresses the formation of too many new aspen stems
(E) protects aspen groves by primarily destroying coniferous trees rather than aspens

3. The author of the passage refers to "the bark of most trees" most likely in order to
emphasize the

(A) vulnerability of aspens to damage from fire when compared to other trees
(B) rapidity with which trees other than aspens succumb to destruction by fire
(C) relatively great degree of difficulty with which aspens catch on fire when compared to other
trees
(D) difference in appearance between the bark of aspens and that of other trees
(E) benefits of fire to the survival of various types of trees
4. According to the passage, which of the following would occur if an aspen failed to
regenerate periodically?

(A) Individual aspens would cease to produce hormones.


(B) Individual aspens would grow outward instead of upward.
(C) The root system of the grove's clone would die.
(D) The grove would lose its access to sunlight.
(E) Soil conditions in the grove would become unfavorable for the growth of aspens.

Mountainside (n) : sườn núi

Bark (n) : vỏ cây

Succumb to (v) : chịu thua

Clone (n) : nhân bản

Simultaneously (adv) : đồng thời


Sep 22 – 1

Numerous investigations have been made to test the validity of the theory that facial behaviors
associated with emotions are universal. Proponents exist for both sides, with some postulating
universals in emotional facial behavior while others arguing against such a possibility.

A team of sociologists studied these contradictory views and hypothesized that the universals are to
be discovered in the relationship between different patterns of facial muscles and specific emotions
such as happiness, sadness, anger, and fear. These sociologists tried to find evidence for their
hypothesis by conducting experiments in which they showed photographs of Western faces to
subjects from various cultures in order to determine whether the same facial behavior would be
judged as the same emotion, irrespective of the observer’s culture. College educated subjects from
countries such as Argentina, Chile, the U.S., and Japan were chosen to identify the same faces with
the same emotion words as were the members of two pre-literature cultures. They interpreted the
results of the experiments as evidence of universals in facial behavior. However, it is arguable as to
how uncontaminated these experiments were. The problem was that all the subjects, even those
from the pre-literature societies, came from cultures that had had exposure to some of the same
mass media portrayals of facial behavior in Western cultures. This handicap was successfully
addressed in a subsequent study by the team in which only subjects who met the criteria
established to screen out all but those who had minimal opportunity to learn or imitate or even
recognize uniquely Western facial behaviors were chosen to participate in the experiment. The
results conclusively supported the hypothesis that specific facial behaviors are associated with
specific emotions across various cultures.

However, the increasing evidence of a pan-cultural element in emotional facial behavior does not
suggest a lack of cultural differences in emotional expression. These differences manifest
themselves in the circumstances that elicit an emotion, with the same emotion being preceded by
different stimuli across cultures.

1. Which of the following statements would the author most likely agree with?

A. One of the two studies undertaken by the team of sociologists was more objective in its approach
since it unknowingly chose specific kinds of subjects.
B. In one culture, happiness could be a follow-up emotion of a situation that is dissimilar
to the circumstances that lead up to it in another culture.
C. Pre-literature societies are normally against contact with any kind of Western exposure.
D. College graduates from Chile and Argentina were more likely to have experienced the same kind
of mass media exposure to portrayals of facial behavior in Western cultures than those from Japan.
E. Sociologists who do not agree that universals exists in facial behavior associated with emotion
across cultures do not have an in depth knowledge of the topic.

This piece of information is supported by the consideration raised by the author in the final
paragraph. Please refer to the extract:
These differences manifest themselves in the circumstances that elicit an emotion, with the same
emotion being preceded by different stimuli across cultures.

2. The author is primarily concerned with


A. To contrast the basis for two contradictory views and side with one
B. To highlight commonalities between two studies while showing how one trumps the other
C. To analyze a hypothesis through a discussion on two studies and to raise a
consideration
D. To introduce a theory and evaluate various studies arguing for and against its validity
E. To criticize an experiment on the basis that it may have been contaminated

Partial Scope. The author does introduce a theory but never discusses studies against it.

3. Which of the following most aptly describes the function of the final paragraph?

A. To warn against a myth


B. To criticize a particular implication of the studies
C. To caution against an overly optimistic conclusion
D. To raise a consideration
E. To initiate a new unrelated discussion

4. Which of the following is mentioned in the passage?

A. The results of the experiments of the first study were tampered with.
B. There are more people who believe in the universals in emotional facial behavior than the other
way around.
C. The subjects of the experiment were not aware of their knowledge of Western culture.
D. Portrayals of facial behavior in mass media are the same in different cultures.
E. The subject of universals in facial behaviors associated with emotions is well
researched.

The first sentence is "Numerous investigations have been made to test the validity of the theory that
facial behaviors associated with emotions are universal.". By saying that numerous investigations
have been made, you can understand that the topic is well researched..

Validity (n) : sự có giá trị, hiệu lực

Postulate (v) : định đề, bổ nhiểm

Sociologist (n) : nhà xã hội học

Contradictory (adj) : mâu thuẫn

Uncontaminate (adj) : không bị nhiễm bẩn

Portrayal (n) : chân dung

Handicap (n) : điều bất lợi

Subsequent (adj) : tiếp theo

Imitate (v) : bắt chước

Manifest (v) : biểu lổ

Elicit (v) : gợi ra

Precede (v) : đứng trước


Sep 22 – 2

The professionalization of the study of history in the second half of the nineteenth century, including
history's transformation from a literary genre to a scientific discipline, had important consequences
not only for historians' perceptions of women but also for women as historians. The disappearance
of women as objects of historical studies during this period has elements of irony to it. On the one
hand, in writing about women, earlier historians had relied not on firsthand sources but rather on
secondary sources; the shift to more rigorous research methods required that secondary sources be
disregarded. On the other hand, the development of archival research and the critical editing of
collections of documents began to reveal significant new historical evidence concerning women, yet
this evidence was perceived as substantially irrelevant: historians saw political history as the general
framework for historical writing. Because women were seen as belonging to the private rather than
to the public sphere, the discovery of documents about them, or by them, did not, by itself, produce
history acknowledging the contributions of women. In addition, genres such as biography and
memoir, those forms of "particular history" that women had traditionally authored, fell into
disrepute. The dividing line between "particular history" and general history was redefined in
stronger terms, widening the gulf between amateur and professional practices of historical research.

1. The passage is primarily concerned with

(A) describing some effects of the professionalization of the study of history on the
writing of women's history
(B) explaining some reasons for the professionalization of the writing of history
(C) discussing the kinds of historical writing traditionally practiced by women
(D) contrasting the approach to the writing of history taken by women with the approach taken by
men
(E) criticizing certain changes that occurred in the writing of history during the second half of the
nineteenth century

2. Which of the following best describes one of the "elements of irony" referred to in the
highlighted text?

(A) Although the more scientific-minded historians of the second half of the nineteenth century
considered women appropriate subjects for historical writing, earlier historians did not.
(B) Although archival research uncovered documentary evidence of women's role in history,
historians continued to rely on secondary sources for information about women.
(C) Although historians were primarily concerned with writing about the public sphere, they
generally relegated women to the private sphere.
(D) The scientific approach to history revealed more information about women, but that
information was ignored.
(E) The professionalization of history, while marginalizing much of women's writing about history,
enhanced the importance of women as historical subjects.

“On the other hand, the development of archival research and the critical editing of collections of
documents began to reveal significant new historical evidence concerning women, yet this evidence
was perceived as substantially irrelevant"
3. According to the passage, the development of archival research and the critical
editing of collections of documents had which of the following effects?

(A) Historians increasingly acknowledged women's contributions to history.


(B) Historians began to debate whether secondary sources could provide reliable information.
(C) Historians began to apply less rigorous scientific research criteria to the study of women's
history.
(D) More evidence concerning women became available to historical researchers.
(E) Women began to study history as professional historians.

Professionalization (n) : chuyên nghiệp hóa

Scientific discipline (n) : kỷ luật khoa học

Irony (n) : trớ trêu

Rigorous (adj) : nghiêm ngặt, chính xác

Disregard (v) : xem thường, không quan tâm

Archival (adj) : lưu trữ

Substantially (adv) : đáng kể, chắc chắn

Irrelevant (adj) : không liên quan

Sphere (n) : khu vực

Biography (n) : tiểu sử

Memoir (n) : hồi ký

Author (v) : tác giả

Dispute (n) : thất vọng, tiếng xấu, quên lãng

Widen (v) : mở rộng

Gulf (n) : hố sâu, vực thẳm

Relegate (v) : xuống hạng


Sep 23 – 1

Researchers bet their bottom dollar on a combination of polar ice cores, tree-rings, geochemistry,
and a medieval chronicle little-known in the West to solve one of vulcanology’s most enduring
mysteries: which peak blew its top in the mid-13th century, causing a catastrophic eruption that
ranks as one of the biggest in the recorded history? As with any investigation, the team had to rule
out other suspects as it followed a trail of clues - and even read palms, or at least palm leaves,
ultimately finding the culprit of the massive 1257 AD eruption, which the researchers say is Samalas
volcano on Lombok Island in Indonesia.

For decades, scientists have been searching for the volcano responsible for the largest spike in
sulfate deposits in the last 7,000 years, which were revealed in the ice cores from Greenland and
Antarctica. The spike indicated a massive eruption around 1257 that may have sent up to eight
times more sulfate into the stratosphere than the 1883 eruption of Karaktau, often held up as an
archetype of volcanoes behaving badly. Researchers say the 1257 mystery spew is comparable in
scope to a second-century AD eruption in the Taupo Volcanic Zone of New Zealand, known as the
most intense historic volcanic event. Multitude of futile attempts for a few decades compelled the
researchers to write the project off as “unsolved”. Some thirty years later, one of the researchers’
tips came from Babad Lombok, a 13th century historical record in Old Javanese, written on palm
leaves, the chronicle referencing a massive eruption of Samalas that created an enormous
caldera.The current research zeroed in on Samalas, part of the Mount Rinjani volcanic complex.

The team was able to accumulate a sizable amount of incriminating evidence, including pyroclastic
deposits from the eruption more than 100 feet thick found more than 15 miles from the ruins of the
volcano. The range of deposits and the volume suggest that the Samalas eruption exceeded that of
the Tambora event in 1815. The team sampled carbonized tree trunks and branches in the Samalas
deposit zone and used radiocarbon dating to confirm a mid 13th-century eruption. Reviewing wind
patterns, researchers were even able to narrow the timeframe for the eruption. The distribution, to
the west, of volcanic ash and other ejecta from Samalas suggest that the dry season’s easterly trade
winds were prevalent, putting the eruption window between May and October of 1257.

1. The author is primarily concerned with:

A. enumerating how an unconventional means solved one of the volcanology’s most enduring
mysteries.
B. explaining how researchers could finally trace the Samalas event.
C. describing one of the deadliest volcanic eruptions that took place in mid-13th century.
D. presenting the nature of volcanic spew of the 1257 volcano eruption in comparison with that of
the other major volcanic event.
E. showing through major volcanic events how deadly the eruption of the 1257 AD was.
2. The author of the passage alludes to the discovery made in Greenland and Antarctica
in order to

A. suggest that the researchers explored these sites to accumulate traces and study effects of the
massive 1257 AD eruption.
B. indicate that the sharp increase in the levels of sulfate in stratosphere must have
been due to massive volcanic eruption in 1257 AD.
C. point massive eruptions can affect vast areas of land.
D. compare the effects of 1257 AD eruption with that of eruption in these two locations.
E. bring in the importance of these places in the resolution of the mystery of 1257 AD eruption.

3. Which of the following statements about “the most intense historic volcanic event” is
supported by information in the passage?

A. The ejecta from this volcanic eruption led to pyroclastic deposits more than 100 feet thick that
were found more than 15 miles away from the site of the volcano.
B. This event took place in 1883 and led to significant increase in sulfate in stratosphere.
C. The event is chronicled in Babad Lombak, a 13th century historical record in Old Javanese.
D. The event refers to Tambora event that took place in 1815 and is included in the history of
massive volcanic eruptions.
E. This event took place in the Taupo Volcanic Zone of New Zealand in 2nd century AD.

Polar ice core (n) : lõi băng cực

Tree-ring (n) : vân gỗ

Geochemistry (n) : địa hóa

Medieval (adj) : trung cổ

Chronicle (n) : biên niên

Vulcanology (n) : lưu hóa

Catastrophic (adj) : thảm khốc

Eruption (n) : phun trào

Palm leaves (n) : lá cọ

Culprit (n) : thủ phạm

Spike (n) : mũi nhọn

Sulfate (n) : sunfat

Deposit (n) : lắng cặn

Stratosphere (n) : tầng bình lưu

Archetype (n) : khuôn mẫu

Comparable (adj) : có thể so sánh

Multitude (n) : vô số
Futile (adj) : vô ích

Compel (v) : buộc

Zeroed out (v) : bỏ không

Incriminate (v) : bắt buộc

Tree trunk (n) : thân cây

Carbonized (v) : đốt thành than

Easterly wind (n) : gió đông


Sep 23 – 2

Many United States companies believe that the rising cost of employees' health care benefits has
hurt the country's competitive position in the global market by raising production costs and thus
increasing the prices of exported and domestically sold goods. As a result, these companies have
shifted health care costs to employees in the form of wage deductions or high deductibles. This
strategy, however, has actually hindered companies' competitiveness. For example, cost shifting
threatens employees' health because many do not seek preventive screening. Also, labor relations
have been damaged: the percentage of strikes in which health benefits were a major issue rose
from 18 percent in 1986 to 78 percent in 1989.

Health care costs can be managed more effectively if companies intervene in the supply side of
health care delivery just as they do with other key suppliers: strategies used to procure components
necessary for production would work in procuring health care. For example, the make/buy
decision—the decision whether to produce or purchase parts used in making a product—can be
applied to health care. At one company, for example, employees receive health care at an on-site
clinic maintained by the company. The clinic fosters morale, resulting in a low rate of employees
leaving the company. Additionally, the company has constrained the growth of health care costs
while expanding medical services.

1. The passage is primarily concerned with

(A) providing support for a traditional theory


(B) comparing several explanations for a problem
(C) summarizing a well-known research study
(D) recommending an alternative approach
(E) criticizing the work of a researcher

2. The author of the passage asserts which of the following about managing health care
costs in an effective manner?

(A) Educating employees to use health care wisely is the best way to reduce health care costs.
(B) Allowing employees to select health care programs is the most effective means of controlling
health care costs.
(C) Companies should pass rising health care costs on to employees rather than to consumers of the
companies' products.
(D) Companies should use strategies in procuring health care similar to those used in
procuring components necessary for production.
(E) Companies should control health care costs by reducing the extent of medical coverage rather
than by shifting costs to employees.

3. Which of the following, if true, would provide the most support for the author's view
about intervening on the supply side of health care?

(A) Most companies do not have enough employees to make on-site clinics cost-effective.
(B) Many companies with on-site clinics offer their employees the option of going outside the
company's system to obtain health care.
(C) The costs of establishing and running an on-site clinic are demonstrably higher than the costs of
paying for health care from an outside provider.
(D) Companies with health care clinics find that employees are unwilling to assist in controlling the
costs of health care.
(E) Employees at companies with on-site clinics seek preventive screening and are thus
less likely to delay medical treatment.

Deduction (n) : giảm bớt

Deductible (n) : khấu trừ

Hinder (v) : cản trở, ngăn cản

Preventitive (adj) : dự phòng, phòng ngừa

Intervene (v) : can thiệp

Procure (v) : mua sắm

Foster (v) : khích lệ, thúc đẩy, bồi dưỡng

Morale (n) : tinh thần

Constraint (v) : hạn chế


Sep 24 – 1

One of the first indications of a distinctive American character came in 1782 with the publication of
Letters From An American Farmer. In it, a French settler observed that Americans tended to
demonstrate a higher level of personal initiative and self-reliance than Europeans and generally had
little regard for social rank.

This viewpoint of the American character is steeped in the philosophical tradition of the
Enlightenment, and the typical American quickly became typecast throughout Europe as an
emancipated, enlightened individual, taking from his surroundings the necessary social and material
resources to ensure his happiness. This idealized portrait of the American citizen corresponded
nicely with the concept of a rational, self-interested individual put forth by Adam Smith and other
political economists. The American personality was the Economic personality, most comfortable in
the world of free markets, of trade and hard work.

To the Europeans, Benjamin Franklin—or at least their mythologized conception of Franklin—best


represented the American character. Here, according to the Europeans, was both the embodiment
of Enlightenment principles, scientist, inventor, statesman, philosopher, and economist, and the
symbol of the American spirit, rational, enterprising, and pragmatic. Franklin was viewed with such
admiration by the Europeans that a virtual cult of personality arose around him.

1. The primary purpose of the passage is to

A. comment on the origin and nature of a national personality type


B. demonstrate why Benjamin Franklin was so admired by the French
C. argue that the idea of an American character is a myth
D. recount the historical and philosophical contributions of Americans in Europe
E. provide a psychological profile of a specific individual

2. The passage suggests which of the following about the Europeans?

A. Europeans lacked personal initiative and self-reliance.


B. Europeans admired Americans because of the American’s rational, self-interested nature.
C. The European conception of the American character might not entirely conform to
reality.
D. Europeans placed a high value on the works of Adam Smith and other political economists.
E. To the Europeans, no individual better embodied the ideas of the Enlightenment than Benjamin
Franklin did. (Here, according to the Europeans, was both the embodiment of Enlightenment
principles, scientist, inventor, statesman, philosopher, and economist)
To the Europeans, Benjamin Franklin—or at least their mythologized conception of Franklin—best
represented the American character.

3. As it is presented in the passage, which of the following best exemplifies the


American personality?

A. An aristocrat who diligently keeps abreast of all the fashionable new trends in society.
B. A pious, hardworking farmer who takes care of his family and remains humble.
C. A cunning statesman who uses his knowledge of human nature to manipulate friends and foes.
D. An absent minded inventor whose brilliant discoveries are counter balanced by his lack of
common sense.
E. A self-made man whose efforts help him create a successful shipping company.

Distinctive (adj) : đặc biệt

Settler (n) : người định cư

Personal initiative (n) : sáng kiến, chủ động cá nhân

Self-reliance (n) : tự lực

Enlightment (n) : giác ngộ

Emancipate (v) : giải phóng

Mythologized (v) : thần thoại hóa

Embodiment (n) : hiện thân

Enterprising (adj) : dám nghĩ dám làm, gan dạ

pragmatic (adj) : thực dụng

cult (n) : sùng bái

virtual (adj) : thật sự


Apr 24 – 2

Dendrochronology, the study of tree-ring records to glean information about the past, is possible
because each year a tree adds a new layer of wood between the existing wood and the bark. In
temperate and subpolar climates, cells added at the growing season's start are large and thin-
walled, but later the new cells that develop are smaller and thick-walled; the growing season is
followed by a period of dormancy. When a tree trunk is viewed in cross section, a boundary line is
normally visible between the small-celled wood added at the end of the growing season in the
previous year and the large-celled spring wood of the following year's growing season. The annual
growth pattern appears as a series of larger and larger rings. In wet years rings are broad; during
drought years they are narrow, since the trees grow less. Often, ring patterns of dead trees of
different, but overlapping, ages can be correlated to provide an extended index of past climate
conditions.

However, trees that grew in areas with a steady supply of groundwater show little variation in ring
width from year to year; these "complacent" rings tell nothing about changes in climate. And trees
in extremely dry regions may go a year or two without adding any rings, thereby
introducing uncertainties into the count. Certain species sometimes add more than one ring in a
single year, when growth halts temporarily and then starts again.

1. In the highlighted text, "uncertainties" refers to

(A) dendrochronologists' failure to consider the prevalence of erratic weather patterns


(B) inconsistencies introduced because of changes in methodology
(C) some tree species' tendency to deviate from the norm
(D) the lack of detectable variation in trees with complacent rings
(E) the lack of perfect correlation between the number of a tree's rings and its age

2. The passage suggests which of the following about the ring patterns of two trees that
grew in the same area and that were of different, but overlapping, ages?

(A) The rings corresponding to the overlapping years would often exhibit similar
patterns.
(B) The rings corresponding to the years in which only one of the trees was alive would not reliably
indicate the climate conditions of those years.
(C) The rings corresponding to the overlapping years would exhibit similar patterns only if the trees
were of the same species.
(D) The rings corresponding to the overlapping years could not be complacent rings.
(E) The rings corresponding to the overlapping years would provide a more reliable index of dry
climate conditions than of wet conditions.

Often, ring patterns of dead trees of different, but overlapping, ages can be correlated to provide an
extended index of past climate conditions

3. The passage is primarily concerned with

(A) evaluating the effect of climate on the growth of trees of different species
(B) questioning the validity of a method used to study tree-ring records
(C) explaining how climatic conditions can be deduced from tree-ring patterns
(D) outlining the relation between tree size and cell structure within the tree
(E) tracing the development of a scientific method of analyzing tree-ring patterns

Erratic (adj) : thất thường

Prevalence (n) : sự đang lưu hành

Glean (v) : lượm lặt

Subpolar (adj) : phân cực

Thin-walled (adj) : vách mỏng

Dormancy (n) : ngủ đông, ngủ mê

Correlate (v) : liên quan, tương quan


Apr 25 - 1

Basic sleep theory holds that while humans are awake, memories are encoded into our active
memory, a process that can lead to temporary synaptic overuse and which eventually leads to
tiredness, and that while they are asleep, those memories are consolidated, a process that includes
returning synapses to their original state. The molecular cause of tiredness should, therefore,
accumulate throughout the day and slowly disappear during sleep. Protein phosphorylation, the
process by which proteins are “activated” for use within the cell by the addition of a phosphate
group transferred from a molecule called adenosine triphosphate (ATP), maybe this molecular
cause. In a recent study, researchers noticed that levels of protein phosphorylation in and around
synapses increase during the day while mice are awake and decrease while mice are asleep,
suggesting that the hyper-phosphorylation of certain proteins may cause tiredness.

Wang et al. further investigated this hypothesis by comparing the phosphorylation tendencies of
regular sleep-deprived mice and of sleep-deprived mice with a single point-mutation in a gene
labeled “Sleepy” in order to differentiate between the effects of stress induced by depriving mice of
sleep and the effects of natural levels of sleepiness. The Sleepy gene causes sleep frequency and
duration to increase, so mice get tired more quickly. Since phosphorylation patterns were consistent
between mice with and without the genetic alteration, it is likely that these phosphorylation patterns
are indeed at least part of the molecular basis of tiredness.

1. It can be inferred from the passage that the "Sleepy" gene


A. does not occur naturally in mice and is only present when introduced by a scientist.
B. has at least some influence on the extent to which a mouse experiences fatigue.
C. is unrelated to the phosphorylation of protein in mice and other rodents.
D. is more susceptible to mutation than are other genes.
E. decreases the amount of sleep that a mouse requires.

2. It can be inferred from the passage that protein phosphorylation

A. is the primary cause of fatigue in mammals.


B. only occurs when an animal is awake.
C. occurs most frequently when an animal is experiencing stress.
D. occurs less frequently in animals that are nocturnal.
E. takes place in the synapses of the brain.

Synapse (n) : khớp thần kinh

Molecular (adj) : thuộc về phân tử

Molecule (n) : phân tử

Mutation (n) : đột biến


Apr 25 – 2

A regimen of intrauterine AZT (zidovudine) as a means of reducing the chances of HIV transmission
from mother to child was first described in a study known as Protocol 076, the results of which were
published in the New England Journal of Medicine in November 1994. The trial found that
administration of AZT to HIV-positive women during pregnancy and delivery, and to their babies
after birth, reduced the transmission of HIV to the infants by two thirds, compared with a placebo.
The study was acclaimed as one of the first successful instances of a prophylaxis preventing HIV
transmission—particularly “vertical” transmission, or transmission from mothers to infants. The
study, prematurely ended so that all the subjects on placebos could be switched to AZT, led to the
recommendation that all pregnant women with HIV take AZT.

The validity of the study’s results, however, is debatable. Since Protocol 076 examined only women
who had fairly high numbers of T-cells—the white blood cells that coordinate immune response—
and who had previously taken AZT for less than six months, the same regimen might not succeed
for other pregnant women with HIV. Moreover, the study administered large doses of AZT without
examining whether lower doses, more economically feasible for uninsured or under insured women,
might be effective. Further, the trial design did not account for important variables: in particular,
whether the subjects’ viral load—the amount of HIV in their bodies—might have contributed to the
difference in transmission rates. Since the exact mechanism and timing of vertical transmission of
the HIV virus are unknown, it cannot be ruled out that viral load influences rates of vertical
transmission.

Also, the study did not consider the long term impact of AZT, which is toxic, on the babies
themselves. There have been few studies of AZT’s effect on HIV-positive infants and none on its
effects on uninfected babies. Since the majority of the babies born to HIV-positive women are HIV-
negative, and since HIV may mutate into drug-resistant strains following a break in medication, the
value of a short term “zap” with AZT becomes suspect.

1. The author questions the results of Protocol 076 for which of the following reasons?

A. The number of T-cells was not considered in choosing subjects for the study.
B. The trial was discontinued, and the control group of women on placebo was shifted to AZT.
C. Seventy-five percent of babies born to women in the placebo group were free of infection with
HIV.
D. Different transmission rates could have been influenced by different amounts of HIV
in the women’s bodies.
E. The exact mechanism and timing of HIV transmission are unknown.

2. The author of this passage would be LEAST likely to challenge the benefits of
intrauterine AZT for which of the following HIV-positive women and/or their future
children?

A. A woman who has used the regimen successfully in a previous pregnancy


B. A woman whose T-cell count is dangerously low and who plans to continue taking AZT after her
baby is born
C. A woman who is matched with the study’s subjects in terms of ethnicity and socioeconomic status
D. A woman who has lost a previous infant to HIV and who wants to be sure that her next child is
healthy
E. A woman whose T-cell count is not low and who has not taken AZT before

‘Since Protocol 076 examined only women who had fairly high numbers of T-cells—the
white blood cells that coordinate immune response—and who had previously taken AZT
for less than six months’
the author mentions in start of second para : high T cells and less than 6 months duration of
AZT (Zero is also less than six )

3. The passage implies that the “viral loads” of the subjects in the study are significant
because

A. all the subjects had relatively high viral loads, so the study’s results may not apply to other
groups of women. (T-cells)
B. the viral loads of babies whose mothers had received the placebo were just as high as those of
babies whose mothers had received AZT.
C. viral load must be measured if researchers are to understand the exact mechanism and timing of
transmission of HIV.
D. the viral loads of women in the placebo group may have been different from those of
women in the AZT group.
E. viral load may be a critical factor in determining transmission, but the researchers wrongly
assumed that it was the only factor.

4. The primary purpose of the passage is to

A. document recent problems with HIV and AIDS treatment.


B. expose medical errors caused by the careless methodology in an important clinical trial.
C. raise doubts about a course of treatment based on a groundbreaking study.
D. describe the process of vertical transmission of HIV and suggest preventive therapy.
E. evaluate treatment possibilities for pregnant women with HIV.

5. The author of the passage would be most likely to agree with which of the following
statements?

A. Medications harmful to one group of patients are probably not harmful to another.
B. Recommendations based on a conditional experiment must be qualified.
C. The study should not have been prematurely ended.
D. Clinical trials that may harm patients are unethical.
E. No HIV-positive woman should take AZT during pregnancy.

6. The primary function of the second paragraph is to

A. refute issues.
B. evaluate solutions.
C. describe results.
D. support action.
E. identify problems.

Regimen (n) : chế độ, quá trình điều trị y tế


Intrauterine (n) : tử cung

Placebo (n) : giả dược

Mutate (v) : đột biến

Strain (n) : chủng, gen, khuynh hướng

Drug-resistant (adj) : khánh thuốc


Apr 26 – 1

Shortly after September 11, 2001, the United States began requesting additional financial
information about persons of interest by subpoenaing records located at the SWIFT banking
consortium. SWIFT, which routes trillions of dollars a day, faced an ethical dilemma: fight the
subpoenas in order to protect member privacy and the group's reputation for the highest level of
confidentiality, or, comply and provide information about thousands of financial communications in
the hope that lives will be saved. SWIFT decided to comply in secret, but in late June 2006, four
major U.S. newspapers disclosed SWIFT's compliance. This sparked a heated public debate over the
ethics of SWIFT's decision to reveal ostensibly confidential financial communications.

Analyzing the situation in hindsight, three ethical justifications existed for not complying with the
Treasury Department's requests. First, SWIFT needed to uphold its long-standing values of
confidentiality, non-disclosure, and institutional trust. The second ethical reason against SWIFT's
involvement came with inadequate government oversight as the Treasury Department failed to
construct necessary safeguards to ensure the privacy of the data. Third, international law must be
upheld and one could argue quite strongly that the government's use of data breached some parts
of international law.

Although SWIFT executives undoubtedly considered the aforementioned reasons for rejecting the
government's subpoena, three ethical justifications for complying existed. First, it could be argued
that the program was legal because the United States government possesses the authority to
subpoena records stored within its territory and SWIFT maintained many of its records in Virginia.
Second, it is entirely possible that complying with the government's subpoena thwarted another
catastrophic terrorist attack that would have cost lives and dollars. Third, cooperating with the
government did not explicitly violate any SWIFT policies due to the presence of a valid subpoena.
However, the extent of cooperation certainly surprised many financial institutions and sparked some
outrage and debate within the financial community.

While SWIFT had compelling arguments both for agreeing and refusing to cooperate with the U.S.
government program, even in hindsight, it is impossible to judge with certitude the wisdom and
ethics of SWIFT's decision to cooperate as we still lack answers to important questions such as:
what information did the government want? What promises did the government make about data
confidentially? What, if any, potentially impending threats did the government present to justify its
need for data?

1) Which of the following can be inferred from the passage?

A) No clear cut answer as to the legality of SWIFT's cooperation existed


B) SWIFT failed to adequately consult its legal staff before deciding to cooperate
C) The volume of money routed through SWIFT declined after its cooperation became public
D) U.S. authorities threatened criminal charges if SWIFT refused their subpoenas
E) Treasury Department officials objected to the publication of information about its classified
program
2) Inferring from the passage, which of the following constituted an ethical justification
for SWIFT complying with the government?

A) The U.S. government can subpoena information that pertains to its citizens
B) SWIFT executives believed another catastrophic attack was imminent
C) Providing data to the government based upon a valid subpoena did not explicitly
violate SWIFT policy
D) Despite ostensibly poor oversight, senior Treasury Department officials assured SWIFT that data
would be kept confidential
E) U.S. officials told SWIFT officials exactly why Treasury needed the information

3) The author suggests which of the following is the most appropriate conclusion of an
analysis of the ethics of SWIFT's decision?

A) SWIFT acted inappropriately as it compromised its long-standing values of integrity, privacy, and
confidentiality
B) SWIFT's actions cannot be judged with perspicuity as answers to important questions
are still unknown
C) SWIFT acted properly as it complied with the requests of a sovereign government in an attempt
to save lives
D) SWIFT's actions endangered the flow of commerce by sparking public outrage at an important
institution
E) SWIFT's actions were appropriate initially, yet should have been discontinued prior to June 2006

4) According to the passage, each of the following describes SWIFT EXCEPT:

A) Had data stored in Virginia


B) Valued confidentiality and non-disclosure
C) Routes trillions of dollars a day
D) Composed of a consortium of banks
E) After over a month of deliberation, complied with the government's subpoena

5) The author implies that which of the following most likely occurred as a result of the
news stories that ran in June 2006:

A) U.S. government officials decried the leaking of classified information


B) SWIFT executives conducted a thorough internal review to assess the legality of SWIFT's actions
C) Some foreign members of the SWIFT consortium demanded answers from SWIFT's executives
D) Many members of the public and financial community debated SWIFT's decision
E) Financial data and transactions slowed as a result of the publication of SWIFT's cooperation

Subpoena (n) : trát hầu tòa

Consortium (n) : tập đoàn, công ty

Route (v) : gửi, chuyển

Ethical dilemma (n) : tiến thoái lưỡng nan về đạo đức

Confidentiality (n) : bảo mật


Comply (v) : tuân theo

Disclose (v) ; tiết lộ

Compliance (n) : tuân thủ

Spark (v) : trâm ngòi

Ostensibly (adv) : bề ngoài, mạo nhận, lấy cớ

Hindsight (n) : nhận thức muộn màng

Oversight (n) : điều vô ý, giám sát

Breach (v) : vi phạm

Aforementioned (adj) : đã nói ở trên

Catastrophic (adj) : thảm khốc

Certitude (n) : sự tin chắc, chứng chỉ, tính xác thực

Impend (v) : đe dọa


Apr 26 – 2

The idea that all mental functions are derived from the brain originated with Hippocrates, but it was
largely neglected until the late 18th century, when Franz Gall attempted to link psychology and brain
science. Gall took advantage of what was already known about the cerebral cortex. He was aware
that it was bilaterally symmetrical and subdivided into four lobes. However, he found that these four
lobes were, by themselves, inadequate to account for the forty-odd distinct psychological functions
that psychologists had characterized by 1790. As a result he began to analyze the heads of
hundreds of musicians, actors, etc., relating certain bony elevations or depressions under the scalp
to the predominant talent or defects of their owners. Based on his skull palpation, Gall subdivided
the cortex into roughly forty regions, each of which served as an organ for a specific mental
function.

While Gall’s theory that all mental processes derive from the brain proved to be correct, his methods
for localizing specific functions were deeply flawed because they were not based on what we would
now consider valid evidence. Gall did not test his ideas empirically by performing autopsies on the
brains of patients and correlating damage to specific regions with defects in mental attributes; he
distrusted the diseased brain and did not think it could reveal anything about normal behavior.
Instead, he developed the notion that as each mental function is used, the particular area of the
brain responsible for that function becomes enlarged. Eventually, a given area may become so bulky
that it pushes out against the skull and produces a bump on the head.

1/ Which of the following is NOT an assumption that Gall makes regarding the relation
between a person’s aptitude and personality traits, and that person’s brain?

A Each bump in the skull corresponds to a distinct part of the brain, which is responsible for a
personality trait.
B The more a person develops a certain ability the more an area of that person’s skull will protrude.
C There are certain personality traits that do not manifest themselves in the shape of a
person’s skull.
D One can judge a person’s character simply by looking at the shape of the person’s skull.
E The diseased brain lends no useful insight into a person’s typical behavior.

2/ The author would agree with all of the following regarding Gall’s work EXCEPT?

A It did not confirm certain assumptions in an empirical manner.


B It relied on surface features of the skull to derive personality traits.
C It deemed the diseased brain as an invalid means of learning about an individual’s personality.
D It postulated that personality traits were not linked to the brain but to depressions
and elevations in the scalp.
E It correctly identified the brain as the source of human behavior.

3/ According to the passage, Gall believes that bumps on the surface of the skull result
from

A defects in a person’s thinking


B a psychological function becoming more pronounced
C a natural physiological progression
D underuse of a specific skill or ability
E processes of change within only one of the four cranial lobes

Underuse (v) : sử dụng kém hiệu quả

Mental function (n) : chức năng tâm thần

Neglect (v) : bỏ bê

Cerebral (adj) : não

Symmetrical (adj) : đối xứng

Bilaterally (adv) : song phương

Subdivide (v) : chia nhỏ

Lobe (n) : thùy

Bony (adj) : thuộc về xương

Elevation (n) : độ cao

Scalp (n) : da đầu

Predominant (adj) : chiếm ưu thế

Defect (v) : phần thiếu, khuyết điểm, tật xấu

Palpation (n) : sờ nắn, xem mạch

Empirically (adv) : theo kinh nghiệm

Autospy (n) : khám nghiệm tử thi


Apr 27 – 1

The Dutch cartographer, Abraham Ortelius, first suggested in 1596 that the Americas were "torn
away from Europe and Africa"; but there was little evidence to support his hypothesis. In England in
1620, Francis Bacon also noted that the similarity of many of the edges of various continents
suggested that they once might have fit together like puzzle pieces. Evidence mounted gradually
over the course of the next few centuries that continents were once joined: fossils of similar plant
and animal species found on widely separated continents, long and linear zones of deformed rocks
occurring at the edges of continents, and certain geologic and glacial features shared across
different continents.

German meteorologist Alfred Wegener proposed in 1912 that the continents were all joined in a
common landmass he named ―Pangaea‖, which began breaking up approximately 200 million years
ago. In fact, precursors of this theory existed in maps depicting the joined continents, which had, it
may be noted, been drawn almost a century earlier, but it was Wegener who was the first to
combine the accumulating evidence for continental drift into a common framework—to weave
seemingly dissimilar, unrelated facts into a theory. His proposal was not well received, however; it
remained unclear how the continents actually moved, and science had not developed accurate
radiometry to date the fossils or the linear belts of rock at the edges of continents. Geologist Arthur
Holmes proposed in 1929 that the hot and melted rocks that made up the mantle of the Earth, the
layer just beneath the Earth‘s thin crust, flowed upward, downward, and laterally, pushing apart
regions of ocean floor or allowing nearby regions to collide and overrun each other; but again little
evidence existed to support the idea. In the following decades, magnetic studies of the ocean floor,
showing that the orientation of rocks had changed over the course of recent geologic time, helped
confirm Holmes‘ ideas that ocean plates were the cause of the rifts and valleys on the ocean floor,
as well as of the larger movement of landmasses.

By the early-1960s, a wealth of new evidence (much of it from studies of the ocean floor) formed a
picture of what caused continents to drift. The sedimentary rocks of an oceanic origin were different
from predial samples previously found, and geologists reasoned from this that continents were not
simply upwellings of ocean floor. Continents are built of blocks of crust varying in age, size, rock
composition, structure, and fossil assemblage (fauna and flora), with relatively stable, older interiors
(the oldest rocks of which are more than 3 billion years old); the sea floors are significantly younger.
The theory of mantle convection currents and sea-floor spreading became the prevailing explanation
of how large plates of the Earth‘s crust continually move upward, downward, and to the side,
allowing the separation of and collision of landmasses well above the moving ocean plates. In 1994,
however, Seiya Uyeda concluded that subduction (the gravity-controlled sinking of a cold, denser
oceanic slab into the subduction zone) ―plays a more fundamental role than seafloor spreading in
shaping the earth's surface features" and "running the plate tectonic machinery." Current analysis of
seismic waves and other geophysical studies continue to vastly expand our understanding of the
Earth‘s interior and the components of plate tectonics theory.

1. The author most likely mentions the work of the Dutch cartographer Abraham
Ortelius in order to:

A. show that the idea of plate tectonics is not new, although most evidence supporting it
dates to the 20th century.
B. compare the state of Dutch and English cartography in the 16 th century.
C. draw a strong contrast between Ortelius‘ pioneering views and those of Wegener and Holmes.
D. show that cartography was sufficiently advanced in the 16 th century that predictions could be
made about continental drift.
E. argue that plate tectonics is a recently developed concept

2. According to the author, the primary significance of the discovery that molten
uprisings continually reshape the ocean floor is that:

A. these uprisings provide a mechanism for the continental drift that has clearly
occurred.
B. it shows how sensitive the Earth‘s crust is to geologic activity taking place beneath it.
C. ocean floor movement lends strong support to the idea that the super-continent Pangea once
existed.
D. the movement of deep ocean plates offers an explanation for magnetic and seismic
measurements that have perplexed scientists for decades.
E. these help explain the phenomena of volcanic eruptions

3. What is the primary concern of the author in writing the passage?

A. to propose that modern maps are inaccurate compared to ancient maps


B. to discuss the gradual development and acceptance of the theory of plate tectonics.
C. to criticise the concept of Pangaea as proposed by some scientists.
D. to argue that all the continents will once again join together and become one
E. to explain that oceanic sedimentary rocks are different from those found on land

4. According to the passage, all of the following statements are true EXCEPT:

A. long, linear zones of rock on continental edges were recognized long before fossils on continental
edges were accurately dated.
B. mantle convection currents help to explain seismic phenomena long measured by oceanographers
and other studying continental drift.
C. fossils of similar plant and animal species can be found on widelyseparated
continents only in the long, linear, coastal rock zones of those continents.
D. the Earth‘s crust is a thin, hard layer of solid rock, while the mantle is a molten, flowing sublayer
of the crust.
E. Sea floors are younger than continental floors

vocab
Apr 27 – 2

The Seventh Symphony (1812) was, at the time, Beethoven’s last and vibrant word on the big style
he had cultivated in the previous decade. In the Eighth Symphony (1814) he does something new
by seeming to return to something old. He writes, that is, a symphony shorter than any since his
First. It is almost as though he wanted to call his entire development throughout that decade into
question. Indeed, over the remaining years of his life he would confidently explore in opposite
directions, writing bigger pieces than before and ones more compressed, his most rhetorical music
and his most inward, his most public and his most esoteric, compositions that plumb the
inexhaustible possibilities of the sonata style and those that propose utterly new ways of organizing
material, music reaching extremes of the centered and the bizarre.

If, however, we think of the Eight as a nostalgic return to the good old days, we misunderstand it.
To say it is 1795 revisited from the vantage point of 1812 is not right either. What interests
Beethoven is not so much brevity for its own sake — and certainly not something called “classicism”
— as concentration. It is as though he were picking up where he had left off in the densely
saturated first movement of the Fifth Symphony to produce another tour de force of tight packing.
He had already done something like this two years earlier in one of his most uncompromising works,
the F-minor String Quartet, Op. 95. But a symphony is not a “private” connoisseur’s music like a
string quartet; by comparison, the Eighth Symphony is Opus 95’s friendly, open-featured cousin,
even though its first and last movements bring us some of the most violent moments in Beethoven.

1) The author implies which of the following is a characteristic of "classicism"?

A.nostalgia
B.lack of focus
C.compromising
D.dense saturation
E.works on a smaller scale

2) Which of the following does the author imply about Beethoven's Eighth Symphony?

A.It is compromising, and not vibrant like a piece typical of classicism


B.In it, for the first time, Beethoven explored concentration as an organizing principle
for an entire symphony
C.It was more inwardly focused, less rhetorical than the F-minor String Quartet, Op. 95
D.It rejected the sonata style used in the Seventh Symphony, exploring completely new ways to
organize music.
E.Compared to his other symphonies, it is a particularly esoteric public work, having both friendly
and violent tendencies.

3)The passage provides support for which of the following?

A.Beethoven's Eighth Symphony would not be appreciated by connoisseurs.


B.The Fifth Symphony is the shortest symphony between the First and the Eighth.
C.In 1795, Beethoven composed works that contained less development than the
majority of pieces over the next 15 years.
D.The F-minor String Quartet, Op. 95 is one of Beethoven's most esoteric works.
E.Later in life, Beethoven developed significant misgivings about the big development characteristic
of Symphonies of his middle period.

Symphony (n) : bản giao hưởng

Vibrant (adj) : sôi động

Cultivate (v) : trau dồi

Inward (adj) : hướng nội

Esoteric (adj) : bí truyền, bí mật

Inexhaustible (adj) : vô tận, vô hạn

Utterly (adv) : hoàn toàn

Bizarre (adj) : kỳ quái, kỳ dị

Nostalgic (adj) : hoài niệm

Vantage point (n) : điểm thuận lợi

Brevity (n) : ngắn gọn

Sake (n) : lợi ích

Densely (adv) : dày đặc

Saturated (adj) : bão hòa

Connoisseur (n) : người sành, người thạo

Violent (adj) : mạnh mẽ, dữ dội


Apr 28 – 1

Retail stores and diversified manufacturing companies have operated under a set of traditional
assumptions that warrant challenge. The most basic assumption that company managers make is
that their companies should provide a high level of service to all their customers. However, acting on
this assumption can lead to loss of market share, less value for some customers, and maintenance
of unwieldy structures for distributing products, which ensures higher fixed costs.

According to Joseph Fuller, James O’Connor, and Richard Rawlinson, one area that needs particular
scrutiny is the way in which companies handle logistics, including transportation costs, handling
costs, management of inventory, storage costs, and order processing. Different customers often
have entirely different needs: designing a logistical structure that provides every customer with the
same level of service is wasteful and inefficient. Fully one-third of a company’s product may be stuck
in the “pipeline” between manufacturer and customer, where it only drains away money through
transportation and storage costs; if a particular customer does not need certain products to be
available immediately, a company does not need to spend money to ensure that all its merchandise
is on hand. Another problem is traditional averaging, in which products that cost the manufacturer
relatively little to produce are given prices similar to those of products that are expensive to
produce. While this means that the retailer is able to move more low-volume products out of
inventory, high-volume products tend to be overpriced, and more specialized products are not
delivered speedily enough and may be underpriced.

Fuller et al. describe a soft drink company’s decision to stop in-store promotions and special sales in
favor of standard pricing because the inconsistent demand caused by the swings in prices
necessitated variability in the manufacturing and distribution systems. Many retail managers tend to
overlook logistics out of a concern for gross margin; that is, they are swayed by the gross profit
made by the sale of a specific item, instead of looking at the net profit that remains after logistics
costs have been subtracted. Low-volume, high-margin products may not ultimately be as profitable
as high-volume, low margin products that are easy to move around, such as T-shirts or calculators.

1. According to the passage, when management assumes that its top priority is
customer service, it risks

A. streamlining its logistical structure to lower fixed costs


B. losing business to competitors
C. weighing inventory in favor of high-margin volume products
D. customer dissatisfaction due to steadily rising prices
E. having a majority of its merchandise stuck in the “pipeline”

2. Which of the following may be inferred from the passage about traditional averaging?

A. Businesses using this pricing method will face declining market share.
B. Companies with large inventories are likely. to employ this method to reduce stock.
C. It leads to significant fluctuations in the inventory levels of specialized products.

leads to is too strong but wait lets keep it and move on..
D. It can create inconsistencies in the pricing of goods with respect to their
manufacturing costs.
E. The technique is only effective for certain nonspecialized products.
3. According to the passage, a soft drink company rejected former marketing methods
that included promotions and special sales because

A. stores complained about having to make special arrangements for storing the extra products
B. high-volume products such as soda are consistently overpriced
C. the company realized it had given too much attention to the gross margin of some of its products
D. the promotions and sales earned the company high profits, but when the promotions ended, the
profits dropped
E. the promotions and sales created uneven demand, which continually forced the
company to change its logistical operations

Fuller et al. describe a soft drink company’s decision to stop in-store promotions and special sales in
favor of standard pricing because the inconsistent demand caused by the swings in prices
necessitated variability in the manufacturing and distribution systems

4. It can be inferred from the passage that which of the following might be the most
profitable item to sell?

A. cassette tapes
B. big-screen television sets
C. bunk beds
D. rowing machines
E. imported wine

Necessitate (v) : bắt buộc, cưỡng bách

Variability (n) : thay đổi


Apr 28 – 2

From the time they were first proposed, the 1962 Amendments to the Food, Drug, and Cosmetic Act
have been the subject of controversy among some elements of the health community and the
pharmaceutical industry. The Amendments added a new requirement for Food and Drug
Administration approval of any new drug: The drug must be demonstrated to be effective by
substantial evidence consisting of adequate and well controlled investigations. To meet this
effectiveness requirement, a pharmaceutical company must spend considerable time and effort in
clinical research before it can market a new product in the United States. Only then can it begin to
recoup its investment. Critics of the requirement argue that the added expense of the research to
establish effectiveness is reflected in higher drug costs, decreased profits, or both, and that this has
resulted in a “drug lag. ”The term drug lag has been used in several different ways.

It has been argued that the research required to prove effectiveness creates a lag between the time
when a drug could theoretically be marketed without proving effectiveness and the time when it is
actually marketed. Drug lag has also been used to refer to the difference between the number of
new drugs introduced annually before 1962 and the number of new drugs introduced each year
after that date. It is also argued that the Amendments resulted in a lag between the time when new
drugs are available in other countries and the time when the same drugs are available in the United
States. And drug lag has also been used to refer to a difference in the number of new drugs
introduced per year in other advanced nations and the number introduced in the same year in the
United States. Some critics have used drug lag arguments in an attempt to prove that the 1962
Amendments have actually reduced the quality of healthcare in the United States and that, on
balance, they have done more harm than good. These critics recommend that the effectiveness
requirements be drastically modified or even scrapped.

Most of the specific claims of the drug lag theoreticians, however, have been refuted. The drop in
new drugs approved annually, for example, began at least as early as 1959, perhaps five years
before the new law was fully effective. In most instances, when a new drug was available in a
foreign country but not in the United States, other effective drugs for the condition were available in
this country and sometimes not available in the foreign country used for comparison. Further,
although the number of new chemical entities introduced annually dropped from more than 50 in
1959 to about 12 to 18 in the 1960s and 1970s, the number of these that can be termed
important—some of them of “breakthrough” caliber—has remained reasonably close to 5 or 6 per
year. Few, if any, specific examples have actually been offered to show how the effectiveness
requirements have done significant harm to the health of Americans. The requirement does ensure
that a patient exposed to a drug has the likelihood of benefiting from it, an assessment that is most
important, considering the possibility, always present, that adverse effects will be discovered later.

1. The author is primarily concerned with

(A) outlining a proposal


(B) evaluating studies
(C) posing a question
(D) countering arguments
(E) discussing a law
2. The passage states that the phrase “drug lag” has been used to refer to all of the
following situations EXCEPT

(A) a lag between the time when a new drug becomes available in a foreign country and its
availability in the United States
(B) the time period between which a new drug would be marketed if no effectiveness research were
required and the time it is actually marketed
(C) the increased cost of drugs to the consumer and the decreased profit margins of the
pharmaceutical industry
(D) the difference between the number of drugs introduced annually before 1962 and the number
introduced after 1962
(E) the difference between the number of new drugs introduced in a foreign country and the
number introduced in the United States

3. The author would most likely agree with which of the following statements?

(A) Whatever “drug lag” may exist because of the 1962 Amendments is justified by the
benefit of effectiveness studies.
(B) The 1962 Amendments have been beneficial in detecting adverse effects of new drugs before
they are released on the market.
(C) Because of the requirement of effectiveness studies, drug consumers in the United States pay
higher prices than consumers in foreign countries.
(D) The United States should limit the number of new drugs which can be introduced into this
country from foreign countries.
(E) Effectiveness studies do not require a significant investment of time or money on the part of the
pharmaceutical industry.

4. The author points out the drop in new drugs approved annually before 1959 in order
to

(A) draw an analogy between two situations


(B) suggest an alternative causal explanation
(C) attack the credibility of an opponent
(D) justify the introduction of statistics
(E) show an opponent misquoted statistics

Most of the specific claims of the drug lag theoreticians, however, have been refuted

Really when it says "opponent" that's talking about the person (or people) and not the argument
itself.

5. The author implies that the non availability of a drug in the United States and its
availability in a foreign country is not necessarily proof of a drug lag because this
comparison fails to take into account

(A) the number of new drugs introduced annually before 1959


(B) the amount of research done on the effectiveness of drugs in the United States
(C) the possible availability of another drug to treat the same condition
(D) the seriousness of possible unwanted side effects from untested drugs
(E) the length of time needed to accumulate effectiveness research

Scrap (v) : bỏ đi

Theoretician (n) : nhà lý luận


Apr 29 – 1

Gautier was indeed a poet and a strongly representative one – a French poet in his limitations even
more than in his gifts; and he remains an interesting example of the manner in which, even when
the former are surprisingly great, a happy application of the latter may produce the most delightful
works. Completeness on his own scale is to our mind the idea he most instantly suggests. Such as
his finished task now presents him, he is almost sole of his kind. He has had imitators who have
imitated everything but his spontaneity and his temper; and as they have therefore failed to equal
him we doubt whether the literature of our day presents a genius so naturally perfect. We say this
with no desire to transfer Gautier to a higher pedestal than he has fairly earned – a poor service, for
the pedestal sometimes sadly dwarfs the figure. His great merit was that he understood himself so
perfectly and handled himself so skilfully. Even more than Alfred de Musset (with whom the speech
had a shade of mock-modesty) he might have said that, if his glass was not large, as least it was all
his own glass.

There are a host of reasons why we should not compare Gautier with such a poet as Browning; and
yet there are several why we should. If we do so, with all proper reservations, we may wonder
whether we are the richer, or, at all events, the better entertained, as a poet‘s readers should before
all things be, by the clear, undiluted strain of Gautier‘s minor key, or by the vast, grossly
commingled volume of utterance. It is idle at all times to point a moral. But if there are sermons in
stones, there are profitable reflections to be made even on Théophile Gautier; notably this one –
that a man‘s supreme use in the world is to master his intellectual instrument and play it in
perfection.

He brought to his task a sort of pagan bonhomie which makes most of the descriptive and pictorial
poets seem, by contrast, a group of shivering ascetics or muddled metaphysicians. He excels them
by his magnificent good temper and the unquestioning serenity of his enjoyment of the great
spectacle of nature and art. His world was all material, and its outlying darkness hardly more
suggestive, morally, than a velvet canopy studded with silver nails. To close his eyes and turn his
back on it must have seemed to him the end of all things; death, for him, must have been as the
sullen dropping of a stone into a well. His observation was so penetrating and his descriptive instinct
so unerring, that one might have fancied grave nature, in a fit of coquetry, or tired of receiving but
half- justice, had determined to construct a genius with senses of a finer strain than the mass of
human family.

1. In the passage, the author suggests that the French poet Théophile Gautier‘s talents
included all of the following EXCEPT:

A. an innovative and unique artistic view of nature.


B. the ability to quickly and immediately compose poetry.
C. extensive training in rhetorical and literary techniques.
D. a strong understanding of his world and himself.
E. he had sharp observation powers

In the 1st para the author speaks about the spontaneity with which Gautier would compose his
poetry is spoken about.

False, refer the word "spontaneity"


2. For what purpose can it reasonably be concluded does the author reference other
writers in this passage, including Musset and Browning?

A. To prove that Gautier, as a poet, was unique among his contemporaries


B. To show that Gautier‘s poetry was representative of French lyricism at the time
C. To criticize Gautier‘s limited talent and creativity
D. To refute the idea that Gautier‘s colleagues could easily imitate his style
E. To prove how inferior the other writers were compared to Gautier

3. The author makes a few noteworthy remarks about Gautier‘s attitude towards death.
This attitude would most support which of the following conclusions?

A. None of Gautier‘s literary works focused on human frailty.


B. Gautier believed that people are inherently linked to the divine.
C. The fleeting passage of time was a common poetic theme that Gautier neglected.
D. In his poetry, Gautier often focused on the vibrancy of human and natural life.
E. Gautier was a strong believer in the theory of life after death

Completeness (n) : đầy đủ cả, trọn vẹn

Imitator (n) : người bắt trước, giả mạo

Spontaneity (n) : sự tự phát

Pedestal (n) : bệ đỡ

Dwarf (v) : làm không lớn

Merit (n) : tài trí, tài cán

Skillfully (adv) : khéo léo

Modesty (n) : khiêm tốn

Strain (n) : sự căng thẳng, quá tải

Undiluted (adj) : không pha loãng

Utterance (n) : cách nói

Grossly (adv) : thô thiển

Commingle (v) : trộn lẫn

Sermon (n) : bài giảng, thuyết giáo

Notably (adv) : đáng chú ý

Pictorial (adj) : hình ảnh

Ascetic (n) : khổ hạnh

Shiver (v) : run rẩy, rùng mình

Muddle (v) : làm hỗn loạn


Metaphysician (n) : nhà siêu hình học

Excel (v) : vượt trội, giỏi hơn, xuất chúng

Magnificent (adj) : đẹp đẽ, hào hoa, tráng lệ

Serenity (n) : thanh thản, êm đềm, yên lặng

Spectacle (n) : cảnh tượng, phong cảnh, quang cảnh

Outlying (adj) : cách xa, đứng tách riêng

Sullen (adj) : buồn rầu

Penetrate (v) : xuyên thấu

Unerring (adj) : chắc chắn

Fancy (v) : suy nghĩ, tưởng tượng

Grave (adj) : nghiêm trọng, nghiêm trang


Sep 29 – 2

Italian economist Vilfredo Pareto’s lasting contribution to economic theory is his notion of Pareto
optimality. Pareto’s concept attempts to provide a theoretical understanding of the rather abstract
concept of desirability. Pareto’s desirability criterion holds that the state of world X is better than
world Y if no one is worse off in Y than X and at least one member of world X is better off in X.
Movement from one state to the other is termed “Pareto-desirable.”

Pareto-desirability as a norm has three important characteristics. The Pareto-desirable state is a


consequentialist theory because it evaluates the world based on the state of the world at some
given time. All institutions, actions, and policies are judged to be desirable based on the effect such
thing has on the world. Furthermore, Pareto desirable states are personal, in the sense that moral
goodness is defined as some better state for some person (even if it is a single person, as long as
everyone else is no worse off). Finally, goodness is defined in terms of a person’s particular
preferences; an individual is better off when achieving more of his or her preferences.

Pareto-desirability overlaps somewhat with the philosophical theory of utilitarianism, which states
that what is best is what causes the most people to be happy. Where Pareto-desirability breaks
ranks, however, is in its ranking of the alternative worlds. Utilitarianism considers and ranks all
possible worlds, good and bad, but the Pareto criterion ranks only those alternatives in which one’s
preference satisfaction is higher. Thus, comparisons between situations in which some people are
better off and some worse off are not possible with the Pareto system. Of course, Pareto desirability
is not a practical tool for use in the real world. Economists mainly use the Pareto criterion when
analyzing systems on a theoretical level.

1. It can be inferred from the passage that economisttheorists

A. regard Pareto desirability as little more than an abstract tool


B. continue to use the concept of Pareto desirability
C. disagree with the philosophical theory of utilitarianism
D. believe that consequentialism is the most appropriate way to view economic policies
E. do not believe the concept of moral goodness is useful in economics

2. The author most probably stated “Of course, Pareto desirability… world,” in order to
emphasize that

A. it is not practical to evaluate only situations in which people are better off because in
the real world someone is always worse off as the result of a policy
B. Pareto desirability has a limited use to economists and is only applicable when analyzing
monetary systems
C. utilitarianism has more application to the real world because it considers all possible worlds
D. Pareto-desirability can only be used as a theoretical tool because no true alternative worlds exist
E. the Pareto criterion would be more useful if theorists could find a way to use it to compare
situations in which some people are better off and some worse off
3. Which of the following statements provides the most logical continuation of the ideas
expressed in the last paragraph of the passage?

A. When examining the way the world actually is, however, economists use utilitarianism.
B. Due to its considerable shortcomings, Pareto-desirability will likely diminish in importance in
economic theory.

This is too extreme a choice to continue the idea.


C. By examining the rules behind particular policies instead of the policies themselves,
economists can determine the properties of the system that created them.
D. All three characteristics of Pareto-desirability are important to economists.
E. Until a more practical tool is developed, Pareto-desirability will continue to be economists’ prime
way of understanding the world.

4. The primary purpose of the passage is to

A. argue for the elevation of utilitarianism over Pareto-desirability in economic theory


B. evaluate the usefulness of the Pareto criterion to practical problems in economics
C. provide examples of some ways that economists attempt to conceptualize abstract concepts
D. suggest a revision to the predominant mode of economic thinking is called for
E. summarize a number of important features of a prevailing economic idea

Criterion (n) : tiêu chí

Abtract (v) : trừu tượng

Preference (n) : sở thích

Utilitarianism (n) : chủ nghĩa thực dụng


Sep 30 – 1

The myth of the noble savage has long dominated the social sciences. According to this myth, man
in his natural state lived in perfect harmony with both his fellow men and with nature. It is only the
corrupting specter of modernity, with its capitalistic rat-race and exploitative mindset, that leads to
war and destruction. While somewhat reassuring as a theory of human nature, the fact of the
matter is that the noble savage has not ever existed.

Consider the extinction of the moa bird, an ostrich-like species indigenous to New Zealand ranging
in size from 3 to 10 feet tall and weighing from 40 to 500 pounds. The last moa bird died roughly
800 hundred years ago, after inhabiting New Zealand for hundreds of thousands of years. It was not
a climatic calamity that wiped the birds out. Nor was it a new disease or a catastrophic event such
as a volcanic eruption. The moa went extinct a mere 200 years after the arrival of the Polynesian
Maoris.

Defenders of the noble savage myth may argue that the Maoris were not responsible for hunting the
moa to extinction, but the evidence suggests otherwise. Carbon dating of moa birds shows
conclusively that all known species of moa were present when the Maori arrived in New Zealand.
And most striking, archeologists have uncovered Maori sites containing between 100 to 500
thousand moa skeletons, roughly 10 times the number of moa living at any one time. The Maori did
not live in peaceful harmony with the environment; instead, they engaged in full-scale genocide of
the moa.

1. The primary purpose of the passage is to

A. argue for the conservation of endangered species


B. debunk a misconception
C. describe Maori hunting practices
D. criticize the imposition of modernity on other cultures
E. question the impact of man on animal extinctions

2. The discovery of the moa skeletons suggests that

A. the Maori not only ate the moa but used their bones in certain rituals
B. the moa were the primary staple of the Maori diet
C. the moa had no other predators besides the Maori
D. Maori society had advanced burial rituals
E. the Maori had been hunting the moa for many generations

3 The author most likely mentions “climatic calamity...volcanic eruption,” in order to

A. rule out possible natural causes of the moa extinction


B. cite common explanations for animal extinctions
C. suggest that extinctions are an inevitable part of biological history
D. argue that the moa was resistant to most diseases and natural disasters
E. mock those who hold to the belief of the noble savage

Noble (adj) : cao quý


Harmony (n) : hòa hợp

Corrupt (v) : đồi bại, suy đồi, hư

Specter (n) : bóng ma

Modernity (n) : hiện đại

Capitalistic (adj) : tư bản

Exploitative (adj) : bóc lột

Reassure (v) : trấn an

Indigenous (adj) : bản địa

Calamity (n) : thiên tai

Catastrophic (adj) : thảm khốc


Sep 30 – 2

Asian American poetry from Hawaii,the Pacific island state of the United States,is generally
characterizable in one of two ways: either as portraying a model multicultural paradise,or as
exemplifying familiar Asian American literary themes such as generational conflict.In this light,the
recent work of Wing Tek Lum in Expounding the Doubtful Points is striking for its demand to be
understood on its own terms. Lum offers no romanticized notions of multicultural life in Hawaii,and
while he does explore themes of family,identity,history,and literary tradition,he does not do so at the
expense of attempting to discover and retain a local sensibility. For Lum such a sensibility is
informed by the fact that Hawaii’s population,unlike that of the continental U.S.,has historically
consisted predominantly of people of Asian and Pacific island descent,making the experience of its
Asian Americans somewhat different than that of mainland Asian Americans.

In one poem,Lum meditates on the ways in which a traditional Chinese lunar celebration he is
attending at a local beach both connects him to and separates him from the past.In the company of
new Chinese immigrants,the speaker realizes that while ties to the homeland are comforting and
necessary,it is equally important to have “a sense of new family” in this new land of Hawaii,and
hence a new identity—one that is sensitive to its new environment. The role of immigrants in this
poem is significant in that,through their presence,Lum is able to refer both to the traditional culture
of his ancestral homeland as well as to the flux within Hawaiian society that has been integral to its
heterogeneity.Even in a laudatory poem to famous Chinese poet Li Po (701–762 A.D.), which partly
serves to place Lum’s work within a distinguished literary tradition,Lum refuses to offer a
stereotypical nostalgia for the past,instead pointing out the often elitist tendencies inherent in the
work of some traditionally acclaimed Chinese poets.

Lum closes his volume with a poem that further points to the complex relationships between
heritage and local culture in determining one’s identity. Pulling together images and figures as vastly
disparate as a famous Chinese American literary character and an old woman selling bread,Lum
avoids an excessively romantic vision of U.S.culture, while simultaneously acknowledging the dream
of this culture held by many newly arrived immigrants.

The central image of a communal pot where each person chooses what she or he wishes to eat but
shares with others the “sweet soup / spooned out at the end of the meal”is a hopeful one;
however,it also appears to caution that the strong cultural emphasis in the U.S.on individual drive
and success that makes retaining a sense of homeland tradition difficult should be identified and
responded to in ways that allow for a healthy new sense of identity to be formed.

1. Which one of the following most accurately expresses the main point of the passage?

(A) The poetry of Lum departs from other Asian American poetry from Hawaii in that it
acknowledges its author’s heritage but also expresses the poet’s search for a new local
identity.
(B) Lum’s poetry is in part an expression of the conflict between a desire to participate in a
community with shared traditions and values and a desire for individual success.
(C) Lum writes poetry that not only rejects features of the older literary tradition in which he
participates but also rejects the popular literary traditions of Hawaiian writers.
(D) The poetry of Lum illustrates the extent to which Asian American writers living in Hawaii have a
different cultural perspective than those living in the continental U.S.
(E) Lum’s poetry is an unsuccessful attempt to manage the psychological burdens of reconciling a
sense of tradition with a healthy sense of individual identity.

2. Given the information in the passage, which one of the following is Lum most likely to
believe?

(A) Images in a poem should be explained in that poem so that their meaning will be widely
understood.
(B) The experience of living away from one’s homeland is necessary for developing a healthy
perspective on one’s cultural traditions.
(C) It is important to reconcile the values of individual achievement and enterprise with
the desire to retain one’s cultural traditions.
(D) One’s identity is continually in transition and poetry is a way of developing a static identity.
(E) One cannot both seek a new identity and remain connected to one’s cultural traditions.

3. The author of the passage uses the phrase “the flux within Hawaiian society”
primarily in order to

(A) describe the social tension created by the mix of attitudes exhibited by citizens of Hawaii
(B) deny that Hawaiian society is culturally distinct from that of the continental U.S.
(C) identify the process by which immigrants learn to adapt to their new communities
(D) refer to the constant change to which the culture in Hawaii is subject due to its
diverse population
(E) emphasize the changing attitudes of many immigrants to Hawaii toward their traditional cultural
norms

4. According to the passage,some Asian American literature from Hawaii has been
characterized as which one of the following?

(A) inimical to the process of developing a local sensibility


(B) centered on the individual’s drive to succeed
(C) concerned with conflicts between different age groups
(D) focused primarily on retaining ties to one’s homeland
(E) tied to a search for a new sense of family in a new land

5. The author of the passage describes Expounding the Doubtful Points as “striking”
primarily in order to

(A) underscore the forceful and contentious tone of the work


(B) indicate that the work has not been properly analyzed by literary critics
(C) stress the radical difference between this work and Lum’s earlier work
(D) emphasize the differences between this work and that of other Asian American
poets from Hawaii
(E) highlight the innovative nature of Lum’s experiments with poetic form

6. With which one of the following statements regarding Lum’s poetry would the author
of the passage be most likely to agree?

(A) It cannot be used to support any specific political ideology.


(B) It is an elegant demonstration of the poet’s appreciation of the stylistic contributions of his
literary forebears.
(C) It is most fruitfully understood as a meditation on the choice between new and old that
confronts any human being in any culture.
(D) It conveys thoughtful assessments of both his ancestral homeland tradition and the
culture in which he is attempting to build a new identity.
(E) It conveys Lum’s antipathy toward tradition by juxtaposing traditional and nontraditional images

Contentious (adj) : gây tranh cãi

Poetry (n) : thơ phú, việc làm của thi sĩ

Characterizable (adj) : đặc trưng

Striking (adj) : nổi bật

Predominantly (adv) : chủ yếu

Descent (n) : con cháu

Meditate (v) : suy ngẫm

Flux (n) : sự tiếp tục thay đổi

Heterogeneity (n) : không đồng nhất

Laudatory (adj) : ca ngợi

Nostalgia (n) : hoài cổ

Elitist (adj) : tinh hoa

Communal (adj) : tài sản chung

Constant (adj) : xảy ra liên tục, không thay đổi

Diverse (adj) : đa dạng, phong phú, khác nhau

Heritage (n) : di sản, gia tài


Oct 1 – 1

Nitroglycerin has long been famous for its relief of angina pectoris attacks but ruled out for heart
attacks on the theory that it harmfully lowers blood pressure and increases heart rate. A heart
attack, unlike an angina attack, always involves some localized, fairly rapid heart muscle death, or
myocardial infarction. This acute emergency happens when the art eriosclerotic occlusive process in
one of the coronary arterial branches culminates so suddenly and completely that the local
myocardium—the muscle area that was fed by the occluded coronary— stops contracting and dies
over a period of hours, to be replaced over a period of weeks by a scar, or “healed infarct.” In 1974,
in experiments with dogs, it was discovered that administration of nitroglycerin during the acute
stage of myocardial infarction consistently reduced the extent of myocardial injury, provided that the
dogs’ heart rate and blood pressure were maintained in the normal range.

Soon after, scientists made a preliminary confirmation of the clinical applicability of nitroglycerin in
acute heart attack in human patients. Five of twelve human subjects developed some degree of
congestive heart failure. Curiously, the nitroglycerin alone was enough to reduce the magnitude of
injury in these five patients, but the other seven patients, whose heart attacks were not complicated
by any congestive heart failure, were not consistently helped by the nitroglycerin until another drug,
phenylephrine, was added to abolish the nitroglycerin-induced drop in blood pressure. One
explanation for this is that the reflex responses in heart rate, mediated through the autonomic
nervous system, are so blunted in congestive heart failure that a fall in blood pressure prompts less
of the cardiac acceleration which otherwise worsens the damage of acute myocardial infarction. It
appears that the size of the infarct that would otherwise result from a coronary occlusion might be
greatly reduced,and vitally needed heart muscle thus saved, by the actions of certain drugs and
other measures taken during the acute phase of the heart attack. This is because the size of the
myocardial infarct is not really determined at the moment of the coronary occlusion as previously
thought.

The fate of the stricken myocardial segment remains largely undetermined, hanging on the balance
of myocardial oxygen supply and demand which can be favorably influenced for many hours after
the coronary occlusion. So it is possible to reduce the myocardial ischemic injury during acute
human heart attacks by means of nitroglycerin, either alone or in combination with phenylephrine.
Other drugs are also being tested to reduce myocardial infarct size, particularly drugs presumed to
affect myocardial oxygen supply and demand, including not only vessel dilators such as nitroglycerin
but also antihypertensives, which block the sympathetic nerve reflexes that increase heart rate and
work in response to exertion and stress. Such measures are still experimental, and there is no proof
of benefit with regard to the great complications of heart attack such as cardiogenic shock, angina,
or mortality. But the drugs for reducing infarct size now hold center stage in experimental
frameworks.

1. According to the passage, the primary difference between a heart attack and an
angina attack is that a heart attack
(A) involves an acceleration of the heartbeat
(B) cannot be treated with nitroglycerin
(C) generally results in congestive heart failure
(D) takes place within a relatively short period of time
(E) always results in damage to muscle tissue of the heart
2. The patients who developed congestive heart failure did not experience cardiac
acceleration because
(A) the nitroglycerin was not administered soon enough after the onset of the heart attack
(B) the severity of the heart attack blocked the autonomic response to the nitroglycerin-
induced drop in blood pressure
(C) administering phenylephrine mitigated the severity of the drop in blood pressure caused by
nitroglycerin
(D) doctors were able to maintain blood pressure, and thus indirectly pulse rate, in those patients
(E) those patients did not experience a drop in blood pressure as a result of the heart attack

3. The passage provides information to answer all of the following questions EXCEPT
(A) What are some of the physiological manifestations of a heart attack?
(B) What determines the size of a myocardial infarct following a heart attack?
(C) What effect does nitroglycerin have when administered to a patient experiencing a heart attack?
(D) What are the most important causes of heart attacks?
(E) What is the physiological effect of phenylephrine?

4. It can be inferred from the passage that nitroglycerin is of value in treating heart
attacks because it
(A) lowers the blood pressure
(B) stimulates healing of an infarct
(C) causes cardiac acceleration
(D) dilates blood vessels
(E) counteracts hypertension

Other drugs are also being tested to reduce myocardial infarct size, particularly drugs presumed to
affect myocardial oxygen supply and demand, including not only vessel dilators such as
nitroglycerin but also antihypertensives,
5. The author’s attitude toward the use of nitroglycerin and other drugs to treat heart
attack can best be described as one of
(A) concern
(B) resignation
(C) anxiety
(D) disinterest
(E) optimism

6. It can be inferred that the phenylephrine is administered in conjunction with


nitroglycerin during a heart attack in order to
(A) prevent the cardiac acceleration caused by a drop in blood pressure
(B) block sympathetic nerve reflexes that increase the pulse rate
(C) blunt the autonomic nervous system which accelerates the pulse rate
(D) reduce the size of a myocardial infarct by increasing oxygen supply
(E) prevent arteriosclerotic occlusion in the coronary arterial branches

Relief (n) : cứu trợ

Angina (n) : đau thắt ngực

Angina pectoris (n) : đau thắt ngực


Heart rate (n) ; nhịp tim

Localize (v) : tụ lại một chỗ

Myocardial (adj) : cơ tim

Infarction (n) : nhồi máu

Acute (adj) : dữ dội, cấp tính

Occlusive (n) : thỉnh thoảng

Coronary (adj) : mạch vành

Arterial (adj) : động mạch

Culminate (v) : lên đến đỉnh điểm

Myocardium (n) : cơ tim

Occlude (v) : bị chặn, hút vào

Preliminary (adj) : sơ bộ

Congestive (adj) : xung huyết

Abolish (v) : bỏ, thủ tiêu

Induce (v) : gây ra

Reflex (adj) : phản xạ

Mediate (v) : làm trung gian

Autonomic (adj) : tự chủ

Blunt (v) : làm cùn, kém sắc bén

Prompt (v) : xúi giục

Cardiac (adj) : thuộc về tim

Acceleration (n) : sự tăng tốc

Occlusion (n) : sự tắc

Stricken (adj) : bị ảnh hưởng

Ischemic (adj) : thiếu máu cục bộ

Dilator (n) : máy giãn, nới rộng, làm trương khí quản

Exertion (n) : gắng sức

Hypertension (n) : tăng huyết áp

Hypotension (n) : giảm huyết áp

Severity (n) : mức độ nghiêm trọng


Oct 1 – 2

Risk is an inherent part of business, and managing its potential consequences means anticipating
those events that could generate adverse and costly outcomes for an organization, while taking
actions to advert and/or diminish the effects of such events. Futures contracts, forward contracts,
swaps, inverse floaters and options are a group of financial instruments called derivatives that are
available to hedge against financial risk. These securities are an agreement to buy or sell an asset
whose value is set by the market price or interest rate of some other security. Derivatives are useful
in corporate risk management, yet because they are complex, highly leveraged and complicated
they can lead to significant losses if not properly exercised.

Inappropriate uses of derivatives have led to highly publicized failures. For instance, Britain’s Barings
Bank, which financed America’s 1803 Louisiana Purchase, collapsed in 1995 when one of its
derivatives traders lost $1.4 billion. For nearly 20 years, California’s Orange County successfully
managed an investment fund that generated outstanding returns as long as interest rates declined;
but when interest rates increased, the purchase of very risky derivative products resulted in the fund
losing about $2 billion. Still when properly implemented, such as hedging rather than speculating to
increase profits, derivatives have significant benefits, which is why a high percentage of American
companies use derivatives regularly.

A particular derivative known as futures is useful for managing and reducing a variety of risks
related to interest rate, stock price and exchange rate fluctuations. Long hedges are futures
contracts that are bought to guard against price increases, while short hedges are futures contracts
that are sold to guard against price declines. The futures markets allow a firm to be protected
against changes that occur between when a decision is made and when a transaction is completed.
A firm’s risk aversion and its ability to assume the risk in consideration influence its decision to
hedge, and the futures markets allow a firm flexibility in the timing of its financial transactions.

1.Company A records its revenue streams in the currency of the country with which it is
doing trade. Believing that the cost of raw materials from one of its domestic suppliers
will decline and the currency rate of one of its foreign trading partners will increase,
which of the following scenarios is ideal based on information contained within the
passage?

a. The company could enter into a long hedge with both its supplier and its foreign trade partner.
b. The company could enter into a long hedge with its foreign trading partner and a
short hedge with its supplier.
c. The company could enter into a long hedge with its supplier and a short hedge with its foreign
trading partner.
d. The company could sell futures contracts to both its supplier and its foreign trading partner.
e. The company could buy futures contract from both its supplier and its foreign trading partner.

2.According to information contained in the passage, which of following is accurately


supported?

a. Derivatives are too complex to be soundly used.


b. Derivatives are one of the best financial tools available for managing the risks associated with
interest rate, stock price and exchange rate variability.
c. Derivatives are beneficial to circumvent potential risks a company may face.
d. A firm needs to have flexibility in the timing of its financial transactions.
e. In spite of their pitfalls, derivatives are necessary to help a company speculate against risks

Anticipate (v) : dự đoán

Adverse (adj) : bất lợi

Derivative (adj) : phái sinh

Hedge (n) : quỹ phòng hộ, bảo hiểm rủi ro

Leverage (v) : đòn bảy


Oct 2 – 1

From time to time history and myth come peculiarly close to one another, casting a new light on old,
and often largely dismissed tales. In various Eastern cultures the notion of the winged serpent and
the dragon have come down from the ages, only to be cast aside by modern society as fantastic,
mythological creations of someone‘s overactive ancient imagination. Now, it seems, this
supernatural beast might have some historical antecedents.

Archaeopteryx lithographica lived during the latter part of the Jurassic period, approximately 150
million years ago, just south of what today is central Germany. This ancient creature combined a
reptilian body and tail with bird-like wings and feathers. This strange amalgamation of traits seems
like something out of ancient mysticism of the Far East.

This beast has provided a wealth of information about the evolution of flight in birds. However, fossil
and skeletal studies indicate that it was not capable of flight. None of the Archaeopteryx fossils
discovered to date, including the most mature specimens, exhibit an ossified or bony sternum, the
wide bone that extends from the chest to the pelvic area in most modern birds. The main purposes
of this structure are to protect internal organs during flight and to act as a sturdy anchoring point
for the enormous pectoral muscles necessary for flight. There is no indication that Archaeopteryx
ever developed strong pectoral muscles, and perhaps this is one reason why it never developed a
sternum. Instead, it retained reptilian gastral ribs, thin braces in the abdominal region, which were
not attached to the skeleton and which served only to support and protect internal organs.
Researchers believe that flight would have been highly unlikely in an animal with such skeletal
characteristics.

Furthermore, the bones in the manus of Archaeopteryx do not seem to have been fused. In modern
birds, these bones are fused in order to support the wing. In addition, the ulna of modern birds is
marked with small knobs where feathers are anchored firmly to the bone by ligaments. The ulna in
Archaeopteryx, however, is smooth, indicating that its feathers were not firmly anchored into the
skeleton.

Finally, the skeletal characteristics of Archaeopteryx seem to indicate that this animal was most
adapted to terrestrial movement. Its hind legs and pelvis closely resemble those of bipedal
theropods and dinosaurs, suggesting that, like these other bipeds, it was adept at running along the
ground. In contrast to the posture of modern birds, whose bodies are suspended at the pelvis like a
seesaw with the thighbones horizontal, it stood up on its hind legs with its long reptilian tail serving
to balance it as well as enhance its ability to coordinate abrupt changes of direction while running.
In modern birds all that remains of the tail is a shrunken, fused structure called a pygostyle.
Although the foot of Archaeopteryx was bird-like, with fused metatarsals, it was also adapted to
running. By way of its peculiar mix of features, it seems to represent a kind of transitionary phase,
illustrating an evolutionary leap from reptile to bird and providing insight into the development of
flight.

1. Suppose that scientists have recently found the skeleton of a bird capable of flight
embedded in pre-Jurassic period rock. What effect would this discovery most likely have
on their thinking about Archaeopteryx lithographica?

A. It would support the view that Archaeopteryx lithographica represented a transitionary species
between reptiles and birds.
B. It would undermine the view that Archaeopteryx lithographica represented a
transitionary species between reptiles and birds.
C. It would neither support nor undermine the view that Archaeopteryx lithographica represented a
transitionary species between reptiles and birds.
D. It would support the view that Archaeopteryx lithographica failed to develop the pectoral muscles
necessary for flight.
E. It would prove beyond doubt that Archaeopteryx lithographica was actually a bird

2. Based on information in the passage, which of the following statements is NOT true?

A. Archaeopteryx lithographica’s skeleton is similar to the skeleton of a modern bird.


B. Archaeopteryx lithographica’s tail played a larger role in its daily life than the tail of a modern bird
plays in its daily life.
C. Scientists have studied Archaeopteryx lithographica in order to learn about the development of
flight.
D. Archaeopteryx lithographica shared some characteristics in common with dinosaurs.
E. Archaeopteryx lithographica lived in what is now Germany

3. Researchers believe that Archaeopteryx differs from modern birds for all of the
following reasons EXCEPT:
A. a lack of feathers.
B. pectoral muscle development.
C. ossification of the sternum.
D. knobs found on the ulna.
E. Fused bones in the manus

This ancient creature combined a reptilian body and tail with bird-like wings and feathers.

None of the Archaeopteryx fossils discovered to date, including the most mature specimens, exhibit
an ossified or bony sternum, the wide bone that extends from the chest to the pelvic area in most
modern birds.

4. The passage is primarily concerned with


A. analysing the factors that led to the extinction of dinosaurs
B. describing the similarities between Archaeopteryx lithographica and modern birds
C. explaining how birds are able to fly
D. discussing how Archaeopteryx lithographica could be an evolutionary link between
reptiles and birds
E. state that bones in the manus of Archaeopteryx lithographica were different from those of
modern birds

Peculiarly (adv) : đặc biệt

Serpent (n) : con rắn

Winged (adj) : có cánh

Antecedent (n) : tiền đề

Reptilian (adj) : bò sát


Reptile (n) : bò sát

Amalgamation (n) : sự hợp nhất

Mysticism (n) : thần bí

Specimen (n) : mẫu vật xét nghiệm

Ossify (v) : hóa thạch, hóa thành xương

Bony (adj) : xương

Sternum (n) : xương ức

Pelvic (adj) : xương chậu

Sturdy (adj) : chắc chắn, mạnh mẽ

Anchoring (v) : neo

Achoring point (n) : điểm neo

Pectoral (adj) : ở trên ngực, trước ngực

Gastral (adj) : dạ dày

Rib (n) : xương sườn

Abdominal (adj) : thuộc về bụng dưới

Brace (n) : vật nối cho chắc

Fuse (v) : dính lại

Manus (n) : bàn tay

Ulna (n) : xương khuỷu, xương tay trụ

Knob (n) : núm, chỗ nhô lên

Ligament (n) : dây chằng, gân

Firmly (adv) : vững chắc, vĩnh viễn

Terrestrial (adj) : mặt đất

Bipedal (adj) : hai chân

Thighbone (n) : xương đùi

Shrunken (adj) : thu nhỏ

Transitionary (adj) : chuyển tiếp

Leap (n) : bước nhảy vọt


Oct 2 – 2

Since the time of Darwin, morphological structures have been used to


identify phylogenetic relations. For example, the similarity between a man‘s arm and a bat‘s wing is
taken as evidence of their common origin. There are innumerable examples of this in nature. From
the whiskers of lions and domestic cats to the bone structure in the fins of a whale and that of a
human hand, it seems one would be hard pressed to fine an attribute in a particular species that did
not illustrate some kind of relationship to another species.

Similarities in behaviour patterns can also serve in reconstructing evolutionary history. It is not
always clear, however, how certain types of innate behaviour evolved through natural selection. In
its modern form the Darwinian interpretation of evolution asserts that evolution consists of changes
in the frequency of appearance of different genes in populations, and that the frequency of the
appearance of a particular gene can only increase if the gene increases the "Darwinian fitness" (the
expected number of surviving offspring) of its possessors.

The discovery of a genetic predisposition to be especially responsive to certain stimuli was an


important contribution to the study of evolution. Genetically determined responses must be subject
to the pressures of natural selection. Hence innate behaviour must evolve. Ethologists were able to
show how a motor pattern employed in a noncommunicatory context such as feeding could evolve
into a ritualized form employed as a signal in, say, courtship.

Differentiation in innate behaviour patterns could be traced to selection pressures arising from the
environment. There are many instances of animal behaviour patterns that seem not to contribute to
the survival of the individual displaying that behaviour. The classic example is the behaviour of the
worker bee: this insect will sting an intruder and thereby kill itself in defense of the hive. The
problem is evident: How can a gene that makes suicide more likely become established? The
concern over this type and other types of apparently anomalous behaviour led to the development
of a new phase in the study of the evolution of behaviour: a marriage of ethology and population
genetics.

Animal behaviour was formerly thought to consist of simple responses, some of them innate and
some of them learned, to incoming stimuli. Complex behaviour, if it was considered at all, was
assumed to be the result of complex stimuli. Over the past 60 years, however, a group of
ethologists, notably Konrad Lorenz, Nikolaas Tinbergen and Karl von Frisch, have established a new
view of animal behaviour. Studying whole patterns of innate animal behaviour in natural
environments (rather tha focusing primarily on learned behaviour, as animal behaviourists do), they
have shown that the animal brain possesses certain specific competences, that animals have an
innate capacity for performing complex acts in response to simple stimuli. As Gould put it in 1982,
"Rather than encompassing merely the rigid and impoverished behavioural repertoire of primitive
organisms, instinct has been shown to possess a stunning flexibility and overwhelming richness. As
a result, we no longer need to invoke the barren behaviouristic tenet of learning as an
'explanation‘ of complexity."
1. In the context of the arguments being made by the author in this passage, the term
"phylogenetic" (line 2) most closely means:

A. structural.
B. inter-species.
C. innate.
D. functional.
E. acquired

2. Which of the following scenarios would be most analogous to the example given by
the author of the worker bee?

A. A male spider reacts to intruding predators by releasing venom that kills both the predator and
itself.
B. A female marsupial abandons her weakest offspring as prey for her natural enemies in order to
protect the rest of her brood.
C. The youngest member of a canine pack sacrifices himself by fatally wounding an
attacking predator so that the pack itself can escape.
D. A drone ant kills an insect preying on his collective by stinging the insect‘s eyes.
E. A young Cheetah tries to hunt and fails repeatedly

3. The author of the passage would be most likely to agree with which of the following
statements?

A. Unusual animal behaviours can be understood in terms of natural selection when they
are studied in the context of procreation patterns and needs for survival of that
particular species.
B. Overpopulated animal colonies often weed out their excess or weak members by abandoning
them to their natural predators.
C. Darwin‘s evolutionary theories of natural selection have been unnecessarily modified by modern
scientists in order to make them accord with servations of animal behaviour patterns.
D. The evolution of certain types of innate animal behaviour demonstrate the inadequacy of the
notion of "Darwinian fitness" as an approach to studying evolution.
E. There are some natural phenomena that cannot be explained by logic

Inter-species (n) : liên loài

Phylogenetic (adj) : phát sinh gen

Morphological (adj) : hình thái

Predisposition (n) : khuynh hướng

Ethologist (n) : nhà đạo đức học

Innate (adj) : bẩm sinh

Encompass (v) : bao gồm

Impoverish (v) : suy nhược, làm hao mòn, nghèo nàn

Repertoire (n) : tiết mục


Primitive (adj) : nguyên thủy

Organism (n) : sinh vật

Invoke (v) : thỉnh nguyện, cầu cứu

Barren (adj) : cằn cỗi

Tenet (n) : nguyên lý, giáo lý

Canine (adj) : răng nanh, thuộc về chó


Oct 3 – 1

In response to the increasing environmental damage wrought by poachers, authorities placed a ban
on ivory in the 1980s. Although the ban resulted in an initial decrease in the sale and trade of illegal
ivory and a concurrent increase in the elephant population, more pressing needs caused most
Western nations to withdraw funding for poaching prevention programs. Without significant financial
support, poorer countries were unable to effectively combat poachers. The resulting explosion in the
ivory trade has seen prices increase to nearly 10 times the $45 per pound price at the beginning of
the decade.

Unfortunately, the countries with the worst poaching problems have also tended to be the ones least
able to combat the problem due to unstable political systems, corruption, lack of comprehensive
enforcement programs, or some combination of all these factors. One primary hindrance to better
enforcement of the ivory ban came from an inability to definitively identify the country of origin of
illegal ivory. Countries used this uncertainty to avoid responsibility for curbing illegal poaching in
their territories by attempting to blame other countries for the oversights in enforcement. Now,
though, zoologists have perfected a new DNA identification system. First, scientists gathered genetic
data from the population of African elephants, an arduous effort that ultimately resulted in a
detailed DNA-based map of the distribution of African elephants. Then, the researchers developed a
method to extract DNA evidence from ivory, allowing them to match the ivory with elephant
populations on the map. Zoologists hope this new method will pinpoint the exact origin of poached
ivory and force countries to accept their responsibility in enforcing the ban.

1. The passage suggests which of the following about the ivory ban?

A. It has been mostly unsuccessful in reducing the trade of illegal ivory.

passage says that the ban was successful, reference text highlighted in green color.
B. It will be successful now that the DNA-based map has been developed.
C. Western countries will increase their funding of the program now that countries are forced to take
responsibility for the poaching occurring within their borders.
D. Western monetary support was a major factor in its success.
E. It will continue to be ineffective unless the problems of political corruption are solved.

Although the ban resulted in an initial decrease in the sale and trade of illegal ivory and a concurrent
increase in the elephant population, more pressing needs caused most Western nations to withdraw
funding for poaching prevention programs

Without significant financial support, poorer countries were unable to effectively combat poachers.
The resulting explosion in the ivory trade has seen prices increase to nearly 10 times the $45 per
pound price at the beginning of the decade.
2. The response of the countries with the worst poaching problems to the situation was
most analogous to

A. a criminal who argues that it is not he but one of his accomplices who is guilty of a
crime
B. a tax evader who uses deceptive accounting practices to hide her income from auditors
C. a white collar criminal who believes that his actions are not a crime because no one has been
hurt
D. an embezzler who steals from her company by pilfering small amounts of money over a long
period of time
E. a con artist who convinces his victim that it is in the victim’s best interest to help him

Countries used this uncertainty to avoid responsibility for curbing illegal poaching in their territories
by attempting to blame other countries for the oversights in enforcement.

3. The passage suggests which of the following about the DNA-based map created by
zoologists?

A. The map is able to pinpoint the exact elephant from which poached ivory was obtained.
B. Its assemblage involved a great deal of work on the part of the researchers.
C. Its creation will force countries to accept responsibility for the ivory poached from within their
borders.

As per the passage the Zoologists hope that the new method will force the countries to accept
responsibility but whether it will happen for sure can't be inferred from the passage.

D. It allows zoologists to keep track of the migration patterns of African elephants.


E. It must be updated constantly to account for changes in elephant populations.

Zoologists hope this new method will pinpoint the exact origin of poached ivory and force
countries to accept their responsibility in enforcing the ban.

4. The passage is chiefly concerned with

A. exposing the weaknesses of the ivory ban


B. criticizing countries for failing to accept responsibility for the proliferation of poachers
C. pleading with Western countries to reinstitute financing for the ivory ban
D. advocating a course of action for countries with poaching problems
E. detailing a way to overcome a problem that has lessened the effectiveness of the
ivory ban

Assemblage (n) : tập hợp

Poacher (n) : kẻ săn trộm

Ivory (n) : ngà voi

work (n) : gây ra - (worked = wrought)

hindrance (n) : trở ngại

arduous (adj) : gian truân, hiểm trở, nặng nề

pinpoint (v) : xác định chính xác


Oct 3 – 2

Once surrounded and protected by vast wilderness, many of the national parks are adversely
affected by activities outside their boundaries. The National Park Organic Act established the
national park system and empowered the Secretary of the Interior to manage activities within the
parks. Conditions outside park boundaries are not subject to regulation by the Park Service unless
they involve the direct use of park resources.

Several approaches to protecting the national parks from external degradation have been proposed,
such as one focusing on enacting federal legislation granting the National Park Service broader
powers over lands adjacent to the national parks. Legislation addressing external threats to the
national parks twice passed the House of Representatives but died without action in the Senate. Also
brought to the table as a possible remedy is giving the states bordering the parks a significant and
meaningful role in developing federal park management policy.

Because the livelihood of many citizens is linked to the management of national parks, local
politicians often encourage state involvement in federal planning. But, state legislatures have not
always addressed the fundamental policy issues of whether states should protect park wildlife.

Timber harvesting, ranching and energy exploration compete with wildlife within the local
ecosystem. Priorities among different land uses are not generally established by current legislation.
Additionally, often no mechanism exists to coordinate planning by the state environmental
regulatory agencies. These factors limit the impact of legislation aimed at protecting park wildlife
and the larger park ecosystem.

Even if these deficiencies can be overcome, state participation must be consistent with existing
federal legislation. States lack jurisdiction within national parks themselves, and therefore state
solutions cannot reach activities inside the parks, thus limiting state action to the land adjacent to
the national parks. Under the supremacy clause, federal laws and regulations supersede state action
if state law conflicts with federal legislation, if Congress precludes local regulation, or if federal
regulation is so pervasive that no room remains for state control. Assuming that federal regulations
leave open the possibility of state control, state participation in policy making must be harmonized
with existing federal legislation.

The residents of states bordering national parks are affected by park management policies. They in
turn affect the success of those policies. This interrelationship must be considered in responding to
the external threats problem. Local participation is necessary in deciding how to protect park
wildlife. Local interests should not, however, dictate national policy, nor should they be used as a
pretext to ignore the threats to park regions.

1. What is the main purpose of the author in writing the passage?

A. argue that rampant timber harvesting is degrading national parks


B. describe a plan of action to resolve an issue
C. discuss different approaches to dealing with a problem
D. suggest that local participation is necessary to solve the problem described
E. to assert that national parks are adversely affected by activities outside their boundaries
2. The passage provides support for which of the following assertions?

A. The National Park Organic Act gave the Secretary of the Interior the right to overrule state
government policy in lands adjacent to national parks.
B. The federal government has been selling national park land to state governments in order to raise
money for wildlife conservation.
C. The actions of state governments have often failed to promote the interests of
national park wildlife.
D. Local politicians want the federal government to turn control of national parks over to state
governments.
E. Timber harvesting and energy exploration have not had any impact on national parks

3. In the context of the passage, the phrase external degradation (lines 8-9) refers to
which of the following:

A. threats to national parks arising from the House of Representative's willingness to address
environmental issues.
B. threats to national parks arising from state government environmental policies.
C. threats to national parks arising from local politicians‘ calls for greater state involvement in
national park planning.
D. threats to national parks arising from the National Park Organic Act.
E. threats to national parks arising from the lack of local support

Conditions outside park boundaries are not subject to regulation by the Park Service unless they
involve the direct use of park resources.

4. According to the passage, which of the following developments is most likely if


environmental cooperation between the federal government and state governments
does not improve?
A. A further decline in the land area of national parks
B. A further increase in federal ownership of land adjacent to national parks
C. A further growth in the powers of the National Park Service
D. A further loss of species in national parks
E. A further increase in timber harvesting activities

The secretary of the interior (n) : bộ trưởng nội vụ

Enact (v) : ban hành

Adjacent (adj) : liền kề

House of representatives (n) : hạ viện

Senate (n) : thượng nghị viện

Timber (n) : gỗ

Harvest (v) : khai thác

Ranch (v) : trại nuôi súc vật, trang trại


Jurisdiction (n) : quyền hạn

Supremacy (n) : quyền tối cao

Clause (n) : điều khoản, điều lệ

Supersede (v) : thay thế

Preclude (v) : ngăn chặn

Pervasive (adj) : phổ biến

Dictate (v) : ra lệnh

Pretext (n) : cái cớ


Oct 4 – 1

In mid-February 1917 a women’s movement independent of political affiliation erupted in New York
City, the stronghold of the Socialist Party in the United States. Protesting against the high cost of
living, thousands of women refused to buy chickens, fish, and vegetables. The boycott shut down
much of the City’s foodstuffs marketing for two weeks, riveting public attention on the issue of food
prices, which had increased partly as a result of increased exports of food to Europe that had been
occurring since the outbreak of the First World War.

By early 1917 the Socialist party had established itself as a major political presence in New York
City. New York Socialists, whose customary spheres of struggle were electoral work and trade union
organizing, seized the opportunity and quickly organized an extensive series of cost-of-living
protests designed to direct the women’s movement toward Socialist goals. Underneath the Socialists’
brief commitment to cost-of-living organizing lay a basic indifference to the issue itself. While some
Socialists did view price protests as a direct step toward socialism, most Socialists ultimately sought
to divert the cost-of-living movement into alternative channels of protest. Union organizing, they
argued, was the best method through which to combat the high cost of living. For others, cost-of-
living or organizing was valuable insofar as it led women into the struggle for suffrage, and similarly,
the suffrage struggle was valuable insofar as it moved United States society one step closer to
socialism.

Although New York’s Socialists saw the cost-of-living issue as, at best, secondary or tertiary to the
real task at hand, the boycotters, by sharp contrast, joined the price protest movement out of an
urgent and deeply felt commitment to the cost-of-living issue. A shared experience of swiftly
declining living standards caused by rising food prices drove these women to protest. Consumer
organizing spoke directly to their daily lives and concerns; they saw cheaper food as a valuable end
in itself. Food price protests were these women’s way of organizing at their own workplace, as
workers whose occupation was shopping and preparing food for their families.

1. The author suggests which of the following about New York Socialists' commitment
to the cost-of-living movement?

(A) It lasted for a relatively short period of time


(B) It was stronger than their commitment to the suffrage struggle.
(C) It predated the cost-of-living protest that erupted in 1917.
(D) It coincided with their attempts to bring more women into union organizing.
(E) It explained the popularity of the Socialist party in NYC.

Underneath the Socialists’ brief commitment to cost-of-living organizing lay a basic indifference to
the issue itself.

He says the commitment was brief, meaning that it only lasted a short time.

2. It can be inferred from the passage that the goal of the boycotting women was the

(A) achievement of an immediate economic outcome


(B) development of a more socialistic society
(C) concentration of a widespread consumer protest on the more narrow issue of food prices.
(D) development of one among a number of different approaches that the women wished to employ
in combating the high cost of price.
(E) attraction of more public interest to issues that the women and the socialist considered
important.

3.Which of the following best states the function of the passage as a whole?

(A) To contrast the views held by the Socialist party and the boycotting women in the cost-of-living
issue.
(B) To analyze the assumption underlying opposing viewpoints with the NY socialist party of 1917
(C) To provide a historical perspective on different approaches to the resolution of cost-of-living
issue.
(D) To chronicle the sequence of events that lead to the NY socialist party's emergency as a political
power.
(E) To analyze the motivations behind the socialist party's involvement in the women's
suffrage movement.

4. According to the passage, most New York Socialists believed which of the following
about the cost-of-living movement?

(A) It was primarily a way to interest women in joining the Socialist Party.
(B) It was an expedient that was useful only insofar as it furthered other goals.
(C) It would indirectly result in an increase in the number of women who belonged to labor unions.
(D) It required a long-term commitment but inevitably represented a direct step toward socialism.
(E) It served as an effective complement to union organizing.

Tertiary (adj) : hạng ba

Urgent (n) : khẩn cấp

Brief (adj) : ngắn gọn (= short)

Expedient (n) : phương tiện

Insofar (adv) : trong chừng mực

Predate (v) : có trước

Complement (n) : bổ sung

Affiliation (n) : liên kết

Customary spheres of struggle (n) : lĩnh vực đấu tranh thông thường

Seize (v) : nắm bắt

Union organizing (n) : tổ chức công đoàn

Oct 4 – 2
Despite their acronymic similarity, LEDs and LCDs represent distinct display technologies. In LEDs,
or light-emitting diodes, two different semiconductor materials are layered together: n-type, in
which mobile electrons carry negative charge, and p-type, in which “holes” in an otherwise bound
sea of electrons carry positive charge. When electric current flows through the p-n junction between
layers, an n-type electron falling into a p-type hole releases a photon, a specifically colored particle
of light.

The dominant technology currently used in most consumer product displays is the active matrix
liquid crystal diode display (LCD). LCDs apply thin-film transistors (TFTs) of amorphous silicon
sandwiched between two glass plates. The TFTs supply voltage to liquid-crystal-filled cells, or pixels,
between the sheets of glass. Liquid crystals can twist the polarization, or wave orientation, of light.
Just as a guitar string can vibrate sideways or up and down, so a light wave can be polarized
horizontally or vertically. Polarizing filters act as selective gates, transmitting light polarized one way
but not the other. Within a pixel, liquid crystals in their relaxed, coiled state rotate the polarization of
ambient light enough to make surrounding filters transparent. Alternatively, applied electrical signals
uncoil the crystals, causing the filters to block light and the pixel to become opaque. LCDs that are
capable of producing color images, such as in televisions and computers, reproduce colors by
blocking out particular color wavelengths from the spectrum of white light until only the desired
color remains. The variation of the intensity of light permitted to pass through the matrix of liquid
crystals enables LCD displays to present images full of gradations of different colors.

The amount of power required to untwist the crystals to display images is much lower than that
required for analogous processes using other technologies, such as plasma. The dense array of
crystals displays images from computer sources extremely well, with full color detail, no flicker, and
no screen burn-in. Moreover, the number of pixels per square inch on an LCD is typically higher than
that for other display technologies; LCD monitors are excellent at displaying large amounts of data
with exceptional clarity and precision.

1. According to the passage, the application of an electrical signal or current to both an


LED and an LCD pixel results in which of the following?

A. Both the LED and the LCD pixel become bright.


B. The LED becomes dark, but the LCD pixel transmits light
C. The LED becomes bright, but the LCD pixel ceases to transmit light.
D. The LED becomes dark, but the liquid crystals in the pixel uncoil.
E. The LED becomes bright, but the liquid crystals in the pixel coil up.

When electric current flows through the p-n junction between layers, an n-type electron falling into
a p-type hole releases a photon, a specifically colored particle of light.
=> LED will bright when electrical signal kicked in

2. The author most likely mentions “plasma” in the third paragraph in order to

A. provide an example of a technology that operates differently than LCDs


B. reinforce the importance of the commercial development of LCDs
C. describe the contrasting workings of another technology
D. indicate the greater number of applications for LCDs
E. explain the features of a competing type of display
The amount of power required to untwist the crystals to display images is much lower than that
required for analogous processes using other technologies, such as plasma.

3. The process through which an LCD monitor displays different colors is most closely
analogous to

A. the partial blocking of an hourglass so that a limited stream of grains of sand fall into the lower
portion
B. the use of rigid sizing boxes at an airport security checkpoint in order to allow the
passage of certain sizes of luggage while excluding other sizes of luggage
C. the soundproofing of a recording studio so that any performances within are muted to those
outside
D. the cutting out of characters from a sheet of paper so that a lamp in front of the paper casts
shadows in the shapes of the characters
E. the emission of warmer air by an air vent on the outside of a building while an air conditioning
system cools the interior of the building

Polarizing filters act as selective gates, transmitting light polarized one way but not the other

LCDs that are capable of producing color images, such as in televisions and computers, reproduce
colors by blocking out particular color wavelengths from the spectrum of white light until only the
desired color remains. The variation of the intensity of light permitted to pass through the matrix of
liquid crystals enables LCD displays to present images full of gradations of different colors.

PT - Similarity is that something can pass while other can be blocked.

4. Which of the following can be inferred about uncoiled liquid crystals in an LCD pixel?

A. Electric currents cause them to release photons.


B. They are in a relaxed state, in comparison to their high-energy coiled state.
C. They are found in one of two wave orientations, horizontal or vertical.
D. They fail to rotate the polarization of surrounding photons enough to allow them to
pass through nearby filters.
E. They cause the pixel to become transparent.

5.In this passage, the author is primarily concerned with

A. delineating the commonalities between two seemingly divergent phenomena


B. contrasting the uses of electrical power for two contrary purposes
C. explicating the scientific underpinnings of two dissimilar technologies
D. describing the economic significance of two processes
E. analyzing the relationship between two methods of displaying information

Semiconductor (n) : chất bán dẫn

Acronymic (adj) : từ viết tắt

Acronym (n) : từ viết tắt

Diode (n) : đi-ốt, đèn lưỡng cực


Emit (v) : phát ra

Liquid crystal (n) : tinh thể lỏng

Transistor (n) : bóng bán dẫn, điện trở

Amorphous (adj) : vô định hình

Polarization (n) : phân cực

Vibrate (v) : rung động

Coil (v) : cuộn, uốn khúc

Ambient (adj) : môi trường xung quanh

Transparent (adj) : trong suốt

Opaque (adj) : mờ mịt, chặn ánh sáng

Array (n) : mảng

Flicker (v) : nhấp nháy

Exceptional (adj) : đặc biệt

Clarity (n) : trong trẻo, rõ ràng

Precision (n) : độ chính xác


Oct 5 – 1

Over the past 20,000 years, the average volume of the human male brain has decreased from 1,500
cubic centimeters to 1,350 cc, losing a chunk the size of a tennis ball. The female brain has shrunk
by about the same proportion. If our brain keeps dwindling at this rate over the next 20,000 years,
it will start to approach the size of the brain found in Homo erectus, a relative that lived half a
million years ago and had a brain volume of only 1,100 cc.

Some believe the erosion of our gray matter means that modern humans are indeed getting
dumber. A common measure of intelligence - the encephalization quotient or EQ, defined as the
ratio of brain volume to body mass - has been found to be decreasing in the recent past. Recent
studies of human fossils suggest the brain shrank more quickly than the body in near-modern times.
More importantly, analysis of the genome casts doubt on the notion that modern humans are simply
daintier but otherwise identical versions of our ancestors, right down to how we think and feel.
Another study concluded that our present EQ is the same as that of the Cro-Magnons - our
ancestors who lived 30,000 years ago in Europe and were known more for brawniness rather than
brilliance.

On the other hand, other anthropologists such as Hawks believe that as the brain shrank, its wiring
became more efficient, transforming us into quicker, more agile thinkers. They explain the shrinking
by arguing that over the very period that the brain shrank, our DNA accumulated numerous adaptive
mutations related to brain development and neurotransmitter systems—an indication that even as
the organ got smaller, its inner workings changed.

This explanation may be plausible, considering that the brain is such a glutton for fuel that it globs
up to 20% of all the calories. To optimize this, the evolution may be moving towards a more
efficient smaller brain that yields the most intelligence for the least energy. A boom in the human
population in the last 20,000 years ago greatly improved the odds of such a fortuitous development
since the more the individuals, the bigger the gene pool, and the greater the chance for an unusual
advantageous mutation to happen.

The man-made product that is closest to the brain, the microprocessor, has seen similar evolution. A
microprocessor consists of transistors- the human equivalent of neuron that participates in decision
making – connected with wires that act as messengers between neurons. The first microprocessors
had extremely simple architectures and were not optimized for a certain set of tasks but were more
general purpose. Consequently, a lot of the power they consumed was dissipated in internal wiring
and not in decision making. With refinements, the architectures became more and more attuned to
the tasks that the microprocessor most commonly needed to do. Consequently, for the same
number of transistors the amount of wiring decreased by a factor of 3 while the microprocessor’s
processing speed increased by a factor of 10. While active research is still to conclude whether the
same holds true in case of the brain, one can only hope that the results are along the lines of the
microprocessor.

1. The passage suggests that the modern microprocessor is more efficient because:

A. It has more transistors and fewer wires.


B. It contains transistors that are smaller in size and consume less power.
C. It contains transistors that became faster with the evolution of technology.
D. Over the years, the organization of wiring and transistors has been better optimized
for tasks that the microprocessor is most commonly required to perform.
E. The prevalence of precision manufacturing has allowed more transistors to be added to the
microprocessor.

2. In paragraph 4 - lines 1 and 2, the author talks about the brain being a glutton for
fuel to:

A. justify the reason that evolution may be moving in a particular direction.


B. stimulate discussion about an alternate hypothesis proving the negative aspects of the reduction
of brain volume.
C. provide supporting evidence for a controversial theory.
D. refute the conclusion of the hypothesis mentioned in the preceding paragraph.
E. draw a parallel to another evolutionary development with a similar impact.

3. According to the passage, the relationship between encephalization quotient and


brain volume is:

A. for the same weight, the higher the brain volume the higher the encephalization
quotient.
B. for the same weight, the lower the brain volume the higher the encephalization quotient.
C. the higher the weight the higher the encephalization quotient.
D. the higher the encephalization quotient the higher the brain volume.
E. the lower the encephalization quotient the higher the brain volume.

4. Which of the following if true would weaken the assertion that humans are getting
dumber with the erosion of brain volume?

A. A study proved that our ancestors with lower encephalization quotient were more
intelligent than our ancestors with higher encephalization quotient.
B. A discovery that showed that through more refined training methods, a 20 year old can perform
many more tasks than a 20 year old could do 500 years back.
C. A study that proves that there is positive but non-linear correlation between encephalization
quotient and human intelligence. (too neutral)
D. A study that discovered that human encephalization quotient increased at a staggering pace
10000 years back.
E. A study that proves that there are measures outside of encephalization quotient that makes
humans more productive.

A- lower EQ with smaller brain  more intelligent  higher EQ with bigger brain

 Smaller brain  smarter  proving that erosion of brain volume can’t lead to dumber

Glutton (n) : tham ăn, háu ăn

Dwindle (v) : suy giảm

Dissipate (v) : tiêu tan, xài phí

Wiring (n) : hệ thống dây điện


Oct 5 – 2

Ironically, it was the ambitious reach of NASA's Viking spacecraft that set back the exploration of
Mars for so many years. As proffered by such enthusiastic visionaries as Carl Sagan, the question
that Viking would answer seemed not to be whether Mars was home to life but to what kind. Would
it be bacteria? Algae? Lichen? Or other, more complex forms? The two landers carried suites of
astrobiological life detection apparatuses that were marvels to behold: tiny growing chambers
packed with nutrients to awaken ancient Martian spores, radioactive tracers to identify the metabolic
activities of microbes, and chemical analysis systems to identify the expected respiratory byproducts.

Alas, save for some narrowly held dissenting opinions, no interpretation of the data supported the
existence of viable life forms on Mars. Instead, the probes found an atmosphere so harshly oxidizing
that biotic matter, as then understood, could scarcely survive. And so, like a rebuffed suitor
retreating across the room to collect his composure, the American exploration of space was left
hunting for new directions to explore. Mars was dead.

But time heals all shame and the allure of the red planet could not long remain at bay. It took a
series of events to bring Martian exploration back into the limelight. The discovery of extremophiles,
forms of life here on Earth that existed in conditions previously considered completely inhospitable
to any biological process, combined with a new understanding of the limitations of Viking's sensors
(which, it turned out, had been somewhat oversold in an effort to secure backing for the missions)
to awaken the possibility that life could still exist on Mars, albeit perhaps in forms we would scarcely
recognize. The competitive spirit of the Reagan presidency then provided the necessary political will
to return to Mars, for the Russians had suffered no shame from the American frustration and were
then pursuing an ambitious program of Martian exploration.

These factors combined to bring about the launch of Mars Observer, a mission almost as grandiose
as Viking had been. Technical shortsightedness caused the Mars Observer craft to explode while
preparing to enter orbit around Mars. But by then the spirit was relit and a sequence of missions,
some humble and some ambitious, some successful and some not, flew throughout the late 1990's
and into the new millennium. Today's goals are less dramatic. No surefire detection of life is
promised or looked for. Instead, a methodical tracing of the biologic, climatologic, and geologic
history of Mars is slowly expanding our understanding of our close planetary neighbor. This
systematic exploration may even one day progress to a reexamination of the great question that has
haunted man's imagination for centuries: Is there life on Mars?

1 : Based on the information in the passage, which of the following best characterizes
the results of the Viking missions?

a) A methodical understanding of the biologic, climatologic, and geologic history of Mars.


b) Undisputed proof that no life could possibly exist on Mars.
c) Widely disputed and inconclusive results regarding the biological potential of Mars.
d) A general belief that life, as then recognized, was unlikely to exist on Mars.
e) Widespread enthusiasm towards continued Martian exploration.

2: The passage discusses which of the following as a reason why the Viking landers
failed to detect signs of life on Mars?
a) Their life detection apparatus was not designed to detect traces of the sort of life now believed to
exist on Mars.
b) Subsequent investigations proved that there was no life present for them to detect.
c) The Viking landers only surveyed two small areas of the planet and might have missed locations
more favorable to biological activity.
d) The instruments were not as sensitive as originally claimed and, if life on Mars does
exist, it may do so in a form they would not have correctly identified.
e) The Viking landers could only identify complex forms of life such as bacteria and algae.

The passage says "combined with a new understanding of the limitations of Viking's sensors (which,
it turned out, had been somewhat oversold in an effort to secure backing for the missions) to
awaken the possibility that life could still exist on Mars, albeit perhaps in forms we would scarcely
recognize.

So A is wrong because it talk about Life detecting instruments where as passage tell us about
limitation of sensors--- The first para list more than one thing (life detection apparatuses, chemical
analysis system etc etc,) that were taken to mars

3 : Which of the following would most effectively and accurately title this article?

a) Viking and Mars Observer - Two Studies in Hubris


b) Back to the Red Planet, One Step at a Time
c) Beat the Russians!
d) Astrobiological Possibilities of Martian Extremophiles
e) Moving On - Why Space Exploration Must Abandon a Dead Mars

4. According to the passage, the competitive spirit of the Reagan presidency played
what role in the exploration of Mars?

a) It provided political impetus to move forward with the launch of the Viking Landers.
b) It ensured the flight of a sequence of missions from the late 1990's through the new millennium.
c) It led to the opposition of Russian efforts to explore Mars.
d) It was like a rebuffed suitor retreating across the room.
e) It contributed to the launch of the Mars Observer mission.

Ironically (adv) : trớ trêu thay

Proffered (v) : được trao

Astrobiological (adj) : sinh vật học

Apparatus (n) : bộ máy, đồ thiết bị, dụng cụ

Allure (n) : quyến rũ

Limelight (n) : ánh đèn sân khấu

Extremophile (n) : một loài vi sinh vật

Inhospitable (adj) : khắc nghiệt

Albeit (conjunction) : mặc dù


Presidency (n) : chủ tịch, hội trưởng, tổng thống

Grandiose (adj) : hoành tráng, đại quy mô, vĩ đại

Relit (v) : mở đèn lại

Sequence (n) : sự nối tiếp

Millennium (n) : thiên niên kỷ

Planetary (adj) : hành tinh


Oct 6 – 1

The prevailing theory of our Moon's origin is that it was created by a giant impact between a large
planet-like object and the proto-Earth very early in the evolution of our solar system. The energy of
this impact was sufficiently high that the Moon formed from melted material that began with a deep
liquid magma ocean. As the Moon cooled, this magma ocean solidified into different mineral
components, the lightest of which floated upwards to form the oldest crust. Although samples of this
presumed ancient crust were brought back to Earth by the Apollo 16 mission in 1972, it was not
until recently that scientists could successfully date them. Recent analysis of one of the samples, a
rock called ferroan anorthosite or FAN, which is believed to be the oldest of the Moon's crustal
rocks, has given scientists new insights into the formation of the Moon, suggesting that the Moon
may be much younger than currently believed.

The sample that had been carefully stored at NASA’s Johnson Space Center had to be extensively
pre-cleaned to remove terrestrial contamination. Once the sample was contamination free, the
researchers were able to study it. The team analyzed the isotopes of the elements lead and
neodymium to place the age of the sample at 4.36 billion years. This figure is significantly younger
than earlier estimates of the Moon's age that range to nearly as old as the age of the solar system
itself at 4.567 billion years. The new, younger age obtained for the oldest lunar crust is similar to
ages obtained for the oldest terrestrial minerals -- zircons from Western Australia -- suggesting that
the oldest crust on both Earth and the Moon formed at approximately the same time.

This study is the first in which a single sample of FAN yielded consistent ages from multiple isotope
dating techniques. This result strongly suggests that these ages pinpoint the time at which this
sample crystallized. The extraordinarily young age of this lunar sample either means that the Moon
solidified significantly later than previous estimates -- and therefore the moon itself is much younger
than previously believed -- or that this sample does not represent a crystallization product of the
original magma ocean. Either scenario requires major revision to existing models for the formation
of the Moon.

1. Which of the following captures the primary purpose of the passage?

A. To explain a recent scientific analysis while analyzing some important implications of


its findings
B. To establish how a prevailing scientific theory is no longer valid
C. To discuss two competing theories regarding a scientific phenomenon
D. To present the details of a recent scientific analysis and its findings
E. To question the outcome of a recent finding

2. The passage states which of the following?

A. The oldest terrestrial minerals can be said to be at least 4.36 billion years old.
B. In the recent analysis, more than one dating mechanism was applied to the FAN
sample.
C. A significant amount of more research is required before anything certain can be concluded
regarding the age of the FAN samples.
D. Zircons from Western Australia corroborated the age of the FAN sample.
E. Terrestrial contamination often occurs in samples brought from space.
This study is the first in which a single sample of FAN yielded consistent ages from multiple isotope
dating techniques

3. The passage supports which of the following statements?

A. In the light of the analysis done on the FAN sample, it is possible that ferroan anorthosite is not a
light mineral.
B. If the Moon was indeed formed 4.36 billion years ago, then the age of the solar system itself
needs to be reconsidered.
C. The fact that the moon was not formed at a much later stage than generally believed
does not affect the requirement for significant changes in the existing theory of Moon
formation.
D. A consistent result from multiple isotope dating techniques had never been achieved in any field
before the mentioned study on the FAN sample.
E. Since the Apollo 16 mission in 1972, scientists have not been able to obtain more samples from
the Moon’s crust.

Solidify (v) : kiên cố hóa, đông đặc lại

Isotope (n) : đồng vị

Crystallize (v) : kết tinh, vón cục


Oct 6 – 2

Some people maintain that the whole of medieval society can be explained by the relationship
between lord and vassal. But while much of Europe was divided into fiefs, the very lands from which
the word feudalism derives, there were parts of the continent that did not fit so nice a definition. In
fact, much of Italy, Spain, and southern France were not “feudal” in this sense.

Still, contractual obligations based on land ownership did define much of the social interaction of the
period. Corporations, such as they were in this time, might be lords or vassals or both. For example,
a monastery might be the lord to the tenant who resides in one of the manor houses on the
monastery grounds, while at the same time the monastery pays its homage to the king. There was
much room in the medieval system for ambiguities, but the exchange of obligations between
superior and inferior was the key element of the society.

As the feudal system increased in scope, new social structures emerged to help maintain the
convoluted networks of relationships required by feudalism. In order to justify the continued
extraction of resources from the peasants, the kings and lords had to provide security. This
exchange led to both the aristocracies that would come to control Europe for centuries to come and
the rise of a dedicated warrior class, whose militaristic tendencies would be partly responsible for
the years and years of warfare that would wrack Europe. Counterbalancing this, though, was the
tradition of courtly behavior and romantic love that would not have arisen without the fighting class.

While the lives of the vassals were not to be envied, it would be remiss to think that medieval kings
led luxurious lives. The great kings of early Europe had more freedom, but that freedom was
tempered by the paucity of options available. There was not much more to do than eat, sleep, pray,
hunt, and watch over the estate.

1. The primary purpose of the passage is to

A. critique the position of those who believe medieval Europe was defined entirely by the feudal
system
B. compare and contrast medieval European social structures with modern ones
C. describe the social conditions prevalent under the feudal system
D. discuss the historical conditions that led to years of European warfare
E. detail the daily lives of typical medieval citizens

2. Each of the following aspects of the feudal system is mentioned in the passage
EXCEPT

A. responsibilities based on land ownership


B. simultaneous occupation of roles
C. complicated societal relationships
D. pastimes of vassals
E. early form of taxation

In order to justify the continued extraction of resources from the peasants, the kings and lords had
to provide security
3. It can be inferred from the author’s discussion of kings in the passage that

A. most medieval kings were highly religious


B. medieval kings were no freer than the vassals that served them
C. medieval kings provided their vassals with protection in exchange for material goods

while at the same time the monastery pays its homage to the king. There was much room in the
medieval system for ambiguities, but the exchange of obligations between superior and inferior was
the key element of the society.
D. kings were partly responsible for the wars that wracked Europe
E. medieval kings paid homage to no one

There was not much more to do than eat, sleep, pray, hunt, and watch over the estate.

4. The author uses the term “feudal” in the first paragraph most probably to emphasize
that

A. social relations in Italy, Spain, and southern France were not based on obligations and land
ownership
B. the word feudal has many different derivations
C. certain parts of Europe need a different word to define the social interactions that took place in
them
D. Italy, Spain, and southern France did not have kings
E. although certain parts of the continent were not divided into fiefdoms, a system of
obligations still existed

Para 1 : But while much of Europe was divided into fiefs

Para 2 : Still, contractual obligations based on land ownership did define much of the social
interaction of the period.

Vassal (n) : địa chủ

Fief (n) : thái ấp dưới sự quản lý của chế độ phong kiến

Derive (v) : chuyển hóa

Contractual (adj) : hợp đồng

Interaction (n) : sự tương tác

Monastery (n) : tu viện

Tenant (n) : người thuê nhà, người ở

Reside (v) : cư trú

Manor house (n) : trang viên

Homage (n) : tôn kính, tôn sùng

Ambiguity (n) : sự mơ hồ
Convoluted (adj) : phức tạp

Extraction (n) : khai thác, rút ra, sự lấy

Peasant (n) : nông dân

Aristocracy (n) : quý tộc

Militaristic (adj) : quân phiệt

Wrack (v) : phá hủy

Counterbalance (v) : đối trọng

Courtly (adj) : lịch sự, nhã nhặn, lễ độ

Envy (v) : đố kỵ, ghen tị

Paucity (n) : sự ít ỏi

Pastime (n) : trò tiêu khiển, sự giải trí

Prevalent (adj) : thịnh hành


Oct 7 – 1

Some reputable astrophysicists propose that the Moon was created simultaneously with Earth from
the primordial accretion disc. Although such a hypothesis agrees with the hypothetical model on
which our solar system is believed to have been built, it nevertheless remains inherently flawed, as it
fails to address the high angular momentum of the Earth-Moon system and the lack of metallic iron
in the Moon's core. Since the amount of metallic iron in the cores of other planetary bodies in our
solar system is considerable, the latter finding is especially damaging to the aforementioned theory's
credibility. Nevertheless, another, more plausible theory has emerged which accounts for the Moon's
formation. Dubbed the Giant Impact Hypothesis, it conjectures that the Moon was created as a
result of a collision of the proto-Earth and another large planetary body roughly the size of Mars.
Computer simulations have confirmed that such a hypothesis would indeed produce the existing
angular momentum of the Earth-Moon system and the small amount of metallic iron that currently
exists in the Moon's core. The Giant Impact Hypothesis is further supported by the findings
indicating that samples of mantle rock collected from the Moon's surface are similar in their
geochemical composition to loose rock on the Earth's surface.

1. The author's primary purpose is to

A. compare competing theories of the Moon's creation


B. applaud reputable astrophysicists for their theory on lunar formation
C. emphasize scientists' lack of knowledge regarding the Moon's formation
D. show why all theories of the Moon's creation are inherently flawed
E. promote a theory of lunar formation over a competing theory

2. Which of the following is mentioned in the passage about the Moon's surface?

A. It is exactly identical to the Earth's surface in geochemical composition.


B. Its geochemical composition puts the Giant Impact Hypothesis in doubt.
C. Computer simulations show that its geochemical composition is similar to that of Earth.
D. It contains small amounts of metallic iron.
E. Some of its components are similar to those on Earth's surface.

3. What can be inferred about Earth from the first hypothesis mentioned in the passage?

A. Its core contains a high concentration of metallic iron.


B. It was created after the Moon.
C. The Moon was originally part of Earth.
D. The high angular momentum of the Earth-Moon system casts doubt on our theories of the age of
the Earth.
E. Scientists do not know how the Earth was created.

4. According to the first hypothesis mentioned in the passage, the Moon and the Earth
are most similar to which of the following?

A. wheat and bread


B. a parent and a child
C. a pair of twins conceived at the same time
D. Earth and the satellites which orbit around it
E. two cars next to each other on an assembly line

5. Which of the following arguments most closely parallels the reasoning behind the
author's analysis of the first hypothesis?

A. Tests revealing that hummingbirds can measure the amount of sugar in nectar have no bearing
on the hypothesis that hummingbirds choose the flowers they pollinate.
B. Despite the empirical evidence that queen termites reproduce with many mates, the hypothesis
that termite colonies contain a monogamous royal couple should be discounted purely on the basis
of its disagreement with the Insect Mating Hypothesis.
C. The hypothesis that dust mites are a cause of asthma should be accepted as fact because it fits
with the hypothesis that the fecal matter of insects contain allergenic enzymes.
D. Despite speculations that blowfish are similar to chameleons, the theory that
blowfish use camouflage to defend themselves has been cast into doubt because of
recent findings that their particular form of movement influences their natural defenses.
E. Although tests have shown that entognatha are similar to secondary insects, scientists need more
evidence to disprove the theory that they are derived from winged ancestors.

Chameleon (n) : tắc kè hoa

Blowfish (n) : cá bơn

Camouflage (n) : ngụy trang, hóa trang, thay đổi hình dạng

Astrophysicist (n) : nhà vật lý thiên văn

Primordial (adj) : ban sơ, nguyên thủy

Accretion (n) : bồi đắp

Angular (adj) : góc cạnh

Momentum (n) : động lượng

Conjecture (v) : phỏng đoán

Collision (n) : va chạm

Geochemical (adj) : địa hóa

Loose (v) : mở ra, bắn ra


Oct 7 – 2

For biologists, the term “eye” describes any lightsensitive organ consisting of more than one cell.
Although most animals have eyes, eye structures vary widely. The compound eyes of insects and
other arthropods, for example, have an architecture strikingly different from the single-lens eyes of
vertebrates and mollusks. Until recently, most biologists believed that all the different kinds of
evolved independently from as many as forty ancestral prototypes, and not from a single ancestral
prototype eye.

Traditional means of tracking the evolutionary development of eyes included examinations of


internal eye structures, which tended to support the multiple origin theory despite some anomalies
such as the resemblance between mammals’ eyes and the eyes of the nautilus mollusk—animals
that are not closely related. Proponents of the multiple origin theory dismissed such examples as
textbook cases of evolutionary convergence: the idea that even strikingly different prototype eyes
could evolve into kinds of eyes remarkably similar to each other.

In support of their theory, these biologists point to the fact that different species inhabiting the
same environment frequently have very different eye structures from one another. This lack of
correlation between eye structures and physical environments has provided what is perhaps the
strongest support for the multiple origin theory. Adherents of the theory argue that if eyes
originated from a single ancestral prototype, then there should be similarity in the patterns of eye
evolution in species that evolved under the same environmental conditions. The inability of biologists
to identify such evolutionary patterns is arguably the primary reason for this theory’s widespread
acceptance.

In 1993, however, a crucial link was discovered: a control gene that activates the many genes
needed for complete eye formation in fruit flies. Analogues to this gene have since been identified in
many organisms, including earthworms, mice, and humans, and are expected to exist in all eye-
bearing organisms. Researchers discovered that inserting the control gene present in mice into fruit
flies results in the formation of functional fruit fly eyes. This suggests that the control genes in mice
and in fruit flies are interchangeable and hence evolved from a single, common ancestral gene. The
same may be true of all of these control genes, which would argue convincingly against the multiple
origin theory and call for a reevaluation of the evidence that seemed to support the theory. For
example, the lack of correlation between eye structures and physical environments may have
resulted from the advantage bestowed by eyes. The survival value of sight is perhaps so great that
even variations in eyes that might be less than optimal for some individuals in a particular
environment are sufficiently advantageous that they allow the individuals to survive and propagate
the variation, thus facilitating the proliferation of variations in eyes even in the absence of an
environmental difference.
1. The author mentions that biologists accepted which one of the following as evidence
for the theory that eyes evolved from multiple origins?

(A) the lack of fossil evidence of a common ancestor for all eye-bearing species
(B) the lack of correlation between eye structures and physical environments
(C) the lack of correlation between physical environments and control genes for eye development
(D) the resemblance between the eyes of mammals and the nautilus mollusk, species that are not
closely related
(E) the obvious evolutionary advantage bestowed by some kinds of eyes as compared with others

2. The primary purpose of the passage is to

(A) distinguish between two theories and explain the theoretical basis for each of those theories
(B) argue that a particular discovery provides insufficient evidence for the rejection of a particular
widely accepted theory
(C) explain how a particular piece of evidence challenges a particular theory that has
been widely accepted
(D) provide grounds for a reexamination of the assumptions underlying a recent challenge to a
commonly held theory
(E) suggest some practical implications of a particular theoretical finding that conflicts with a
particular commonly held theory

3. The passage provides the strongest support for the inference that the author’s
attitude regarding the discovery of a control gene responsible for activating eye
formation in fruit flies can be accurately described as

(A) concern that biologists have accepted the single origin theory without carefully evaluating the
assumptions it is based on
(B) concern that biologists may have prematurely abandoned the multiple origin theory
(C) confidence that the discovery will imminently lead to the complete abandonment of the multiple
origin theory
(D) optimism that its discovery might foster a reevaluation of the assumptions biologists utilize when
researching evolution in general
(E) anticipation that the discovery will foster further efforts to determine whether all
varieties of eyes have evolved from a single ancestral prototype eye

4. According to the author, control genes have been found that


(A) determine how each species’ characteristic eye structure will differ from that of other species
(B) probably occur in all animals but only activate eye formation in just a few
(C) may be interchangeable among mice, fruit flies, and humans without causing
members of those species to develop eyes that are atypical for their species
(D) regulate specific aspects of eye evolution in certain species, suggesting that those species need
not be assumed to stem from a common ancestor
(E) activate eye formation in species including humans and mice, and this helps explain how
evolutionary convergence occurs – not inferrring question

This suggests that the control genes in mice and in fruit flies are interchangeable and hence evolved
from a single, common ancestral gene
5. Proponents of the multiple origin theory would likely regard the relationship between
the development of human eyes and the development of the eyes of the nautilus
mollusk as most analogous to which one of the following?

(A) Traditional Italian pizza is very similar in form and ingredients to a traditional Southern French
food, which, like Italian pizza, has its origins in earlier Mediterranean cultures.
(B) Mexican traditional cuisine uses round, flat tortillas made from corn or wheat, and
traditional cuisines of India use tortilla-like flat, round bread made from wheat flour,
even though the Mexican and Indian cultures have no traditional connection with each
other.
(C) Yogurt is superficially unlike cheese, although both are made almost entirely of milk and both
are traditional ingredients in Middle Eastern and European cuisines.
(D) Mozzarella cheese is traditionally made from cow’s milk in the United States, even though Italian
mozzarella, which is the original model for the US version and has almost the same flavor, color, and
texture, is traditionally made from buffalo’s milk.
(E) The culinary use of corn—a vegetable that was originally available only to various Native
American cultures—has spread to many cultures throughout the world, some of which are very
distant and different from one another.

Traditional means of tracking the evolutionary development of eyes included examinations of


internal eye structures, which tended to support the multiple origin theory despite some anomalies
such as the resemblance between mammals’ eyes and the eyes of the nautilus mollusk—animals
that are not closely related.

Anticipation (n) : dự đoán, chờ đợi

Atypical (adj) : không điển hình

Prototype (n) : nguyên mẫu

Arthropod (n) : động vật chân đốt

Vertebrate (n) : động vật có xương sống

Mollusk (n) : nhuyễn thể, động vật thân mềm

Anomaly (n) : dị thường, vô quy tắc

Resemblance (n) : sự giống nhau

Nautilus (n) : ốc anh vũ

Convergence (n) : hội tụ

Correlation (n) : sự tương quan, quan hệ, liên quan

Adherents (n) : tín đồ

Optimal (adj) : tối ưu

Propagate (v) : làm lan ra, tuyên truyền, sanh sôi nảy nở, truyền giống

Proliferation (n) : sinh sôi nảy nở, nảy mầm, sanh sản
Oct 8 – 1

In 1979, John A. Eddy of the High Altitude Observatory and Aram A. Boornazian, a mathematician in
Boston, concluded that the sun is contracting at a considerable rate, based on daily measurements
of the sun’s image. They estimated the reduction of the sun’s horizontal diameter to be about two
seconds of arc, or about 0.1 percent, per century. These findings sparked renewed debate over
solar measurement and the possibility that our sun may be shrinking.

Heliologists contend that the brilliance of the sun’s disk, and the imperfect and variable transparency
of the earth’s atmosphere, make observations from the earth’s surface unreliable. A better source of
data regarding solar shrinkage is the duration of total solar eclipses, while a second statistically
independent check is provided by the time the planet Mercury is visible against the solar disk. These
observations allow a monitoring of changes in the sun’s diameter at reasonably regular intervals
back to about the start of the 18th century, or roughly twice the number of years covered by the
data used by Eddy and Boornazian.

When the observations of total solar eclipses over the past three centuries are analyzed in
conjunction with the evidence of the transits of Mercury, the data indicate only a negligible change
in the sun’s diameter. The percentage of decrease per century has been calculated to be only 0.007,
with a 0.0008 percent margin of error.

1. The author argues that Eddy and Boornazian have

a. misrepresented experimental results in order to gain support for their conclusions


b. incorrectly surmised that the sun's diameter is independent of Mercury's orbit
c. probably drawn an incorrect conclusion due to their reliance on unreliable data
d. made several noteworthy breakthroughs in the field of solar measurement
e. wilfully ignored the existence of data pertaining to the duration of solar eclipses

2. According to the author, evidence that the sun is shrinking only at a negligible rate is
based on

I. measurement of the length of solar eclipses


II. measurement of the time during which Mercury can be observed crossing the sun's path
III. daily measurements of the sun's image

a. I only
b. II only
c. III only
d. I and II only
e. I, II and III
3. It can be inferred from the passage that all of the following cast doubt on Eddy and
Boornazian's conclusion EXCEPT

a. the non-uniformity of atmospheric clarity

"the imperfect and variable transparency of the earth’s atmosphere"

b. the calculations used to determine the sun's horizontal diameter

They estimated the reduction of the sun’s horizontal diameter to be about two seconds of arc, or
about 0.1 percent, per century.

c. atmospheric distortion even during periods of clear skies

"the imperfect and variable transparency of the earth’s atmosphere"

d. the sheer intensity of the sun's radiance

"the brilliance of the sun’s disk"

e. the comparatively brief period of time their study covers

as it mentioned in P2, they must observed for a longer duration.

Altitude (n) : độ cao

Contract (v) : co rút

Spark (v) : trâm ngòi

Heliologist (n) : chuyên gia trị liệu

Brilliance (n) : sáng chói, sáng rực

Transparency (n) : tính trong suốt

Eclipse (n) : nhật thực

Negligible (adj) : không đáng kể

Non-uniformity (n) : sự không đồng đều

Clarity (n) : sự trong trẻo, trong suốt

Distortion (n) : biến thể, méo mó

Radiance (n) : rạng rỡ, ánh sáng chói lọi, bức xạ

Sheer intensity (n) : cường độ cao

Comparatively (adv) : so sánh


Oct 8 – 2

Symptoms of Parkinson’s Disease, such as tremors, are thought to be caused by low dopamine
levels in the brain. Current treatments of Parkinson’s disease are primarily reactionary, aiming to
replenish dopamine levels after dopamine-producing neurons in the brain have died. Without a more
detailed understanding of the behavior of dopamine-producing neurons, it has been impossible to
develop treatments that would prevent the destruction of these neurons in Parkinson’s patients.

Recent research provides insight into the inner workings of dopamine-producing neurons, and may
lead to a new drug treatment that would proactively protect the neurons from decay. By examining
the alpha-synuclein protein in yeast cells, scientists have determined that toxic levels of the protein
have a detrimental effect on protein transfer within the cell. More specifically, high levels of alpha-
synuclein disrupt the flow of proteins from the endoplasmic reticulum, the site of protein production
in the cell, to the Golgi apparatus, the component of the cell that modifies and sorts the proteins
before sending them to their final destinations within the cell. When the smooth transfer of proteins
from the endoplasmic reticulum to the Golgi apparatus is interrupted, the cell dies.

With this in mind, researchers conducted a genetic screen in yeast cells in order to identify any gene
that works to reverse the toxic levels of alpha-synuclein in the cell. Researchers discovered that such
a gene does in fact exist, and have located the genetic counterpart in mammalian nerve cells, or
neurons. This discovery has led to new hopes that drug therapy could potentially activate this gene,
thereby suppressing the toxicity of alpha-synuclein in dopamine-producing neurons.

While drug therapy to suppress alpha-synuclein has been examined in yeast, fruitflies, roundworms,
and cultures of rat neurons, researchers are hesitant to conclude that such therapies will prove
successful on human patients. Alpha-synuclein toxicity seems to be one cause for the death of
dopamine-producing neurons in Parkinson’s patients, but other causes may exist. Most scientists
involved with Parkinson’s research do agree, however, that such promising early results provide a
basis for further testing.

1) One function of the third paragraph of the passage is to

(A) highlight the many similarities between yeast cells and mammalian nerve cells
(B) explain in detail the methods used to conduct a genetic screen in yeast cells
(C) further explain the roles of various cellular components of yeast cells
(D) identify the genes in yeast cells and mammalian nerve cells that work to reverse the toxic levels
of alpha-synuclein
(E) clarify the relevance of genetic testing in yeast cells to the search for a new
treatment for Parkinson’s disease

2) It can be inferred from the passage that a yeast cell with toxic levels of alpha-
synuclein will die because

(A) low levels of dopamine will disrupt the flow of proteins from the endoplasmic reticulum to the
Golgi aparatus

High levels of alpha-synuclein disrupt the flow of proteins from the endoplasmic reticulum
(B) the gene that suppresses alpha-synuclein is missing or is not functioning properly in such yeast
cells
(C) drug therapy has proven to be ineffective in yeast cells
(D) the normal distribution of proteins to the different cell components outside the Golgi
apparatus will be affected
(E) alpha-synuclein is by nature a toxic protein

3) It can be inferred from the passage that current treatments of Parkinson’s Disease

(A) repair damaged cells by replenishing dopamine levels in the brain


(B) are ineffective in their treatment of Parkinson’s symptoms, such as tremors
(C) were developed without a complete understanding of dopamine-producing neurons
(D) will inevitably be replaced by new drug therapy to suppress alpha-synuclein toxicity
(E) were not developed through research on yeast cells

Current treatments of Parkinson’s disease are primarily reactionary, aiming to replenish dopamine
levels after dopamine-producing neurons in the brain have died. Without a more detailed
understanding of the behavior of dopamine-producing neurons, it has been impossible to develop
treatments that would prevent the destruction of these neurons in Parkinson’s patients.

4. According to the passage, which of the following represents the chronology of a


typical protein life in a healthy yeast cell?

(A) Protein is made in the endoplasmic reticulum, sent to the Golgi apparatus, processed
and altered in the Golgi apparatus, and then delivered to other parts of the cell.
(B) Protein is created in the Golgi apparatus, modified and delivered to other parts of the cell, then
decomposed by alpha-synuclein.
(C) Protein is produced in the endoplasmic reticulum, sent to the Golgi apparatus, and then
decomposed by alpha-synuclein.
(D) Protein is produced in the Golgi apparatus, modified by the Golgi apparatus, distributed to the
neuron, and then sent to the endoplasmic reticulum.
(E) Protein is produced by alpha-synuclein, transferred to the endoplasmic reticulum, sent to the
Golgi apparatus for modification, and then distributed to the rest of the cell.

Tremor (n) : run rẩy

Yeast (n) : nấm men, men

Detrimental (adj) : bất lợi, có hại

Endoplasmic (adj) : nội chất

Reticulum (n) : mạng lưới

Mammalian (n) : động vật có vú


Oct 9 – 1

Logical arguments are usually classified as either deductive or inductive, depending on the process
used to arrive at them. In the process of deduction, you begin with some statements, called
premises, which are assumed to be true, and you then determine what else would have to be true if
the premises are true. For example, in mathematics, you can begin with some axioms and then
determine what you can prove to be true given those axioms are true. With deduction, you can
provide absolute proof of your conclusions, since your premises are considered correct. The
premises themselves, however, remain unproven and unprovable; they must be accepted on face
value, or by faith, or for the purpose of exploration.

On the other hand, in the process of induction, you begin with some data, and then determine what
general conclusion(s) can logically be derived from that data. In other words, you determine what
theory or theories could explain the data. For example, you note that the probability of becoming
schizophrenic is greatly increased if at least one parent is schizophrenic, and from that you conclude
that schizophrenia may be inherited. That is certainly a reasonable hypothesis given the data. Note,
however, that induction does not prove that the theory is correct. There are often alternative
theories that are also supported by the data. For example, the behavior of the schizophrenic parent
may cause the child to be schizophrenic, not the genes. What is important in induction is that the
theory does indeed offer a logical explanation of the data. To conclude that the parents have no
effect on the schizophrenia of the children is not supportable given the data, and would not be a
logical conclusion.

Both deduction and induction by themselves are inadequate for a scientific approach. While
deduction gives absolute proof, it never makes contact with the real world; there is no place for
observation or experimentation - no way to test the validity of the premises. And, while induction is
driven by observation, it never approaches actual proof of a theory. Accordingly, a synthesis of
these two logical approaches is required for an actual scientific method.

1. The author’s primarily concerned with

A. describing two modes of constructing a logical argument


B. explaining two processes while evaluating them on a specific criterion
C. discussing how neither one of the two mentioned processes is relevant for a scientific approach
D. establishing how logical arguments are more or less flawed, no matter through which process
they are arrived at
E. establishing the supremacy of one process over the other

Partial Scope. This choice barely captures the scope of the first two paragraphs and fails to account
for the final one. In addition to describing the two processes, the author also explains them in detail
and then evaluates them in the end with respect to how they fare on a particular criterion.

2. Which one of the following statements can be inferred about the induction process of
arriving at a logical argument?

A. There is no way to be certain whether the theories it provides are fully logical.
B. A theory arrived at through it is not likely to be the only possible explanation of the
observed facts
C. One can provide a hundred percent proof for the conclusion drawn through it.
D. The conclusion arrived at through it are very likely to be causal in nature.
E. One of the reasons it fails to provide a single theory is that no two individuals are likely to
interpret a given set of information in the same way.

3. Which one of the following statements is true as per the information given in the
passage?

A. Deduction as a process does not allow for experimentation.


B. There are only two ways to classify a logical argument.
C. The conclusions arrived at through the process of induction do not take in to account any actual
reference points.
D. If the premises of the arguments derived through the deduction process are tested, the validity
of most such arguments will be jeopardized.
E. The process of deduction takes in to account the possibility that the basis of the conclusion is
questionable.

deductive (adj) : suy diễn

inductive (adj) : quy nạp

axiom (n) : định lý

schizophrenic (adj) : tâm thần phân liệt


Oct 9 – 2

The international team of scientists, whose research was conducted under the European
Commission-funded CARBO-Extreme Project, contends that extreme climate events, such as
droughts, heat waves and storms, weaken the buffer effect that terrestrial ecosystems have on the
climate system. The researchers discovered that drought has a particular capacity to reduce the
amount of carbon that is absorbed by forests, meadows and agricultural land. Their findings suggest
that by diminishing the natural environment’s ability to absorb carbon, extreme weather
simultaneously results from, and perpetuates, climate change. During the course of their study, the
CARBO-Extreme team, headed by Dr Reichstein, examined how extreme weather affects forests,
bogs, grassy landscapes and arable areas across the planet. The team, whose findings have been
published in the latest issue of the journal “Nature”, enlisted the help of complex computer models
to calculate the global effect of extreme climate events on the carbon balance. They discovered that
drought, as opposed to heat, has the most pronounced effect on an ecosystem’s ability to remove
carbon dioxide (CO2) from the atmosphere. As climate change continues to gather pace, many
experts believe that extreme weather events will occur more frequently than they do at present. If
the CARBO-Extreme team is correct and extreme weather is a significant driver of climate change,
this relationship could turn out to be a self-fulfilling prophesy. Unfortunately, it is very difficult for
scientists to categorically state whether or not extreme weather is becoming more frequent. Whilst
there is a great deal of evidence to suggest that heat waves and droughts will increase in frequency
over the course of the next century, data pertaining to other extreme climate events remains
inconclusive.

1.The author of this passage is primarily concerned with

a) providing a list of evidences that suggests a correlation between drought and climate change
b) resolving a contention between two schools of thought regarding a perceived natural occurrence
c) arguing for a change in National policies to incorporate a more ecologically conscientious view to
the use of natural resources
d) evaluating a theory put forth by a team of scientists regarding a possible
phenomenon
e) detailing the research findings of the CARBO-Extreme team

2.Which of the following can be best inferred from the passage?

a) Increase in droughts is the cause for extreme climate changes.  can’t confirm the accuracy

If the CARBO-Extreme team is correct and extreme weather is a significant driver of climate change,
this relationship could turn out to be a self-fulfilling prophesy

) Extreme climate changes are the sole causative factors for drought and other extreme climate
events
c) Accumulation of carbon-dioxide in the environment can cause climatic changes that
are deleterious
d) Apart from droughts, other extreme climate events have no apparent connection with climatic
change.
e) Aquatic ecosystems have a less pronounced mitigatory effect on climate change than do
terrestrial ecosystems
3.The journal “Nature” is introduced in the 2nd paragraph primarily to

a) cite the source used by the CARBO-extreme team to substantiate their hypotheses.
b) forestall potential criticisms that might call into question the authenticity of the teams findings
c) provide a recorded instance of the teams work from which information about the
teams findings could be obtained.
d) show the magnitude to which the teams research findings have had an impact.
e) highlight the fact that - extreme climate change is an issue that even the common-man has
become concerned about.

4.Which of the following best describes the organization of the passage

a) An opinion is discussed; evidences suggesting the fallaciousness of the opinion are then provided.
b) A phenomenon is discussed, this phenomenon is then compared to another to understand the
similarities shared.
c) A teams perspective about a natural occurrence is discussed; some evidence
suggesting the validity of the theory are provided.
d) An explanation to a phenomenon is provided; evidences provided to substantiate this explanation
are evaluated.
e) An assumption is put forth and the validity of this assumption is analyzed in light of new scientific
findings.

Prophesy (v) : đoán trước, tiên tri

Self-fulfilling (adj) : tự hoàn thành

Pertain (v) : liên quan

Inconclusive (adj) : không kết luận, không đi đến kết quả

Substantiate (v) : chứng minh

Obtain (v) : kiếm được, thu được, thông dụng

Put forth (v) : đặt ra


Oct 10 – 1

American companies may find the solution to their performance related problems in their own
backyard. A recently conducted independent study shows that in the business world, social and
political skills have become the real key to getting ahead in organisations, skills that successful
managers use to their advantage. The study found out that successful managers, those who get
promoted relatively quickly vis-à-vis effective managers, perform day to day activities that are more
or less dissimilar to the ones conducted by effective managers or those who have satisfied,
committed subordinates, in addition to high performing units. Accordingly, it comes as no surprise
that networking, which consists of socializing or politicking and interacting with others, was one
activity that, out of the core four activities among the plethora of activities performed by managers,
topped the list for successful managers but was ranked the lowest in the list of activities performed
by the effective ones. Indeed the findings of the study do not negate the reality that there are
managers who strike a balance between the activities performed by both types of managers and
hence are successful and effective at the same time, but the meagre percentage such managers
formed of the study’s sample, barely ten percent, affirms the general divide between successful and
effective managers.

These findings clearly belie the traditional assumption typically suggested by formal personnel
policies that promotions are based purely on performance. In effect, the study’s implications affirm
the cynical, yet what now seems real, view that people who are not necessarily the most
accomplishing in terms of performing well in the other three key activity areas, namely
communication, traditional management, and human resource management, are being promoted to
the top level. Therefore, American companies looking to improve their performance and productivity
need to ensure that formal rewards, especially promotions, are tied to performance. This way
companies will be promoting a work-culture that turns effective managers in to successful managers
and gives the currently successful managers a chance to effectively focus on productivity and not
just on socializing and politicking.

1. Each of the following can be inferred from the passage EXCEPT

A. Managers in general perform some activities that are more important than others.
B. Successful managers are inherently good at networking whereas effective managers
are not.
C. Some managers do not fit exclusively in one of the two categories.
D. Some company policies are inconsistent with the actual workings of the company.
E. American companies looking to enhance their performance must look within their set-up for scope
of improvement.

2. The author is primarily concerned with

A. discussing a trend and its implications


B. criticising some companies for an unfair practice followed by them
C. evaluating a situation and warning against its possible implications
D. describing the findings of a study
E. uncovering the gap between the perception and the reality of a situation

The author discusses the trend observed in American companies; the trend is that some managers
(successful ones) not performing better than others (effective managers)on most of the four core
activities are promoted relatively quickly. This trend is given to us in the form of the study findings.

The second paragraph talks about the implications of the findings/trend in terms of what they mean
for the current situation in companies and how companies can use this information to institute
measures enhancing their productivity and performance.

3. Which of the following is mentioned in the passage?

A. Socializing and politicking are activities not worthy of a manager's time.


B. The study's findings shed light on many unstated assumptions that are part of some of the
policies followed by American companies.
C. Out of the four core activities, it is communication in which effective managers perform the best.
D. There is a gap between what appraisal related policies suggest and what the actual
case is.
E. American companies looking to improve their performance and productivity need to ensure that
the most effective managers are also promoted once in a while.

These findings clearly belie the traditional assumption typically suggested by formal personnel
policies that promotions are based purely on performance.

4. Which of the following most aptly describes the function of the second paragraph?

A. To criticize the false picture companies portray with respect to appraisals


B. To close the gap between perception and reality

Covers only the first two sentences of the last paragraph and not the whole scope of the paragraph.

C. To take forward the discussion on the strict measures American companies need to take
D. To discuss the implications of a study
E. To motivate a kind of managers to become successful

Belie (v) : từ chối, mâu thuẫn

Plethora (n) : tình trạng quá nhiều

Appraisal (n) : thẩm định, sự đánh giá


Oct 10 – 2

As scientists developed the Standard Model of physics (the generally accepted framework that
explains how subatomic particles interact) in the 1960s and 1970s, they began to ask: How do
subatomic particles acquire mass? The Standard Model gave satisfactory results in other areas in
which it had been tested, but the mathematics behind the model seemed to prohibit subatomic
particles from having any mass at all, which meant that the model was incomplete. Scientists
speculated that a mechanism must exist that causes such particles to gain mass, but does not
violate the principles of the Standard Model. They theorized that a field known as the Higgs field
would allow such a mechanism to function, and that the Higgs field would have an accompanying
Higgs boson particle.

In attempting to find this missing mechanism, physicists have focused on the Higgs boson. If
experiments prove that the Higgs boson exists, then scientists can be satisfied that this part of the
Standard Model is accurate. On the other hand, if results demonstrate that the Higgs boson does
not exist, then scientists will need to consider alternative versions of the theory. Finding the Higgs
boson, however, has proven challenging. If it were similar to lighter, more commonly found bosons,
such as photons, it would be easily identifiable. However, this particle can only be created through
highly energetic particle collisions achieved by massive particle accelerators, and even then, such
particle collisions are just as likely to produce particles other than the Higgs boson. Further
complicating matters, any Higgs bosons that are created will only exist for mere fractions of a
second before quickly decaying, which means that rather than studying the particles directly,
scientists must analyze the patterns of the decay in order to determine whether the particle was in
fact a Higgs boson.

Recent findings have nevertheless been promising. In July 2012, scientists at the European
Organization for Nuclear Research announced that they had discovered evidence of a boson with
energy and other properties consistent with those predicted for the Higgs boson. While the data is
not yet conclusive, if the particle can indeed be shown to be a Higgs boson, then scientists will be
able to take the next step of considering whether its characteristics match those predicted by the
Standard Model.

1. Based on the information presented by the author, it can be inferred that if Higgs
bosons did not exist

A) the current theory of how particles gain mass would have to be adapted

How do subatomic particles acquire mass?

B) subatomic particles would not be able to gain mass


C) the Standard Model would be proved erroneous
D) scientists would be forced to look for another, lighter particle
E) more experiments using highly energetic particle collisions would have to be conducted

2) The primary purpose of the passage is to

A) present a disputed theory and defend its accuracy


B) criticize a conventional model and argue for an alternative theory
C) outline an issue and demonstrate the unlikelihood of resolving it
D) describe a problem and examine its possible solutions
E) contrast two different models and show that one is more accurate

3) The author’s claim that “if experiments prove that the Higgs boson exists, then
scientists can be satisfied that this part of the Standard Model is accurate” would be
most undermined by the discovery that

A) many subatomic particles produce decay signatures that are similar to those predicted for the
Higgs boson
B) in order to create mass, the Higgs field must work with another unidentified
mechanism that requires its own accompanying particle
C) Higgs bosons and photons are both highly energetic and can decay very rapidly when created by
particle accelerators
D) the particle discovered at the European Center for Nuclear Research has some properties that are
not consistent with those predicted for the Higgs boson
E) most collisions created by particle accelerators produce particles other than Higgs bosons, so the
probability of discovering the Higgs boson is very low

4. According to the passage, if Higgs bosons exist, then they possess all of the following
properties EXCEPT

A) a fast decay rate


B) the ability to work in conjunction with the Higgs field
C) a difficulty to be detected if they were created
D) be created by massive particle detectors
E) relatively little mass

Adapt (v) : sửa đổi

Particle (n) : các hạt

Subatomic (n) : hạ nguyên tử

Acquire (v) : thu được, có được

Accelerator (n) : máy ra tốc

Accompany (v) : đi cùng


Oct 11 – 1

In the field of historiography—the writing of history based on a critical examination of authentic


primary information sources—one area that has recently attracted attention focuses on the
responses of explorers and settlers to new landscapes in order to provide insights into the
transformations of the landscape itself has undergone as a result of settlement. In this endeavor
historiographers examining the history of the Pacific Coast of United States have traditionally
depended on the records left by European American explorers of the nineteenth century who, as
commissioned agents of the U.S. government, were instructed to report thoroughly their findings in
writing.

But in furthering this investigation some historiographers have recently recognized the need to
expand their definition of what a source is. They maintain that the sources traditionally accepted as
documenting the history of the Pacific Coast have too often omitted the response of Asian settlers to
this territory. In part this is due to the dearth of written records left by Asian settlers; in contrast to
the commissioned agents, most of the people who first came to western North America from Asia
during this same period did not focus on developing a self-conscious written record of their
involvement with the landscape. But because a full study of a culture’s historical relationship to its
land cannon confine itself to a narrow record of experience, these historiographers have begun to
recognize the value of other kinds of evidence, such as the actions of Asian settlers.

As a case in point, the role of Chinese settlers in expanding agriculture throughout the Pacific Coast
territory is integral to the history of the region. Without access to the better land, Chinese settlers
looked for agricultural potential in this generally arid region where other settlers did not. For
example, where settlers of European descent looked at willows and saw only useless, untillable
swamp, Chinese settlers saw fresh water, fertile soil, and the potential for bringing water to more
arid areas via irrigation. Where other settlers who looked at certain weeds, such as wild mustard,
generally saw a nuisance, Chinese settlers saw abundant raw material for valuable spices from a
plant naturally suited to the local soil and climate.

Given their role in the labor force shaping this territory in the nineteenth century, the Chinese
settlers offered more than just a new view of the land. Their vision was reinforced by specialized
skills involving swamp reclamation and irrigation systems, which helped lay the foundation for the
now well-known and prosperous agribusiness of the region. That 80 percent of the area’s cropland is
now irrigated and that the region is currently the top producer of many specialty crops cannot be
fully understood by historiographers without attention to the input of Chinese settlers as
reconstructed from their interactions with that landscape.
1. Which one of the following most accurately states the main point of the passage?

A. The history of settlement along the Pacific Coast of the U.S., as understood by most
historiographers, is confirmed by evidence reconstructed from the actions of Asian settlers.
B. Asian settlers on the Pacific Coast of the U.S. left a record of their experiences that traditional
historiographers believed to be irrelevant.
C. To understand Asian settlers’ impact on the history of the Pacific Coast of the U.S.,
historiographers have had to recognize the value of non-traditional kinds of
historiographical evidence.
D. Spurred by new findings regarding Asian settlement on the Pacific Coast of the U.S.
historiographers have begun to debate the methodological foundations of historiographer.
E. By examining only written information, historiography as it is traditionally practiced has produced
inaccurate historical accounts.

2. Which one of the following most accurately describes the author’s primary purpose in
discussing Chinese settlers in the third paragraph?

A. To suggest that Chinese settlers followed typical settlement patterns in this region during the
nineteenth century.
B. To argue that little written evidence of Chinese settlers’ practice survives
C. To provide examples illustrating the unique view Asian settlers had of the land
D. To demonstrate that the history of settlement in the region has become a point of contention
among historiographers.
E. To claim that the historical record provided by the actions of Asian settlers is inconsistent with
history as derived from the traditional sources. ( Not mentioned )

3. The passage states that the primary traditional historiographic sources of information
about the history of the Pacific Coast of the U.S. have which one of the following
characteristics?

A. They were written both before and after Asian settlers arrived in the area
B. They include accounts by Native Americans in the area
C. They are primarily concerned with potential agricultural uses of the land.
D. They focus primarily on the presence of water sources in the region
E. They are accounts left by European American explorers.

4. The author would most likely disagree with which one of the following statements?

A. Examining the actions not only of Asian settlers but of other cultural groups of the Pacific Coast of
the U.S. is necessary to a full understanding of the impact of settlement on the landscape there.
B. The significance of certain actions to the writing of history may be recognized by one group of
historiographers but not another.
C. Recognizing the actions of Asian settlers adds to but does not complete the writing of the history
of the Pacific Coast of the U.S.
D. By recognizing as evidence the actions of people, historiographers expand the definition of what
a source is
E. The expanded definition of a source will probably not be relevant to studies of regions
that have no significant immigration of non-Europeans.
5. According to the passage, each of the following was an aspect of Chinese settlers’
initial interactions with the landscape of the Pacific Coast of the U.S. EXCEPT:

A. New ideas for utilizing local plants


B. A new view of the land
C. Specialized agricultural skills
D. Knowledge of agribusiness practices
E. Knowledge of irrigation systems

6. Which one of the following can most reasonably be inferred from the passage?

A. Most Chinese settlers came to the Pacific Coast of the U.S. because the climate was similar to
that which they were familiar
B. Chinese agricultural methods in the nineteenth century included knowledge of
swamp reclamation
C. Settlers of European descent used wild mustard seed as a spice
D. Because of the abundance of written source available, it is not worthwhile to examine the actions
of European settlers.
E. What written records were left by Asian settlers were neglected and consequently lost to scholarly
research.

7. Which one of the following, if true, would most help to strengthen the author’s main
claim in the last sentence of the passage?

A. Market research of agribusinesses owned by descendants of Chinese settlers shows that the
market for the region’s specialty crops has grown substantially faster than the market for any other
crops in the last decade.
B. Nineteenth-century surveying records indicate that the lands now cultivated by
specialty crop businesses owned by descendants of Chinese settlers were formerly
swamp lands.
C. Research by university agricultural science departments proves that the formerly arid lands now
cultivated by large agribusinesses contain extremely fertile soil when they are sufficiently irrigated
D. A technological history tracing the development of irrigation systems in region reveals that their
efficiency has increased steadily since the nineteenth century
E. Whether records compiled over the previous century demonstrate that the weather patterns in
the region are well-suited to growing certain specialty crops as long as they are irrigated.

Descendant (n) : con cháu, hậu duệ

Historiography (n) : biên soạn lịch sử

Endeavor (n) : nỗ lực

Irrigation (n) : thủy lợi

Willow (n) : cây liễu

Untillable (adj) : không thể

Swamp (n) : đầm lầy


Fertile (adj) : phì nhiêu, màu mỡ

Nuisance (n) : phiền toái

Reclamation (n) : khai hoang, khai phá

Prosperous (adj) : thịnh vượng


Oct 11 – 2

The study of the analog position of mental representation has many fascinating branches which help
illuminate the inner workings of our minds and how we perceive images in our mind‘s eye. This
theory points to the link between the time it takes to solve mental problems and their complexity.

In a now-famous study, Stephen Kosslyn asked subjects to imagine an animal, such as a rabbit,
next to either an elephant or a fly. When the image was formed, Kosslyn would ask whether or not
the target animal had a particular attribute. For example, Kosslyn might say, ―elephant, rabbit,‖
and then ―leg.‖ He found that it took subjects longer to answer when the target animal was next to
the large animal than when it was next to the small animal. Kosslyn interpreted this to mean that
subjects had to zoom in on the image to detect the particular feature. Just as one has difficulty
seeing details on small objects, so the subjects could not simply mentally ―see‖ details on the
smaller object in their mental image.

Second, Kosslyn and colleagues demonstrated that the time it takes to scan between two points
depends on the distance between the two points [in a memorized image]. In one experiment,
subjects memorized an array of letters separated by different distances. Kosslyn found that the
farther apart the letters were from each other, the longer it took to answer questions about one of
the letters. One of the principal hypotheses of the analog position of mental representation, which is
the idea that mental processing requires one to move sequentially through all intervening steps to
solve a problem, is that mental images have regular properties.

In a similar experiment, Kosslyn had subjects memorize pictures of objects like a plane or a
motorboat. Then he had them focus on one part of the object (e.g., the motor) and move to
another (e.g., the anchor). He found that the time it took to determine whether the second part was
present depended on the distance between the two parts in the memorized picture.

Using a completely different paradigm, Shepard and Feng tested the amount of time that it would
take for subjects to specify whether two arrows on unfolded blocks matched up. They found a linear
relationship between the number of folds between the arrows and the time it took to make this
judgment, suggesting that subjects went through a discrete series of organized steps in order to
solve this problem.

The final type of experiment showing that mental images have regular properties is perhaps the
most famous: mental rotation experiments. In 1971, Shepard and Metzler tested subjects‘ abilities to
make complex figure comparisons. They presented subjects with a three dimensional ―standard‖
figure and a comparison figure which was either identical to the standard figure, or its mirror image;
the comparison stimulus was rotated, either clockwise or into the third dimension. Shepard and
Metzler found that the time needed to judge whether the comparison stimulus was identical or a
mirror image depended directly on the size of the angle between the target orientation and the
orientation of the standard.

1. According to the way it is presented by the author in the passage, the analog position
of mental representation argues that:

A. mental processing requires one to go sequentially through all intervening steps to


solve a problem.
B. one typically uses short cuts to solve mental problems.
C. it should take longer to solve more complex problems.
D. most problems are not able to be solved by people without help.
E. the closer two points are the more time it takes to mentally scan between them

2. According to the scanning experiments mentioned in the passage, it should take


longer to scan longer distances because the subjects:

A. believe that there is no relationship between distance and time.


B. have to keep time with a metronome set up by the experimenter.
C. form a mental picture of the scene and go through all the intervening positions in the
picture.
D. are tricked by the experimenter into taking a longer time.
E. tend to forget things quickly

3. According to the passage, why does Kosslyn say it takes longer to identify attributes
of objects when they are next to a bigger object than when they are next to a smaller
object?

A. Because one scans objects in order of size from larger to smaller


B. Because the larger object covers the smaller object and one must move it out of the way
C. Because large and small objects have all the same features and so interfere with each other
D. Because one must zoom in to see parts of the smaller object when it is next to a
larger object
E. Because the larger object looks more visually imposing

Analog (adj) : tương tự

Illuminate (v) : biện minh, giải thích

Sequentially (adv) : tuần tự

Intervene (v) : xảy đến, xen vào

Anchor (n) : mỏ neo

Paradigm (n) : mô hình


Oct 12 – 1

Before there were books, before, even, there was the written word in civilization, there must surely
have been stories told. Relating stories to one another is a unique way that we, as humans,
communicate thoughts, needs, desires, and instruction. Whether it be the true story of what
happened on the way to the well yesterday—a story meant to instruct about the latest water
situations—or a dramatic retelling of a long-ago battle—a cautionary tale meant to warn against
unnecessary warfare—stories have the unique ability to bring home information and instruct in a
way a mere recitation of the facts cannot.

The Tale, the Parable, and the Fable are all common and popular modes of conveying instruction—
each being distinguished by its own special characteristics. The true Fable, if it rises to its high
requirements, ever aims at one great end and purpose: the representation of human motive, and
the improvement of human conduct, and yet it so conceals its design under the disguise of fictitious
characters, by clothing with speech the animals of the field, the birds of the air, the trees of the
wood, or the beasts of the forest, that the reader receives the advice without perceiving the
presence of the adviser. Thus the superiority of the counsellor, which often renders counsel
unpalatable, is kept out of view, and the lesson comes with the greater acceptance when the reader
is led, unconsciously to himself, to have his sympathies enlisted on behalf of what is pure,
honourable, and praiseworthy, and to have his indignation excited against what is low, ignoble, and
unworthy.

The true fabulist, therefore, is charged with a most important function. He is neither a narrator, nor
an allegorist, he is a great teacher, a corrector of morals, a censor of vice, and a commender of
virtue. In this consists the superiority of the Fable over the Tale or the Parable. The fabulist is to
create a laugh, but yet, under a merry guise, to convey instruction. Phaedrus, the great imitator of
Aesop, plainly indicates this double purpose to be the true office of the writer of fables.

The Fable partly agrees with, and partly differs from the Tale and the Parable. It will contain, like
the Tale, a short but real narrative; it will seek, like the Parable, to convey a hidden meaning, not so
much by the use of language, as by the skilful introduction of fictitious characters; and yet unlike to
either Tale or Parable, it will ever keep in view, as its high prerogative, and inseparable attribute,
the great purpose of instruction, and will necessarily seek to inculcate some moral maxim, social
duty, or political truth.
The Tale consists simply of the narration of a story either founded on facts, or created solely by the
imagination, and not necessarily associated with the teaching of any moral lesson. The Parable is
the designed use of language purposely intended to convey a hidden and secret meaning other than
that contained in the words themselves; and which may or may not bear a special reference to the
hearer, or reader.

1. The passage suggests that the fable is superior to the parable and the tale for none of
the following reasons EXCEPT:

I. the fable contains a moral lesson within its narrative.


II. the parable‘s message may be too enigmatic for a reader to comprehend.
III. the tale is a chronicle of recent historical events.

A. I only
B. I and II
C. II and III
D. I, II, and III
E. None of the above

The Parable is the designed use of language purposely intended to convey a hidden and secret
meaning other than that contained in the words themselves; and which may or may not bear a
special reference to the hearer, or reader.

2. According to the passage, which of the following is NOT a requirement for a narrative
text to be classified as a fable?

A. Use of fictional characters, such as personified animals and natural objects


B. Inclusion of social, moral, or political references relevant to contemporary readers
C. Constant awareness of and attention to a particular instructional goal
D. Figurative or poetic language to demonstrate the author‘s creative talent
E. Every fable must have a ‗moral‘ at the end

3. Which of the following best characterizes the claim that the fabulist is a ―great
teacher, a corrector of morals, a censor of vice, and a commender of virtue?‖

A. It is an analysis of the importance of the fabulist‘s role in society.


B. It is a conclusion that fabulists should be honoured above writers of parables or tales.
C. It is appreciation for the fabulist‘s ability to multi-task.
D. It advocates increased honour and respect for the fabulist.
E. It suggests that more and more people should become fabulists

The true fabulist, therefore, is charged with a most important function


Oct 12 – 2

Though he left us with numerous great works and, to be sure, is widely regarded as America‘s first
internationally renowned author, Washington Irving‘s sometimes enigmatic tendencies and
techniques have left literary critiques and academics to ponder his motives more than 140 years
after his death. One such trait that raises the proverbial eyebrow of the community of readers and
critiques is Irving‘s repeated, and varied, use of pseudonyms throughout his career.

One of the most well-known female writers to adopt a pen name was George Sand, born Aurore
Dupin in 1804, who became one of the most prolific and admired French authors – female or male –
during the nineteenth century. The true identity of George Sand did not remain a secret for long, for
after 1830 the author used this name in her everydaylife, and close friends commonly referred to
her as ―George

Most portraits of the author as an adult are entitled simply George Sand and make no reference to
her given name. Her son, too, adopted this new last name even though association with his famous
authormother did not bring him any obvious benefits, other than to indicate that his relationship
with his mother was closer than that of his sister. Given that the name ―George Sand‖ is radically
different from Aurore Dupin‘s birth name, many readers have wondered how the author formulated
her masculine pen name.

At least two possible answers spring to mind. The first, as indicated in Curtis Cate‘s biography
George Sand, is that the idea for this pseudonym arose from a collaboration with her first lover,
Jules Sandeau, with whom she co-authored several articles as well as a full-length novel entitled
Rose et Blanche. Since her own literary output was a great success in the 1830s-1850s, she quickly
became known by this name, and began to use her pen-name on a daily basis.

By continuing to use the name initially assigned to collaborative writings with her lover, perhaps
Aurore hoped to maintain her connection to Sandeau. Perhaps she fondly remembered their time
together and wished to have a permanent reminder of their relationship. Or perhaps she simply
realized that it would be much more expedient to continue to write under a name which was already
familiar to her audience thanks to the joint works she and Sandeau had published.

Given that George Sand began writing under this masculine name at around the same time as she
began to roam around Paris in pants and a jacket – typically male clothing – it is not hard to
understand why she chose a masculine pseudonym, since, like her choice of clothes, this male
identity gave her more freedom of expression, both literally and figuratively. Writing under a false
name allowed her to distance parts of her character – her roles as wife, mother, and lover – from
the creative and literary parts that formed the basis for her role as an author. Using a male name
set her apart and added to her persona as an unusual and fascinating woman. And in the end, the
reason why she chose this particular pen-name is not nearly as important as the vast quantity of
writing – articles, letters, novels, plays – that forms her legacy to the field of French literature.

1. The author‘s attitude towards the use of male pseudonyms by female authors as
noted in the passage can best be described as:

A. skeptical of the usefulness of pseudonyms.


B. critical of the women‘s adoption of a male name.
C. appreciative of female authors‘ efforts to be published at any cost.
D. intrigued by the creation of a pseudonym.
E. disillusionment at the idea of discrimination against women

2. According to the passage, the following were all possible reasons for George Sand to
create a pseudonym EXCEPT:

A. she began publishing collaborative works with Jules Sandeau.


B. her new name reflected important parts of her life.
C. she was not able to publish any works under her own given name.
D. the works published under her pen name sold well.
E. the male identity gave her greater freedom of expression

3. What is the main purpose of the passage?

A. to criticise females writers who use male names


B. to discuss why some female writers use male names
C. to applaud female writers for using male names
D. to encourage female writers to use their own names
E. to suggest that male writers should use females names

4. With which of the following statements would the author most likely agree?

A. Aurore Dupin should have written works under her own name once the secret of her pseudonym
was revealed.
B. By writing under a pseudonym, George Sand created for herself a new identity which
allowed her to transcend the limitations of society.
C. George Sand owed her early success to her partner, Jules Sandeau.
D. The choice of a masculine pseudonym was restrictive for George Sand and forced her to live as a
man throughout her life.
E. The use of pseudonyms in general by authors of both the sexes should be avoided

Renowned (adj) : nổi tiếng, trứ danh

Ponder (v) : suy ngẫm

Pseudonym (n) : bút danh

Pen name (n) : tên giả

Intrigue (v) : hấp dẫn, khêu gợi

Disillusionment (n) : vỡ mộng, hết mê muội, giác ngộ


Oct 13 – 1

Information is the essence of universe and means distinction between things. It is the very basic
principle of physics that distinctions never disappear even though they might get scrambled or
mixed away even after a seemingly irreversible change – say a magazine gets dissolved into pulp at
a recycling plan, the information on the pages of the magazines will be re-organized and not
eliminated and in theory the decay can be reversed; the pulp reconstructed into words and
photographs. The only exception to this principle in physics is if the magazine were thrown into a
black hole, a singular object in this regard, since nothing can emerge out of it after all. Even after
Stephen Hawking showed in 1975 that black holes can radiate away matter and energy, the
radiation seemed devoid of any structure, indicating that all information is lost in a black hole – a
conclusion that has been hotly contested by physicists all over the world who argue that the entire
structure of theoretical physics will disintegrate once you accept the notion that information can be
lost, even if in a black hole.

Even though Hawking was not easily convinced, the physicists adopted a new theory called the
holograph principle that states that when an object falls inside a black hole the stuff inside it may be
lost but the objects information may be imprinted on the surface of black hole and with the right
tools you may reconstruct the magazine from the black hole just as you would have reconstructed it
from the pulp. This principle which may sound like an accounting trick has some serious implications
if true. It implies that all information about 3 dimensional objects is stored in 2 dimensions and that
there is a limit to how much information can be stored on a given surface area. While this theory
plugs a key gap in Hawkins assertion its corollaries spring some interesting implications that may
have a tough time standing up to the scrutiny.

1.According to the passage, prior to 1975 it was believed that black holes were unique
because:

A.They had the ability to absorb and incinerate everything.


B.They could cause changes that were not easily reversible.
C.Once an object goes inside the black hole, it is impossible to extract and reconstruct
it.
D.Their existence was hotly contested by physicists all over the world.
E. No one had actually detected a black hole.

2.Why does the author imply that the holographic principle “may sound like an
accounting trick”?

A.Because it was added as an afterthought or compensation to ensure that laws of


theoretical physics don’t fall apart.
B.Because it was added by scientists who worked more like accountants rather than scientists.
C.Because Stephen Hawking who proved that black holes radiate was not entirely convinced.
D.Because it puts a limit on how much information can be stored on the surface of a black hole.
E.Because it does not seem likely that information about a 3-dimensional object can be stored in 2
dimensions.

a conclusion that has been hotly contested by physicists all over the world who argue that the entire
structure of theoretical physics will disintegrate once you accept the notion that information can be
lost, even if in a black hole.

3.Which of the following best describes author’s feelings regarding Holograph principle?

A.One of relief.
B.One of despair.
C.One of skepticism.
D.One of joy.
E.One of frustration.

4.According to the passage, the hotly contested debate about black holes was:

A.whether black holes radiate or not.


B.how easy is it to reconstruct an object that has fallen in a black hole.
C.what are the tools required to reconstruct an object that has fallen in a black hole.
D.whether an object’s information when it falls in a black hole is lost to an extent that
the object can never be reconstructed.
E.whether the information about an object falling in the black hole is stripped out during the fall.

Scramble (v) : xô đẩy

Irreversible (adj) : không thể đảo ngược

Dissolve (v) : giải thể, hủy bỏ, làm tan

Pulp (n) : bột giấy

Radiate (v) : tỏa ra, bắn ra, phóng xạ

Devoid (adj) : không có

Disintegrate (v) : tan rã, phân tán

Holography (n) : hình ba chiều

Imprint (v) : in dấu

Corollary (n) : hệ luận, hệ quả

Spring (v) : làm bung lên, nhảy

Scrutiny (n) : xem xét kỹ lưỡng

Afterthought (n) : suy nghĩ lại


Oct 13 – 2

The palette of sights and sounds that reach the conscious mind are not neutral perceptions that
people then evaluate: they come with a value already tacked onto them by the brain‘s processing
mechanisms. Tests show that these evaluations are immediate and unconscious and applied even to
things people have never encountered before, like nonsense words: "juvalamu" is intensely pleasing
and "bargulum" moderately so, but "chakaka" is loathed by English-speakers. These conclusion
come from psychologists who have developed a test for measuring the likes and dislikes created in
the moment of perceiving a word, sound or picture. The findings, if confirmed, have possibly
unsettling implications for people‘s ability to think and behave objectively. This is all part of
preconscious processing, the mind‘s perception and organization of information that goes on before
it reaches awareness—these judgments are lightning fast in the first moment of contact between
the world and the mind.

Some scientists disagree with the claim that virtually every perception carries with it an automatic
judgment, though they, too, find that such evaluations are made in many circumstances. These
scientists believe that people don‘t have automatic attitudes for everything, but rather, for areas of
interest.

In responding to a stimulus, a signal most likely travels first to the verbal cortex, then to the
amygdala, where the effect is added, and then back. The circuitry involved can do all this in a
matter of a hundred milliseconds or so, long before there is conscious awareness of the word. This
creates an initial predisposition that gets things off on a positive or negative footing. These reactions
have the power to largely determine the course of a social interaction by defining the psychological
reality of the situation from the start.

The "quick-and-dirty" judgment tends to be more predictive of how people actually behave than is
their conscious reflection on the topic. This may represent a new, more subtle tool for research on
people‘s attitudes, allowing scientists to assess what people feel without their having any idea of
what exactly is being tested. You could detect socially sensitive attitudes people are reluctant to
admit, like ethnic biases because these 35 automatic judgments occur outside a person‘s awareness,
as part of an initial perception. They are trusted in the same way senses are trusted, not realizing
that seemingly neutral first perceptions are already biased.

Conclusions from both camps are based on a method that allows them to detect subtle evaluations
made within the first 250 milliseconds—a quarter of a second—of perception of words. The
measurement of liking can be made outside the person‘s awareness because if the first word is
presented in less than a quarter of a second the reaction to it never registers in consciousness,
though it can still be read.

1. According to the passage and with regards to words like bargulum, juvalamu, and
chakaka, "preconscious processing" (line 12) would most influence which of the
following?

A. Subconscious memories concerning traumatic childhood events


B. Perception of a stranger on first sighting
C. Formulation of arguments after intense research
D. Thought processes involved in creating an intricate novel
E. Reuniting with one‘s children after a long trip overseas

2. Scientists that disagree with the idea that humans place a value on all perceptions
would most likely agree with which of the following statements?

A. Most perceptions receive a value due to a familiarity with the stimulus.


B. The mind cannot possibly interpret information in an interval as short as a quarter of a second.
C. Preconscious processing would have no effect on behaviour patterns.
D. The senses are not used when placing a value on stimuli presented during an experiment.
E. Some perceptions are too valuable to actually put a value on

3. Based on information in the passage, in the author's view, which of the following
statements is NOT true?

A. Information regarding external stimuli is processed so quickly that it does not become part of our
conscious awareness.
B. Automatic judgments occur on stimuli with which there is great familiarity.
C. Nonsense words have little or no effect on a person‘s mood.
D. Ethnic biases may be influenced by attitudes of which we are unaware.
E. The measurement of liking could be made outside of a person‘s awareness

Palette (n) : bảng màu

Loath (v) : không ưa, ghê tởm, không thích

Verbal cortex (n) : vỏ não

Amygdala (n) : hạch hạnh nhân, là một trong hai nhóm nhân hình quả hạnh nhân nằm ở giữa sâu
bên trong thùy thái dương của não ở các loài động vật có xương sống phức tạp, bao gồm cả con
người.

Circuitry (n) : mạch điện

Predisposition (n) : khuynh hướng

Subtle (adj) : tế nhị, khôn khéo

Reluctant (adj) : lưỡng lự, không sẵn lòng, miễn cưỡng

Liking (n) : ý thích


Oct 14 – 1

Feminism is an essentially pluralist movement. Early historians of the feminist movement claim that
the movement was borne from the experiences of every woman who has resisted or tried to resist
domination. From this proposition, three major premises follow. First, feminism places great
importance on subjective experience. This implies that much of feminist theory is inductive, moving
from the particularities of an individual’s perception of the world to general statements about the
nature of the world. Second, feminism is not easily reducible to generalities. The diversity of
viewpoints incorporated into the movement resists systematization, but the difficulty of forming a
coherent whole from the parts of the theory should be viewed as detrimental. Third, feminism
attempts to transform the personal into the political. The individual’s experiences of gender
domination transcend the typical boundaries that divide the public and private sphere. Predictably,
feminist political theory assumes the intermingling of these domains.

Differences among feminist theories arise from two major sources. The first is from the differences
inherent in various interpretations of events and the second is from the differences in the nature
and character of experiences. Varieties of interpretations led to the establishment of several major
schools of feminist thought, including liberal feminists, radical feminists, and socialist feminists, while
the diversity of experiences accounts for the cultural feminist, psychoanalytic feminist, and black
feminist movements. These experiential movements should not be seen as complete, for as the
feminist ethos takes root in other countries with different traditions, new theoretical perspectives will
develop.

1. The passage is primarily concerned with

A. evaluating a theory
B. tracing the history of a movement
C. suggesting the likely progression of an idea
D. describing a perspective
E. rejecting a criticism

2. Which of the following would most undermine the author’s conclusion in the final
paragraph?

A. Although the traditions in other countries may differ, the experiences of women in
those countries are fairly similar.
B. The societal structure of some countries will make it difficult for feminism to take hold.
C. Most feminist scholars identify themselves with interpretationbased schools of feminism.
D. Because women’s experiences in society are always changing, so too will the theoretical
perspectives gained from them.
E. Many other countries already have the seeds of a feminist movement.

3. The author implies which of the following about feminism?

A. It offers a systematic and coherent view of the world.


B. Its primary characteristic is its rejection of male hierarchical structures.
C. It seeks to intertwine the political and theoretical worlds.
D. It values diversity.
E. It uses the experiences of women to attempt to transform the world.
The diversity of viewpoints incorporated into the movement resists systematization, but the difficulty
of forming a coherent whole from the parts of the theory should be viewed as detrimental.

Pluralist (n) : đa nguyên

Borne (adj) : chịu đựng

Proposition (n) : mệnh đề

Premise (n) : cơ sở

Particularity (n) : đặc thù

Reducible (adj) : giảm

Generality (n) : phần đông, tổng quát

Coherent (adj) : mạch lạc, dính liền

Detrimental (adj) : bất lợi

Transcend (v) : vượt qua

Intermingle (v) : xen kẽ, pha trộn

Interpretation (n) : diễn giải, thuyết minh, giải thích

Radical (n) : cấp tiến

Psychoanalytic (adj) : phân tâm học

Ethos (n) : tập quán, đạo đức, phong tục

Value (v) : coi trọng

Intertwine (v) : đan xen, kết với nhau


Oct 14 – 2

The internal temperatures of an animal that uses a variety of physiological and behavioral
mechanisms to regulate its body temperature do not vary by more than a few degrees centigrade
over the animal’s lifespan. At temperatures below which such an endotherm loses heat to the
environment, an endotherm requires heat production, and therefore metabolism, increases and,
obversely, at temperatures above which it gains heat from the environment, an endotherm requires
heat dissipation. However, these processes are not achieved in an equally effective manner; the
responses used by an endotherm to decrease its body temperature contribute to the situation the
animal is attempting to surmount.

Even though such thermo-regulation allows endotherms to maintain an internal temperature not
equal to the temperature of the air, the temperature of the air is not an insignificant factor in
determining the animal’s metabolic functioning, which is directly correlated with the amount of
resources the animal needs. Moreover, an endotherm does not have an infinite capability to
dissipate or produce heat, and enough exposure to extreme temperatures can ultimately lead to the
animal’s death. However, the effects of temperature on endotherms far exceed in scope those which
directly affect the animals themselves, as temperature is a vital factor in determining the amount of
energy available in any given ecosystem and all the biotic elements, which all interact with one
another, are subjected to the same conditions.

During the Paleocene-Eocene maximum, when the overall temperature of the planet increased
between four to seven degrees centigrade over 20,000 years, almost half of the benthic foraminifera
disappeared while several modern endothermic species appeared. Data collected in the modern era,
however, between the beginnings of the industrial revolution and the present day, show that climate
change has had even more drastic and detrimental results. Because the time span covered by these
studies is relatively short, scientists cannot discriminate with certainty between naturally occurring
phenotypic plasticity and evolutionary responses to climate or even determine which results occur in
reaction to climate change and which occur from any number of other factors that vary over time
such as habitat succession, density-dependence or predation.

1. What is the function of the second paragraph in relation to the passage as a whole?

A. It discusses how a process described in the first paragraph occurs when an additional factor is
considered, a factor which, when altered, creates negative effects on certain creatures.
B. It explains the mechanisms which creatures mentioned in the first paragraph have at
their disposal to cope with the process mentioned in the third paragraph.
C. It clarifies a possible misconception that could be deduced from the information in the first
paragraph and explains why the results presented in the third paragraph occur.
D. It presents facts that allow the author to argue that a biological theory presented in the first
paragraph is refuted by the events described in the third paragraph.
E. It provides more details about an argument about the relative efficiencies of two mechanisms
presented in the first paragraph, facts which clarify the processes that allow the events described in
the third paragraph to occur.
2. Which of the following best states the main idea of the passage?

A. The metabolic functioning and resource consumption of an endotherm, which regulates its own
body temperature, are determined by air temperature which, when altered, creates drastic effects
on those animals.
B. Although, in the past, changes in climate had both negative and positive effects, today climate
change lacks any positive effects on animals who regulate their own internal temperatures by
dissipating and producing heat.
C. Climate change in the past and present has influenced the life course of animals because their
metabolic functioning and overall resource consumption are affected by the temperature of the air
surrounding them.
D. Since animals who regulate their own body temperature are still affected by the air temperature
in their environments, significant changes in that air temperature have affected those animals in the
past.
E. Because an animal which regulates its own body temperature by producing or
dissipating heat is still susceptible to the surrounding air temperature, modern changes
in climate have proved damaging to those animals.

Data collected in the modern era, however, between the beginnings of the industrial revolution and
the present day, show that climate change has had even more drastic and detrimental results.

Endotherm (n) : nhiệt nội

Surmount (v) : vượt qua, khắc phục

Thermo (n) : nhiệt

Insignificant (adj) : tầm thường

Scope (n) : phạm vi

Benthic (adj) : sinh vật đáy

Foraminifera (n) : chuẩn đoán

Discriminate (v) : phân biệt

Succession (n) : sự nối tiếp

Predation (n) : ăn thịt

Disposal (n) : sự xử lý, hành động

Susceptible (adj) : mẫn cảm, ảnh hưởng xấu


Oct 15 – 1

As companies tend to innovate faster than their customers’ needs evolve, most organizations
eventually end up producing products or services that are actually overly sophisticated, extremely
expensive, and rather complicated for many customers in their market. These innovations fall under
the category of sustaining innovations, innovations that simply improve existing products.
Companies pursue sustaining innovations at the higher tiers of their markets because this is what
has historically helped them succeed: by charging the highest prices to their most demanding and
sophisticated customers at the top of the market, companies achieve the greatest profitability.
However, by doing so, companies unwittingly open the door to another category of innovations -
“disruptive innovations”. In contrast to sustaining innovations, disruptive innovations lie at the
bottom of the market. They are made not only by harnessing new technologies but also by
developing new business models and exploiting old technologies in new ways.

An innovation that is disruptive allows a whole new population of consumers at the bottom of a
market access to a product or service that was historically only accessible to consumers with a lot of
money or a lot of skill. Personal computers, for instance, were disruptive innovations because they
created a new mass market for computers - previously, expensive mainframe computers were sold
only to big companies and research universities. Characteristics of disruptive businesses, at least in
their initial stages, can include: lower gross margins, smaller target markets, and simpler products
and services that may not appear as attractive as existing solutions when compared against
traditional performance metrics. Because these lower tiers of the market offer lower gross margins,
they are unattractive to other firms moving upward in the market, creating space at the bottom of
the market for new disruptive competitors to emerge.

1. Which of the following statements is supported by the information given in the


passage?

A. A disruptive business is less likely to face competition in the market than a business pursuing
sustaining innovations.
B. Companies following innovations that are improvements of existing products do not
knowingly leave market space for disruptive businesses to emerge.
C. Most technological innovations can be considered disruptive innovations.
D. It is likely that to produce products, disruptive businesses engage more in using old technologies
in new ways than using new technologies.
E. Companies typically pursuing sustaining innovations do not find disruptive products attractive
since these products are typically not profitable.

2. The author’s primarily concerned with

A. differentiating between two kinds of innovations


B. explaining two kinds of innovations while focusing more on one
C. explaining how different kinds of innovations roll out different kinds of products in the market
D. describing how newer needs are created in the market even with the same old technology
E. Highlighting the difference in the profits that companies can gain from two different kinds of
innovations
3. The passage supports which of the following statements about disruptive
innovations?

A. They are not expensive to produce.


B. They do not add as much research value as sustaining innovations do.
C. Because they are targeted at smaller market, they never become mass scale products.
D. They make a service easier to consume for consumers who possess limited
specialized knowledge in the field the innovations are made in.
E. The products under them gradually move to sustaining innovations.

Unwittingly (adv) : vô tình

Sustaining innovation (n) : đổi mới bền vững

Disruptive innovation (n) : đổi mới đột phá

Harness (v) : khai thác

Mainframe (n) : máy tính lớn

Gross margin (n) tỉ suất lợi nhuật gộp

Metric (n) : số liệu

Knowingly (adv) : cố ý
Oct 15 – 2

The public generally expects reputable, business-related newspapers and magazines to be valuable,
objective and well-researched sources of information. My research suggests, however, that most
publications that address current financial-related topics distort the facts. In some newspapers, for
example, journalists report verbatim what they learned in an interview or read in another
publication, only to write a different article days later, completely refuting what they originally wrote.
In essence, the value of reporting by many journalists, even in what some consider the most
reputable publications worldwide, is exceptionally limited by their interests in exerting the necessary
effort to conduct independent due diligence on the topics upon which they report. Further, many
financial reporters have little to no formal business education or practical experience, and thus lack
ability to conduct meaningful research. Although the public continually holds these publications in
high regard and depends upon their published information to make informed and current decisions,
I contend such journalism alone is of little true value for significant financial-related decisions and, in
many cases, a meaningful detriment to investors' decisions.

One argument against my contentions asserts that, by nature, business-related newspapers and
magazines must report on a large breadth of information with limited resources and, therefore, only
publish information that is cursory in nature, dependent largely upon unverified information gleaned
through interviews, and without the foundation of significant independent research. Some even
claim that the public generally recognizes this and reads all publications with a firm understanding of
these limitations. Yet, substantial evidence exists to the contrary and recent events have proven the
public's dependence on information generally learned through business-related publications. For
example, even after a start of a recent extreme decline in the global financial markets, many
financial reporters who lacked a fundamental understanding of the market dynamics continued to
publish stories based on the false assumption that markets would continue to experience positive
growth indefinitely. Many in the general public depended upon both the specific content and the
mood• of such stories in making important investment decisions, only to learn the truth from the
very same reporters in stories written a short time later, after they had already lost significant value
in their investments. Given this, an ongoing, careful review and publication of the credentials,
knowledge base and practical experience of reporters of reputable business newspapers and
magazines are certainly warranted.

1. In the second paragraph, the author mentions an example concerning a recent


decline in the global financial markets most likely in order to

A. suggest that the decisions of the general public often are based solely upon the reporting in
business-related publications.
B. counter the claim that the general public recognizes the limitations of reporting in
business-related publications and takes such limitations into account when reading such
reports.
C. suggest that the general public tends to interpret the contents of business-related publications
more literally than do professional investors.
D. demonstrate that business-related publications carry political messages meant to influence their
readers.
E. prove that business-related publications are not limited with regard to their objectiveness and
accuracy.
2. Which of the following would most logically be the topic of the paragraph
immediately following the passage?

A. Specific ways to evaluate the abilities of business reporters


B. The centrality of the business reporter's role in society
C. Alternative sources of information to financial-related newspapers and magazines
D. Other non-financial-related subjects on which business newspapers and magazines report
E. Ways in which business publications influence the political attitudes of the general public

The last sentence of the passage states, an ongoing, careful review and publication of the
credentials, knowledge base and practical experience of reporters are certainly warranted•.
Therefore, ways to evaluate reporters' capabilities is a sound continuation of the passage, making
(A) the correct answer choice. Further, this answer choice is the only choice consistent with the
strong, contentious tone of the passage. Choices B, C, D, and E all have scopes that are inconsistent
with the passage.

3. The primary purpose of the passage is to

A. describe in detail one example regarding the impact of financial reporting in business-related
newspapers on the investment decisions of the general public.
B. describe changes that should be made to the way in which journalists conduct their reporting.
C. discuss the difficulty business-related newspapers and magazines have in accurately reporting
financial-related topics.
D. argue that business-related publications do not publish valuable and well-researched
information.
E. summarize ways in which some business-related publications give distorted pictures of financial
topics.

Reputable (adj) : có uy tín

Distort (v) : xuyên tạc

Verbatim (adv) : nguyên văn, đúng y từng chữ

Essence (n) : bản chất

Exceptionally (adv) : đặc biệt

Exert (v) : gắng sức

Detriment (n) : thiệt hại


Oct 16 – 1

After thirty years of investigation into cell genetics, researchers made startling discoveries in the
1960s and early 1970s which culminated in the development of processes, collectively known as
recombinant deoxyribonucleic acid (rDNA) technology, for the active manipulation of a cell’s genetic
code. The technology has created excitement and controversy because it involves altering DNA—
which contains the building blocks of the genetic code.

Using rDNA technology, scientists can transfer a portion of the DNA from one organism to a single
living cell of another. The scientist chemically “snips” the DNA chain of the host cell at a
predetermined point and attaches another piece of DNA from a donor cell at that place, creating a
completely new organism.

Proponents of rDNA research and development claim that it will allow scientists to find cures for
disease and to better understand how genetic information controls an organism’s development.
They also see many other potentially practical benefits, especially in the pharmaceutical industry.
Some corporations employing the new technology even claim that by the end of the century all
major diseases will be treated with drugs derived from microorganisms created through rDNA
technology. Pharmaceutical products already developed, but not yet marketed, indicate that these
predictions may be realized.

Proponents also cite nonmedical applications for this technology. Energy production and waste
disposal may benefit: genetically altered organisms could convert sewage and other organic material
into methane fuel. Agriculture might also take advantage of rDNA technology to produce new
varieties of crops that resist foul weather, pests, and the effects of poor soil.

A major concern of the critics of rDNA research is that genetically altered microorganisms might
escape from the laboratory. Because these microorganisms are laboratory creations that, in all
probability, do not occur in nature, their interaction with the natural world cannot be predicted with
certainty. It is possible that they could cause previously unknown perhaps incurable, diseases. The
effect of genetically altered microorganisms on the world’s microbiological predator-prey
relationships is another potentially serious problem pointed out by the opponents of rDNA research.
Introducing a new species may disrupt or even destroy the existing ecosystem. The collapse of
interdependent relationships among species, extrapolated to its extreme, could eventually result in
the destruction of humanity.

Opponents of rDNA technology also cite ethical problems with it. For example, it gives scientists the
power to instantly cross evolutionary and species boundaries that nature took millennia to establish.
The implications of such power would become particularly profound if genetic engineers were to
tinker with human genes, a practice that would bring us one step closer to Aldous Huxley’s grim
vision in Brave New World of a totalitarian society that engineers human beings to fulfill specific
roles.
1. In the passage, the author is primarily concerned with doing which one of the
following?

(A) explaining the process and applications of rDNA technology


(B) advocating continued rDNA research and development
(C) providing evidence indicating the need for regulation of rDNA research and development
(D) summarizing the controversy surrounding rDNA research and development
(E) arguing that the environmental risks of rDNA technology may outweigh its medical benefits

2. According to the passage, which one of the following is an accurate statement about
research into the genetic code of cells?

(A) It led to the development of processes for the manipulation of DNA.


(B) It was initiated by the discovery of rDNA technology.
(C) It led to the use of new treatments for major diseases.
(D) It was universally heralded as a great benefit to humanity.
(E) It was motivated by a desire to create new organisms.

3. The potential benefits of rDNA technology referred to in the passage include all of the
following EXCEPT
(A) new methods of waste treatment
(B) new biological knowledge
(C) enhanced food production
(D) development of less expensive drugs
(E) increased energy production

4. Which one of the following, if true, would most weaken an argument of opponents of
rDNA technology?

(A) New safety procedures developed by rDNA researchers make it impossible for
genetically altered microorganisms to escape from laboratories.
(B) A genetically altered microorganism accidentally released from a laboratory is successfully
contained.
(C) A particular rDNA-engineered microorganism introduced into an ecosystem attracts predators
that keep its population down.
(D) Genetically altered organisms designed to process sewage into methane cannot survive outside
the waste treatment plant.
(E) A specific hereditary disease that has plagued humankind for generations is successfully
eradicated.

5. The author’s reference in the last sentence of the passage to a society that engineers
human beings to fulfill specific roles serves to

(A) emphasize the potential medical dangers of rDNA technology


(B) advocate research on the use of rDNA technology in human genetics
(C) warn of the possible disasters that could result from upsetting the balance of nature
(D) present Brave New World as an example of a work of fiction that accurately predicted
technological developments
(E) illustrate the sociopolitical ramifications of applying genetic engineering to humans
6. Which one of the following, if true, would most strengthen an argument of the
opponents of rDNA technology?

(A) Agricultural products developed through rDNA technology are no more attractive to consumers
than are traditional crops.
(B) Genetically altered microorganisms have no natural predators but can prey on a
wide variety of other microorganisms.
(C) Drugs produced using rDNA technology cost more to manufacture than drugs produced with
traditional technologies.
(D) Ecosystems are impermanent systems that are often liable to collapse, and occasionally do so.
(E) Genetically altered microorganisms generally cannot survive for more than a few hours in the
natural environment.

(D) would weaken, rather than strengthen, the critics’ case. If, as (D) says, the environment is
unstable to start with, and prone to sudden collapse, then would releasing laboratory-created
organisms into it be that much of an additional risk? Maybe not.

Initiate (v) : khởi xướng

Occasionally (adv) : thỉnh thoảng

Impermanent (adj) : vô thường

Liable (adj) : chịu trách nhiệm, dễ bị

Recombinant (n) : tái tổ hợp

Startling (adj) : giật mình

Culminate (v) : lên đến đỉnh điểm

Interdependent (adj) : phụ thuộc lẫn nhau

Extrapolate (v) : ngoại suy

Profound (adj) : dữ dội

Tinker (v) : sửa

Grim (adj) : nghiệt ngã, ghê gớm

Totalitarian (adj) : chuyên chế, toàn trị

Fulfill (v) : làm đúng theo


Oct 16 – 2

The Grameen Bank, founded in Bangladesh by Nobel Peace Prize Laureate, Dr. Muhammad Yunus,
who is also credited for giving life to the concept of microcredit/microfinance, is one of the most
popular models of social business, a business that operates primarily for addressing social needs and
not for-profit. Based on the concept of microfinance, Grameen provides microloans, small loans, to
the poor and those living beyond the reach of traditional finance, in order to promote income
generating activities, avoiding the more traditional form of poverty alleviation - aid donations. The
original focus of the Grameen Bank was on rural people with no credit history and no collateral.
However, the bank rapidly expanded, giving the illusion of a win-win situation that yielded both
profits for Grameen Bank and huge improvements in development for Bangladesh. Private, for-profit
organizations around the world wrongly judged the situation, thinking that they could first and
foremost profit from this type of venture while developing the country in the process. This view was
indeed tainted since Grameen’s goal was development and any profits were an added bonus.
Because of these wrong expectations, the Grameen model has achieved limited success in countries
other than those in South-East Asia, though the concept of microfinance has had better luck, with
many countries now running profitable microfinance institutions that typically charge considerably
higher interest rates for zero collateral loans than Grameen ever would.

1.Which of the following best captures the main idea of the passage?

A.To explain a model and a reason behind its limited success in some countries
B.To show how the focus of a concept changed as it achieved success with time
C.To explain the reason behind the success of a concept in South East Asian countries
D.To highlight how a model is a not very ideal one for-profit oriented microfinance oriented
organizations
E.To discuss the various models of microfinance operational in countries within and outside South
East Asia

2. Which one of the following statements can be inferred about the for-profit
microfinance institutions?

A.The coexistence of profit and development under the Grameen model was not
interpreted correctly by such institutions
B.They are profitable models of social business.
C.They were responsible for the complete failure of Grameen as a model of microfinance.
D.Such institutions are not operational in Bangladesh.
E.Such institutions are not likely to offer loans to applicants who do not have any collateral security
to offer.

3.According to the passage, which one of the following statements is true?

A.When Grameen started functioning, there were fewer people in the urban areas of Bangladesh
who applied for a loan from it than there were when it became more successful
B.There is a limited scope of Grameen’s success in countries outside South East Asia.

limited success is there but scope limited is not mentioned

C.Microfinance as a concept is more successful for for-profit oriented microfinance institutions than
for Grameen.
D.Granting a sum of money as charity to the poor is not a part of Grameen’s operations.
E.Institutions offering traditional loans charge much higher interest rates than Grameen does.

Alleviation (n) : giảm nhẹ, sự an ủi

Collateral (n) : tài sản thế chấp

Yield (v) : mang lại

Foremost (adj) : quan trọng nhất

Indeed (adv) : thật sự

Taint (v) : hủ bại, bại hoại, làm dơ bẩn


Oct 17 – 1

“Strange Bedfellows!” lamented the title of a recent letter to Museum News, in which a certain
Harriet Sherman excoriated the National Gallery of Art in Washington for its handling of tickets to
the much-ballyhooed “Van Gogh’s van Goghs” exhibit. A huge proportion of the 200,000 free tickets
were snatched up by homeless opportunists in the dead of winter, who then scalped those tickets at
$85 apiece to less hardy connoisseurs.

Yet, Sherman’s bedfellows are far from strange. Art, despite its religious and magical origins, very
soon became a commercial venture. From bourgeois patrons funding art they barely understood in
order to share their protegee’s prestige, to museum curators stage-managing the cult of artists in
order to enhance the market value of museum holdings, entrepreneurs have found validation and
profit in big-name art. Speculators, thieves, and promoters long ago created and fed a market
where cultural icons could be traded like commodities.

This trend toward commodification of high-brow art took an ominous, if predictable, turn in the
1980's during the Japanese “bubble economy.” At a time when Japanese share prices more than
doubled, individual tycoons and industrial giants alike invested record amounts in some of the
West’s greatest masterpieces. Ryoei Saito, for example, purchased van Gogh’s Portrait of Dr. Gachet
for a record-breaking $82.5 million. The work, then on loan to the Metropolitan Museum of Modern
Art, suddenly vanished from the public domain. Later learning that he owed the Japanese
government $24 million in taxes, Saito remarked that he would have the paining cremated with him
to spare his heirs the inheritance tax. This statement, which he later dismissed as a joke, alarmed
and enraged many. A representative of the Van Gogh museum, conceding that he had no legal
redress, made an ethical appeal to Mr. Saito, asserting, “a work of art remains the possession of the
world at large.”

Ethical appeals notwithstanding, great art will increasingly devolve into big business. Firstly, great
art can only be certified by its market value. Moreover, the “world at large” hasn’t the means of
acquisition. Only one museum currently has the funding to contend for the best pieces–the J. Paul
Getty Museum, founded by the billionaire oilman. The art may disappear into private hands, but its
transfer will disseminate once static fortunes into the hands of various investors, collectors, and
occasionally the artist.

1. Which of the following would be the most appropriate title for the passage?

A. Art of Art’s Sake: A Japanese Ideal


B. Van Gogh: Breaking New Ground
C. Museums and the Press: Strange Bedfellows
D. Money vs. Art: An Ethical Mismatch
E. Great Art: Business as Usual

the author clearly mentioned its not an ethical mismatch. Refer "Ethical appeals notwithstanding,
great art will increasingly devolve into big business"
2. It can be inferred from the passage that Harriet Sherman would be most likely to
agree with which of the following statements regarding admission to museum exhibits?

A. Tickets should be available on a first-come-first-served basis.

Harriet Sherman excoriated the National Gallery of Art in Washington

B. Those with a genuine interest in art should not have to pay inflated prices.
C. Museums need the income from ticket sales in order to buy great art.
D. Tickets should be distributed without prior announcement.
E. No one should be able to purchase more than one or two tickets.

3. The passage supplies information for answering which of the following questions?

A. Who owned van Gogh’s Portrait of Dr. Gachet prior to its purchase by Saito?
B. Where did Saito exhibit van Gogh’s Portrait of Dr. Gachet?
C. Which museum proposed to purchase van Gogh’s Portrait of Dr. Gachet from Saito?
D. Did the Van Gogh Museum threaten legal action in response to reports that Saito
intended to destroy van Gogh’s Portrait of Dr. Gachet?

A representative of the Van Gogh museum, conceding that he had no legal redress, made an ethical
appeal to Mr. Saito, asserting, “a work of art remains the possession of the world at large.”
E. Did Saito actually intend to destroy van Gogh’s Portrait of Dr. Gachet?

4. The author of the passage mentions which of the following connections between art
and business?

I. Entrepreneurs sometimes try to enhance their reputations by supporting artists.

From bourgeois patrons funding art they barely understood in order to share their protegee’s
prestige, to museum curators stage-managing the cult of artists in order to enhance the market
value of museum holdings, entrepreneurs have found validation and profit in big-name art

II. Artworks are often bought and sold as investments.

‘’individual tycoons and industrial giants alike invested record amounts in some of the West’s
greatest masterpieces."
III. Great artworks can best be recognized by their high market value.

A. I
B. II
C. I and II only
D. III
E. I,II, and III

Mismatch (n) : không phù hợp

Inflated (adj) : thổi phồng

Lament (v) : thương tiếc

Excoriate (v) : chỉ trích


Snatch (v) : giật, chộp lấy

Scalp (v) : bán giá cạnh tranh

Connoisseur (n) : người sành sỏi

Bourgeois (adj) : tư sản

Patron (n) : khách quen

Curator (n) : người quản lý

Vanish (v) : biến mất

Cremate (v) : hỏa táng

Spare (v) : dung tha, dung thứ, tiết kiệm

Heir (n) : người thừa kế

Inheritance tax (n) : thuế di sản

Enrage (v) : tức giận

Redress (n) : sự khôi phục, sự chỉnh lại

Disseminate (v) : phổ biến, truyền bá

Ominous (adj) : đáng ngại, điềm xấu

Tycoon (n) : ông trùm


Oct 17 – 2

It has recently been discovered that many attributions of paintings to the seventeenth-century
Dutch artist Rembrandt may be false. The contested paintings are not minor works, whose removal
from the Rembrandt corpus would leave it relatively unaffected: they are at its very center. In her
recent book, Svetlana Alpers uses these cases of disputed attribution as a point of departure for her
provocative discussion of the radical distinctiveness of Rembrandt's approach to painting.

Alpers argues that Rembrandt exercised an unprecedentedly firm control over his art, his students,
and the distribution of his works. Despite Gary Schwartz's brilliant documentation of Rembrandt's
complicated relations with a wide circle of patrons, Alpers takes the view that Rembrandt refused to
submit to the prevailing patronage system. He preferred, she claims, to sell his works on the open
market and to play the entrepreneur. At a time when Dutch artists were organizing into professional
brotherhoods and academies, Rembrandt stood apart. In fact, Alpers portrait of Rembrandt shows
virtually every aspect of his art pervaded by economic motives. Indeed, so complete was
Rembrandt's involvement with the market, she argues, that he even presented himself as
commodity, viewing his studio's products as extensions of himself, sent out into the world to earn
money. Alpers asserts that Rembrandt's enterprise is found not just in his paintings, but in his
refusal to limit his enterprise to those paintings he actually painted. He marketed Rembrandt.

Although there may be some truth in the view that Rembrandt was an entrepreneur who made
some aesthetic decisions on the basis of what he knew the market wanted, Alpers' emphasis on
economic factors sacrifices discussions of the aesthetic qualities that make Rembrandt's work
unique. For example, Alpers asserts that Rembrandt deliberately left his works unfinished so as to
get more money for their revision and completion. She implies that Rembrandt actually wished the
Council of Amsterdam to refuse the great Claudius Civilis, which they had commissioned for their
new town hall, and she argues that he must have calculated that he would be able to get more
money by retouching the painting.• Certainly the picture is painted with very broad strokes but
there is no evidence that it was deliberately left unfinished. The fact is that the look of a work like
Claudius Civilis must also be understood as the consequence of Rembrandt's powerful and profound
meditations on painting itself. Alpers makes no mention of the pictorial dialectic that can be
discerned between, say, the lessons Rembrandt absorbed from the Haarlem school of paintings and
the styles of his native Leiden. The trouble is that while Rembrandt's artistic enterprise may indeed
not be reducible to the works he himself painted, it is not reducible to marketing practices either.

1. Which one of the following best summarizes the main conclusion of the author of the
passage?

(A) Rembrandt differed from other artists of his time both in his aesthetic techniques and in his
desire to meet the demands of the marketplace.

other artists are not discussed, and so there isn't enough information to agree with that
(B) The aesthetic qualities of Rembrandt's work cannot be understood without consideration of how
economic motives pervaded decisions he made about his art.
(C) Rembrandt was one of the first artists to develop the notion of a work of art as a commodity
that could be sold in an open marketplace.
(D) Rembrandt's artistic achievement cannot be understood solely in terms of decisions
he made on the basis of what would sell in the marketplace.
(E) Rembrandt was an entrepreneur whose artistic enterprise was not limited to the paintings he
actually painted himself.

2. According to the passage, Alpers and Schwartz disagree about which one of the
following?

(A) the degree of control Rembrandt exercised over the production of his art
(B) the role that Rembrandt played in organizing professional brotherhoods and academies
(C) the kinds of relationships Rembrandt had with his students
(D) the degree of Rembrandt's involvement in the patronage system
(E) the role of the patronage system in seventeenth-century Holland

3. In the third paragraph, the author of the passage discusses aesthetic influences on
Rembrandt's work most probably in order to

(A) suggest that many critics have neglected to study the influence of the Haarlem school painters
on Rembrandt's work
(B) suggest that Claudius Civilis is similar in style to many paintings from the seventeen century
(C) suggest that Rembrandt's style was not affected by the aesthetic influences that Alpers points
out
(D) argue that Rembrandt's style can best be understood as a result of the influences of his native
Leiden
(E) indicate that Alpers has not taken into account some important aspects of
Rembrandt's work

Although there may be some truth in the view that Rembrandt was an entrepreneur who made
some aesthetic decisions on the basis of what he knew the market wanted, Alpers' emphasis on
economic factors sacrifices discussions of the aesthetic qualities that make Rembrandt's work unique

4. Which one of the following, if true, would provide the most support for Alpers'
argument about Claudius Civilis?

(A) Rembrandt was constantly revising his prints and paintings because he was never fully satisfied
with stylistic aspects of his earlier drafts.
(B) The works of many seventeenth-century Dutch artists were painted with broad strokes and had
an unfinished look.

that support the authors argument against the Alper's argument, i.e. that the painting depicted
aesthetic details
(C) Many of Rembrandt's contemporaries eschewed the patronage system and sold their works on
the open market.
(D) Artists were frequently able to raise the price of a painting if the buyer wanted the
work revised in some way.

example, Alpers asserts that Rembrandt deliberately left his works unfinished so as to get more
money for their revision and completion. She implies that Rembrandt actually wished the Council of
Amsterdam to refuse the great Claudius Civilis, which they had commissioned for their new town
hall, and she argues that he must have calculated that he would be able to get more money by
retouching the painting
(E) Rembrandt did not allow his students to work on paintings that were commissioned by public
officials.

5. It can be inferred that the author of the passage and Alpers would be most likely to
agree on which one of the following?

(A) Rembrandt made certain aesthetic decision on the basis of what he understood
about the demands of the marketplace.
(B) The Rembrandt corpus will not be affected if attributions of paintings to Rembrandt are found to
be false.
(C) Stylistic aspects of Rembrandt's painting can be better explained in economic terms than in
historical or aesthetic terms.
(D) Certain aesthetic aspects of Rembrandt's art are the result of his experimentation with different
painting techniques.
(E) Most of Rembrandt's best-known works were painted by his students, but were sold under
Rembrandt's name.

Dutch (n) : người hà lan

Corpus (n) : văn thể

Provocative (adj) : khiêu khích, trêu chọc

Patronage (n) : bảo trợ của khách hàng

Pervade (v) : thấm vào, xâm nhập, tràn ngập

Extension (n) : tiện ích mở rộng

Deliberately (adv) : xét đoán kỹ lưỡng

Commission (v) : đặt mua

Town hall (n) : tòa thị chính

Profound (adj) : sâu sắc, uyên thâm, thâm thúy

Meditation (n) : suy tính

Pictorial (adj) : hình ảnh

Dialectic (n) : biện chứng

Discern (v) : phân biệt, nhận thức


Oct 18 – 1

Oppression has been no less fatal to the Indian than to the Negro race, but its effects are different.
Before the arrival of white men in the New World, the inhabitants of North america lived quietly in
the woods, enduring the vicissitudes and practicing the virtues and vices common to the savage
nations. The Europeans, having dispersed the Indian tribes and driven them into the deserts,
condemned them to a wandering life, full of inexpressible sufferings.

Savage nations are only controlled by opinion and custom. When the North American Indians had
lost the sentiment of attachment to their country; when their families were dispersed, their
traditions obscured, and the chain of their recollections broken; when all their habits were changed,
and their wants increased beyond measure, European tyranny rendered them more disorderly and
less civilized than they were before. The moral and physical condition of these tribes grew worse,
and they became more wretched. Nevertheless, the Europeans have not been able to change the
character of the Indians; and though they have had the power to destroy, they have never been
able to subdue and civilize them.

The lot of the Negro is placed on the extreme limit of servitude, while that of the Indian lies on the
utmost verge of liberty; and slavery does not produce more fatal effects upon the first than
independence upon the second. The Negro has lost all property in his own person, and he cannot
dispose of his existence without committing a sort of fraud. But the savage is his own master as
soon as he is able to act; parental authority is scarcely known to him; he has never bent his will to
that of any of his kind, nor learned the difference between voluntary obedience and shameful
subjection; and the very name of law is unknown to him. As he delights in this barbarous
independence and would rather perish than sacrifice the least part of it, civilization has little hold
over him.

The Negro makes a thousand fruitless efforts to insinuate himself among men who repulse him; he
conforms to the taste of his oppressors, adopts their opinions, and hopes by imitating them to form
a part of their community. Having been told from infancy that his race is naturally inferior to that of
the whites, he ascents to the proposition and is ashamed of his own nature. In each of his features,
he discovers a trace of slavery, and if it were in his power, he would willingly rid himself of
everything that makes him what he is.

The Indian, on the contrary has his imagination inflated with the pretended nobility of his origin, and
lives and dies in the midst of these dreams of pride. Far from desiring to conform his habits to ours,
he loves his savage life as the distinguishing mark of his race and repels every advance to
civilization, less, perhaps, from hatred of it than from a dread of resembling the Europeans.

While he has nothing to oppose to our perfection in the arts but the resources of the wilderness, to
our tactics nothing but undisciplined, while our well-digested plans are met only by spontaneous
instincts of savage life, who can wonder if he fails in this unequal contest?

The Negro, who earnestly desires to mingle his race with that of the European, cannot do so, while
the Indian, who might succeed to a certain extent, disclaims to make the attempt. The servility of
the one dooms him to slavery, the pride of the other to death.
1. As inferred from the passage, the Negro

(a) though born a slave was still happy in his life of servitude
(b) had no greater aspiration than to be accepted by the Europeans
(c) sought to imitate his master in order to secure his freedom
(d) has been brainwashed into believing his concomitant subordination
(e) had no pride in his individuality or identity as he was ignorant of his history

Having been told from infancy that his race is naturally inferior to that of the whites, he ascents to
the proposition and is ashamed of his own nature.

2. Which of the following is not in line with the author's comparison of the Negro and
the Indian?

(a) Both suffered tyranny that though different originated from the same author
(b) While the one became servile, the other refused to bow even at the cost of his own life
(c) While the one admired his oppressor and sought to be like him, the other spurned civilization
(d) While oppression caused one to despise himself, it could not quell the fierce pride of the other
(e) The differences in their attitudes helped to save one and became fatal to the other

3. According to the author, all of the following were the effects of European despotism
on the Indians EXCEPT

(a) The bond that held them together was broken and they become isolated
(b) They became more unruly and barbaric as the glue of custom and tradition was lost
(c) They were forced into submission by the need to get their basic requirements
(d) They were forced to roam around in inhospitable terrain facing a lot of hardship
(e) As their habits were changed, they began to desire new things that they could not afford

when all their habits were changed, and their wants increased beyond measure, European tyranny
rendered them more disorderly and less civilized than they were before

4. Regarding the conflict between the Europeans and the Indians, the author opines
that

(a) it was an unequal match, tilted heavily against the Indians


(b) if the Indiand had been a little more like the Negros, then peace would have prevailed in the
continent
(c) all efforts taken by the Whites to bridge the gap was rejected by the Indians
(d) the Indians hated the Europeans and refused to have anything to do with them
(e) the only admirable quality in the Indian was his refusal to bow down in the face of adversity

5. The passage

(a) is critical of the Europeans for the misery they inflicted on the other races
(b) delineates how the future of a race is determined by its historical past
(c) analyses the history of the U.S. to account for the condition of the races as it is today
(d) examines dispassionately the relationship between three major races in the U.S. in a
historical context
(e) seeks to account for the emergence of the whites as the dominant race in the U.S.

6. Which of the following points to the irony of the race relations as depicted by the
author?

(a) Neither the extreme submissiveness of one, nor the fierce independence of the other
benefited them

The servility of the one dooms him to slavery, the pride of the other to death.

(b) Both the desire to please and the desire to oppose was doomed
(c) The one who sought to ingratiate was repulsed while the one who spurned was sought after
(d) One accepted slavery as an attendant of birth, the other loved freedom more than his life
(e) The one who wanted to identify with the rulers could not do so while the other who could have,
did not want to do so.

Endure (v) : tồn tại

Vicissitude (n) : thăng trầm

Savage (adj) : chưa được khai phá, man rợ

Disperse (v) : phân tán

Condemn (v) : lên án, kết án

Wandering (adj) : lang thang, du mục

Inexpressible (adj) : không thể diễn tả, đau khổ

Sentiment (n) : tình cảm

Attachment (n) : sự gắn bó

Obscure (v) : che khuất

Recollection (n) : hồi tưởng, hồi ức

Tyranny (n) : chuyên chế, quyền bạo quân

Wretch (v) : khốn khổ

Subdue (v) : khuất phục, chinh phục, dập tắt

Civilize (v) : khai hóa

Servitude (n) : phục vụ, hình phạt khổ sai, thân phận tôi tớ

Utmost (adj) : hết sức, vô cùng

Verge (n) : bờ vực

Fraud (n) : sự man trá, xảo trá

Delight (v) : vui mừng


Barbarous (adj) : dã man, tàn nhẫn

Perish (v) : diệt vong, chết

Insinuate (v) : ngấm ngầm, lẻn vào

Repulse (v) : đẩy lùi, đánh lui

Conform (v) : làm cho hợp

Infancy (n) : thời thơ ấu

Ascent (v) : đi lên, tiến bộ

Rid (v) : phóng thích, thoát khỏi

Nobility (n) : quý tộc, thanh cao

Midst (n) : ở giữa

Repel (v) : bài xích, khước từ, xô đuổi, cự tuyệt

Hatred (n) : sự thù ghét

Dread (n) : kinh sợ, oai nghiêm, đáng kính

Resemble (v) : giống như

Spontaneous (adj) : tự phát, tự ý

Mingle (v) : trộn lẫn, hòa nhập

Earnestly (adv) : tha thiết

Disclaim (v) : từ chối trách nhiệm, từ bỏ

Servility (n) : sự phục vụ, tình trạng nô lệ, thân phận tôi tớ

Submissiveness (n) : sự phục tùng

Fierce (adj) : dữ dội, hung tợn

Subordination (n) : cấp dưới, sự phụ thuộc

Concomitant (adj) : đồng thời, tự nhiên đi kèm


Oct 18 – 2

The sensation of pain cannot accurately be described as “located” at the point of an injury, or, for
that matter, in any one place in the nerves or brain. Rather, pain signals—and pain relief—are
delivered through a highly complex interacting circuitry.

When a cell is injured, a rush of prostaglandins sensitizes nerve endings at the injury. Prostaglandins
are chemicals produced in and released from virtually all mammalian cells when they are injured:
these are the only pain signals that do not originate in the nervous system. Aspirin and other similar
drugs (such as indomethacin and ibuprofen) keep prostaglandins from being made by interfering
with an enzyme known as prostaglandin synthetase, or cyclooxygenase. The drugs’ effectiveness
against pain is proportional to their success in blocking this enzyme at the site of injury.

From nerve endings at the injury, pain signals move to nerves feeding into the spinal cord. The long,
tubular membranes of nerve cells carry electrical impulses. When electrical impulses get to the
spinal cord, a pain-signaling chemical known as substance P is released there. Substance P then
excites nearby neurons to send impulses to the brain. Local anesthetics such as novocaine and
xylocaine work by blocking the electrical transmission along nerves in a particular area. They inhibit
the flow of sodium ions through the membranes, making the nerves electrically quiescent; thus no
pain signals are sent to the spinal cord or to the brain.
Recent discoveries in the study of pain have involved the brain itself—the supervising organ that
notices pain signals and that sends messages down to the spinal cord to regulate incoming pain
traffic. Endorphins—the brain’s own morphine—are a class of small peptides that help to block pain
signals within the brain itself. The presence of endorphins may also help to explain differences in
response to pain signals, since individuals seem to differ in their ability to produce endorphins. It
now appears that a number of techniques for blocking chronic pain—such as acupuncture and
electrical stimulation of the central brain stem—involve the release of endorphins in the brain and
spinal cord.

1. The passage is primarily concerned with

(A) analyzing ways that enzymes and other chemicals influence how the body feels pain
(B) describing the presence of endorphins in the brain and discussing ways the body blocks pain
within the brain itself
(C) describing how pain signals are conveyed in the body and discussing ways in which
the pain signals can be blocked
(D) demonstrating that pain can be influenced by acupuncture and electrical stimulation of the
central brain stem
(E) differentiating the kinds of pain that occur at different points in the body’s nervous system

2. According to the passage, which of the following is one of the first things to occur
when cells are injured?

(A) The flow of electrical impulses through nerve cells at the site of the injury is broken.
(B) The production of substance P traveling through nerve cells to the brain increases.
(C) Endorphins begin to speed up the response of nerve cells at the site of the injury.
(D) A flood of prostaglandins sensitizes nerve endings at the site of the injury.
(E) Nerve cells connected to the spinal cord become electrically quiescent.
3. Of the following, which is most likely attributable to the effect of endorphins as
described in the passage?

(A) After an injection of novocaine, a patient has no feeling in the area where the injection was
given.
(B) After taking ibuprofen, a person with a headache gets quick relief.
(C) After receiving a local anesthetic, an injured person reports relief in the anesthetized area.
(D) After being given aspirin, a child with a badly scraped elbow feels better.
(E) After acupuncture, a patient with chronic back pain reports that the pain is much
less severe.

4. It can be inferred from the passage that if the prostaglandin synthetase is only
partially blocked, which of the following is likely to be true?

(A) Some endorphins will be produced, and some pain signals will be intensified.
(B) Some substance P is likely to be produced, so some pain signals will reach the brain.
(C) Some sodium ions will be blocked, so some pain signals will not reach the brain.
(D) Some prostaglandins will be produced, but production of substance P will be prevented.
(E) Some peptides in the brain will receive pain signals and begin to regulate incoming pain traffic.
Oct 19 – 1

A recent generation of historians of science, far from portraying accepted scientific views as
objectively accurate reflections of a natural world, explain the acceptance of such views in terms of
the ideological biases of certain influential scientists or the institutional and rhetorical power such
scientists wield. As an example of ideological bias, it has been argued that Pasteur rejected the
theory of spontaneous generation not because of experimental evidence but because he rejected
the materialist ideology implicit in that doctrine. These historians seem to find allies in certain
philosophers of science who argue that scientific views are not imposed by reality but are free
inventions of creative minds, and that scientific claims are never more than brave conjectures,
always subject to inevitable future falsification. While these philosophers of science themselves
would not be likely to have much truck with the recent historians, it is an easy step from their views
to the extremism of the historians.

While this rejection of the traditional belief that scientific views are objective reflections of the world
may be fashionable, it is deeply implausible. We now know, for example, that water is made of
hydrogen and oxygen and that parents each contribute one-half of their children’s complement of
genes. I do not believe any serious-minded and informed person can claim that these statements
are not factual descriptions of the world or that they will inevitably be falsified.

However, science’s accumulation of lasting truths about the world is not by any means a
straightforward matter. We certainly need to get beyond the naive view that the truth will
automatically reveal itself to any scientist who looks in the right direction; most often, in fact, a
whole series of prior discoveries is needed to tease reality’s truths from experiment and observation.
And the philosophers of science mentioned above are quite right to argue that new scientific ideas
often correct old ones by indicating errors and imprecision (as, say, Newton’s ideas did to Kepler’s).
Nor would I deny that there are interesting questions to be answered about the social processes in
which scientific activity is embedded. The persuasive processes by which particular scientific groups
establish their experimental results as authoritative are themselves social activities and can be
rewardingly studied as such. Indeed, much of the new work in the history of science has been
extremely revealing about the institutional interactions and rhetorical devices that help determine
whose results achieve prominence.

But one can accept all this without accepting the thesis that natural reality never plays any part at
all in determining what scientists believe. What the new historians ought to be showing us is how
those doctrines that do in fact fit reality work their way through the complex social processes of
scientific activity to eventually receive general scientific acceptance.

1. It can be inferred from the passage that the author would be most likely to agree
with which one of the following characterizations of scientific truth?
(A) It is often implausible.
(B) It is subject to inevitable falsification.
(C) It is rarely obvious and transparent.
(D) It is rarely discovered by creative processes.
(E) It is less often established by experimentation than by the rhetorical power of scientists.
2. According to the passage, Kepler’s ideas provide an example of scientific ideas that
were
(A) corrected by subsequent inquiries
(B) dependent on a series of prior observations
(C) originally thought to be imprecise and then later confirmed
(D) established primarily by the force of an individuals rhetorical power
(E) specifically taken up for the purpose of falsification by later scientists

3. In the third paragraph of the passage, the author is primarily concerned with
(A) presenting conflicting explanations for a phenomenon
(B) suggesting a field for possible future research
(C) qualifying a previously expressed point of view
(D) providing an answer to a theoretical question
(E) attacking the assumptions that underlie a set of beliefs

4. The use of the words “any serious-minded and informed person’ (lines 28-29) serves
which one of the following functions in the context of the passage?
(A) to satirize chronologically earlier notions about the composition of water
(B) to reinforce a previously stated opinion about certain philosophers of science
(C) to suggest the author’s reservations about the “traditional belief” mentioned in line 22
(D) to anticipate objections from someone who would argue for an objectively accurate description
of the world
(E) to discredit someone who would argue that certain scientific assertions do not
factually describe reality

5. It can be inferred from the passage that the author would most likely agree with
which one of the following statements about the relationship between the views of
“certain philosophers of science” (lines l2-13) and those of the recent historians?
(A) These two views are difficult to differentiate.
(B) These two views share some similarities.
(C) The views of the philosophers ought to be seen as the source of the historians’ views.
(D) Both views emphasize the rhetorical power of scientists.
(E) The historians explicitly acknowledge that their views are indebted to those of the philosophers.

6. Which one of the following best characterizes the author’s assessment of the opinions
of the new historians of science, as these opinions are presented in the passage?
(A) They lack any credibility.
(B) They themselves can be rewardingly studied as social phenomena.
(C) They are least convincing when they concern the actions of scientific groups.
(D) Although they are gross overstatements, they lead to some valuable insights.
(E) Although they are now popular, they are likely to be refused soon.

7. In concluding the passage, the author does which one of the following?
(A) offers a prescription
(B) presents a paradox
(C) makes a prediction
(D) concedes an argument
(E) anticipates objections
8. The authors attitude toward the “thesis” mentioned in line 56 is revealed in which
one of the following pairs of words?
(A) “biases” (line 5) and “rhetorical” (line 6)
(B) “wield” (line 7) and “falsification” (line 17)
(C) “conjectures” (line l6) and “truck with” (line 19)
(D) “extremism” (line 20) and “implausible” (line 24)
(E) “naive” (line 35) and “errors’ (line 42
Oct 19 – 2

Transportation developments have greatly influenced the pace and course of business growth in
America. The early turnpike and the canal systems each broadened the market area by lowering
costs and speeding distribution. But the influence of railroads dwarfed all of these previous
developments. Railroads pioneered many aspects of business administration and enhanced some
land values enormously. They also had an important impact on the growth of certain cities. Atlanta,
for example, was transformed from a spot in the wilderness to a thriving metropolis as a result of
the construction of the Western and Atlantic Railroad.

Railroads also provided a large outlet for savings.Their capital requirements were so great that they
provided the first big opening for the investment banker – who by the end of the 19th century was
in control of many railroads. Railroads, too, because of the stress of competition, both in
construction and operation, were the first big firms to experiment with new forms of business
organization such as pools and consolidation. It followed – because railroads were so vital to the
nation and because their performance was tied in with the business cycle – that the highways which
came later were the first form of business to have their operations regulated in large degree by the
government.

Finally, railroads were responsible for a great many jobs, at one time more than 2,000,000 workers.
Railroads were easily the nation’s largest employers during the post-Civil War, pre-World War I
period. In addition they were responsible, indirectly, for tens of thousand of other jobs in the coal,
iron, steel, and engineering industries – in such big enterprises, for example, as the Pullman Palace
Car Company and the Westinghouse Air Brake Company. \ The Pullman Company, in 1909, was the
eighth largest firm in the nation in terms of assets, and practically all of its output went to American
railroads. Railroads, actually, were to the period 1850-1915 what the auto industry is to today in
terms of being the pacesetter or bellwether of the economy. The biggest difference is that no one
railroad ever dominated the rail industry as Ford and GM once dominated the auto industry.

1. Which of the following can be logically inferred from the passage?

A) Early turnpike and canal systems were not largely regulated by the U.S. government
B) Current automakers directly employ more workers than did the railroad industry
C) The Pullman Car Company had more assets than the Westinghouse Brake Company
D) Railroads dramatically increased the land values in Atlanta
E) Highways are more regulated today than railroads.

that the highways which came later were the first form of business to have their operations
regulated in large degree by the government
 We can infer that before the highways, there was no regulation about turnpike and canal system

2. It can be inferred from the passage that the railroad system:

A. Had a greater influence on American markets than the American highway system
B. Required more capital than any other business before
C. Increased all land values across the U.S.
D. Involved strong competition among the companies that formed it
E. Created more employment than any industry before.
3. During the post-Civil War, pre-World War I period, if the railroad business was
declining it could be expected that:

A. Investment banking would decline


B. Land values would decline
C. Production at the Pullman Company would decline
D. U.S. savings would decline
E. Car production would increase

Canal (n) : con kênh, sông đào

Turnpike (n) : đường thu phí

Metropolis (n) : đô thị, thành phố lớn, thủ đô, thủ phủ, tâm điểm hoạt động

Pool (n) : vốn chung

Bellwether (n) : người tạo ra xu hướng, người dẫn đầu

Pacesetter (n) : a person or organization that is the first to do new or different things and
so provides anexample for others to follow
Oct 20 – 1

Existentialism is a term applied to the work of certain late 19th- and 20th-century philosophers who,
despite profound doctrinal differences, shared the belief that philosophical thinking begins with the
human subject—not merely the thinking subject, but the acting, feeling, living human individual. In
existentialism, the individual's starting point is characterized by what has been called "the existential
attitude", or a sense of disorientation and confusion in the face of an apparently meaningless or
absurd world. Many existentialists have also regarded traditional systematic or academic
philosophies, in both style and content, as too abstract and remote from concrete human
experience.

A central proposition of existentialism is that existence precedes essence, which means that the
most important consideration for individuals is the fact that they are an individual—an independently
acting and responsible, conscious being ("existence")—rather than what labels, roles, stereotypes,
definitions, or other preconceived categories the individual fits ("essence"). The actual life of the
individual constitutes what could be called their "true essence" instead of an arbitrarily attributed
essence others use to define them. Thus, human beings, through their own consciousness, create
their own values and determine a meaning to their life.

1. It can be inferred from the passage that:

(A) Most late 19th and 20th century philosophers had profound doctrinal differences
(B) Style is more important than value in existential philosophy.
(C) Existentialism favors a deeper connection to human experience than traditional
philosophies.
(D) Existential philosophers were frequently confused by traditional philosophies.
(E) A core notion of existentialism is that “true essence” supersedes the individual.

‘Existentialism is a term applied to the work of certain late 19th- and 20th-century philosophers who,
despite profound doctrinal differences, shared the belief that philosophical thinking begins with the
human subject—not merely the thinking subject, but the acting, feeling, living human individual..."

2. Which of the following situations best illustrates the “existential attitude”?

(A) A confused man wanders through an orderly and structured parking lot trying to find his car
(B) A confused woman contemplates charges for something that she never did and that
never happened.
(C) A disoriented dog gets lost in a large, well-manicured park, but is found by his owners later.
(D) A purposeful student sits in an outdoor art gallery with no boundaries or logic and contemplates
life.
(E) A disoriented sailor finishes 2nd in an outcome that was expected by some and doubted by
others.

3. The author uses the term “true essence” in order to:

(A) identify the preconceived categories in which the individual fits


(B) emphasize the importance of individualism over consciousness
(C) show how human beings create their own consciousness
(D) define the true life of an individual in contrast to that created by others
(E) contrast the individual with the true self

Existentialism (n) : chủ nghĩa hiện sinh, hiện hữu, tồn tại

Thinking subject (n) : môn học tư duy

Disorientation (n) : mất phương hướng

Absurd (adj) : vô lý

Abstract (adj) : trừu tượng

Proposition (n) : mệnh đề

Precede (v) : đứng trước

Essence (n) : bản chất

Preconceive (v) : định kiến

Arbitrarily (adv) : tùy tiện


Oct 20 – 2

Bernard Bailyn has recently reinterpreted the early history of the United States by applying new
social research findings on the experiences of European migrants. In his reinterpretation, migration
becomes the organizing principle for rewriting the history of preindustrial North America. His
approach rests on four separate propositions.

The first of these asserts that residents of early modern England moved regularly about their
countryside; migrating to the New World was simply a “natural spillover.” Although at first the
colonies held little positive attraction for the English—they would rather have stayed home—by the
eighteenth century people increasingly migrated to America because they regarded it as the land of
opportunity. Secondly, Bailyn holds that, contrary to the notion that used to flourish in America
history textbooks, there was never a typical New World community. For example, the economic and
demographic character of early New England towns varied considerably.

Bailyn’s third proposition suggests two general patterns prevailing among the many thousands of
migrants: one group came as indentured servants, another came to acquire land. Surprisingly,
Bailyn suggests that those who recruited indentured servants were the driving forces of transatlantic
migration. These colonial entrepreneurs helped determine the social character of people who came
to preindustrial North America. At first, thousands of unskilled laborers were recruited; by the
1730’s, however, American employers demanded skilled artisans.

Finally, Bailyn argues that the colonies were a half-civilized hinterland of the European culture
system. He is undoubtedly correct to insist that the colonies were part of an Anglo-American empire.
But to divide the empire into English core and colonial periphery, as Bailyn does, devalues the
achievements of colonial culture. It is true, as Bailyn claims, that high culture in the colonies never
matched that in England. But what of seventeenth-century New England, where the settlers created
effective laws, built a distinguished university, and published books? Bailyn might respond that New
England was exceptional. However, the ideas and institutions developed by New England Puritans
had powerful effects on North American culture.

Although Bailyn goes on to apply his approach to some thousands of indentured servants who
migrated just prior to the revolution, he fails to link their experience with the political development
of the United States. Evidence presented in his work suggests how we might make such a
connection. These indentured servants were treated as slaves for the period during which they had
sold their time to American employers. It is not surprising that as soon as they served their time
they passed up good wages in the cities and headed west to ensure their personal independence by
acquiring land. Thus, it is in the west that a peculiarly American political culture began, among
colonists who were suspicious of authority and intensely anti-aristocratic.
1.Which of the following statements about migrants to colonial North America is
supported by information in the passage?

(A) A larger percentage of migrants to colonial North America came as indentured servants than as
free agents interested in acquiring land.
(B) Migrants who came to the colonies as indentured servants were more successful at making a
livelihood than were farmers and artisans.
(C) Migrants to colonial North America were more successful at acquiring their own land during the
eighteenth century than during the seven-tenth century.
(D) By the 1730’s, migrants already skilled in a trade were in more demand by American
employers than were unskilled laborers.
(E) A significant percentage of migrants who came to the colonies to acquire land were forced to
work as field hands for prosperous American farmers.

2.The author of the passage states that Bailyn failed to

(A) give sufficient emphasis to the cultural and political interdependence of the colonies and England
(B) describe carefully how migrants of different ethnic backgrounds preserved their culture in the
united States
(C) take advantage of social research on the experiences of colonists who migrated to colonial North
America specifically to acquire land
(D) relate the experience of the migrants to the political values that eventually shaped
the character of the United States
(E) investigate the lives of Europeans before they came to colonial North America to determine more
adequately their motivations for migrating

3.Which of the following best summarizes the author’s evaluation of Bailyn’s fourth
proposition?

(A) It is totally implausible.


(B) It is partially correct.
(C) It is highly admirable.
(D) It is controversial though persuasive.
(E) It is intriguing though unsubstantiated.

4.According to the passage, Bailyn and the author agree on which of the following
statements about the culture of colonial New England?

(A) High culture in New England never equaled the high culture of England.
(B) The cultural achievements of colonial New England have generally been unrecognized by
historians.
(C) The colonists imitated the high culture of England, and did not develop a culture that was
uniquely their own.
(D) The southern colonies were greatly influenced by the high culture of New England.
(E) New England communities were able to create laws and build a university, but unable to create
anything innovative in the arts.
5.According to the passage, which of the following is true of English migrants to the
colonies during the eighteenth century?

(A) Most of them were farmers rather than trades people or artisans.
(B) Most of them came because they were unable to find work in England.
(C) They differed from other English people in that they were willing to travel.
(D) They expected that the colonies would offer them increased opportunity.
(E) They were generally not as educated as the people who remained in England.

6.The author of the passage is primarily concerned with

(A) comparing several current interpretations of early American history


(B) suggesting that new social research on migration should lead to revisions in current
interpretations of early American history
(C) providing the theoretical framework that is used by most historians in understanding early
American history
(D) refuting an argument about early American history that has been proposed by social historians
(E) discussing a reinterpretation of early American history that is based on new social
research on migration

7.It can be inferred from the passage that American history textbooks used to assert
that

(A) many migrants to colonial North America were not successful financially
(B) more migrants came to America out of religious or political conviction that came in the hope of
acquiring land
(C) New England communities were much alike in terms of their economics and
demographics
(D) many migrants to colonial North America failed to maintain ties with their European relations
(E) the level of literacy in New England communities was very high

8.The author of the passage would be most likely to agree with which of the following
statements about Bailyn’s work?

(A) Bailyn underestimates the effects of Puritan thought on North American culture.
(B) Bailyn overemphasizes the economic dependence of the colonies on Great Britain.
(C) Bailyn’s description of the colonies as part of an Anglo-American empire is misleading and
incorrect.
(D) Bailyn failed to test his propositions on a specific group of migrants to colonial North America.
(E) Bailyn overemphasizes the experiences of migrants to the New England colonies, and neglects
the southern and the western parts of the New World.

Bailyn might respond that New England was exceptional. However, the ideas and institutions
developed by New England Puritans had powerful effects on North American culture

Reinterpret (v) : diễn giải lại

Organizing principle (n) : nguyên tắc tổ chức

Spillover (n) : lan tỏa


Demographic (n) : nhân khẩu học

Servant (n) : người hầu

Indenture (v) : làm hợp đồng

Trans-atlantic (n) : xuyên đại tây dương

Hinterland (n) : nội địa

Periphery (n) : ngoại vi

Devalue (v) : làm giảm giá trị

Overemphasize (v) : quá tập trung, quá coi trọng

Peculiarly (adv) : đặc biệt, dị thường

Aristocratic (adj) : quý tộc


Oct 21 – 1

One of the best sources modern scholars have for learning about Hellenistic Egypt is the large
supply of papyrus fragments that have turned up in the Egyptian desert over the last century.
Papyrus is a thick type of paper made from a reedy plant found in Egypt. Papyrus is much tougher
than the wood-pulp paper used in modern society; whereas a book produced today will most likely
fall apart within a century, there are papyrus fragments that are still legible over 2,000 years after
scribes wrote on them. This is primarily by accident that any of these fragments have survived. Most
of the surviving fragments have been found in ancient garbage dumps that were covered over by
the desert and preserved in the dry heat. The benefit of this type of archeological find is that these
discarded scraps often give us a more accurate picture of the daily lives of ancient Egyptians—their
business affairs, personal correspondence, and religious pleas—than the stone engravings and
recorded texts that were intended to be passed down to later generations.

One of the most important papyrus discoveries of recent years was the revelation in 2001 that a
scrap of papyrus that had been discarded and used to wrap a mummy contained 110 previously
unknown epigrams (short, witty poems) by the Hellenistic poet Posidippus (ca. 280–240 B.C.).
Posidippus lived in Alexandria and benefited from the support of King Ptolemy II Philadelphus (ruled
284–246 B.C.). These new epigrams. have yielded fascinating insight into the court culture and
literary sensibilities of early Hellenistic Egypt. King Ptolemy, of course, was also a sponsor of the
famous library of Alexandria, the greatest depository of knowledge in the ancient world. According
to the twelfth-century Byzantine writer John Tzetzes, the ancient library contained nearly half a
million papyrus scrolls. If that library had not burned down, maybe archeologists today would not
have to spend so much of their time sorting through ancient trash!

1. The author’s primary intention in this passage appears to be which of the following?

A. To shed light on the underappreciated work of the Hellenistic poet Posidippus


B. To compare the relative merits of papyrus and wood-pulp paper as media for recording
information
C. To discuss the ways in which papyrus fragments help scholars learn about Hellenistic
Egypt
D. To answer the questions regarding the burning of the library of Alexandria, one of the great
mysteries of the ancient world
E. To suggest possibly fruitful paths for future archeological research into Hellenistic Egypt

2. Which of the following would best illustrate how a discarded fragment of papyrus
might give us a more accurate picture of the daily lives of ancient Egyptians than a
record intended to be permanent?

A. A poet such as Posidippus may have composed rough drafts of his epigrams on papyrus
fragments prior to writing them in their final form.
B. Grocery lists, which give insights into the diets of ancient people, would never be
included in stone inscriptions but could be scribbled on scraps of papyrus.
C. The Hellenistic monarchs employed some of the finest historians of the Greek world to provide
chronicles of their reigns.
D. Some papyrus fragments may have been used for purposes other than writing, such as binding
wounds or wrapping small packages.
E. Stone inscriptions describing military events often embellish the truth to favor whoever is paying
for the inscription.

3. The mention of the discovery of 110 previously unknown epigrams by the poet
Posidippus serves what purpose in the passage?

A. Revealing insights into the nuances of court culture in Hellenistic Egypt


B. Demonstrating how durable a material papyrus can be
C. Arguing for a greater appreciation of this little-known Hellenistic poet
D. Highlighting the importance of royal patronage in the development of arts and literature in the
Hellenistic world
E. Illustrating the kind of discovery that can be made from researching papyrus
fragments

4. According to information given in the passage, which of the following locations would
probably yield the highest probability of finding a previously undiscovered papyrus
fragment?

A. The ship of a royal messenger that sank off the Egyptian coast of the Mediterranean Sea in the
third century B.C.
B. The charred remnants of an ancient Egyptian palace that was burned by Roman troops in the first
century B.C.
C. The refuse heap of an ancient Egyptian town that was buried in the desert in the fifth
century A.D.
D. The private collections of French and British explorers from the nineteenth century who first
uncovered many of the principal sites of Egyptian archaeology
E. The library of a Hellenistic fishing village that sank into the marshes of the Nile Delta in the third
century A.D.

5. What does the author imply by the final statement: “If that library had not burned
down, maybe archeologists today would not have to spend so much of their time sorting
through ancient trash!”?

A. The author implies that if the library had not burned down, archeologists would be able to
appreciate the full cultural legacy of King Ptolemy II Philadelphos.
B. The author implies that if the library had not burned down, scholars today would have not only
the full works of Posidippus, but also those of Aeschylus, Sophocles, and Euripides.
C. The author implies that if the library had not burned down, the scrolls contained within the library
would have decomposed before modern times in any event, because they would not have been
preserved in the dry heat of the desert.
D. The author implies that if the library had not burned down, it might have contained
more complete details about the life and culture of Hellenistic Egypt than can be found
in the papyrus fragments from ancient refuse dumps.
E. The author implies that if the library had not burned down, the cultural awakening of the
Renaissance might have occurred centuries earlier.

Papyrus (n) : giấy cói

Reedy (adj) : làm bằng sậy


Scribe (n) : người ghi chép, thơ ký

Discard/ed (v/adj) : loại bỏ/ bị bỏ đi

Correspondence (n) : thư tín, giao thiệp, tương hợp

Pleas (n) : những lời cầu xin

Epigram (n) : lời chế nhạo

Witty (adj) : dí dỏm

Yield (v) : mang lại

Depository (n) : chỗ chứa, lưu ký


Oct 21 – 2

Many aspects of coral reefs remain puzzling to scientists. One mystery concerns the relationship
between Scleractinia, the coral type whose colonization produces reefs, and their symbiotic partners,
a unicellular algae present in the coral's endodermic tissues. It is known that both organisms play an
integral part in the formation of a reef's foundation by together secreting and depositing calcium
carbonate, which reacts with sea salt to form a hard limestone underlayer. Scientists also know that,
because of algal photosynthesis, the reef environment is oxygen-rich, while similarly high amounts
of carbon dioxide are removed rapidly. All of this accounts for the amazing renewability of coral
reefs despite the erosion caused by waves. The precise manner in which one symbiotic organism
stimulates the secretion of calcium carbonate by the other, however, remains unclear.

In addition to the above unanswered question, scientists have proposed various theories to explain
the transformation of "fringing reefs" (those connected above sea level to land masses) into "barrier
reefs" (those separated from shorelines by lagoons) and finally into island atolls. Although Darwin's
view of the transformation is considered partially correct, some scientists feel that the creation of
reef formations has more to do with the rise in sea level that occurred at the end of the last Ice Age
than with a gradual submergence of the volcanic islands to which fringing reefs were originally
attached. However, recent drillings at one atoll have revealed a substantial underlayer of volcanic
rock, which suggests that Darwin's explanation may be largely correct. The term "coral reef" is
something of a misnomer. The Scleractiniathemselves generally comprise only 10 percent of the
total mass of life forms of an average reef community: algae, along with foraminifera, annelid
worms and assorted molluscs, can account for up to 90 percent of the reef mass. Moreover, the
conditions under which reef growth occurs are determined by the needs of the algae, not those of
the coral. Reefs flourish only in shallow, highly saline waters above 70° F, because the algae require
such an environment. Non-reef-building coral, meanwhile, occur worldwide.

1. It can be inferred from the passage that Darwin

A. believed that reefs became atolls through the sinking of volcanoes.


B. should have expanded his studies of reefs to include those found at atolls
C. theorized that each reef was formed by an entirely different process.
D. is less persuasive on the topic of reef formation in light of recent discoveries
E. was more interested in algae and coral than in other organisms living at reefs

2. The passage does NOT discuss the relationship between

A. algal photosynthesis and high oxygen content


B. Darwin's views and evidence supplied by recent research
C. volcanic rock and the life forms found at reefs
D. sea salt and calcium carbonate
E. wave action and the renewal of reefs

Coral (n) : san hô

Reef (n) : đá ngầm

Colonization (n) : thuộc địa

Simbiotic (adj) : cộng sinh


Algae (n) : rong biển

Endodermic (adj) : nội tiệt

Secrete (v) : tiết ra

Deposit (v) : lắng cặn

Limestone (n) : đá vôi

Photosynthesis (n) : quang hợp

Renewability (n) : tái tạo

Manner (n) : cách thức, phương pháp, thói quen

Precise (adj) : rõ ràng, tỉ mỉ, chắc chắn

Stimulate (v) : kích thích

Lagoon (n) : hồ nước mặn ở gần biển

Atoll (n) : đảo san hô

Misnomer (n) : hiểu sai

Saline (adj) : nước mặn


Oct 22 – 1

Corporate law firms have traditionally billed their clients on an hourly basis. However, in the current
economy, it is increasingly common for law firms worldwide to offer clients alternative billing
structures. Seeking a competitive advantage, many law firms now earn much of their compensation
in the form of an unguaranteed bonus which is paid upon completion of the firm's work and the
amount of which depends on the success of that work. The terms of these alternative compensation
schemes are documented in engagement agreements that specify the bonus the client will pay the
firm given various outcomes of the case. Particularly with first-time clients, a compensation
structured on such a bonus can be a highly effective marketing tool, especially if the client is very
cautious in selecting legal service providers, a high hourly billing rate is a barrier to client
engagement, the firm often loses potential clients to firms with lower hourly rates, or the
consequences to the client of a negative outcome are severe.

On the other hand, these bonus-based compensation structures can sometimes hinder law firms'
marketing efforts. Sophisticated clients may be wary of firms offering such structures, understanding
that the firm may have less incentive to provide independent, objective advice and instead advise
the client to take undue risks in the hope of winning a larger bonus payout. Such structures may
mislead less sophisticated clients by suggesting that the outcomes of certain cases will necessarily
be favorable. A firm's willingness to work for a bonus may even signal to potential clients that a firm
is desperate for business. Indeed, law firms with exceptional reputations and a track record of
success have little to gain by offering an alternative to the long standing norm of hourly billing.

1. All of the following hypothetical situations exemplify a potential problem noted in the
second paragraph EXCEPT which of the following?

a. A law firm's offer of a bonus-based compensation structure leads a client to believe that the
success of his or her case is a foregone conclusion.
b. A law firm's offer of a bonus-based compensation structure makes a client suspect that the firm
has few existing clients.
c. A law firm's offer of a bonus-based compensation structure makes a client suspect that the firm
may give riskier advice than it would if it were working on an hourly basis.
d. A law firm's offer of a bonus-based compensation structure makes a client suspect
that the firm is trying to charge more than it would if it were working on an hourly
basis.
e. A law firm's offer of a bonus-based compensation structure makes a client suspect that a firm's
objectivity may be compromised.

2. The primary purpose of this passage is to

a. justify the popularity of a behavior


b. evaluate the usefulness of a practice
c. demonstrate how to implement a program
d. discuss the ethics of a business activity
e. explain the reasons for pursuing a strategy

3. The passage's description of the issue raised by a bonus-based fee structure for law
firms most clearly implies that which of the following is true?
a. Success-based bonuses violate the legal profession's standard of practice.
b. Lawyers working on behalf of a client cannot always determine the outcome of a
lawsuit in advance.
c. The dignity of the legal profession is undermined by new legal services that differ from those
typically offered in the past.
d. Clients whose cases are not successful cannot be adequately compensated by financial
settlements alone.
e. Predicting the cost of legal services is more difficult than predicting the costs of other professional
services.

" Such structures may mislead less sophisticated clients by suggesting that the outcomes of certain
cases will necessarily be favorable. .."

Wary (adj) : cảnh giác, đề phòng

Undue risk (n) : rủi ro không đáng có

Mislead (v) : đánh lạc hướng, lừa dối

Desperate (adj) : tuyệt vọng

Norm (n) : quy tắc, định mức


Oct 22- 2

The premise with which the multiculturalists begin is unexceptional: that it is important to recognize
and to celebrate the wide range of cultures that exist in the United States. In what sounds like a
reflection of traditional American pluralism, the multiculturalists argue that we must recognize
difference, that difference is legitimate; in its kindlier versions, multiculturalism represents the
discovery on the part of minority groups that they can play a part in molding the larger culture even
as they are molded by it. And on the campus multiculturalism, defined more locally as the need to
recognize cultural variations among students, has tried with some success to talk about how a
racially and ethnically diverse student body can enrich everyone’s education.

Phillip Green, a political scientist at Smith and a thoughtful proponent of multiculturalism, notes that
for a significant portion of the students the politics of identity is all-consuming. Students he says
“are unhappy with the thin gruel of rationalism. They require a therapeutic curriculum to overcome
not straightforward racism but ignorant stereotyping.”

But multiculturalism’s hard-liners, who seem to make up the majority of the movement, damn as
racism any attempt to draw the myriad of American groups into a common American culture. For
these multiculturalists, differences are absolute, irreducible, intractable—occasions not for
understanding but for separation. The multiculturalist, it turns out, is not especially interested in the
great American hyphen, in the syncretistic (and therefore naturally tolerant) identities that allow
Americans to belong to more than a single culture, to be both particularists and universalists.

The time-honored American mixture of assimilation and traditional allegiance is denounced as a


danger to racial and gender authenticity. This is an extraordinary reversal of the traditional liberal
commitment to a “truth” that transcends parochialisms. In the new race/class/gender formation,
universality is replaced by, among other things, feminist science Nubian numerals (as part of an
Afro-centric science), and what Marilyn Frankenstein of the University of Massachusetts-Boston
describes as “ethno-mathematics,” in which the cultural basis of counting comes to the fore.

The multiculturalists insist on seeing all perspectives as tainted by the perceiver’s particular point of
view. Impartial knowledge, they argue, is not possible, because ideas are simply the expression of
individual identity, or of the unspoken but inescapable assumptions that are inscribed in a culture or
a language. The problem, however, with this warmed-over Nietzscheanism is that it threatens to
leave no ground for anybody to stand on. So the multiculturalists make a leap, necessary for their
own intellectual survival, and proceed to argue that there are some categories, such as race and
gender, that do in fact embody an unmistakable knowledge of oppression. Victims are at least
epistemologically lucky. Objectivity is a mask for oppression. And so an appalled former 1960s
radical complained to me that self-proclaimed witches were teaching classes on witchcraft. “They’re
not teaching students how to think,” she said, “they’re telling them what to believe.”

1.“Multiculturalist relativism” is the notion that there is no such thing as impartial or


objective knowledge. The author seems to be grounding his criticism of this notion on

(A) the clear evidence that science has indeed discovered “truths” that have been independent of
both language and culture.
(B) the conclusion that relativism leaves one with no clear notions of any one thing that
is true.
(C) the absurdity of claiming that knowledge of oppression is more valid than knowledge of scientific
facts.
(D) the agreement among peoples of all cultures as to certain undeniable truths—e.g., when the sky
is clear, day is warmer than night.
(E) the fact that “truth” is not finitely definable and therefore that any discussion of impartial or
objective truth is moot.

2.Which one of the following best describes the attitude of the writer toward the
multicultural movement?

(A) Tolerant. It may have some faults, but it is well-meaning overall.


(B) Critical. A formerly admirable movement has been taken over by radical
intellectuals.
(C) Disinterested. He seems to be presenting an objective report.
(D) Enthusiastic. The author embraces the multiculturalist movement and is trying to present it in a
favorable light.
(E) Ambivalent. Like a moth to a flame he is simultaneously attracted and repulsed by the
movement.

3.The author speaks about the “politics of identity” that Phillip Green, a political
scientist at Smith, notes is all-consuming for many of the students. Considering the
subject of the passage, which one of the following best describes what the author
means by “the politics of identity”?

(A) The attempt to discover individual identities through political action


(B) The political agenda that aspires to create a new pride of identity for Americans
(C) The current obsession for therapy groups that help individuals discover their inner selves
(D) The trend among minority students to discover their identities in their ethnic groups
rather than in their individuality
(E) The increased political activism of minorities on college campuses

Phillip Green, a political scientist at Smith and a thoughtful proponent of multiculturalism, notes that
for a significant portion of the students the politics of identity is all-consuming. Students he says
“are unhappy with the thin gruel of rationalism. They require a therapeutic curriculum to overcome
not straightforward racism but ignorant stereotyping.”

Interpretation:
Students are unhappy about indirect (thin) racism. Students feel that there's no direct racism or no
one is directly differentiating them based on their race or ethnicity but there's a sense of indirect
racism that prevails amongst students. Let's consider another example: if there's a group of 5-6 Afro
students, there are good chances that these members could form their own group and express such
indirect racial experiences with each other. So author here is referring to such grp of students and
their activity of discussing the racism among themselves as 'politics of identity'.

Author believes that if these students are taught that this thin racism is NOT deliberately done then
there are good chances that students will not feel unhappy. Author says that Phillip Green
recommended having a therapeutic curriculum that 'perhaps' teaches other race ppl that racism is
happening due to ignorance. At the same time, this curriculum can also teach how to avoid such
'undeliberate' ways of racism.
4.The author states that in a “kindlier version” of multiculturalism, minorities discover
“that they can play a part in molding the larger culture even as they are molded by it.” If
no new ethnic groups were incorporated into the American culture for many centuries to
come, which one of the following would be the most probable outcome of this “kindlier
version”?

(A) At some point in the future, there would be only one culture with no observable
ethnic differences.
(B) Eventually the dominant culture would overwhelm the minority cultures, who would then lose
their ethnic identities.
(C) The multiplicity of ethnic groups would remain but the characteristics of the different ethnic
groups would change.
(D) The smaller ethnic groups would remain, and they would retain their ethnic heritage.
(E) The minority cultures would eventually overwhelm the dominant culture, which would then lose
its identity.

In what sounds like a reflection of traditional American pluralism, the multiculturalists argue that we
must recognize difference, that difference is legitimate; in its kindlier versions, multiculturalism
represents the discovery on the part of minority groups that they can play a part in molding the
larger culture even as they are molded by it.

1) when a group of different race/cast/ethnic members join the larger American culture --> they
adapt themselves to larger American culture.
2) At the same time, larger American culture can be adapted /changed/molded to the trends OR
nuances offered by these smaller ethnic groups as each grp represents their own unique
characteristics.
3) As per question, if no ethnic groups are incorporated into larger American culture, then the larger
culture will prevail. - very simple, isn't it?

5.According to a hard-line multiculturalist, which one of the following groups is most


likely to know the “truth” about political reality?

(A) Educated people who have learned how to see reality from many different perspectives.
(B) A minority group that has suffered oppression at the hands of the majority.
(C) High government officials who have privileged access to secret information.
(D) Minorities who through their education have risen above the socioeconomic position occupied by
most members of their ethnic group.
(E) Political scientists who have thoroughly studied the problem.

6.Which one of the following ideas would a multiculturalist NOT believe?

(A) That we should recognize and celebrate the differences among the many cultures in the United
States.
(B) That we can never know the “truth” because “truth” is always shaped by one’s culture.
(C) That “difference” is more important than “sameness.”
(D) That a school curriculum should be constructed to compensate for institutionalized racism.
(E) That different cultures should work to assimilate themselves into the mainstream
culture so that eventually there will be no excuse for racism.

Kindlier (adj) : tốt hơn


Mold (v) : đúc

Diverse (adj) : phong phú, đa dạng

Racially (adv) : chủng tộc

Ethnically (adv) : chủng tộc

All-consuming (adj) : tất cả tiêu thụ

Gruel (n) : độc ác, mệt mỏi

Rationalism (n) : chủ nghĩa duy lý (a belief or theory that opinions and actions should be based
on reason and knowledge rather than on religious belief or emotional response.)

Therapeutic (adj) : trị liệu

Curriculum (n) : giáo trình, chương trình giáo dục

Ignorant (adj) : dốt nát, vô học, thiếu hiểu biết

Hard-liner (n) : a member of a group, typically a political group, who adheres uncompromisingly
to a set of ideas or policies.

Myriad (adj) : vô số

Irreducible (adj) : không thể giả

Intractable (adj) : không thể kiểm soát


Hyphen (n) : dấu nối

Syncretistic (adj) : đồng bộ hóa

Tolerant (adj) : khoan dung

Particularist (n) : chuyên gia, đặc thù

Universalist (n) : phổ quát

Assimilation (n) : đồng hóa

Allegiance (n) : lòng trung thành

Denounce (v) : tố cáo

Authenticity (n) : tính xác thực

Transcend (v) : vượt qua, đi quá giới hạn

Parochialism (n) : chủ nghĩa địa phương, sự phân biệt địa phương

Afrocentric (adj) : phi thường

Ethno (prefix) : dân tộc học

Fore (n) : mũi thuyền


Impartial (adj) : công bình, vô tư

Inescapable (adj) : không thể bỏ qua, không thể chối cãi

Leap (n) : bước nhảy vọt

Embody (v) : hiện thân

Epistemologically (adv) : nhận thức luận

Appall (v) : kinh khủng, ghê gớm, kinh hoàng

Witchcraft (n) : ma thuật

Radical (n) : đảng cấp tiến


Oct 23- 1

Critics have long been puzzled by the inner contradictions of major characters in John Webster’s
tragedies. In his The Duchess of Malfi, for instance, the Duchess is “good” in demonstrating the
obvious tenderness and sincerity of her love for Antonio, but “bad” in ignoring the wishes and
welfare of her family and in making religion a “cloak” hiding worldly self-indulgence. Bosola is “bad”
in serving Ferdinand, “good” in turning the Duchess’ thoughts toward heaven and in planning to
avenge her murder. The ancient Greek philosopher Aristotle implied that such contradictions are
virtually essential to the tragic personality, and yet critics keep coming back to this element of
inconsistency as though it were an eccentric feature of Webster’s own tragic vision.

The problem is that, as an Elizabethan playwright, Webster has become a prisoner of our critical
presuppositions. We have, in recent years, been dazzled by the way the earlier Renaissance and
medieval theater, particularly the morality play, illuminates Elizabethan drama. We now understand
how the habit of mind that saw the world as a battleground between good and evil produced the
morality play. Morality plays allegorized that conflict by presenting characters whose actions were
defined as the embodiment of good or evil. This model of reality lived on, overlaid by different
conventions, in the most sophisticated Elizabethan works of the following age. Yet Webster seems
not to have been as heavily influenced by the morality play’s model of reality as were his
Elizabethan contemporaries; he was apparently more sensitive to the more morally complicated
Italian drama than to these English sources. Consequently, his characters cannot be evaluated
according to reductive formulas of good and evil, which is precisely what modern critics have tried to
do. They choose what seem to be the most promising of the contradictor values that are dramatized
in the play, and treat those values as if they were the only basis for analyzing the moral
development of the play’s major characters, attributing the inconsistencies in a character’s behavior
to artistic incompetence on Webster’s part. The lack of consistency in Webster’s characters can be
better understood if we recognize that the ambiguity at the heart of his tragic vision lies not in the
external world but in the duality of human nature. Webster establishes tension in his plays by
setting up conflicting systems of value that appear immoral only when one value system is viewed
exclusively from the perspective of the other. He presents us not only with characters that we
condemn intellectually or ethically and at the same time impulsively approve of, but also with
judgments we must accept as logically sound and yet find emotionally repulsive. The dilemma is not
only dramatic: it is tragic, because the conflict is irreconcilable, and because it is ours as much as
that of the characters.

1. The primary purpose of the passage is to

(A) clarify an ambiguous assertion


(B) provide evidence in support of a commonly held view
(C) analyze an unresolved question and propose an answer
(D) offer an alternative to a flawed interpretation)
(E) describe and categorize opposing viewpoints

2. The author suggests which one of the following about the dramatic works that most
influenced Webster’s tragedies?
(A) They were not concerned with dramatizing the conflict between good and evil that
was presented in morality plays.
(B) They were not as sophisticated as the Italian sources from which other Elizabethan tragedies
were derived.
(C) They have never been adequately understood by critics.
(D) They have only recently been used to illuminate the conventions of Elizabethan drama.
(E) They have been considered by many critics to be the reason for Webster’s apparent artistic
incompetence.

3. The author’s allusion to Aristotle’s view of tragedy in lines 11-13 serves which one of
the following functions in the passage?
(A) It introduces a commonly held view of Webster’s tragedies that the author plans to defend.
(B) It supports the author’s suggestion that Webster’s conception of tragedy is not
idiosyncratic.
(C) It provides an example of an approach to Webster’s tragedies that the author criticizes.
(D) It establishes the similarity between classical and modern approaches to tragedy.
(E) It supports the author’s assertion that Elizabethan tragedy cannot be fully understood without
the help of recent scholarship.

4. It can be inferred from the passage that modern critics’ interpretations of Webster’s
tragedies would be more valid if
(A) the ambiguity inherent in Webster’s tragic vision resulted from the duality of human nature
(B) Webster’s conception of the tragic personality were similar to that of Aristotle
(C) Webster had been heavily influenced by the morality play
(D) Elizabethan dramatists had been more sensitive to Italian sources of influence
(E) the inner conflicts exhibited by Webster’s characters were similar to those of modern audiences

5. With which one of the following statements regarding Elizabethan drama would the
author be most likely to agree?
(A) The skill of Elizabethan dramatists has in recent years been overestimated.
(B) The conventions that shaped Elizabethan drama are best exemplified by Webster’s drama.
(C) Elizabethan drama, for the most part, can be viewed as being heavily influenced by
the morality play.
(D) Only by carefully examining the work of his Elizabethan contemporaries can Webster’s
achievement as a dramatist be accurately measured.
(E) Elizabethan drama can best be described as influenced by a composite of Italian and classical
sources.

We have, in recent years, been dazzled by the way the earlier Renaissance and medieval theater,
particularly the morality play, illuminates Elizabethan drama

6. It can be inferred from the passage that most modern critics assume which one of the
following in their interpretation of Webster’s tragedies?

(A) Webster’s play tended to allegorize the conflict between good and evil more than did those of
his contemporaries.
(B) Webster’s plays were derived more from Italian than from English sources.
(C) The artistic flaws in Webster’s tragedies were largely the result of his ignorance of the classical
definition of tragedy.
(D) Webster’s tragedies provide no relevant basis for analyzing the moral development of their
characters.
(E) In writing his tragedies, Webster was influenced by the same sources as his
contemporaries.

They choose what seem to be the most promising of the contradictor values that are dramatized in
the play, and treat those values as if they were the only basis for analyzing the moral development
of the play’s major characters, attributing the inconsistencies in a character’s behavior to artistic
incompetence on Webster’s part.

7. The author implies that Webster’s conception of tragedy was

(A) artistically flawed


(B) highly conventional
(C) largely derived from the morality play
(D) somewhat different from the conventional Elizabethan conception of tragedy
(E) uninfluenced by the classical conception of tragedy

Puzzle (v) : lúng túng

Contradiction (n) : mâu thuẫn

Tragedy (n) : bi kịch

Duchess (n) : nữ công tước

Tenderness (n) : dịu dàng

Sincerity (n) : sự chân thành

Welfare (n) : sự an toàn, sự an lạc

Cloak (n) : áo choàng

Worldly (adj) : trần tục

Self-indulgence (n) : tự sướng

Avenge (v) : báo thù

Inconsistency (n) : mâu thuẫn

Eccentric (adj) : kỳ dị, khác thường, quái gở

Supposition (n) : giả định

Dazzle (v) : lóa mắt

Allegorize (v) : biểu hiện, cáo buộc

Overlay (v) : phủ lên

Convention (n) : tục lệ, tập quán, lễ nghi

Incompetence (n) : không đủ sức, bất tài

Artistic (adj) : nghệ thuật

Duality (n) : tính hai mặt


Exclusively (adv) : duy nhất

Condemn (v) : lên án

Impulsively (adv) : bốc đồng

Repulsive (adj) : phản cảm

Irreconcilable (adj) : không thể hòa giải

Allusion (n) : ám chỉ

Idiosyncratic (adj) : phong cách riêng, đặc trưng, tính riêng biệt
Oct 23 – 2

While it may seem simple and unobjectionable to non-specialists, one of the many issues that
illustrates the divergence of post-Keynesian economics from mainstream economics is the idea of
time. Most models of neoclassical economics utilize the notion of "logical time," in which markets
(whether they be capital, goods, or labor markets) return to equilibrium after a disturbance is
introduced and then overcome. While few economists object to this general model, post-Keynesians
try to shift focus away from the "long run"-level analysis and analyze instead events in "historical
time," emphasizing the real-world effect of deviations from equilibrium.

However, post-Keynesian economics cannot be understood simply as a reaction to, or adjustment


of, the ideas of Keynes. In fact, because many post-Keynesians believe that the movement known
as Keynesianism actually represents a severe divergence from the ideas of Keynes, they are in the
position of both resuscitating and updating his theories. One example of this phenomenon is the
labor market, on which traditional Keynesians follow the classical theory (that is, pre-Keynesian) of
unemployment. While traditional and post-Keynesians both support [deficit spending] (known as
"pump-priming") as a solution to short- and medium-term inefficiencies in the labor market,
traditional Keynesians believe that, so long as wages and prices are perfectly flexible, those
inefficiencies will disappear. That is the extent of the story for the traditional Keynesian, but viewed
through the lens of "historical time," such inefficiencies are problems worthy of the economist's
further study.

1. The passage suggests which of the following about the deficit spending mentioned in
the highlighted text?

(A) To a traditional Keynesian, it is a way to eliminate inefficiencies in the labor market.


(B) To a post-Keynesian, it solves all of the problems associated with inefficiencies in the labor
market.
(C) To both traditional and post-Keynesians, it is a key aspect of the classical theory of
unemployment.
(D) To a traditional Keynesian, it is a way to turn attention away from "long run"-level analysis.
(E) To a post-Keynesian, it is a superior strategy to that of "pump-priming."

Deficit spending and "pump-priming" are two terms for the same concept.

2. The author of the passage would be most likely to agree with which of the following
statements about the idea of time mentioned in the first sentence of the passage?

(A) It underlies the most trenchant critiques of the classical theory of unemployment.
(B) It applies most directly to theories of the labor market, which has long been misunderstood by
economists.
(C) It represents the most serious problem with the positions held by post-Keynesians.
(D) It is the leading cause of the inefficiencies that wreak havoc with economic theories.
(E) It illustrates an important distinction between streams of economic thought.

3. It can be inferred from the passage that which of the following is true about
traditional Keynesians?

(A) When studying the labor market, they are mainly concerned with eliminating short-
and medium-term inefficiencies.
(B) In criticizing the classical theory of unemployment, they understate the importance of "pump-
priming."
(C) They favor the notion of "historical time" to that of "logical time."
(D) They support more of the theories of Keynes than do post-Keynesian economists.
(E) As neoclassical economists, they study the real-life impact of inefficiencies that appear in the
capital, goods,and labor markets.

That is the extent of the story for the traditional Keynesian, but viewed through the lens of
"historical time," such inefficiencies are problems worthy of the economist's further study.

 traditional Keynesians use "logical time."

Unobjectionable (adj) : không thể chối cãi

Divergence (n) : khác nhau

Neoclassical (adj) : tân cổ điển

Overcome (v) : thắng thế

Deviation (n) : sai lệch

Resuscitate (v) : hồi sức, làm cho sống lại

Deficit spending (n) ; chi tiêu thâm hụt

Pump-priming (n) : bơm mồi

Inefficiency (n) : không hiệu quả


Oct 24 – 1

Until recently many astronomers believed that asteroids travel about the solar system
unaccompanied by satellites. These astronomers assumed this because they considered asteroid-
satellite systems inherently unstable. Theoreticians could have told them otherwise: even minuscule
bodies in the solar system can theoretically have satellites, as long as everything is in proper scale.
If a bowling ball were orbiting about the Sun in the asteroid belt, it could have a pebble orbiting it as
far away as a few hundred radii (or about 50 meters) without losing the pebble to the Sun’s
gravitational pull.

Observations now suggest that asteroid satellites may exists not only in theory but also in reality.
Several astronomers have noticed, while watching asteroids pass briefly in front of stars, that
something besides the known asteroid sometimes blocks out the star as well. Is that something a
satellite? The most convincing such report concerns the asteroid Herculina, which was due to pass in
front of a star in 1978. Astronomers waiting for the predicted event found not just one occultation,
or eclipse, of the star, but two distinct drops in brightness. One was the predicted occultation,
exactly on time. The other, lasting about five seconds, preceded the predicted event by about two
minutes. The presence of a secondary body near Herculina thus seemed strongly indicated. To
cause the secondary occultation, an unseen satellite would have to be about 45 kilometers in
diameter, a quarter of the size of Herculina, and at a distance of 990 kilometers from the asteroid at
the time. These values are within theoretical bounds, and such an asteroid-satellite pair could be
stable.

With the Herculina event, apparent secondary occultations became “respectable”—and more
commonly reported. In fact, so common did reports of secondary events become that they are now
simply too numerous for all of them to be accurate. Even if every asteroid has as many satellites as
can be fitted around it without an undue number of collisions, only one in every hundred primary
occultations would be accompanied by a secondary event (one in every thousand if asteroid
satellites system resembled those of the planets).
Yet even astronomers who find the case for asteroid satellites unconvincing at present say they
would change their minds if a photoelectric record were made of a well-behaved secondary event.
By “well-behaved” they mean that during occultation the observed brightness must drop sharply as
the star winks out and must rise sharply as it reappears from behind the obstructing object, but the
brightness during the secondary occultation must drop to that of the asteroid, no higher and no
lower. This would make it extremely unlikely that an airplane or a glitch in the instruments was
masquerading as an occulting body.
1. Which one of the following best expresses the main idea of the passage?

(A) The observation of Herculina represented the crucial event that astronomical observers and
theoreticians had been waiting for to establish a convincing case for the stability of asteroid-satellite
systems.
(B) Although astronomers long believed that observation supports the existence of stable asteroid-
satellite systems, numerous recent reports have increased skepticism on this issue in astronomy.
(C) Theoreticians’ views on the stability of asteroid-satellite systems may be revised in the light of
reports like those about Herculina.
(D) Astronomers continue to consider it respectable to doubt the stability of asteroid-satellite
systems, but new theoretical developments may change their views.
(E) The Herculina event suggests that theoreticians’ views about asteroid-satellite
systems may be correct, and astronomers agree about the kind of evidence needed to
clearly resolve the issue.

2. Which one of the following is mentioned in the passage as providing evidence that
Herculina has a satellite?

(A) the diameter of a body directly observed near Herculina


(B) the distance between Herculina and planet nearest to it
(C) the shortest possible time in which satellites of Herculina, if any, could complete a single orbit
(D) the occultation that occurred shortly before the predicted occultation by Herculina
(E) the precise extent to which observed brightness dropped during the occultation by Herculina

3. According to the passage, the attitude of astronomers toward asteroid satellites since
the Herculina event can best described as

(A) open-mindedness combined with a concern for rigorous standards of proof


(B) contempt for and impatience with the position held by theoreticians
(C) bemusement at a chaotic mix of theory, inadequate or spurious data, and calls for scientific rigor
(D) hardheaded skepticism, implying rejection of all data not recorded automatically by state-of-the-
art instruments
(E) admiration for the methodical process by which science progresses from initial hypothesis to
incontrovertible proof

4. The author implies that which one of the following was true prior to reports of the
Herculina event?

(A) Since no good theoretical model existed, all claims that reports of secondary occultations were
common were disputed.
(B) Some of the reported observations of secondary occultations were actually observations of
collisions of satellites with one another.
(C) If there were observations of phenomena exactly like the phenomena now labeled
secondary occultations, astronomers were less likely than to have reported such
observations.
(D) The prevailing standards concerning what to classify as a well-behaved secondary event were
less stringent than they are now.
(E) Astronomers were eager to publish their observations of occultations of stars by satellites of
asteroids.
5. The information presented in the passage implies which one of the following about
the frequency of reports of secondary occultations after the Herculina event?

(A) The percentage of reports of primary occultations that also included reports of secondary
occultations increased tenfold compared to the time before the Herculina event.
(B) Primary occultations by asteroids were reported to have been accompanied by secondary
occultations in about one out of every thousand cases.
(C) The absolute number of reports of secondary occultations increased tenfold compared to the
time before the Herculina event.
(D) Primary occultations by asteroids were reported to have been accompanied by
secondary occultations in more than one out of every hundred cases.
(E) In more than one out of every hundred cases, primary occultations were reported to have been
accompanied by more than one secondary occultation.

Even if every asteroid has as many satellites as can be fitted around it without an undue number of
collisions, only one in every hundred primary occultations would be accompanied by a secondary
event (one in every thousand if asteroid satellites system resembled those of the planets).

Unaccompanied (adj) : không có người đi theo

Asteroid (n) : tiểu hành tinh

Minuscule (adj) : nhỏ

Pebble (n) : đá cuội

Occultation = eclipse (n) : nhật thực

Masquerade (v) : giả trang


Oct 25 – 1

For once, the whys are turning out to be more important than the hows. A study shows that
consumers who become fixated on how to achieve a goal probably face more difficulty in achieving
their aims than people who think abstractly about why they want to do something. The authors of
the study conducted four experiments to examine consumer behavior when it came to the goal of
saving money. In one such experiment, some people were asked to make a specific plan to save
money, whereas others were not asked to plan. Then some people were asked to focus on why they
wanted to save money. Later on all participants were given the opportunity to buy candy. Subjects
who were thinking concretely and formed a specific plan were less able to avoid the candy purchase
than those who had not formed a plan at all.

To elucidate the results, the authors cite the instance of person who wants to save money and
hence makes a plan for achieving this goal. This person plans to save money by purchasing fewer
clothing items at the mall. On investigating how this plan influences the person's response to other
money-saving opportunities, for example, would the person be more likely to order a cheaper meal
at a restaurant, avoid making an impulse purchase, or combine errands to save money on gas etc.,
the authors found that when people focus on concrete aspects of how they want to achieve goals,
they become more closed-minded and consequently less likely to take advantage of opportunities
that fall outside their plans. In contrast, people who focus on the why aspect are more likely to
consider out-of-plan opportunities to achieve their goals.

Furthermore, in the same experiment, the group that performed the best was the one that was
asked to focus on the “why” and had formed a plan implicitly. Accordingly, the authors conclude that
planning is more effective when people plan with the complete understanding of “why”. They
suggest that this strategy is particularly effective when the original plan turns out to be unrealistic in
terms of feasibility or when other goal-directed activities become available.

1. The author is primarily concerned with

A addressing a key issue in consumer psychology and behavior


B. making the readers understand how they can save money
C. describing a theory that deals with goal determination
D. advocating consumers to follow a particular strategy while making and executing plans
E. presenting a study, explaining its findings, and outlining its final conclusion.

2. Which of the following statement can be derived on the basis of the analysis done by
the study authors?

A. Fixating on how to achieve a goal makes the plan less feasible


B. Planning can be more effective in certain circumstances
C. A good strategy is one that always accounts for every possible opportunity available to achieve a
particular goal
D. People who want to save money get fixated on little details and lose sight of the big picture
E. Out of all the aspects of a plan, the why aspect is the most important to achieve the desired goal.

Accordingly, the authors conclude that planning is more effective when people plan with the
complete understanding of “why”.
3. Through the second paragraph, the author of the passage intends to

A. to describe a new aspect of the study.


B. to present an explanation.
C. to evaluate the merits of one strategy while discussing the demerits of another.
D. to generate awareness about a particular aspect of consumer psychology.
E. to discuss a feature of the study

4. Each of the following is a statement that is mentioned with respect to the study
described in the passage EXCEPT

A. The feasibility of a plan is dependent on whether a person has an understanding of


why the plan is devised.
B. In the experiment to save money, those who planned explicitly did not perform better than those
who understood why but planned implicitly.
C. The strategy to keep an open mind and focus on the reason behind achieving the goal can help
when alternate means of achieving the goal exist.
D. People focusing on the reason behind achieving a particular goal are likely to consider other goal-
directed activities that may not have been a part of the original plan.
E. People fixated on the how aspect of a plan tend to ignore such opportunities that are not a part
of their original plan.

5. Which of the following is an example of a scenario in which the strategy suggested by


the authors of the study in the last sentence of the passage would be most effective?

A. The consumer is not able to determine the reason behind the pursuit of the goal.
B. The consumer is not able to pursue the desired goal because of reasons not unrelated to the lack
of out of plan goal-directed activities.
C. The consumer realizes that the plan is not clear as the definition of the goal is not clear.
D. The consumer is able to perceive issues related to practicality of the plan.
E. The original plan becomes less attractive to the consumer as he/she moves closer to the desired
goal.

They suggest that this strategy is particularly effective when the original plan turns out to be
unrealistic in terms of feasibility or when other goal-directed activities become available.

Practicality (n) : tính thực tiện, thực dụng, khả thi

Particularly (adv) : đặc biệt

Fixate (v) : cố định

Elucidate (v) : làm rõ

Impulse (n) : thúc đẩy

Errand (n) : việc vặt

Closed-minded (adj) : bảo thủ

Implicity (adv) : ngầm

Feasibility (n) : tính khả thi


Oct 26 – 2

In August 1348 the bubonic plague, or Black Death, suddenly appeared in England. Its germs were
carried by the fleas on black rats that came into the country on ships from abroad. The first
outbreak of the plague was of intense ferocity, for the people had no immunity and persons living
close to the margin of subsistence fell victims to the disease.

Returning in 1361, the plague caused high mortality among children born since 1348; there were
other visitations in 1368 and 1375. High farming in the thirteenth century had been based on the
scarcity of land, a large population, and a great demand for food—conditions that had forced the
peasants to remain on their holdings and to accept the burdens of serfdom. But when the demand
for food was less, the profits of agriculture shrank. High farming, which had already been slipping
before 1348, came to an end.

The startling fact about those figures is the amazing drop in population between 1348 and 1377. It
may be the number of people in overcrowded England already was beginning to decline before the
coming of the Black Death. There were floods and famines in the years between 1315 and 1317.
Certainly the plague caused a high mortality. In some monasteries the monks all but disappeared (it
is thought that 20 half the clergy in England fell victims to the pestilence). The Black Death had its
most striking effect on the rural economy. The balance between the number of labourers and the
amount of land under cultivation and the relations between lord and peasant were quickly altered.
There were deserted villages and many unoccupied peasant holdings. After the first 25 visitation
widows and widowers remarried quickly and produced as many children as before; but because of
the high mortality among young people this population increase was not maintained later in the
century.

The work of the manor could not be performed by the villeins who had survived the plague; the lord
had to employ casual labor at wages that doubled within a decade. Moreover, a villein, once tied to
his holding by economic necessity, could easily run away to another manor where employment
would be offered to him with no questions asked.

Landowners complained bitterly of the labour shortage and of the wages they had to pay. In 1351
they obtained the Statute of Laborers, which fixed wages at the rates before the plague, declared
that all landless men must accept work when it was offered to them, and prohibited peasants from
moving from one manor to another. For a time the statute had some effect, but in the long run it
was useless, for wages continued to rise and employers had to pay them. There was also a scarcity
of tenants. Few manors were without vacant holdings; hence the yield was less and income from
the land declined. Agricultural products no longer fetched high prices. Yet the cost of luxuries and of
manufactured goods was rising.

Thereafter the plague subsided in the rural areas but remained endemic in London and other towns,
where it could become active at any time and could spread along lines of communication into the
country. It remained in England for more than 300 years.

1. Which of the following was NOT a contributing factor in the dependence of the
peasantry on high farming as a means of subsistence?
A. A large population
B. A widespread outbreak of plague
C. A great demand for food
D. A scarcity of land
E. Too many mouths to feed

High farming, which had already been slipping before 1348, came to an end.

 Widespread outbreak of plague happened in August 1348, it means that Black Death is not the
contributing factor to an end of high farming.

2. According to information brought forth by the author in the passage, the economic
difficulties brought on by the Black Death were not quickly resolved because:

A. potential workers were afraid to leave their homes due to the fear of contracting disease.
B. population gains that might have been made by remarriages were offset by a high
infant mortality rate.
C. many landholdings were left unoccupied, often without recourse.
D. the Statute of Laborers fixed wages at the pre-plague levels.
E. there was no money in the economy

The Black Death had its most striking effect on the rural economy. The balance between the
number of labourers and the amount of land under cultivation and the relations between
lord and peasant were quickly altered.

3. Which of the following claims would, if true, most substantially weaken the author‘s
claim that the plague brought an end to the practice of high farming?

A. The practice of high farming was reinforced after the floods and famines in the 1310s reduced
the amount of arable land.
B. Immediately following the plague, the profits of agriculture would see a rebound due
to the stabilization in wages and food prices.
C. The numbers of peasants working on English farms decreased throughout much of the years of
plague.
D. The Statute of Laborers began to be strictly enforced when it became apparent that wages were
still rising.
E. Over the next few years following the plague, the incomes of agriculturists kept falling lower and
lower

Bubonic (adj) : hạch

Plague (n) : bệnh dịch

Germs (n) : vi trùng

Fleas (n) : bọ chét

Ferocity (n) : hung ác, dã man, vô nhân đạo

Subsistence (n) : sự tồn tại

Mortality (n) : tỷ lệ tử vong


Visitation (n) : cuộc thanh tra

Serfdom (n) : chế độ nông nô

Shrink (v) : co lại (shrank)

Startling (adj) : giật mình, hoảng hốt

Famine (n) : nạn đói

Monastery (n) : tu viện

Monk (n) : nhà sư, thầy tu

Clergy (n) : giáo sĩ

Pestilence (n) : sâu bênh, bệnh dịch hạch

Manor (n) : lãnh địa

Villein (n) : nông nô

Necessity (n) : sự cưỡng bách

Tenant (n) : người thuê nhà

Fetch (v) : lấy, dẫn đến, đem đến

Subside (v) : giảm dần, lắng xuống

Arable (adj) : trồng trọt


Oct 27 – 1

DNA degrades quickly after an animal dies, so researchers once believed it impossible to find ancient
genetic material. The search for primeval vestiges of DNA took off in the late 1980s after the
development of a technique called polymerase chain reaction (PCR), which copies minute quantities
of DNA. Armed with PCR, scientists could look for tiny fragments of DNA that might have weathered
the millennia unharmed.

In recent years, researchers have isolated DNA from 20-million-year-old magnolia leaves and
extracted DNA from a 135-million-year-old weevil found in amber. Recently, a team extract DNA
from bone dating back millions of years for the first time. In the frenzied hunt for ancient DNA,
microbiologist Scott R. Woodward may have bagged the biggest quarry. Drawing on lessons learned
while growing up among the fossil-rich rocks of eastern Utah, Woodward and his team became the
first people to find genetic material belonging to a dinosaur.

Woodward, whose grandfather was a coal miner, knew that mines in the area often contained
dinosaur traces. After six months of looking Woodward pulled two bone fragments from a
Cretaceous siltstone layer directly atop a coal seam. Impeded by an unstable mine roof, Woodward‘s
team could not recover any more bone samples. The siltstone apparently inhibited fossilization and
preserved much of the original cell structure in the bone. Researchers isolated strands of DNA from
both fragments and used PCR to copy a segment that codes for a protein called cytochrome b. Once
they had made many copies, they could determine the DNA sequence.

Throughout their work, the biologists took precautions to avoid contaminating the samples with
modern DNA or ancient material found within the coal. According to Woodward, circumstantial
evidence indicates that the bone fragments belong to one or two species of dinosaurs. Dinosaur
tracks are abundant in this coal formation, and the bones visible in the mine were larger than those
of a crocodile—the biggest non-dinosaur known in these rocks.
Woodward explains variation found in the DNA as a result of damage to the ancient DNA, which
caused the PCR technique to alter the original sequence. Scientists had hoped to use the DNA to
resolve debate about the relationship among birds, dinosaurs, and other reptiles. But the
cytochrome b fragments were too short to offer meaningful phylogenetic information, says
Woodward. Utah‘s state palaeontologist believes that the fragments found by Woodward could
definitely be dinosaur in origin. Other researchers, however, question the identity of the DNA
strands. Because the copies of the cytochrome b sequence varied considerably, they wonder
whether the DNA comes from several organisms.

1. Researchers who believe that the DNA isolated by Woodward did not come from a
dinosaur would most likely use which of the following discoveries as support?

A. Damage to the dinosaur DNA causes the PCR technique to alter the original sequence.
B. Comparison of the discovered DNA with that of modern DNA reveals a variation in sequence.
C. Birds, dinosaurs, and reptiles have no phylogenetic relationship.
D. The cytochrome b sequence comprises DNA from several different animals.
E. DNA cannot survive for such long periods of time
2. The passage suggests that researchers continue to look for dinosaur DNA because:

A. the DNA found by Woodward derived from several different species.


B. the amount of DNA retrieved was too small to copy using PCR.
C. the DNA fragments produced by PCR were too insignificant to determine substantial
information about bird, dinosaur, and reptile phylogeny.
D. the sites where Woodward excavated had never been highly populated with dinosaurs.
E. they were hoping they might chance upon the DNA of a different dinosaur

3. The findings of Woodward are by no means universally accepted. Which of the


following findings, if true, would MOST contradict the researchers who question the
identity of Woodward‘s dinosaur DNA?

A. Variations in the cytochrome b sequence of Woodward‘s DNA have been directly linked to hybrid
DNA.
B. Carbon dating proved that the bone fragments retrieved by Woodward were from the Cretaceous
era.
C. More elaborate PCR traced the cytochrome b sequence in Woodward‘s sample to one
species of dinosaur.
D. Utah‘s state palaeontologist confirmed that dinosaurs were abundant in the areas where the
researchers excavated.
E. An independent study reported that the DNA discovered by Woodward in fact belongs to a hen

Primeval (adj) : nguyên thủy

Vestige (n) : dấu tích, vết tích, di tích

Weather (v) : phong hóa

Magnolia (n) : mộc lan

Weevil (n) : mọt lúa

Amber (n) : hổ phách

Frenzied (adj) : điên cuồng

Bag (v) : bỏ túi

Quarry (n) : hầm đá, mỏ đá

Coal (n) : than đá

Impede (v) : cản trở

Siltstone (n) : đá sa thạch

Circumstantial (adj) : đầy đủ chi tiết


Oct 27 – 2

Recently the focus of historical studies of different ethnic groups in the United States has shifted
from the transformation of ethnic identity to its preservation. Whereas earlier historians argued
that the ethnic identity of various immigrant groups to the United States blended to form an
American national character, the new scholarship has focused on the transplantation of ethnic
cultures to the United States. Fugita and O’Brien’s Japanese American Ethnicity provides an example
of this recent trend; it also exemplifies a problem that is common to such scholarship.

In comparing the first three generations of Japanese Americans (the Issei, Nisei, and Sansei), Fugita
and O’Brien conclude that assimilation to United States culture increased among Japanese
Americans over three generations, but that a sense of ethnic community endured. Although the
persistence of community is stressed by the authors, their emphasis in the book could just as easily
have been on the high degree of assimilation of the Japanese American population in the late
twentieth century, which Fugita and O’Brien believe is demonstrated by the high levels of education,
income, and occupational mobility achieved by Japanese Americans. In addition, their data reveal
that the character of the ethnic community itself changed: the integration of Sanseis into new
professional communities and nonethnic voluntary associations meant at the very least that ethnic
ties had to accommodate multiple and layered identities. Fugita and O’Brien themselves
acknowledge that there has been a “weakening of Japanese American ethnic community life.”

Because of the social changes weakening the bonds of community, Fugita and O’Brien maintain that
the community cohesion of Japanese Americans is notable not for its initial intensity but because
“there remains a degree of involvement in the ethnic community surpassing that found in most
other ethnic groups at similar points in their ethnic group life cycle.” This comparative difference is
important to Fugita and O’Brien, and they hypothesize that the Japanese American community
persisted in the face of assimilation because of a particularly strong preexisting sense of
“peoplehood”. They argue that this sense of peoplehood extended beyond local and family ties.

Fugita and O’Brien have explained persistence of ethnic community by citing a preexisting sense of
national consciousness that is independent of how a group adapts to United States culture.
However, it is difficult to prove as Fugita and O’Brien have attempted to do that a sense of
peoplehood is a distinct phenomenon. Historians should instead attempt to identify directly the
factors that sustain community cohesion in generations that have adapted to United States culture
and been exposed to the pluralism of American life.

1. Which one of the following best summarizes the main point of the author of the
passage?

(A) Fugita and O’Brien’s study provides a comparison of the degree of involvement in ethnic
community of different groups in the United States.
(B) Fugita and O’Brien’s study describes the assimilation of three generations of Japanese Americans
to United States culture.
(C) Fugita and O’Brien’s study illustrates both a recent trend in historical studies of
ethnic groups and a problem typical of that trend.
(D) Historical studies of ethnic preservation among Japanese Americans have done much to define
the interpretive frameworks for studies of other ethnic groups.
(E) Historical studies are more concerned with the recent development of ethnic communities in the
United States than with the process of adaptation to United States culture.

2. According to the passage, Fugita and O’Brien’s data indicate which one of the
following about the Japanese American ethnic community?

(A) Community bonds have weakened primarily as a result of occupational mobility by Japanese
Americans.
(B) The community is notable because it has accommodated multiple and layered identities without
losing its traditional intensity.
(C) Community cohesion is similar in intensity to the community cohesion of other ethnic groups
that have been in the United States for the same period of time.
(D) Community involvement weakened during the second generation, but strengthened as the third
generation regained an interest in cultural traditions.
(E) The nature of the community has been altered by Japanese American participation in
new professional communities and nonethnic voluntary associations.

3. Which one of the following provides an example of a research study that has
conclusion most analogous to that argued for by the historians mentioned
in blue highlighted text?

(A) a study showing how musical forms brought from other countries have persisted in the United
States
(B) a study showing the organization and function of ethnic associations in the United States
(C) a study showing how architectural styles brought from other counties have merged
to form an American style
(D) a study showing how cultural traditions have been preserved for generations in American ethic
neighborhoods
(E) a study showing how different religious practices brought from other countries have been
sustained in the United States

Whereas earlier historians argued that the ethnic identity of various immigrant groups to the
United States blended to form an American national character

4. According to the passage, which one of the following is true about the focus of
historical studies on ethnic groups in the United States?

(A) Current studies are similar to earlier studies in claiming that a sense of peoplehood helps
preserve ethnic community.
(B) Current studies have clearly identified factors that sustain ethnic community in generations that
have been exposed to the pluralism of American life.
(C) Current studies examine the cultural practices that make up the American national character.
(D) Earlier studies focused on how ethnic identities became transformed in the United
States.
(E) Earlier studies focused on the factors that led people to immigrate to the United States.

..., the new scholarship has focused on the transplantation of ethnic cultures to the United States.
5. The author of the passage quotes Fugita and O’Brien in lines 36-39 most probably in
order to

(A) point out a weakness in their hypothesis about the strength of community ties among Japanese
Americans
(B) show how they support their claim about the notability of community cohesion for Japanese
Americans
(C) indicate how they demonstrate the high degree of adaptation of Japanese Americans to United
States culture
(D) suggest that they have inaccurately compared Japanese Americans to other ethnic groups in the
United States
(E) emphasize their contention that the Japanese American sense of peoplehood
extended beyond local and family ties

Fugita and O’Brien maintain that the community cohesion of Japanese Americans is notable not for
its initial intensity but because “there remains a degree of involvement in the ethnic community
surpassing that found in most other ethnic groups at similar points in their ethnic group life cycle.”

6. The passage suggests that the author would be most likely to describe the hypothesis
mentioned in line 47 as

(A) highly persuasive


(B) original but poorly developed
(C) difficult to substantiate
(D) illogical and uninteresting
(E) too similar to earlier theories

Fugita and O’Brien have explained persistence of ethnic community by citing a preexisting sense of
national consciousness that is independent of how a group adapts to United States culture.
However, it is difficult to prove as Fugita and O’Brien have attempted to do that a sense of
peoplehood is a distinct phenomenon

7. The passage suggests which one of the following about the historians mentioned
in green highlighted text?

(A) They have been unable to provide satisfactory explanations for the persistence of
European ethnic communities in the United States.
(B) They have suggested that European cultural practices have survived although the community
ties of European ethnic groups have weakened.
(C) They have hypothesized that European ethnic communities are based on family ties rather than
on a sense of national consciousness.
(D) They have argued that European cultural traditions have been transformed in the United States
because of the pluralism of American life.
(E) They have claimed that the community ties of European Americans are still as strong as they
were when the immigrants first arrived.
8. As their views are discussed in the passage, Fugita and O’Brien would be most likely
to agree with which one of the following?

(A) The community cohesion of an ethnic group is not affected by the length of time it has been in
the United States.
(B) An ethnic group in the United States can have a high degree of adaptation to United
States culture and still sustain strong community ties.
(C) The strength of an ethnic community in the United States is primarily dependent on the strength
of local and family ties.
(D) High levels of education and occupational mobility necessarily erode the community cohesion of
an ethnic group in the United States.
(E) It has become increasingly difficult for ethnic groups to sustain any sense of ethnic identity in
the pluralism of United States life.

Transplantation (n) : cấy ghép

Assimilation (n) : đồng hóa

Endure (v) : tồn tại

Persistence (n) : kiên trì

High degree of (n) : mức độ cao

Accommodate (v) : chứa, giúp đỡ

Cohesion (n) : sự gắn kết

Notable (adj) : đáng chú ý

Peoplehood (n) : tình người

Pluralism (n) : chủ nghĩa đa nguyên


Oct 28 – 1

A common misconception is that color refers only to a wavelength of light in the visual spectrum,
from about 400 nanometers (violet) to about 700 nanometers (red). When an object reflects light of
a given wavelength, we see that object as the corresponding color. So, for example, we might see a
Braebum apple as red and a Granny Smith apple as green because they reflect light of different
wavelengths. However, color is not merely a property of an external physical object but rather the
result of an interaction among that object, the light that shines on it, and, finally but most
significantly, the manner in which the human eye and brain make sense of the reflected light
stimulus. Thus, the study of color can properly fall as much within the realm of psychology as that of
physics.

Experience is one psychological factor that informs our perception of color. For example, a child
eating by a campfire that emits a great deal of yellow light may believe that the melted Cheddar
cheese served on white bread on a white paper plate is actually a white cheese like Swiss or
Monterey jack. This occurs because the yellow light reflects off both the plate and the bread, which
the child knows are white, and off the cheese, which the child isn't sure about. All the objects
therefore appear to be the same color, and the child assumes that color is white. On the other hand,
an adult with experience viewing things in firelight would intuitively adjust her perception to account
for the yellow light and would not make the same mistake.

Color is also perceived differently depending on its context. The noted abstract painter Josef Albers
produced an influential body of work based on this phenomenon, including his series Homage to the
Square featuring nested squares of different colors. In one psychological experiment testing
perception, the letter Xis presented against two colored backgrounds. Although the letter is identical
each time it is presented, it appears olive green in one context and lavender in the other context.
This effect is achieved when the X is given a low-saturation blue color, or gray-blue, and the
backgrounds are also low-saturation colors with hues on either side of blue on the color wheel.
Because blue falls between purple and green on the color wheel, a gray-blue X against a gray-purple
background will look gray-green, or olive, and the same X against an olive background will look
gray-purple, or lavender. In a similar manner, an intermediate color will look different against
different primary color backdrops; teal, for instance, will look green against a blue background and
blue against a green background.

Other subjective factors also influence the experience of color. These include cultural norms
(Westerners most often name blue as their favorite color, whereas in China red is preferred) and
simply what we learn about color. Consider that if a child learns that stop signs are "red," the child
will call them "red." Another person in that society will also have learned to call stop signs "red."
However, whether the two people are experiencing the same color is unknown since that experience
exists only in the mind. Therefore, if one were to tell an interior designer that color is an immutable
physical property of objects, one would meet with skepticism. Before placing the electric blue sofa in
a client's living room, the designer considers the color of light the various light fixtures will emanate,
the colors of the carpet and walls, and her client's feelings about electric blue, which after all may
not even be the same color in the client's mind as it is in the designer's.
1) Which of the following statements best expresses the main idea of the passage?

A) Color is primarily a psychological construct, and therefore the study of physics is not relevant to
an understanding of how color is perceived.
B) The phenomenon of color is a combined effect of the wavelength of light that shines
on an object, the wavelength of light reflected by the object, and the human mind's
perception of the light stimulus that comes to the eye.
C) Scientists have determined that although people may perceive color differently in different
situations, color is an immutable characteristic of objects.
D) Creative professionals, such as artists and interior designers, view color significantly differently
than do scientists.
E) To say that an object is a particular color is meaningless because color is a subjective perception
influenced by experience, culture, and context and cannot therefore be ascertained to be a specific
physical characteristic.

2) The author would be most likely to agree with which of the following ideas?

A) When attempting to achieve a particular aesthetic effect, a graphic designer should


consider how the color used for the border of an advertisement will appear next to the
color of the text.
B) A decorator working for a client in China would not purchase an electric blue sofa for that
individual's living room, because blue is not a preferred color in China.
C) Companies designing packaging for their products should avoid using gray tones because these
would cause different customers to see the colors differently, thereby rendering the brand message
inconsistent.
D) Because red is a primary color, a wall should not be painted red if a sofa of an intermediate color
will be placed against it, as the sofa's color may be distorted by its proximity to the wall.
E) Artists often explore the interaction of adjacent colors when juxtaposing different forms in the
composition of their paintings.

Before placing the electric blue sofa in a client's living room, the designer considers the color of light
the various light fixtures will emanate, the colors of the carpet and walls, and her client's feelings
about electric blue, which after all may not even be the same color in the client's mind as it is in the
designer's.

3) The author mentions Josef Albers in paragraph 3 in order to

A) argue that artists are aware of how humans perceive color and use this phenomenon to enhance
the impact of their work.
B) illustrate the idea that color is fundamentally a subjective, aesthetic phenomenon rather than a
scientific one.
C) demonstrate that a child would probably see a painting in the Homage to the Square series
differently than would an adult.
D) explain that humans perceive the color of regular shapes, such as squares, differently than they
perceive the color of less regular shapes, such as food on a plate or a letter of the alphabet.
E) provide an example that reinforces the importance of the concept that color is a
subjective experience manufactured in part within the human mind.
Proximity (n) : gần như

Hues (n) : màu sắc

Realm of (n) : lĩnh vực

Intuitively (adv) : bằng trực giác

Low-saturation (n) : độ bão hòa thấp

Nest (v) : lồng vào nhau

Immutable (adj) : bất biến, không thể phai

Fixture (n) : đồ đạc

Emanate (v) : phát ra


Oct 28 – 2

Our current system of unemployment compensation has increased nearly all sources of adult
unemployment: seasonal and cyclical variations in the demand for labor, weak labor force
attachment and unnecessarily long durations of unemployment. First, for those who are already
unemployed, the system greatly reduces the cost of extending the period of unemployment. Second,
for all types of unsteady work—seasonal, cyclical, and casual—it raises the net wage to the
employee, relative to the cost of the employer. As for the first, consider a worker who earns $500
per month or $6,000 per year if she experiences no unemployment. If she is unemployed for one
month, she loses $500 in gross earnings but only $116 in net income. How does this occur? A
reduction of $500 in annual earnings reduces her federal, payroll and state tax liability by $134.
Unemployment compensation consists of 50% of her wage or $250. Her net income therefore falls
from $366 if she is employed, to $250 paid as unemployment compensation.

Moreover, part of the higher income from employment is offset by the cost of transportation to work
and other expenses associated with employment; and in some industries, the cost of unemployment
is reduced further or even made negative by the supplementary unemployment benefits paid by
employers under collective bargaining agreements. The overall effect is to increase the duration of a
typical spell of unemployment and to increase the frequency with which individuals lose jobs and
become unemployed. The more general effect of unemployment compensation is to increase the
seasonal and cyclical fluctuations in the demand for labor and the relative number of short-lived
casual jobs. A worker who accepts such work knows she will be laid off when the season ends. If
there were no unemployment compensation, workers could be induced to accept such unstable jobs
only if the wage rate were sufficiently higher in those jobs than in the more stable alternative. The
higher cost of labor, then, would induce employers to reduce the instability of employment by
smoothing production through increased variation in inventories and delivery lags, by additional
development of off-season work, and by the introduction of new production techniques, e.g., new
methods of outdoor work in bad weather.

Employers contribute to the state unemployment compensation fund on the basis of the
unemployment experience of their own previous employees. Within limits, the more benefits that
those former employees draw, the higher is the employer’s tax rate. The theory of experience rating
is clear. If an employer paid the full cost of the unemployment benefits that his former employees
received, unemployment compensation would provide no incentive to an excess use of unstable
employment. In practice, however, experience rating is limited by a maximum rate of employer
contribution. For any firm that pays the maximum rate, there is no cost for additional unemployment
and no gain from a small reduction in unemployment. The challenge at this time is to restructure the
unemployment system in a way that strengthens its good features while reducing the harmful
disincentive effects. Some gains can be achieved by removing the ceiling on the employer’s rate of
contribution and by lowering the minimum rate to zero. Employers would then pay the full price of
unemployment insurance benefits and this would encourage employers to stabilize employment and
production. Further improvement could be achieved if unemployment insurance benefits were taxed
in the same way as other earnings. This would eliminate the anomalous situations in which a
worker’s net income is actually reduced when he returns to work.
1. The author’s primary concern is to

(A) defend the system of unemployment compensation against criticism


(B) advocate expanding the benefits and scope of coverage of unemployment compensation
(C) point to weaknesses inherent in government programs that subsidize individuals
(D) suggest reforms to eliminate inefficiencies in unemployment compensation
(E) propose methods of increasing the effectiveness of government programs to reduce
unemployment

2. The author cites the example of a worker earning $500 per month in order to

(A) show the disincentive created by unemployment compensation for that worker to
return to work
(B) demonstrate that employers do not bear the full cost of worker compensation
(C) prove that unemployed workers would not be able to survive without unemployment
compensation
(D) explain why employers prefer to hire seasonal workers instead of permanent workers for short-
term jobs
(E) condemn workers who prefer to live on unemployment compensation to taking a job

3. The author recommends which of the following changes be made to the


unemployment compensation system?

(A) Eliminating taxes on benefits paid to workers


(B) Shortening the time during which a worker can draw benefits
(C) Removing any cap on the maximum rate of employer contribution
(D) Providing workers with job retraining as a condition of benefits
(E) Requiring unemployed workers to accept public works positions

4. The author mentions all of the following as ways by which employers might reduce
seasonal and cyclical unemployment EXCEPT

(A) developing new techniques of production not affected by weather


(B) slowing delivery schedules to provide work during slow seasons
(C) adopting a system of supplementary benefits for workers laid off in slow periods
(D) manipulating inventory supplies to require year-round rather than short-term employment
(E) finding new jobs to be done by workers during the off-season

The higher cost of labor, then, would induce employers to reduce the instability of employment by
smoothing production through increased variation in inventories and delivery lags, by additional
development of off-season work, and by the introduction of new production techniques, e.g., new
methods of outdoor work in bad weather.

5. With which of the following statements about experience rating would the author
most likely agree?

(A) Experience rating is theoretically sound, but its effectiveness in practice is


undermined by maximum contribution ceilings.
(B) Experience rating is an inefficient method of computing employer contribution because an
employer has no control over the length of an employee’s unemployment.
(C) Experience rating is theoretically invalid and should be replaced by a system in which the
employee contributes the full amount of benefits he will later receive.
(D) Experience rating is basically fair, but its performance could be improved by requiring large firms
to pay more than small firms.
(E) Experience rating requires an employer to pay a contribution that is completely unrelated to the
amount his employees draw in unemployment compensation benefits.

6. The author makes which of the following criticisms of the unemployment


compensation system?

(A) It places an unfair burden on firms whose production is cyclical or seasonal.


(B) It encourages out-of-work employees to extend the length of time they are
unemployed.
(C) It constitutes a drain on state treasuries, which must subsidize unemployment compensation
funds.
(D) It provides a source of income for employees who have no income or have only reduced income
from employment.
(E) The experience rating system means that employers responsible for higher than-average
turnover in staff pay higher-than-average premiums.

Our current system of unemployment compensation has increased nearly all sources of adult
unemployment: seasonal and cyclical variations in the demand for labor, weak labor force
attachment and unnecessarily long durations of unemployment

"The overall effect is to increase the duration of a typical spell of unemployment and to increase the
frequency with which individuals lose jobs and become unemployed. The more general effect of
unemployment compensation is to increase the seasonal and cyclical fluctuations in the demand for
labor and the relative number of short-lived casual jobs."

Unemployment compensation (n) : trợ cấp thất nghiệp

Cyclical (adj) : theo chu kỳ

Supplementary (adj) : bổ sung

Collective bargaining agreement (n) : thỏa ước lao động tập thể

Induce (v) : gây ra, thuyết phục, ảnh hưởng

Former employee (n) : nhân viên cũ

Anomalous (adj) : dị thường


Oct 29 – 1

In the past decade, rapid technological progress and a greater demand for high-quality digital
imaging have led to dramatic advances in video display technology. The dominant technology
currently used in most consumer product displays is the active matrix liquid crystal diode display
(LCD).

LCDs apply thin-film transistors (TFTs) of amorphous or polycrystalline silicon sandwiched between
two glass plates. The TFTs supply voltage to liquid-crystal-filled cells, or pixels, between the sheets
of glass. When hit with an electric charge, the liquid crystals untwist to an exact degree to filter
white light generated by a lamp. This filtered light shines directly on the viewing screen or, in the
case of projection televisions, is projected through a small chip that acts as a lens. LCDs that are
capable of producing color images, such as in televisions and computers, reproduce colors through a
process of subtraction, blocking out particular color wavelengths from the spectrum of white light
until only the desired color remains. It is the variation of the intensity of light permitted to pass
through the matrix of liquid crystals that enables LCD displays to present images full of gradations of
different colors.

The nature and functioning of LCD displays present many advantages relative to other display
technologies. The amount of power required to untwist the crystals to display images, even dark
ones, is much lower than that required for analogous processes using other technologies, such as
plasma. The dense array of crystals displays images from computer or other video graphics sources
extremely well, with full color detail, no flicker, and no screen burning. Moreover, the number of
pixels per square inch on an LCD display is typically higher than that for other display technologies,
so LCD monitors are particularly good at displaying large amounts of data with exceptional clarity
and precision. As a result, LCD TVs are considered the best display platform for video games, high
definition television, movie special effects, and other graphics intensive uses.

1. The process through which an LCD monitor displays different colors is most closely
analogous to

A. An hourglass partially blocked such that a limited stream of grains of sand fall into the lower
portion

although it does involve subtraction and filtering, but the subtraction is of no use or good, and since
all the particles are of similar size and shape so there is no bias among which particles are allowed
and which ones are stopped.

B. A series of filters that separate all of the components of a mixture according to size
C. A recording studio soundproofed so that any performances within are muted to those outside
D. A piece of construction paper with outlines of characters cut out such that a lamp in
front of the paper casts shadows in the shapes of the characters
E. An air vent that emits warmer air outside of a building while an air conditioning system cools the
interior of the building

" LCDs that are capable of producing colour images, such as in televisions and computers, reproduce
colours through a process of subtraction, blocking out particular colour wavelengths from the
spectrum of white light until only the desired colour remains."
2. It can be inferred from the passage that an LCD monitor would utilize the most
electrical power when displaying

A. a movie scene of a NASCAR race with many computergenerated special effects


B. the introduction sequence for a World War II video game
C. a bright daytime television broadcast of a parade
D. a photorealistic screen saver of a bouquet of flowers
E. a filmed scene set inside a cave with minimal lighting

The amount of power required to untwist the crystals to display images, even dark ones, is much
lower than that required for analogous processes using other technologies, such as plasma.

 it means that darker  more electrical consuming, a high degree of darkness, and thus a high
level of electricity use.

3. The passage indicates that each of the following may be considered an advantage of
LCD displays relative to other display technologies EXCEPT

A. they consume less power


B. they generally have more pixels per square inch of the display
C. they are able to display continuous video graphics images with no interruption
D. they reflect a widely adopted standard
E. they represent the latest, most advanced technology

4. The tone of the passage could best be described as

A. supportive advocacy
B. historical discussion
C. objective explanation
D. intellectual exploration
E. qualified support

To get this question you should be clear that NO OPINION of author is presented throughout the
passage, even If the author hates LCD displays he still may write such a passage

Subtraction (n) : sự khử đi

Piece of construction (n) : giấy thủ công

Air vent (n) : lỗ thông hơi

Hourglass (n) : đồng hồ cát


Oct 29 – 2

Green Fluorescent Protein (GFP) is a protein that emits bright light in the green segment of the
visible spectrum upon exposure to blue light. Its properties were first researched in 1962 by Osamu
Shimomura, a Japanese scientist who studied the bioluminescence of Crystal Jellyfish, or Aequorea
victoria. Shimomura discovered that such biologically-generated glow is produced when the jellyfish
produces calcium ions. The ionized calcium is absorbed by a protein that produces blue
luminescence upon binding with calcium, called Aequorin. Crystal jellyfish typically have nodes of
Aequorin on the rim of their umbrella, but Shimomura found that these photosensitive organs are
composed of another type of protein. The blue light resulting from the Aequorin calcium binding is
received by the protein now known as GFP. Since GFP's major excitation peak is at a wavelength of
395nm, or in other words - blue light, it becomes active and emits visible light whose peak is at
509nm, i.e., green light. This creates a circle of greenish light around the jellyfish's umbrella.
Shimomura et al received a Nobel Prize in Chemistry for the discovery and development of GFP.

GFP has numerous uses in cellular and molecular biology, most commonly as a reporter of
expression. In layman's terms, Green Fluorescent Protein is used as an indicator to the activation of
certain genes into which it is spliced. This means that only the cells in which those specific genes are
activated will fluoresce green under a blue light. This has been tremendously useful in mapping out
the connection between DNA strings in genes and the biological systems which they control.

Another field where GFP was found to be profoundly useful is fluorescence microscopy. Before the
discovery of GFP and its derivatives, fluorescent microscopists used synthetic fluorophores to dye
the target sample. However, such fluorophores were found to be highly phototoxic, as the light they
emitted was transferred to oxygen molecules in the cells under examination. This provoked the
creation of oxygen radicals that could kill a living cell in seconds. GFP has proven to be dramatically
less phototoxic, thereby revolutionizing this field of science.

1. The relationship between cells under examination in fluorescence microscopy and


oxygen molecules is most similar to which of the following?

A. the relationship between human beings and the air they breathe
B. the relationship between the ecosystem and industrial pollution poisoning it
C. the relationship between a sovereign state in which the citizens are deliberating a
revolt and those citizens
D. the relationship between a machine and a faulty cogwheel
E. the relationship between two enemy armies ready for war

This provoked the creation of oxygen radicals that could kill a living cell in seconds.

2. The author refers to oxygen radicals in order to point out

A. the potential danger to human life when using a certain kind of fluorophore
B. that jellyfish can use their poison to introduce toxins to their victim's system
C. the reason for which GFP is no longer used in fluoresence microscopy
D. a negative effect that the use of GFP as an alternative dye in fluoresence microscopy
has significantly reduced
E. the importance of GFP in the field of cellular biology
3. It can be inferred from the passage that Crystal Jellyfish

A. must have a constant supply of calcium to survive


B. have enough GFP to illuminate their way in the dark
C. do not require anything to spread a green glow
D. do not require an external blue light source to spread a green glow
E. use the circle of light on their umbrella to deter underwater predators

4. Which of the following best describes the relation of the second paragraph to the
passage as a whole?

A. It creates a logical connection between the first and third paragraphs.


B. It serves an an example of the theory presented in the first paragraph.
C. It presents a certain application, which is contrasted in the third paragraph.
D. It is the first of two paragraphs describing applications of the information presented
in the first parargraph.
E. It repeats the technical information presented in the first paragraph in simpler terms.

Bio-luminescene (n) : phát quang sinh học

Node (n) : hạch

Rim (n) : vành

Photosensitive (adj) : cảm quang

Cellular (adj) : tế bào

Molecular (adj) : phân tử

Flouresce (n) : huỳnh quang

Derivatives (n) : các dẫn xuất

Radical (n) : gốc tự do


Oct 30 – 1

In the 1960s, Northwestern University sociologist John McKnight coined the term redlining, the
practice of denying or severely limiting service to customers in particular geographic areas, often
determined by the racial composition of the neighborhood. The term came from the practice of
banks outlining certain areas in red on a map; within the red outline, banks refused to invest. With
no access to mortgages, residents within the red line suffered low property values and landlord
abandonment; buildings abandoned by landlords were then more likely to become centers of drug
dealing and other crime, thus further lowering property values.

Redlining in mortgage lending was made illegal by the Fair Housing Act of 1968, which prohibited
such discrimination based on race, religion, gender, familial status, disability, or ethnic origin, and by
community reinvestment legislation in the 1970s. However, redlining has sometimes continued in
less explicit ways, and can also take place in the context of constrained access to health care, jobs,
insurance, and more. Even today, some credit card companies send different offers to homes in
different neighborhoods, and some auto insurance companies 15 offer different rates based on zip
code.

Redlining can lead to reverse redlining, which occurs when predatory businesses specifically target
minority or low income consumers for the purpose of charging them more than would typically be
charged for a particular service. When mainstream retailers refuse to serve a certain area, people in
that area can fall prey to opportunistic smaller retailers who sell inferior goods at higher prices.

1. Which of the following, not mentioned in the passage, would qualify as an example of
reverse redlining as defined in the passage?

(A) A bank refuses to offer mortgages to consumers in certain neighborhoods.


(B) Residents of low-income neighborhoods are less likely to be hired for positions than residents of
higher-income neighborhoods, even when the applicants have the same qualifications.
(C) Police respond to reports of crimes more quickly in some neighborhoods than in others.
(D) A grocery store in a low-income neighborhood sells low-quality produce for high
prices, knowing that most residents do not have the ability to buy elsewhere.
(E) An auto insurance company hires an African American spokesperson in a bid to attract more
African American consumers.

2. Which correctly describes a sequence of events presented in the passage?

(A) Subprime mortgages lead to widespread defaults, which lead to landlord abandonment.
(B) Reverse redlining leads to landlord abandonment, which leads to the use of buildings for crime
and drug dealing.
(C) Landlord abandonment leads to redlining, which leads to crime and drug dealing.
(D) Redlining leads to reverse redlining, which leads to constrained access to health care, jobs,
insurance, and more.
(E) Redlining leads to landlord abandonment, which leads to the use of buildings for
crime and drug dealing.

3. Which of the following can be inferred from the passage?


i) Redlining ceased with the passing of the Fair Housing Act in 1968.
ii)Providing services based on zip code may be a form of redlining.
iii)Access to mortgages is related to higher property values

a) i only
b) ii only
c) iii only
d) i & ii only
e) ii and iii Only

cease (v) : ngưng, không làm, chấm dứt

Coin (v) : đặt ra

Predatory (adj) : ăn cướp, cướp bóc


Oct 30 – 2

One of the most studied senses is vision. Scientists have carefully unravelled the connections of
brain cells in the visual system and have studied how they respond to light, so we have many clues
about how the brain takes visual images apart. What is particularly elusive, however, is how the
brain puts the pieces back together, turning two-dimensional patterns of light on the retinas into our
perception of the visual world. In one case, however, the perception of color, we are beginning to
get a good idea of how the brain operates.

Most people think that the balance of red, green and blue light reflected from an object into the eye
determines the object's color. It is easy to demonstrate that this notion is not true, however, simply
by noting that objects remain the same colour in daylight, fluorescent light and incandescent light,
each of which contains a mix of wavelengths of light very different from the others. Edwin Land,
inventor of the instant camera, has provided an explanation of this phenomenon in what he calls the
retinex theory, a term that combines 'retina' and 'cortex' to suggest that both pars of the visual
system are involved in perceiving color.

Retinex theory proposes that the retina and the cortex co-operate to perform some complex
computations on the basis of light received from all areas within the visual landscape. A separate
computation is carried out for each of three wavelengths of light that correspond to what we
normally think of as red, green, and blue: the wavelengths to which the three types of receptors in
the retina are most sensitive. According to the theory. the color we perceive at a particular location
is determined by three numbers, computed by dividing the amount of light received from that
location at each wavelength by a weighted average of the amount of light at that wavelength
received from all parts of the field of vision. The weighted average gives more weight to light
coming from close to the location than to that coming from far away. The three numbers, co-
ordinates in a color space of three dimensions, uniquely determine the color we see just as the three
dimensions of physical space uniquely define the location of an object Land has conducted a number
of experiments showing that the numbers computed in this way correctly predict what color an
observer will see under a number of unusual lighting conditions.

This remarkable theory suggests that our visual systems evolved so that we see the colors of objects
as the same. regardless of the mix of wavelengths of light falling on our retinas. Furthermore. this
complex computation is carried out virtually instantaneously without our even being aware of it.

Q1: The passage suggests that Edwin Land created the name retinex for his optical
theory in order to

a) distinguish his theory from rival theories of the retina's operation.


b) indicate that both the retina and the cortex are involved in color perception.
c) differentiate between the actions of the retina and the actions of the cortex.
d) imply that properties ascribed to the retina actually belong to the cortex.
e) indicate that the cortex and the retina work together in perceiving location.
Q2: It can be inferred from the passage that if the balance of red, green and blue light
entering the eye determined color, the apparent color of an object could be expected to
change if the object were moved

a) from a blue background to a bright yellow background.


b) from a sunlit room to a room with fluorescent lights.
c) to a different set of coordinates in physical space.
d) close enough to take up the viewer's entire field of vision.
e) to a new area in the viewer's visual landscape.

Q3: According to the passage, the proportions of red, green and blue light reflected by
an object cannot be the sole determinants of the object's color because:

a) color information about three wavelengths is not sufficient to produce the full spectrum of
possible colors.
b) the perceived color of an object changes with the ambient lighting of the object's environment.
c) the image of an object is formed not by light coming from the object itself, but from other parts
of the field of vision.
d) variations in the mix of wavelengths illuminating an object do not affect its color.
e) this information varies according to the object's proximity to the viewer.

Most people think that the balance of red, green and blue light reflected from an object into the eye
determines the object's color. It is easy to demonstrate that this notion is not true, however, simply
by noting that objects remain the same colour in daylight, fluorescent light and incandescent
light, each of which contains a mix of wavelengths of light very different from the others.

Incandescent (adj) : sợi đốt, chói sáng

Instantaneously (adv) : tức thời


Oct 31 – 1

No one can say where the bones of Machiavelli rest, but modern Florence has decreed him a stately
cenotaph in Santa Croce, by the side of her most famous sons, recognizing that, whatever other
nations may have found in his works, Italy found in them the idea of her unity and the germs of her
renaissance among the nations of Europe. Whilst it is idle to protest against the world-wide and evil
signification of his name, it may be pointed out that the harsh construction of his doctrine which this
sinister reputation implies was unknown to his own day, and that the researches of recent times
have enabled us to interpret him more reasonably. It is due to these inquiries that the shape of an
"unholy necromancer," which so long haunted men's vision, has begun to fade.

Machiavelli was undoubtedly a man of great observation, acuteness, and industry noting with
appreciative eye whatever passed before him, and with his supreme literary gift turning it to account
in his enforced retirement from affairs. He does not present himself, nor is he depicted by his
contemporaries, as a type of that rare combination, the successful statesman and author, for he
appears to have been only moderately prosperous in his several embassies and political
employments. He was misled by Catherina Sforza, ignored by Louis XII, overawed by Cesare Borgia;
several of his embassies were quite barren of results; his attempts to fortify Florence failed, and the
soldiery that he raised astonished everybody by their cowardice. In the conduct of his own affairs he
was timid and time-serving; he dared not appear by the side of Soderini, to whom he owed so
much, for fear of compromising himself; his connection with the Medici was open to suspicion, and
Giuliano appears to have recognized his real forte when he set him to write the "History of
Florence," rather than employ him in the state. It is on the literary side of his character, and there
alone, that we find no weakness and no failure.

Q1) What is the primary purpose of the second paragraph in the passage?

A. To highlight major contributions of Machiavelli


B. To praise Machiavelli‘s keen observation skills
C. To discuss contrasting aspects of Machiavelli‘s personality
D. To conclude that Machiavelli was a great author
E. To state that Machiavelli was not as wicked as a person as he has been historically made out to
be

Q2) Which of the following can be inferred about Machiavelli from the passage?

A. He was renowned for his oratory skills


B. The people of his time did not view him as harshly as did later generations
C. He was a great statesman and author
D. He was a poor soldier
E. Machiavelli was buried in Italy

Q3) According to the information in the passage, why did Italy provide Machiavelli a
cenotaph next to her most famous sons?

A. Machiavelli was responsible for freeing Italy from the clutches of slavery
B. Machiavelli‘s works made Italy famous amongst the European nations
C. Machiavelli‘s works were responsible for the re-emergence of Italy as a unified nation
D. Machiavelli had been unjustly treated by other countries of the world
E. Machiavelli started the Italian renaissance by encouraging Italians to be united
No one can say where the bones of Machiavelli rest, but modern Florence has decreed him a stately
cenotaph in Santa Croce, by the side of her most famous sons, recognizing that, whatever other
nations may have found in his works, Italy found in them the idea of her unity and the germs of
her renaissance among the nations of Europe

Q4) The passage implies each of the following EXCEPT:

A. Several of Machiavelli‘s assignments as a statesman did not bear any results


B. At some point of time Machiavelli was assigned the task of protecting Florence
C. Machiavelli was not very aggressive in his conduct
D. Machiavelli was a better writer than his contemporaries
E. Modern research has helped diminish the negative aura around Machiavelli

Unify (v) : thống nhất

Unity (n) : đoàn kết

Germ (n) : mầm mống

Renaissance (n) : phục hưng, khôi phục

Decree (v) : ra sắc lệnh

Stately (adj) : trang nghiêm, oai vệ

Cenotaph (n) : bia mộ, đài kỷ niệm

Whilst (con) : trong khi

Harsh (adj) : khắc nghiệt

Sinister (adj) : nham hiểm, độc ác, xấu xa

Inquiry (n) : điều tra

Unholy (adj) : không vui, báng bổ thần thánh

Necromancer (n) : thầy chiêu hồn, kẻ phá hoại

Acuteness (n) : sự nhạy bén

Enforced retirement (n) : nghỉ hưu cưỡng chế

Affair (n) : công việc

Depict (v) : miêu tả

Statesman (n) : chính khách

Prosperous (adj) : thịnh vượng, phát đạt, thành công

Mislead (v) : lừa dối

Overawe (v) : giám sát, bảo vệ

Barren (adj) : cằn cỗi


Fortify (v) : củng cố, làm mạnh thêm

Soldiery (n) : quân nhân, quân đội

Astonish (v) : ngạc nhiên, kinh ngạc

Cowardice (n) : hèn nhát

Timid (adj) : nhút nhát

Compromise (v) : thỏa hiệp, hòa giải

Forte (n) : sở trường, ưu điểm


Oct 31 – 2

Freud is considered an early expert on the subject of grief. In his writings, one of the primary tasks
faced by the bereaved individual was reclaiming the energy that had been invested in the loved one.
Later theorists added to our understanding of grief by reflecting on the role of socialization and the
relationship with the deceased in the experience of grief. Bowlby identified four phases through
which the grieving person passes: numbing, yearning and searching, disorganization and despair,
and reorganization. Still others have suggested that responses to grief are predictable, regardless of
the individual’s personality characteristics and personal coping skills. A recent theory also suggests
that grief is not only psychological but also biological, and represents the individual’s movement
toward reintegration and balance.

One concern frequently expressed by critics of the stage models of grief is general misunderstanding
about how the stages are experienced. The average person and even many mental health
practitioners tend to see the stages of grief as linear and as descriptions of how grief should be
experienced. This can make grieving individuals feel as if they must ‘progress’ through the stages in
a timely manner and that something is wrong if they do not do so. The authors of the various stage
models did not make this assumption, however. Instead, they acknowledged that the way grief is
expressed, the timing and sequence of stages, the duration for which grief lasts, and the coping
responses used were varied. Another criticism is that these models assume that the outcome of the
grief process will be a return to a more ‘normal’ psychological state. Some experts, on the contrary,
suggest that a person does not ever return to the state of mind prior to the loss, and long-term grief
may be normal for some people. This aspect needs further examination of theories before the merit
of this criticism can be objectively judged.

1.What is the author’s attitude on the theories suggesting stage models of grief?

A. The author considers these theories plausible.

Nowhere the author disagrees with the theories discussed.


B. The author finds the stages to be predictable.
C. The author questions the chronological nature of the theories.
D. The author is convinced that a final state of normalcy exists.
E. The author finds similarity among the various stage models.

Another criticism is that these models assume that the outcome of the grief process will be a return
to a more ‘normal’ psychological state. Some experts, on the contrary, suggest that a person does
not ever return to the state of mind prior to the loss, and long-term grief may be normal for some
people. This aspect needs further examination of theories before the merit of this criticism can
be objectively judged.

2.Which of the following statements is the author most likely to agree with?

A. The latest theories of grief find their base in the work of Freud.
B. The critics of stage models have incorrectly made certain assumptions about the
stage model theories.
C. The ‘coming to normalcy’ idea of stage model theories is incorrectly interpreted by the critics of
these theories.
D. The stages of grief are linear and time-bound.
E. Socialization can help in refocusing energy invested in the relationship with the deceased.

One concern frequently expressed by critics of the stage models of grief is general
misunderstanding about how the stages are experienced

Grief (n) : đau buồn

Bereave (v) : mất người thân

Deceased (adj) : người chết

Numb (v) : tê cóng, không cảm giác

Yearn (v) : khao khát

Despair (v) : tuyệt vọng

Reintegration (n) : tái hòa nhập

Linear (adj) : tuyến tính

Merit (n) : xứng đáng, chất lượng

Objectively (adv) : khách quan


Nov 1 – 1

Some writers have so confounded society with government, as to leave little or no distinction
between them, whereas they are not only entirely different, but have different origins. Society is a
blessing brought forth naturally by our wants, uniting our affections and promoting our happiness.
Government is a necessary evil originating from the need to restrain our vices.

Suppose a small number of persons represent the first peopling of any country, or of the world. In
this state of natural liberty, a thousand motives will excite them to society: the strength of one is so
unequal to his wants, and his mind so unfitted for perpetual solitude, that he is soon obliged to seek
assistance and relief from another, who in turn requires the same. Considering the slavish times in
which it developed, the form of government known as ―constitutional monarchy‖ is granted to have
been a noble creation. When the world was overrun with tyranny, the least remove therefrom was a
glorious rescue. However, government, if unchecked, evolves over time to a form so complex that a
nation may suffer for years without being able to discover in which part the fault lies; and every
political physician will advise a different medicine.

Four or five united in a society would be able to raise a dwelling, but one might labour out the
period of life without accomplishing anything. Disease or misfortune could soon reduce an individual
to a state in which he could easily perish. As time passes, however, in proportion as they surmount
their early difficulties, the people will inevitably relax in their duty and attachment to each other;
and this laxity will point out the necessity for each to surrender a part of his property in order to
establish some form of government to protect the rest. Here then is the origin of government: the
inability of moral virtue to govern the world; here, too, is the design and end of government:
freedom and security.

And it unanswerably follows that whatever form of government which appears most likely to ensure
the protection which constitutes government‘s essential purpose, with the least expense, is
preferable to all others. As the community expands, public concerns will increase and the distance at
which the members are separated may render it inconvenient for all to meet on every occasion.
Thus the members may consent to leave the legislative part to be managed by a number of chosen
representatives, who are supposed to have the same concerns as those who appointed them, and
who will act in the same manner as the whole would, if present.

That the interest of every part of the colony may be attended to, the whole may be divided into
convenient parts, each part sending its proper number. And so that there be assured a common
interest with every part of the community, on which the strength of government depends, prudence
will point to the need for frequent elections, thereby assuring that the elected return and mix often
with the community.

1. As evidenced by the arguments posed by the author in each paragraph, the primary
purpose of the passage is to:

A. chronicle the development of a particular form of government.


B. advocate a simple form of representative government.
C. contrast society and government.
D. distinguish representative government from constitutional monarchy.
E. criticise all forms of government as an unnecessary burden on people
The para doesn't contract between S & G. Yes, author starts with a difference between S & G, but
then explains how S helps in forming G --> goes beyond and explains key aspects of G.

2. The author concluded in the passage that the essential purpose of government is
protection of property. In doing so the author assumes that:

I. there actually existed a time in which the disparity between an individual‘s needs and wants
motivated cooperation, and not transgressions against property.
II. the part of property surrendered to establish some form of government is less than that which
would be lost if it were left unprotected.
III. the moral laxity resulting from reduction in hardship results in acts against property, rather than
failure to assist those experiencing disease or misfortune.

A. I, II, and III


B. II and III only
C. I and II only
D. I and III only
E. II only

3. It can be inferred from the passage that its author would most probably respond to
the view that the resources of government should be employed to relieve the effects of
poverty by stating that:

A. since the strength of an individual must be recognized to, at times, be unequal to his needs, it is
natural for government, once it has evolved, to perform such functions.
B. these activities should be performed by individuals or associations outside of
government.
C. since poverty is correlated with crime against property, government must perform these functions
if non-governmental efforts are not fully effective.
D. this should be decided by the representatives elected by the people as a whole.
E. relieving poverty would be impossible unless efforts were taken to reduce illiteracy

Confound (v) : bối rối, làm hỏng, làm sai

Affection (n) : tình cảm

Vices (n) : tệ nạn, tật xấu, thói xấu

Excite (v) : kích thích

Perpetual (adj) : vĩnh viễn, mãi mãi

Solitude (n) : cô đơn

Oblige (v) : bắt buộc, ép buộc

Slavish (adj) : thuộc về nô lệ

Constitutional monarchy (n) : chế độ quân chủ lập hiến

Grant (v) : thừa nhận, ban cho

Noble (adj) : cao quý, vĩ đại


Overrun (v) : tràn ngập, vượt qua, vượt khỏi

Tyranny (n) : chuyên chế, bạo quân

Evolve (v) : tiến hóa

Dwelling (n) : nhấn mạnh vấn đề

Perish (v) : chết, suy đồi, diệt vong, làm hư hỏng

Surmount (v) : vượt qua, khắc phục

Inevitably (adv) : chắc chắn

Laxity (n) : lỏng lẻo, không rõ ràng, tình trạng suy đồi

Consent (v) : bằng lòng, tán thành

Legislative (adj) : lập pháp

Prudence (n) : sự thận trọng, cẩn thận, khôn ngoan


Nov 1 – 2

Enacted on March 11, 1941, the Lend-Lease Act authorised the United States president to provide
Great Britain and its allies with weapons and military aid during World War II. In lobbying for the
act, U.S. President Franklin Roosevelt compared the proposed law to lending one’s neighbor a hose
to help put out a house fire, quoting: "I don't say... 'Neighbor, my garden hose cost me $15; you
have to pay me $15 for it.’ I don't want $15. I want my garden hose back after the fire is over."

Many isolationist opponents of the act saw it less as a neighbourly gesture and more as a
commitment to ultimately join the war, noting that it tied American finances directly to Allied success
against Germany. Most historians tend to agree, but view it positively and note that it was an
important turning point in a war that at the time was heavily controlled by the Germans. The act,
they explain, was the only way that the American wartime industry would have been prepared to
supply its own military effort when the U.S. entered the war later that year.

While the Lend-Lease Act formally ended late in 1945, its legacy lived on in the postwar aftermath
and beyond. The congressional fear of increased presidential power may have come to fruition;
while Congress declined to authorize the Lend-Lease products as gifts, maintaining that Britain
would indeed need to pay for them, the executive branch granted discounts as high as 90% to
effectively render much of the material gifts. Even so, the final debt remained unpaid until 2006,
when the British, after many deferrals, made their final payment to the American government.

1.According to the passage, which of the following would likely have occurred had the
Lend-Lease Act not been passed?

A Germany would have maintained control of World War II well into 1942.
B The United States would have had no financial interest in Allied success in World War II.
C The United States would not have been willing to join the Allied effort in World War II.
D American industry would have been unprepared to support the American war effort.
E Congress would not have approved another act offering to donate military supplies directly to the
British.

2.The primary purpose of the passage is to:

A Criticize the isolationists who opposed the Lend-Lease Act


B Point out that the Lend-Lease Act was a positive aspect of Franklin Roosevelt’s legacy
C Discuss the cases made by both those who favored and those who opposed the Lend-Lease Act
D Provide an overview of the history of a significant piece of legislation
E Argue that the British took too long to repay their debts from World War II

3.Which of the following can be inferred from the passage?

A. The British were among the nations that could claim victory in World War II.
B. The passage of the Lend-Lease Act was controversial.
C. The Lend-Lease Act served as a catalyst for American industry to produce military
supplies.
D. Historians believe that the Allies could not have won the war without the Lend-Lease Act.
E. The British economy was shaken enough by World War II that it could not afford to repay its
Lend-Lease Act debts.
Enact (v) : ban hành

Lobbying (n) : vận động hành lang

Hose (n) : vòi nước

Isolationist (adj) : cô lập

Opponent (n) : đối thủ

Aftermath (n) : hậu quả

Fruition (n) : kết quả, hệ quả nhận thấy

Deferral (n) : trì hoãn


Nov 2 – 1

The recurring theme of equality in the United States has flared into a fervent moral issue at crucial
stages: the Revolutionary and Jacksonian periods, the Civil War, the populist and progressive eras,
the New Deal, and the 1960s and 1980s. The legitimacy of American society is challenged by some
set of people unhappy with the degree of equality. New claims are laid, new understandings are
reached, and new policies for political or economic equality are instituted. Yet the equality
issueendures outside these moments of fervour. Ideologies in favour of
extending equality are arrayed against others that would limit its scope; advocates of social justice
confront defenders of liberty.

In the moments of egalitarian ascendancy, libertarians are on the defensive. In the moments of
retrenchment, egalitarians cling to previous gains. And in either period the enemy is likely to be the
"special interests" that have too much power. In egalitarian times, these are the moneyed interests.
In times of retrenchment, these are labour or big government and its beneficiaries.

The moments of creedal passion, in Samuel Huntington‘s words, have usually been outbursts of
egalitarianism. In part, the passion springs from the self-interest of those who would benefit from a
more equal distribution of goods or political influence. But the passion also springs from ideology
and values, including deep religious justifications for equality.

The passion accompanying the discovery or rediscovery that ideals do not match reality is
particularly intense when the ideal is as deeply felt as is equality. Yet there can be passion on the
non-egalitarian side as well. The self-interested passion to protect an established position may be
even more powerful than the passion to redress inequality, though its expression may be more
muted.

Devotion to inequality may also be based on ideals, such as liberty, individualism, and the free
market, which are no less ancient and venerable. Like the ideals of equality, these alternative ideals
serve as yardsticks for measuring whether society has moved away from its true principles.

Yet the spirit of reform during Reconstruction dissipated in the face of spent political struggles,
sluggish social institutions, and outright mendacity. Society‘s entrepreneurial energy was channelled
into economic activity, and the courts failed to endorse many of the reformers‘ grandest visions. The
egalitarian thrust of the Populists around the turn of the century inspired an anti-egalitarian
counterthrust over the next two decades.

Americans do not have an ideology that assigns clear priority to one value over any other. At every
historical juncture where equality was an issue, its proponents failed to do all that they had set out
to do. Swings in the equality of social conditions are restrained not just by institutional obstacles but
by fundamental conflicts of values that are a traditional element of American politics. Faith in the
individualistic work ethic and belief in the legitimacy of unequal wealth retard progression to the
egalitarian left. As for conservatism, the indelible tenet of political equality firmly restrains the right
and confirms a commitment to the disadvantaged. In seeking equal opportunity over equal result,
Americans forego a ceiling, not a floor.
1. Suppose there is a government plan to raise taxes to pay for more social programs for
the disadvantaged. If the information that the author presents in the passage about
libertarians is correct, how would libertarians be expected to react this plan?

A. They would support the plan because they think that the government should help the
disadvantaged.
B. They would condemn the plan because they do not think that the government should
use its power to redistribute wealth.
C. They would neither support nor condemn the plan because it does not address political values.
D. They would call on the government to let private welfare agencies look after the disadvantaged.
E. The would partly support and partly condemn the plan

2. The existence of which of the following would most strongly challenge the author‘s
view about the American public‘s ideology?

A. A study that demonstrates that Americans have always favoured equality above all
other political values
B. A book that asserts that Americans have always believed in the economic principle of unequal
wealth
C. An article that suggests that Americans are willing to support the taxation of the rich in order to
assist the poor
D. A lecture that shows that Americans have grown increasingly tolerant of minority political views
since the turn of the century
E. a report stating that Americans value capitalism over everything else

Americans do not have an ideology that assigns clear priority to one value over any other.

3. According to the passage, none of the following statements are true EXCEPT:

A. the political upheaval of the Civil War increased the popularity of progressive ideals among the
American public.
B. eras of egalitarian reform in American history have been followed by eras of
retrenchment.
C. those who endorse non-egalitarian ideals have generally been less committed to their position
than those who endorse egalitarian ideals.
D. special interests have always had too much political power within the American government.
E. very soon a third group of people is likely to emerge which will be opposed to both egalitarians
and libertarians

the author writes between egalitarian reform and retrenchment in contrast to each other along with
the behaviour of various stakeholders so it is safer to imply that one followed the other.Also BEST
OPTION through Process of Elimination

Recurring (adj) : định kỳ

Theme (n) : chủ đề

Flare (v) : bùng phát

Fervent (adj) : nhiệt thành, nóng dữ


Populist (n) : dân túy

Progressive (adj) : cấp tiến, cải cách, tiến bộ

Legitimacy (n) : tính hợp pháp

Endure (v) : tồn tại

Ascendancy (n) : quyền thế, thế lực

Egalitarian (adj/n) : bình đẳng

Libertarian (n) : người theo chủ nghĩa tự do

Retrenchment (n) : sự xây đắp thành lũy

Cling (v) : bám lấy

Moneyed (adj) : tiền, giàu có

Beneficiary (n) : người thụ hưởng

Creedal (adj) : tín ngưỡng

Outburst (n) : sự bộc phát, bùng nổ, trồi lên

Redress (v) : phục hồi

Venerable (adj) : đáng kính, kính trọng

Yardstick (n) : thước đo

Reconstruction (n) : tái thiết

Dissipate (v) : tiêu tan

Sluggish (adj) : chậm chạp

Outright (adv) : hoàn toàn

Mendacity (n) : dối trá, sai lầm

Endorse (v) : kiểm nhận, thừa nhận

Thrust of (n) : lực đẩy

Counterthrust (n) : chống lại

Juncture (n) : thời cơ

Indelible (adj) : không thể xóa bỏ

Tenet (n) : nguyên lý, giáo lý, chủ nghĩa


Nov 2 – 2

In 1887 the Dawes Act legislated wide-scale private ownership of reservation lands in the United
States for Native Americans. The act allotted plots of 80 acres to each Native American adult.
However, the Native Americans were not granted outright title to their lands. The act defined each
grant as a “trust patent,” meaning that the Bureau of Indian Affairs (BIA), the governmental agency
in charge of administering policy regarding Native Americans, would hold the allotted land in trust
for 25 years, during which time the Native American owners could use, but not alienate (sell) the
land. After the 25-year period, the Native American allottee would receive a “fee patent” awarding
full legal ownership of the land.

Two main reasons were advanced for the restriction on the Native Americans’ ability to sell their
lands. First, it was claimed that free alienability would lead to immediate transfer of large amounts
of former reservation land to non-Native Americans, consequently threatening the traditional way of
life on those reservations. A second objection to free alienation was that Native Americans were
unaccustomed to, and did not desire, a system of private landownership. Their custom, it was said,
favored communal use of land.

However, both of these arguments bear only on the transfer of Native American lands to non-Native
Americans: neither offers a reason for prohibiting Native Americans from transferring land among
themselves. Selling land to each other would not threaten the Native American culture. Additionally,
if communal land use remained preferable to Native Americans after allotment, free alienability
would have allowed allottees to sell their lands back to the tribe.

When stated rationales for government policies prove empty, using an interest-group model often
provides an explanation. While neither Native Americans nor the potential non-Native American
purchasers benefited from the restraint on alienation contained in the Dawes Act, one clearly
defined group did benefit: the BIA bureaucrats. It has been convincingly demonstrated that
bureaucrats seek to maximize the size of their staffs and their budgets in order to compensate for
the lack of other sources of fulfillment, such as power and prestige. Additionally, politicians tend to
favor the growth of governmental bureaucracy because such growth provides increased opportunity
for the exercise of political patronage. The restraint on alienation vastly increased the amount of
work, and hence the budgets, necessary to implement the statute. Until allotment was ended in
1934, granting fee patents and leasing Native American lands were among the principal activities of
the United States government. One hypothesis, then, for the temporary restriction on alienation in
the Dawes Act is that it reflected a compromise between non-Native Americans favoring immediate
alienability so they could purchase land and the BIA bureaucrats who administered the privatization
system.

1. Which one of the following best summarizes the main idea of the passage?

(A) United States government policy toward Native Americans has tended to disregard their needs
and consider instead the needs of non-Native American purchasers of land.
(B) In order to preserve the unique way of life on Native American reservations, use of Native
American lands must be communal rather than individual.
(C) The Dawes Act’s restriction on the right of Native Americans to sell their land may
have been implemented primarily to serve the interests of politicians and bureaucrats.
(D) The clause restricting free alienability in the Dawes Act greatly expanded United States
governmental activity in the area of land administration.
(E) Since passage of the Dawes Act in 1887, Native Americans have not been able to sell or transfer
their former reservation land freely.

2. Which one of the following statements concerning the reason for the end of
allotment, if true, would provide the most support for the author’s view of politicians?

(A) Politicians realized that allotment was damaging the Native American way of life.
(B) Politicians decided that allotment would be more congruent with the Native American custom of
communal land use.
(C) Politicians believed that allotment’s continuation would not enhance their
opportunities to exercise patronage.
(D) Politicians felt that the staff and budgets of the BIA had grown too large.
(E) Politicians were concerned that too much Native American land was falling into the hands of
non-Native Americans.

3. Which one of the following best describes the organization of the passage?

(A) The passage of a law is analyzed in detail, the benefits and drawbacks of one of its clauses are
studied, and a final assessment of the law is offered.
(B) The history of a law is narrated, the effects of one of its clauses on various populations are
studied, and repeal of the law is advocated
(C) A law is examined, the political and social backgrounds of one of its clauses are characterized,
and the permanent effects of the law are studied.
(D) A law is described, the rationale put forward for one of its clauses is outlined and
dismissed, and a different rationale for the clause is presented.
(E) The legal status of an ethnic group is examined with respect to issues of landownership and
commercial autonomy, and the benefits to rival groups due to that status are explained.

4. The author’s attitude toward the reasons advanced for the restriction on alienability
in the Dawes Act at the time of its passage can best be described as

(A) completely credulous


(B) partially approving
(C) basically indecisive
(D) mildly questioning
(E) highly skeptical

5. It can be inferred from the passage that which one of the following was true of Native
American life immediately before passage of the Dawes Act?

(A) Most Native Americans supported themselves through farming.


(B) Not many Native Americans personally owned the land on which they lived.
(C) The land on which most Native Americans lived had been bought from their tribes.
(D) Few Native Americans had much contact with their non-Native American neighbors.
(E) Few Native Americans were willing to sell their land to non-Native Americans.
6. According to the passage, the type of landownership initially obtainable by Native
Americans under the Dawes Act differed from the type of ownership obtainable after a
25-year period in that only the latter allowed

(A) owners of land to farm it


(B) owners of land to sell it
(C) government some control over how owners disposed of land
(D) owners of land to build on it with relatively minor governmental restrictions
(E) government to charge owners a fee for developing their land

7. Which of the following, if true, would most strengthen the author’s argument
regarding the true motivation for the passage of the Dawes Act?

(A) The legislators who voted in favor of the Dawes Act owned land adjacent to Native American
reservations.
(B) The majority of Native Americans who were granted fee patents did not sell their land back to
their tribes.
(C) Native Americans managed to preserve their traditional culture even when they were
geographically dispersed.
(D) The legislators who voted in favor of the Dawes Act were heavily influenced by BIA
bureaucrats.
(E) Non-Native Americans who purchased the majority of Native American lands consolidated them
into larger farm holdings.

Disperse (v) : phân tán, rải rác khắp nơi

Allot (v) : phân bổ, giao cho, cấp cho, phân phối

Alienate (v) : chuyển sang người khác

Communal (adj) : tài sản chung

Compromise (n) : thỏa hiệp

Privatization (n) : tư nhân hóa


Nov 3 – 1

Metaphysics – the philosophical investigation of the nature of the universe and existence – had
become unfashionable by the time Alfred North Whitehead began writing in earnest about it in the
1920s. The ever-more impressive accomplishments of empirical science had led to a general
consensus in academia that the development of comprehensive metaphysical systems was a waste
of time because they were not subject to empirical testing.

Whitehead was unimpressed by this objection. In Whitehead's view, scientists and philosophers
make metaphysical assumptions about how the universe works all the time, but such assumptions
are not easily seen precisely because they remain unexamined and unquestioned. Perhaps foremost
among what Whitehead considered faulty metaphysical assumptions was the idea of substance
materialism, which he rejected in favor of an opposite, event-based “process" ontology in which
transient events are primary and material substances/beings are abstractions. He also argued that
the most basic elements of reality have experiential features, that in fact everything is constituted by
its experience (i.e., even things like electrons experience). Whitehead referred to his metaphysical
system as "philosophy of organism," but it would become known more widely as "process
philosophy."

This philosophy of Whitehead was highly original, and soon garnered interest in philosophical circles.
After publishing The Concept of Nature in 1920, he served as president of the Aristotelian Society
from 1922 to 1923, and Henri Bergson was quoted as saying that Whitehead was "the best
philosopher writing in English." So impressive and different was Whitehead's philosophy that in 1924
he was invited to join the faculty at Harvard University as a professor of philosophy at 63 years of
age, even though his educational background was in a very different field.

1. It can be inferred from the passage that those adhering to the idea of “substance
materialism” would view material substances and beings as:

(A) abstract
(B) concrete
(C) event-based
(D) experiential
(E) process-driven

Now, if you will refer the passage it states:"Whitehead considered faulty metaphysical assumptions
was the idea of substance materialism, which he rejected in favor of an opposite, event-based
“process" ontology in which transient events are primary and material substances/beings are
abstractions..."

Substance Materialism(SM) is OPPOSITE of Process Ontology(PO)


opposite – hai ý kiến sẽ đảo ngược nhau
In PO, Transient events-Primary
Material Substances- Abstractions.

Now, as we are told that PO is opposite of SM

Therefore, in SM, Transient Events- Abstractions


Material Substances-Primary

Now, we have to proceed to the answer choice and select the choice that is closest to the word
"Primary",So in Nut-shell you have to do POE!

2. The primary purpose of this passage is to:

(A) highlight one unique contribution of an important philosopher.


(B) discuss the importance of metaphysics as a field in science.
(C) provide details on the concept of “process philosophy.”
(D) emphasize Whitehead’s place among other philosophers.
(E) prove that “process philosophy” was superior to “substance materialism.”

3. Which one of the following perspectives would best capture Whitehead’s “process
philosophy”?

(A) A building is viewed through its size, materials, and aesthetics.


(B) A car is viewed through its horsepower, design, and speed.
(C) A school is viewed through its students, teachers, and staff.
(D) A business is viewed through its mission, advertising, and headquarters.
(E) A person is viewed through his experiences, interactions, and memories.

4. The author would most likely describe Whitehead’s philosophy in the early 1920s as:

(A) highly controversial


(B) unimpressive
(C) never before considered
(D) poorly understood
(E) unique and respected

5. The author of the passage would most likely agree with which of the following
statements?

(A) Metaphysics is a field worthy of considerably more attention and respect.


(B) Alfred North Whitehead was successful as a challenger of conventional wisdom.
(C) Empirical science is a more respected field than is metaphysics.
(D) Alfred North Whitehead was a controversial figure among academics in the early 20th century.
(E) Alfred North Whitehead restored the prestige of metaphysics as an academic subject.

Concrete (adj) : cụ thể

Metaphysic (n) : siêu hình học, sự trừu tượng

Existence (n) : thực thể

Earnest (adj) : nghiêm trang, đứng đắn, hết lòng

Event-based (adj) : dựa trên sự kiện

Ontology (n) : bản thể học


Transient (adj) : tạm thời, thoáng qua, ngắn ngủi

Abstraction (n) : trừu tượng


Nov 3 – 2

The great white shark is considered to be an apex predator, or without any predators of their own.
It has been known to hunt a variety of different marine life and consume non-biological items (tires,
metals, etcs.). While primarily hunting alone, Great Whites have been known to hunt in clans of 2 to
6. Their unique power and pattern of eating undigestible items has earned them a reputation of
indiscriminate eating machines; however, its diet consists primarily of marine life.

One hunting technique, which has been observed in a least one other shark species, that great
white sharks employ is called spy-hopping. In this technique, the shark will lift its head entirely out
of water. Two explanations are offered as reasons for this behavior. One is that this is a learned
behavior from humans and the other is that lifting their head above water allows them to smell
better as smell travels faster through air then it does water.

Some suggest that despite the fact that great whites are known to have an exceptional sense of
smell, spy-hopping is likely a learned behavior from humans. Studies have shown that when
presented with two objects of different shape, they are more likely to attack shapes similar to
common prey than others. Specifically, all else being equal, a great white is more likely to attack
shapes similar to seals than to attack round shapes. Thus, vision is very important to a great white
when hunting and is something that has been learned as humans invade the hunting grounds of
great whites.
Other viewpoints use the variety of hunting techniques of a great white as evidence that spy-
hopping helps sharks track prey via scent. As sub-surface and surface charge attacks, which involve
the shark breaching the water's surface, have been observed, it would stand to reason that sharks
use spy-hopping to locate prey via their sense of smell.
Even without knowing the true use for spy-hopping, it is clear that the great white is a versatile
hunter using a variety of techniques.

1)The primary purpose of this passage is to:

A)Compare hunting techniques of the Great White


B)Compare contrasting views for the explanation of a behavior
C)Inform the reader about a subject
D)Explain one way that the Great White hunts for food
E)Analyze the cause of a certain behavior

2)What best describes the purpose of the third paragraph of the passage?

A)Offers support for the authors viewpoint


B)Describes a contrasting viewpoint the a previously mentioned theory
C)Give support for the prevailing theory
D)Evaluates the validity of a purposed theory
E)Offers reasons why a viewpoint cannot be true

Undigestible (adj) : khó tiêu hóa

Tires (n) : lốp xe

Scent (n) : mùi hương

Versatile (adj) : đa năng


Nov 4 – 1

The labor force is often organized as if workers had no family responsibilities. Preschool-age children
need full-time care; children in primary school need care after school and during school vacations.
Although day-care services can resolve some scheduling conflicts between home and office, workers
cannot always find or afford suitable care. Even when they obtain such care, parents must still cope
with emergencies, such as illnesses, that keep children at home. Moreover, children need more than
tending; they also need meaningful time with their parents. Conventional full-time workdays,
especially when combined with unavoidable household duties, are too inflexible for parents with
primary child-care responsibility.

Although a small but increasing number of working men are single parents, those barriers against
successful participation in the labor market that are related to primary child-care responsibilities
mainly disadvantage women. Even in families where both parents work, cultural pressures are
traditionally much greater on mothers than on fathers to bear the primary child-rearing
responsibilities.

In reconciling child-rearing responsibilities with participation in the labor market, many working
mothers are forced to make compromises. For example, approximately one-third of all working
mothers are employed only part-time, even though part-time jobs are dramatically underpaid and
often less desirable in comparison to full-time employment. Even though part-time work is usually
available only in occupations offering minimal employee responsibility and little opportunity for
advancement or self-enrichment, such employment does allow many women the time and flexibility
to fulfill their family duties, but only at the expense of the advantages associated with full-time
employment.

Moreover, even mothers with full-time employment must compromise opportunities in order to
adjust to barriers against parents in the labor market. Many choose jobs entailing little challenge or
responsibility or those offering flexible scheduling, often available only in poorly paid positions, while
other working mothers, although willing and able to assume as much responsibility as people
without children, find that their need to spend regular and predictable time with their children
inevitably causes them to lose career opportunities to those without such demands. Thus, women in
education are more likely to become teachers than school administrators, whose more conventional
full-time work schedules do not correspond to the schedules of school-age children, while female
lawyers are more likely to practice law in trusts and estates, where they can control their work
schedules, than in litigation, where they cannot. Nonprofessional women are concentrated in
secretarial work and department store (department store: n. sales, where their absences can be
covered easily by substitutes and where they can enter and leave the work force with little loss,
since the jobs offer so little personal gain. Indeed, as long as the labor market remains hostile to
parents, and family roles continue to be allocated on the basis of gender, women will be seriously
disadvantaged in that labor market.

1. Which one of the following best summarizes the main idea of the passage?

(A) Current trends in the labor force indicate that working parents, especially women, may not
always need to choose between occupational and child-care responsibilities.
(B) In order for mothers to have an equal opportunity for advancement in the labor force, traditional
family roles have to be reexamined and revised.
(C) Although single parents who work have to balance parental and career demands, single mothers
suffer resulting employment disadvantages that single fathers can almost always avoid.
(D) Although child-care responsibilities disadvantage many women in the labor force, professional
women (such as teachers and lawyers) are better able to overcome this problem than are
nonprofessional women.
(E) Traditional work schedules are too inflexible to accommodate the child-care
responsibilities of many parents, a fact that severely disadvantages women in the labor
force.

2. Which one of the following statements about part-time work can be inferred from the
information presented in the passage?

(A) One-third of all part-time workers are working mothers.


(B) Part-time work generally offers fewer opportunities for advancement to working mothers than to
women generally.
(C) Part-time work, in addition to having relatively poor wages, often requires that employees work
during holidays, when their children are out of school.
(D) Part-time employment, despite its disadvantages, provides working mothers with an
opportunity to address some of the demands of caring for children.
(E) Many mothers with primary child-care responsibility choose part-time jobs in order to better
exploit full-time career opportunities after their children are grown.

3. It can be inferred from the passage that the author would be most likely to agree
with which one of the following statements about working fathers in two-parent
families?

(A) They are equally burdened by the employment disadvantages placed upon all parents—male and
female—in the labor market.
(B) They are so absorbed in their jobs that they often do not see the injustice going on around
them.
(C) They are shielded by the traditional allocation of family roles from many of the
pressures associated with child-rearing responsibilities.
(D) They help compound the inequities in the labor market by keeping women form competing with
men for career opportunities.
(E) They are responsible for many of the problems of working mothers because of their insistence
on traditional roles in the family.

4. Of the following, which one would the author most likely say is the most troublesome
barrier facing working parents with primary child-care responsibility?

(A) the lack of full-time jobs open to women


(B) the inflexibility of work schedules
(C) the low wages of part-time employment
(D) the limited advancement opportunities for nonprofessional employees
(E) the practice of allocating responsibilities in the workplace on the basis of gender

5. The passage suggests that day care is at best a limited solution to the pressures
associated with child rearing for all of the following reasons EXCEPT:
(A) Even the best day care available cannot guarantee that children will have meaningful time with
their parents.
(B) Some parents cannot afford day-care services.
(C) Working parents sometimes have difficulty finding suitable day care for their children.
(D) Parents who send their children to day care still need to provide care for their
children during vacations.
(E) Even children who are in day care may have to stay home when they are sick.

Preschool-age children need full-time care; children in primary school need care after school and
during school vacations.==> Supporting the previous conclusion.

6. According to the passage, many working parents may be forced to make any of the
following types of career decisions EXCEPT

(A) declining professional positions for nonprofessional ones, which typically have less
conventional work schedules
(B) accepting part-time employment rather than full-time employment
(C) taking jobs with limited responsibility, and thus more limited career opportunities, in order to
have a more flexible schedule
(D) pursuing career specializations that allow them to control their work schedules instead of
pursuing a more desirable specialization in the same field
(E) limiting the career potential of one parent, often the mother, who assumes greater child-care
responsibility

7. Which one of the following statements would most appropriately continue the
discussion at the end of the passage?

(A) At the same time, most men will remain better able to enjoy the career and salary
opportunities offered by the labor market.
(B) Of course, men who are married to working mothers know of these employment barriers but
seem unwilling to do anything about them.
(C) On the other hand, salary levels may become more equitable between men and women even if
the other career opportunities remain more accessible to men than to women.
(D) On the contrary, men with primary child-rearing responsibilities will continue to enjoy more
advantages in the workplace than their female counterparts.
(E) Thus, institutions in society that favor men over women will continue to widen the gap between
the career opportunities available for men and for women.
Nov 4 – 2

Although postcolonial theory does not propose a simple causal relation between literature and
political action, it nonetheless views literature as an enabler of nationalism and in turn political
action: literature imagines the community of the nation, giving it a virtually mythical status. The
power of the national imaginary to inspire political action is never in doubt, but the precise
mechanisms by which this happens or whether indeed any kind of direct causality is involved
remains unclear. It may very well be that literature and political action are reciprocating parts of a
complex process for which such questions are peripheral.

An example of such reciprocation is an early play by Yeats, The King’s Threshold (1904), in which an
ancient Irish poet goes on hunger strike to protest against his king. This play might be thought to
have engendered the hunger strikes Irish republicans resorted to, first during the struggle with
Britain from 1916 to 1920, then in the civil war that followed as they fought against the Free State
government forces, and finally in 1981 as they resisted continuing British rule in Northern Ireland.
Similarly in reciprocation, when the Republican mayor of Cork, Terence MacSwiney, died in hunger
strike in 1920, Yeats revised his play, and gave it a revised conclusion – a tragic ending, attributing
the tragic ending as a great improvement & much more representative of the then political climate.

1. The passage mentions the instance of death of Republican mayor of Cork, Terence
MacSwiney to illustrate

A. That Yeats was not an obstinate poet who set his play in stone and was willing to make changes
to the same as long it doing so ensured popularity.
B. That Yeats’ The King’s Threshold was influential enough to have inspired an important person like
Terence MacSwiney to give up his life.
C. An instance of the reciprocal relationship between Literature and Political action in
which political action inspires changes to a literary work.
D. That even a play once popular enough to rally the entire nation may have to be adapted to fit the
then present national scenery to prevent it from running out of favor.
E. How literary works can sometimes misguide individuals into fatal consequences.

Enabler (n) : kẻ gây rối

Nationalism (n) : chủ nghĩa dân tộc

Virtually (adv) : hầu như

Mythical (adj) : hoang đường, truyền kỳ, huyền thoại

Reciprocate (v) : đối ứng, trao đổi lẫn nhau


Nov 5 – 1

Great comic art is never otherwordly, it does not seek to mystify us, and it does not deny ambiguity
by branding as evil whatever differs from good. Great comic artists assume that truth may bear all
lights, and thus they seek to accentuate contradictions in social action, not gloss over or transcend
them by appeals to extrasocial symbols of divine ends, cosmic purpose, or laws of nature. The
moment of transcendence in great comic art is a social moment, born out of the conviction that we
are human, even though we try to be gods. The comic community to which artists address
themselves is a community of reasoning, loving, joyful, compassionate beings, who are willing to
assume the human risks of acting rationally. Without invoking gods or demons, great comic art
arouses courage in reason, courage which grows out of trust in what human beings can do as
humans.

1) The passage suggests that great comic art can be characterized as optimistic about
the ability of humans to

a) rid themselves of pride


b) act rationally
c) transcend the human condition
d) differentiate clearly between good and evil
e) avoid social conflicts

"The comic community to which artists address themselves is a community of reasoning, loving,
joyful, compassionate beings, who are willing to assume the human risks of acting rationally..."

2) It can be inferred from the passage that the author admires great comic artists
primarily for their

a) ability to understand the frequently subtle differences between good and evil
b) ability to reconcile the contradictions in human behaviour
c) ability to distinguish between rational and irrational behaviour
d) insistence on confronting the truth about the human condition
e) insistence on condemning human faults and weaknesses

3) Which of the following is the most accurate description of the organization of the
passage?

a) A sequence of observations leading to a prediction


b) A list of inferences drawn from facts stated at the beginning of the passage
c) A series of assertions related to one general subject
d) A statement of the major idea, followed by specific examples

no specific examples
e) A succession of ideas moving from specific to general

inference (n) : suy luận

drawn (adj) : rút ra

assertion (n) : khẳng định

Beings (n) : chúng sinh


Compassionate (adj) : động lòng thương, từ bi, thương xót

Arouse (v) : khơi dậy, đánh thức

Reason (n) : lý trí

Courage (n) : can đảm, dũng khí

Mystify (v) : làm bối rối

Accentuate (v) : nhấn mạnh, làm nổi bật lên

Contradiction (n) : mâu thuẫn

Gloss (v) : chú giải, chú thích, chỉ trích, phê bình

Transcend (v) : vượt qua, đi quá giới hạn

Appeal (v) : chống án, kháng cáo

Extra-social (adj) : ngoài xã hội

Transcendence (n) : siêu việt

Conviction (n) : lòng tin chắc, sự kết án, phán quyết


Nov 5 – 2

Transgenic agriculture is the use of genetically modified crops to enhance the production of food
and other farm products. Genes from another species are added to a crop to introduce new traits,
such as resistance to herbicides, that are not present in the original. One widespread application
involves the addition of genes from Bacillus thuringiensis, a soil bacterium that produces insecticidal
toxins, to corn, producing a hybrid that is resistant to insect pests.

Proponents of this agricultural biotechnology argue that it has enormous potential benefits. The corn
hybrids produce delta endotoxins, which are generally thought to be harmless to humans and other
nonpest species, but toxic to certain insect pests. Production of the insecticide within the plant
reduces crop losses from the pests and also reduces the need for application of conventional
insecticides and other agrochemicals, which can harm humans and cause other environmental
damage.

Traditional insecticide treatments involve spraying of the delta endotoxins on the vulnerable crops.
However, the chemicals break down rapidly when exposed to ultraviolet light, a problem avoided
when the toxins are produced internally to the plant.

The growth of genetically modified crops has sparked opposition on a number of counts. The toxins
produced in the plant remain in the soil when the crop is plowed under, accumulating over time with
unknown effects at high levels of concentration. The insecticide has the potential to spread upwards
through the food chain, starting with predators that consume contaminated insects and, in turn,
become prey for other species. Transgenes can spread to nearby wild plants through crop-weed
hybridization, a process that is extremely difficult to control, transmitting superior traits to
unintended recipients. Fears of these “superweeds” motivate a number of the opponents of genetic
modification. Finally, transgenic agriculture encourages monocultures, genetically homogeneous
fields, which are ironically more vulnerable to pests and disease because they lack the genetic
diversity that provides the natural defense of evolution.

1. According to the passage, delta endotoxins

A. are added to a crop, such as corn, to introduce new traits found in another species
B. are one of several types of herbicide
C. are generally perceived to be harmful to humans
D. decompose quickly under certain forms of radiation
E. can spread to nearby plants through crop-weed hybridization
2. With which of the following statements would the author of the passage be most
likely to agree?

A. The dangers of transgenic agriculture outweigh any possible benefits.


B. Genetic modification can create crops that are particularly vulnerable to the very
dangers it was trying to reduce.
C. Traditional insecticide treatment carries less risk to other species than does transgenically
produced insecticide.

D. Proponents of transgenic agriculture cite the creation of “superweeds” as evidence of the


technology’s potential benefits.
E. Plants that are contaminated with delta endotoxins should be plowed under the soil.

Finally, transgenic agriculture encourages monocultures, genetically homogeneous fields, which are
ironically more vulnerable to pests and disease because they lack the genetic diversity that
provides the natural defense of evolution.

Monoculture (n) : độc canh (the cultivation of a single crop in a given area.)

Homogenenous (adj) : đồng nhất


Herbicide (n) : thuốc diệt cỏ

Endotoxin (n) : nội độc tố

Insecticide (n) : thuốc trừ sâu

Spark (v) : châm ngòi

Opposition (n) : sự đối lập, tương phản

Plow (v) : cày

Recipient (n) : người nhận


Nov 6 – 1

The discoveries of the white dwarf, the neutron star, and the black hole, coming well after the
discovery of the red giant are among the most exciting developments in decades because they may
be well present physicists with their greatest challenge since the failure of classical mechanics. In
the life cycle of the star, after all of the hydrogen and helium fuel has been burned, the delicate
balance between the outer nuclear radiation pressure and the stable gravitational force becomes
disturbed and slow contraction begins. As compression increases, a very dense plasma forms. If the
initial star had mass of less than 1.4 solar masses (1.4 times the mass of our sun), the process
ceases at the density of 1,000 tons per cubic inch, and the star becomes the white dwarf. However,
if the star was originally more massive, the white dwarf plasma can’t resist the gravitational
pressures, and in rapid collapse, all nuclei of the star are converted to a gas of free neutrons.
Gravitational attraction compresses this neutron gas rapidly until a density of 10 tons per cubic inch
is reached; at this point the strong nuclear force resists further contraction. If the mass of the star
was between 1.4 and a few solar masses, the process stops here, and we have a neutron star.

But if the original star was more massive than a few solar masses, even the strong nuclear forces
cannot resist the gravitational crunch. The neutrons are forced into one another to form heavier
hadrons and these in turn coalesce to form heavier entities, of which we as yet know nothing. At
this point, a complete collapse of the stellar mass occurs; existing theories predict a collapse to
infinite density and infinitely small dimensions Well before this, however, the surface gravitational
force would become so strong that no signal could ever leave the star - any photon emitted would
fall back under gravitational attraction – and the star would become black hole in space.

This gravitational collapse poses a fundamental challenge to physics. When the most widely
accepted theories predict such improbable things as infinite density and infinitely small dimensions,
it simply means that we are missing some vital insight. This last happened in physics in the 1930’s,
when we faced the fundamental paradox concerning atomic structure. At that time, it was
recognized that electrons moved in table orbits about nuclei in atoms. However, it was also
recognized that if charge is accelerated, as it must be to remain in orbit, it radiates energy; so,
theoretically, the electron would be expected eventually to spiral into the nucleus and destroy the
atom. Studies centered around this paradox led to the development of quantum mechanics. It may
well be that an equivalent t advance awaits us in investigating the theoretical problems presented by
the phenomenon of gravitational collapse.

1. The primary purpose of the passage is to

A. offer new explanations for the collapse of stars.


B. explain the origins of black holes, neutron stars, and white dwarfs.
C. compare the structure of atoms with the structure of the solar system.
D. explain how the collapse of stars challenges accepted theories of physics.
E. describe the imbalance between radiation pressure and gravitational force.

2. According to the passage, in the final stages of its development our own sun is likely
to take the form of a

A. white dwarf
B. neutron star
C. red giant
D. gas of free neutrons
E. black hole

If the initial star had mass of less than 1.4 solar masses (1.4 times the mass of our sun), the
process ceases at the density of 1,000 tons per cubic inch, and the star becomes the white dwarf.

3. According to the passage, an imbalance arises between nuclear radiation pressure


and gravitational force in stars because

A. the density of a star increases as it ages


B. radiation pressure increases as a star increases in mass
C. radiation pressure decreases when a star’s fuel has been consumed
D. the collapse of a star increases its gravitational force.
E. a dense plasma decreases the star’s gravitational force.

4. The author asserts that the discoveries of the white dwarf, the neutron star, and the
black hole are significant because these discoveries.

A. demonstrate the probability of infinite density and infinitely small dimensions


B. pose the most comprehensive and fundamental problem faced by physicists in
decades
C. clarify the paradox suggested by the collapse of electrons into atomic nuclei.
D. establish the relationship between the mass and gravitational pressure.
E. assist in establishing the age of the universe by tracing the life histories of stars.

5. The passage contains information that answers which of the following questions?

I. What is the density limit of the gravitational collapse of neutron stars?


II. At what point in its life cycle does a star begin to contract?
III . What resists the gravitational collapse of a star?

A. I only
B. III only
C. I and II only
D. II and III only
E. I, II, and III

6. The author introduces the discussion of the paradox concerning atomic structures(in
highlighted text) in order to

A. Show why it was necessary to develop quantum mechanics


B. Compare the structure of an atom with the structure of star
C. Demonstrate by analogy that a vital insight in astrophysics is missing
D. Illustrate the contention that improbable things do happen in astrophysics
E. Argue that atoms can collapse if their electrons do not remain in orbit.
7. According to the passage, paradoxes are useful in a scientific investigation because
they

(A) point to the likelihood of impending discoveries


(B) assist scientists in making comparisons with other branches of knowledge
(C) disprove theories that have been called into question
(D) call attention to inadequacies of existing theory
(E) suggest new hypotheses that can be tested by observation

Two paradoxes are described in the passage, the atomic structure problem of the 1930s and the
gravitational collapse problem, and both are depicted as calling attention to problems in existing
scientific theories. In lines 57-58 the author notes that work in atomic structure physics focused on
the inconsistencies of the existing theories, and in lines 44-48 that the discoveries of such
paradoxical phenomena as infinite density and infinitely small dimensions tell physicists that "some
vital insight" is missing within the existing theory. Thus in both cases, the paradoxes serve to point
out problems with existing theories and to draw the attention of physicists toward solving them.

White dwarf (n) : sao lùn trắng

Delicate (adj) : tinh tế

Radiation (n) : sự bức xạ

Contraction (n) : co thắt

Compression (n) : sự nén lại

Cease (v) : ngưng lại

Coalesce (v) : kết hợp, hợp nhất, liên hiệp

Spiral (v) : xoắn ốc


Nov 6 – 2

Most economists in the United States seem captivated by the spell of the free market. Consequently,
nothing seems good or normal that does not accord with the requirements of the free market. A
price that is determined by the seller or, for that matter, established by anyone other than the
aggregate of consumers seems pernicious. Accordingly, it requires a major act of will to think of
price-fixing (the determination of prices by the seller) as both “normal” and having a valuable
economic function. In fact, price-fixing is normal in all industrialized societies because the industrial
system itself provides, as an effortless consequence of its own development, the price-fixing that it
requires. Modern industrial planning requires and rewards great size. Hence, a comparatively small
number of large firms will be competing for the same group of consumers. That each large firm will
act with consideration of its own needs and thus avoid selling its products for more than its
competitors charge is commonly recognized by advocates of free-market economic theories. But
each large firm will also act with full consideration of the needs that it has in common with the other
large firms competing for the same customers. Each large firm will thus avoid significant price-
cutting, because price-cutting would be prejudicial to the common interest in a stable demand for
products. Most economists do not see price-fixing when it occurs because they expect it to be
brought about by a number of explicit agreements among large firms; it is not.

Moreover, those economists who argue that allowing the free market to operate without
interference is the most efficient method of establishing prices have not considered the economies
of non-socialist countries other than the United states. These economies employ intentional price-
fixing, usually in an overt fashion. Formal price-fixing by cartel and informal price-fixing by
agreements covering the members of an industry are commonplace. Were there something
peculiarly efficient about the free market and inefficient about price-fixing, the countries that have
avoided the first and used the second would have suffered drastically in their economic
development. There is no indication that they have.

Socialist industry also works within a framework of controlled prices. In the early 1970’s, the Soviet
Union began to give firms and industries some of the flexibility in adjusting prices that a more
informal evolution has accorded the capitalist system. Economists in the United States have hailed
the change as a return to the free market. But Soviet firms are no more subject to prices established
by a free market over which they exercise little influence than are capitalist firms; rather, Soviet
firms have been given the power to fix prices.

1. The primary purpose of the passage is to

(A) refute the theory that the free market plays a useful role in the development of industrialized
societies
(B) suggest methods by which economists and members of the government of the United States can
recognize and combat price-fixing by large firms
(C) show that in industrialized societies price-fixing and the operation of the free market are not
only compatible but also mutually beneficial
(D) explain the various ways in which industrialized societies can fix prices in order to stabilize the
free market
(E) argue that price-fixing, in one form or another, is an inevitable part of and benefit to
the economy of any industrialized society
2. The passage provides information that would answer which of the following
questions about price-fixing?

I. What are some of the ways in which prices can be fixed?


II. For what products is price-fixing likely to be more profitable that the operation of the free
market?
III. Is price-fixing more common in socialist industrialized societies or in non-socialist industrialized
societies?

(A) I only
(B) III only
(C) I and II only
(D) II and III only
(E) I, II, and III
 Each large firm will thus avoid significant price-cutting
 they expect it to be brought about by a number of explicit agreements among large firms
 These economies employ intentional price-fixing, usually in an overt fashion. Formal price-fixing
by cartel and informal price-fixing by agreements covering the members of an industry are
commonplace.
 the Soviet Union began to give firms and industries some of the flexibility in adjusting prices
that a more informal evolution has accorded the capitalist system.

3. The author’s attitude toward “Most economists in the United States” (line 1) can best
be described as

(A) spiteful and envious


(B) scornful and denunciatory
(C) critical and condescending
(D) ambivalent but deferential
(E) uncertain but interested

Most economists in the United States seem captivated by the spell of the free market.
Consequently, nothing seems good or normal that does not accord with the requirements of
the free market.

4. It can be inferred from the author’s argument that a price fixed by the seller “seems
pernicious” (line 7) because

(A) people do not have confidence in large firms


(B) people do not expect the government to regulate prices
(C) most economists believe that consumers as a group should determine prices
(D) most economists associate fixed prices with communist and socialist economies
(E) most economists believe that no one group should determine prices

5. The suggestion in the passage that price-fixing in industrialized societies is normal


arises from the author’s statement that price-fixing is
(A) a profitable result of economic development
(B) an inevitable result of the industrial system
(C) the result of a number of carefully organized decisions
(D) a phenomenon common to industrialized and non-industrialized societies
(E) a phenomenon best achieved cooperatively by government and industry

. In fact, price-fixing is normal in all industrialized societies because the industrial system itself
provides, as an effortless consequence of its own development, the price-fixing that it requires.

6. According to the author, price-fixing in non-socialist countries is often

(A) accidental but productive


(B) illegal but useful
(C) legal and innovative
(D) traditional and rigid
(E) intentional and widespread

7. According to the author, what is the result of the Soviet Union’s change in economic
policy in the 1970’s?

(A) Soviet firms show greater profit.


(B) Soviet firms have less control over the free market.
(C) Soviet firms are able to adjust to technological advances.
(D) Soviet firms have some authority to fix prices.
(E) Soviet firms are more responsive to the free market.

8. With which of the following statements regarding the behavior of large firms in
industrialized societies would the author be most likely to agree?

(A) The directors of large firms will continue to anticipate the demand for products.
(B) The directors of large firms are less interested in achieving a predictable level of profit than in
achieving a large profit.
(C) The directors of large firms will strive to reduce the costs of their products.
(D) Many directors of large firms believe that the government should establish the prices that will be
charged for products.
(E) Many directors of large firms believe that the price charged for products is likely to increase
annually.

9. In the passage, the author is primarily concerned with

(A) predicting the consequences of a practice


(B) criticizing a point of view
(C) calling attention to recent discoveries
(D) proposing a topic for research
(E) summarizing conflicting opinions

Deferential (adj) : bảo vệ, cung kính

Spiteful (adj) : cay cú, khó chịu, trái ý


Envious (adj) : ghen tị

Scornful (adj) : khinh bỉ

Denunciatory (adj) : tố cáo, buộc tội

Condescend (v) : bỏ qua, chiếu cố, hạ mình, nhường nhịn

Ambivalent (adj) : cảm xúc lẫn lộn, trái ngược

Captivate (v) : say mê, quyến rũ

Pernicious (adj) : nguy hiểm

Prejudicial (adj) : làm tổn hại

Intentional (adj) : cố ý

Overt (adj) : công khai

Fashion (n) : cách làm

Peculiarly (adv) : đặc biệt, dị thường

Hail (v) : ca ngợi


Nov 7 – 1

At the time Jane Austen's novels were published-between 1811 and 1818-English literature
was not part of any academic curriculum. In addition, fiction was under strenuous attack.(5)
Certain religious and political groups felt novels had the power to make so-called immoral
characters so interesting that young readers would identify with them; these groups also
considered novels to be of little practical use. Even Cole-(10) ridge, certainly no literary
reactionary" spoke for many when he asserted that "novel-reading occasions the destruction of
the mind's powers." These attitudes toward novels help explain why Austen received little
attention from early nine-(15)teenth-century literary critics. (In any case, a novelist published
anonymously, as Austen was, would not be likely to receive much critical attention.) The
literary response that was accorded her, however, was often as incisive as twentieth-(20)
century criticism. In his attack in 1816 on novelistic portrayals "outside of ordinary experience,"
for example, Scott made an insightful remark about the merits of Austen's fiction. Her novels,
wrote Scott, "present to the reader an accurate and exact (25) picture of ordinary everyday
people and places, reminiscent of seventeenth-century Flemish painting." Scott did not use the
word "realism," but he undoubtedly used a standard of realistic probability in judging novels.
The critic Whately (30) did not use the word realism either, but he expressed agreement with
Scott's evaluation and went on to suggest the possibilities for moral instruction in what we have
called Austen's realistic method. Her characters, wrote Whately, (35) are persuasive agents for
moral truth since they are ordinary persons "so clearly evoked that we feel an interest in their
fate as if it were our own." Moral instruction, explained Whately, is more likely to' be effective
when conveyed through recog-(40)nizably human and interesting characters than when
imparted by a sermonizing narrator. Whately especially praised Austen's ability to create
characters who "mingle goodness and villainy, weakness and virtue, as in life they are always
(45) mingled." Whately concluded his remarks by comparing Austen's art of characterization to
Dickens', stating his preference for Austen's. Yet the response of nineteenth-century literary
critics to Austen was not always so laudatory and (50) often anticipated the reservations of
twentieth-century critics. An example of such a response was Lewes' complaint in 1859 that
Austen's range of subjects and characters was too narrow. Praising her verisimilitude, Lewes
added that nonethe-(55) less, her focus was too often upon only the unlofty and the
commonplace. (Twentieth-century Marxists, on the other hand, were to complain about
what they saw as her exclusive emphasis on a lofty upper-middle class.) In any case, having
been (60) rescued by some literary critics from neglect and indeed gradually lionized by them,
Austen steadily reached, by the mid-nineteenth century, the enviable pinnacle of being
considered controversial.

1. The primary purpose of the passage is to

(A) demonstrate the nineteenth-century preference for realistic novels rather than romantic ones
(B) explain why Jane Austen's novels were not included in any academic curriculum in the early
nineteenth century
(C) urge a reassessment of Jane Austen's novels by twentieth-century literary critics
(D) describe some of the responses of nineteenth-century critics to Jane Austen's novels
as well as, to fiction in general
(E) argue that realistic character portrayal is the novelist's most difficult task as well as the aspect of
a novel most likely to elicit a critical response
2. The passage supplies information for answering which of the following questions?

(A) Was Whately aware of Scott's remarks about Jane Austen's novels?
(B) Who is an example of a twentieth-century Marxist critic?
(C) Who is an example of a twentieth-century critic who admired Jane Austen's novels?
(D)What is the author's judgment of Dickens?
(E) Did Jane Austen express her opinion of those nineteenth-century critics who admired her novels?

The critic Whately (30) did not use the word realism either, but he expressed agreement with
Scott's evaluation and went on to suggest the possibilities for moral instruction in what we have
called Austen's realistic method

3. The author mentions that English literature "was not part of any academic
curriculum" in the highlighted text in the early nineteenth century in order to

(A) emphasize the need for Jane Austen to create ordinary, everyday characters in her novels
(B) give support to those religious and political groups that had attacked fiction
(C) give one reason why Jane Austen 's novels received little critical attention in the
early nineteenth century
(D) suggest the superiority of an informal and unsystematized approach to the study of literature
(E) contrast nineteenth-century attitudes toward English literature with those toward classical
literature

4. The passage supplies information to suggest that the religious and political groups
mentioned in highlighted text and Whately might have agreed that a novel

(A) has little practical use


(B) has the ability to influence the moral values of its readers
(C) is of most interest to readers when representing ordinary human characters
(D) should not be read by young readers
(E) needs the sermonizing of a narrator in order to impart moral truths

5. The author quotes Coleridge "novel-reading occasions the destruction of the mind's
powers" in order to

(A) refute the literary opinions of certain religious and political groups
(B) make a case for the inferiority of novels to poetry
(C) give an example of a writer who was not a literary reactionary
(D) illustrate the early nineteenth-century belief that fiction was especially appealing to young
readers
(E) indicate how widespread was the attack on novels in the early nineteenth century

6. The passage suggests that twentieth-century Marxists would have admired Jane
Austen's novels more if the novels, as the Marxists understood them, had

(A) described the values of upper-middle-class society


(B) avoided moral instruction and sermonizing
(C) depicted ordinary society in a more flattering light
(D) portrayed characters from more than one class of society
(E) anticipated some of the controversial social problems of the twentieth century

7. It can be inferred from the passage that Whately found Dickens' characters to be

(A) especially interesting to young readers


(B) ordinary persons in recognizably human situations
(C) less liable than Jane Austen's characters to have a realistic mixture of moral
qualities
(D) more often villainous and weak than virtuous and good
(E) less susceptible than Jane Austen's characters to the moral judgments of a sermonizing narrator

Moral instruction, explained Whately, is more likely to' be effective when conveyed through
recognizably human and interesting characters than when imparted by a sermonizing
narrator. Whately especially praised Austen's ability to create characters who "mingle goodness
and villainy, weakness and virtue, as in life they are always (45) mingled." Whately concluded his
remarks by comparing Austen's art of characterization to Dickens', stating his preference for
Austen's.

8. According to the passage, the lack of critical attention paid to Jane Austen can be
explained by all of the following nineteenth-century attitudes toward the novel EXCEPT
the

(A) assurance felt by many people that novels weakened the mind
(B) certainty shared by many political commentators that the range of novels was too
narrow
(C) lack of interest shown by some critics in novels that were published anonymously
(D) fear exhibited by some religious and political groups that novels had the power to portray
immoral characters attractively
(E) belief held by some religious and political groups that novels had no practical value

9. The author would most likely agree that which of the following is the best measure of
a writer's literary success?

(A) The inclusion of the writer 's work in an academic curriculum


(B) Publication of the writer's work in the writer's own name
(C) The existence of debate among critics about the writer's work
(D) Praise of the writer 's work by religious and political groups
(E) Ability of the writer's work to appeal to ordinary people

Author’s judgment (n) : phán đoán, đánh giá, nhận xét của tác giả

Remark (n) : nhận xét

Curriculum (n) : chương trình giáo dục

Strenuous (adj) : kịch liệt

Incisive (adj) : sâu sắc, minh mẫn


Sermonize (v) : thuyết pháp, truyền bá bài giảng, thuyết giáo

Narrator (n) : người dẫn truyện, người thuật lại

Villainy (n) : nhân vật phản diện

Laudatory (adj) : ca ngợi, tán thưởng

Verisimilitude (n) : sự minh bạch

Unlofty (n) : không có lợi

Lionize (v) : làm nổi danh

Enviable (adj) : đáng ghen tị

Pinnacle (n) : đỉnh cao


Nov 7 – 2

Recent years have brought minority-owned businesses in the United States unprecedented
opportunities—as well as new and significant risks. Civil rights activists have long argued that one of
the principal reasons why Blacks, Hispanics, and other minority groups have difficulty establishing
themselves in business is that they lack access to the sizable orders and subcontracts that are
generated by large companies. Now Congress, in apparent agreement, has required by law that
businesses awarded federal contracts of more than $500,000 do their best to find minority
subcontractors and record their efforts to do so on forms filed with the government. Indeed, some
federal and local agencieshave gone so far as to set specific percentage goals for apportioning parts
of public works contracts to minority enterprises.

The corporate response appears to have been substantial. According to figures collected in 1977,
the total of corporate contracts with minority businesses rose from $77 million in 1972 to $1.1 billion
in 1977. The projected total of corporate contracts with minority businesses for the early 1980’s is
estimated to be over 53 billion per year with no letup anticipated in the next decade. Promising as it
is for minority businesses, this increased patronage poses dangers for them, too. First, minority
firms risk expanding too fast and overextending themselves financially, since most are small
concerns and, unlike large businesses, they often need to make substantial investments in new
plants, staff, equipment, and the like in order to perform work subcontracted to them. If, thereafter,
their subcontracts are for some reason reduced, such firms can face potentially crippling fixed
expenses. The world of corporate purchasing can be frustrating for small entrepreneurs who get
requests for elaborate formal estimates and bids. Both consume valuable time and resources, and a
small company’s efforts must soon result in orders, or both the morale and the financial health of
the business will suffer.

A second risk is that White-owned companies may seek to cash in on the increasing apportionments
through formation of joint ventures with minority-owned concerns. Of course, in many instances
there are legitimate reasons for joint ventures; clearly, White and minority enterprises can team
up to acquire business that neither could acquire alone. But civil rights groups and minority business
owners have complained to Congress about minorities being set up as “fronts” with White backing,
rather than being accepted as full partners in legitimate joint ventures.

Third, a minority enterprise that secures the business of one large corporate customer often runs
the danger of becoming—and remaining—dependent. Even in the best of circumstances, fierce
competition from larger, more established companies makes it difficult for small concerns to broaden
their customer bases: when such firms have nearly guaranteed orders from a single corporate
benefactor, they may truly have to struggle against complacency arising from their current success.

1. The primary purpose of the passage is to

(A) present a commonplace idea and its inaccuracies


(B) describe a situation and its potential drawbacks
(C) propose a temporary solution to a problem
(D) analyze a frequent source of disagreement
(E) explore the implications of a finding
2. The passage supplies information that would answer which of the following
questions?

(A) What federal agencies have set percentage goals for the use of minority-owned businesses in
public works contracts?
(B) To which government agencies must businesses awarded federal contracts report their efforts to
find minority subcontractors?
(C) How widespread is the use of minority-owned concerns as “fronts” by White backers seeking to
obtain subcontracts?
(D) How many more minority-owned businesses were there in 1977 than in 1972?
(E) What is one set of conditions under which a small business might find itself
financially overextended?

3. According to the passage, civil rights activists maintain that one disadvantage under
which minority-owned businesses have traditionally had to labor is that they have

(A) been especially vulnerable to governmental mismanagement of the economy


(B) been denied bank loans at rates comparable to those afforded larger competitors
(C) not had sufficient opportunity to secure business created by large corporations
(D) not been able to advertise in those media that reach large numbers of potential customers
(E) not had adequate representation in the centers of government power

civil rights activists have long argued that a problem for members of minority groups who are
attempting to establish businesses has been that minority groups "lack access to the sizable orders
and subcontracts that are generated by large companies."

4. The passage suggests that the failure of a large business to have its bids for
subcontracts result quickly in orders might cause it to

(A) experience frustration but not serious financial harm


(B) face potentially crippling fixed expenses
(C) have to record its efforts on forms filed with the government
(D) increase its spending with minority subcontractors
(E) revise its procedure for making bids for federal contracts and subcontracts

First, minority firms risk expanding too fast and overextending themselves financially, since most are
small concerns and, unlike large businesses, they often need to make substantial investments in
new plants, staff, equipment, and the like in order to perform work subcontracted to them. If,
thereafter, their subcontracts are for some reason reduced, such firms can face potentially
crippling fixed expenses.

5. The author implies that a minority-owned concern that does the greater part of its
business with one large corporate customer should

(A) avoid competition with larger, more established concerns by not expanding
(B) concentrate on securing even more business from that corporation
(C) try to expand its customer base to avoid becoming dependent on the corporation
(D) pass on some of the work to be done for the corporation to other minority-owned concerns
(E) use its influence with the corporation to promote subcontracting with other minority concerns
6. It can be inferred from the passage that, compared with the requirements of law, the
percentage goals set by “some federal and local agencies” in highlighted text are

(A) more popular with large corporations


(B) more specific
(C) less controversial
(D) less expensive to enforce
(E) easier to comply with

7. Which of the following, if true, would most weaken the author’s assertion that, in the
1970’s, corporate response to federal requirements (highlighted text) was substantial

(A) Corporate contracts with minority-owned businesses totaled $2 billion in 1979.


(B) Between 1970 and 1972, corporate contracts with minority-owned businesses declined by 25
percent.
(C) The figures collected in 1977 underrepresented the extent of corporate contracts with minority-
owned businesses.
(D) The estimate of corporate spending with minority-owned businesses in 1980 is approximately
$10 million too high.
(E) The $1.1 billion represented the same percentage of total corporate spending in
1977 as did $77 million in 1972.

8. The passage most likely appeared in

(A) a business magazine


(B) an encyclopedia of Black history to 1945
(C) a dictionary of financial terms
(D) a yearbook of business statistics
(E) an accounting textbook

9. The author would most likely agree with which of the following statements about
corporate response to working with minority subcontractors?

(A) Annoyed by the proliferation of “front” organizations, corporations are likely to reduce their
efforts to work with minority-owned subcontractors in the near future.
(B) Although corporations showed considerable interest in working with minority businesses in the
1970’s, their aversion to government paperwork made them reluctant to pursue many government
contracts.
(C) The significant response of corporations in the 1970’s is likely to be sustained and
conceivably be increased throughout the 1980’s.
(D) Although corporations are eager to cooperate with minority-owned businesses, a shortage of
capital in the 1970’s made substantial response impossible.
(E) The enormous corporate response has all but eliminated the dangers of over-expansion that
used to plague small minority-owned businesses.

Sizable (adj) : khá lớn

Order (n) : đơn hàng


Letup (n) : sự suy giảm, ngừng lại, chấm dứt

Unprecedented (adj) : chưa từng có

Subcontract (n) : hợp đồng thầu phụ

Apportion (v) : phân bổ

Substanial (adj) : đáng kể

Patronage (n) : sự bảo trợ, sự bảo hộ

Over-extend (v) : vượt quá

Cripple (v) : làm tê liệt

Elaborate (v) : nghiên cứu, kỹ lưỡng

Bid (n) : hồ sơ đấu thầu, dự thầu

Morale (n) : tinh thần

Fierce (adj) : dữ dội, hung tợn, tàn nhẫn

Benefactor (n) : ân nhân

Complacency (n) : sự tự mãn, thỏa mãn


Nov 8 – 1

In strongly territorial birds such as the indigo bunting, song is the main mechanism for securing,
defining, and defending an adequate breeding area. When population density is high, only the
strongest males can retain a suitable area. The weakest males do not breed or are forced to nest on
poor or marginal territories.

During the breeding season, the male indigo bunting sings in his territory; each song lasts two or
three seconds with a very short pause between songs. Melodic and rhythmic characteristics are
produced by rapid changes in sound frequency and some regularity of silent periods between
sounds. These modulated sounds form recognizable units, called figures, each of which is
reproduced again and again with remarkable consistency. Despite the large frequency range of
these sounds and the rapid frequency changes that the bird makes, the number of figures is very
limited. Further, although we found some unique figures in different geographical populations, more
than 90 percent of all the figures of birds from different regions are alike. Indigo bunting figures are
extremely stable on a geo- graphic basis. In our studies of isolated buntings we found that male
indigo buntings are capable of singing many more types of figures than they usually do. Thus, it
would seem that they copy their figures from other buntings they hear singing.

Realizing that the ability to distinguish the songs of one species from those of another could be an
important factor in the evolution of the figures, we tested species recognition of a song. When we
played a tape recording of a lazuli bunting or a painted bunting, male indigo buntings did not
respond, even when a dummy of a male indigo bunting was placed near the tape recorder. Playing
an indigo bunting song, however, usually brought an immediate response, making it clear that a
male indigo bunting can readily distinguish songs of its own species from those of other species.

The role of the song figures in intraspecies recognition was then examined . We created
experimental songs composed of new figures by playing a normal song backwards, which changed
the detailed forms of the figures without altering frequency ranges or gross temporal features. Since
the male indigos gave almost a full response to the backward song, we concluded that a wide range
of figure shapes can evoke positive responses. It seems likely, therefore, that a specific
configuration is not essential for intraspecies recognition' but it is clear that song figures must
conform to a particular frequency range, must be within narrow limits of duration, and must be
spaced at particular intervals.

There is evidence that new figures may arise within a population through a slow process of Change
and selection. This variety is probably a valuable adapta- tion for survival: if every bird sang only a
few types of figures, in dense woods or underbrush a female might have difficulty recognizing her
mate's song, and a male might not be able to distinguish a neighbor from a stranger. Our studies
led us to conclude that there must be a balance between song stability and conservatism, which
lead to clear-cut species recognition, and song variation which leads to individual recognition.

1. The primary purpose of the passage is to

(A) raise new issues


(B) explain an enigma
(C) refute misconceptions
(D) reconcile differing theories
(E) analyze a phenomenon

2. According to the passage, which of the following is true about the number and
general nature of figures sung by the indigo bunting?

(A) They are established at birth.


(B) They evolve slowly as the bird learns.
(C) They are learned from other indigo buntings.
(D) They develop after the bird has been forced onto marginal breeding areas.
(E) They gradually develop through contact with prospective mates.

3. It can be inferred that the investigation that determined the similarity among more
than 90 percent of all the figures produced by birds living in different regions was
undertaken to answer which of the following questions?
I. How much variation, if any, is there in the figure types produced by indigo buntings in different
locales?

Further, although we found some unique figures in different geographical populations,


more than 90 percent of all the figures of birds from different regions are alike.
II. Do local populations of indigo buntings develop their own dialects of figure types?
III . Do figure similarities among indigo buntings decline with increasing geographic separation?

(A) II only
(B) III only
(C) I and II only
(D) II and III only
(E) I, II, and III

4. It can be inferred from the passage that the existence of only a limited number of
indigo bunting figures serves primarily to

(A) ensure species survival by increasing competition among the fittest males for the females
(B) increase population density by eliminating ambiguity in the figures to which the females must
respond
(C) maintain the integrity of the species by restricting the degree of figure variation and change
(D) enhances spices recognition by decreasing the number of figure patterns to which
the bird must respond
(E) avoid confusion between species by clearly demarcating the figure patterns of each species

The third paragraph indicates that the songs serve as a means of recognition for members of the
same species. The fourth paragraph discusses the strict limitations on the ways in which figures are
produced. The last paragraph indicates that "song stability and conservatism," that is, limits to the
numbers of figures and variations, are essential for clear-cut species recognition. Choice D is a
statement of that idea.
5. It can be inferred that a dummy of a male indigo bunting was placed near the tape
recorder that played the songs of different species in order to try to

(A) simulate the conditions in nature


(B) rule out visual cues as a factor in species recognition
(C) supply an additional clue to species recognition for the indigo bunting
(D) provide data on the habits of bunting species other than the indigo bunting
(E) confound the indigo buntings in the experiment

When we played a tape recording of a lazuli bunting or a painted bunting, male indigo buntings did
not respond, even when a dummy of a male indigo bunting was placed near the tape
recorder.

6. According to the passage , the authors played a normal indigo bunting song
backwards in order to determine which of the following?
(A) What are the limits of the frequency range that will provide recognition by the indigo bunting?
(B) What is the time duration necessary for recognition by the indigo bunting?
(C) How specific must a figure shape be for it to be recognized by the indigo bunting?
(D) How does variation in the pacing of song figures affect the indigo bunting's recognition of the
figures ?
(E) Is the indigo bunting responding to cues other than those in the song figures ?

7. According to the passage , the indigo buntings' songs function in which of the
following ways?
I. To delineate a breeding area
II. To defend a breeding area
III. To identify the birds to their mates
(A) I only
(B) II only
(C) I and III only
(D) II and III only
(E) I, II, and III

Integrity (n) : tính toàn vẹn

Demarcate (v) : phân giới, chia biên giới

Cues (n) : tín hiệu, vai trò

Visual (adj) : thị giác

Delineate (v) : phân định, miêu tả, phác họa

Rhythmic (adj) : phách, nhịp

Modulate (v) : chuyển điệu, ngâm nga, điều chế

Dummy (n) : hình nộm

Intra-species (n) : nội loại

Conservatism (n) : thuộc về bảo tồn, bảo thủ


Nov 8 – 2

Despite their many differences of temperament and of literary perspective, Emerson, Thoreau,
Hawthorne, Melville, and Whitman share certain beliefs. Common to all these writers is their (5)
humanistic perspective. Its basic premises are that humans are the spiritual center of the universe
and that in them alone is the clue to nature, history, and ultimately the cosmos itself. Without
denying outright the existence either of a deity or of brute (10) matter, this perspective nevertheless
rejects them as exclusive principles of interpretation and prefers to explain humans and the world in
terms of humanity itself. This preference is expressed most clearly in the Transcendentalist principle
(15) that the structure of the universe literally duplicates the structure of the individual self;
therefore, all knowledge begins with self-knowledge. This common perspective is almost always
universalized. Its emphasis is not upon the (20) individual as a particular European or American, but
upon the human as universal, freed from the accidents of time, space, birth, and talent. Thus, for
Emerson, the" American Scholar" turns out to be simply "Man Thinking"; while, for Whitman, (25)
the "Song of Myself" merges imperceptibly into a song of all the "children of Adam", where "every
atom belonging to me as good belongs to you." Also common to all five writers is the belief that
individual virtue and happiness depend upon (30) self-realization, which, in turn, depends upon the
harmonious reconciliation of two universal psychological tendencies: first, the self-asserting impulse
of the individual to withdraw, to remain unique and separate, and to be responsible only to (35)
himself or herself, and second, the self-transcending impulse of the individual to embrace the whole
world in the experience of a single moment and to know and become one with that world. These
conflicting impulses can be seen in the (40) democratic ethic. Democracy advocates individualism,
the preservation of the individual's freedom and self-expression. But the democratic self is torn
between the duty to self, which is implied by the concept of liberty, and the duty to society, (45)
which is implied by the concepts of equality and fraternity. A third assumption common to the five
writers is that intuition and imagination offer a surer road to the truth than does abstract logic or
scientific (50) method. It is illustrated by their emphasis upon introspection-their belief that the clue
to external nature is to be found in the inner world of individual psychology-and by their
interpretation of experience as, in essence, symbolic. 80th these stresses (55) presume an organic
relationship between the self and the cosmos of which only intuition and imagination can properly
take account. These writers' faith in the imagination and in themselves as practitioners of
imagination led them to conceive (60) of the writer as a seer and enabled them to achieve supreme
confidence in their own moral and metaphysical insights.

1. The author's discussion of Emerson, Thoreau, Hawthorne, Melville, and Whitman is


primarily concerned with explaining

(A) some of their beliefs about the difficulties involved in self-realization


(B) some of their beliefs concerning the world and the place that humanity occupies in
the universal order
(C) some of their beliefs concerning the relationship between humanism and democracy
(D) the way some of their beliefs are shaped by differences in temperament and literary outlook
(E) the effects of some of their beliefs on their writings

2. According to the passage, the humanistic perspective of the five writers presupposes
which of the following?
1. The structure of the universe can be discovered through self-knowledge.
II. The world can be explained in terms of humanity.
III. The spiritual and the material worlds are incompatible.
(A) I only
(B) II only
(C) I and II only
(D) II and III only
(E) I, II, and III

3. The author quotes Whitman primarily in order to

(A) show that the poet does not agree with Emerson
(B) indicate the way the poet uses the humanist ideal to praise himself
(C) suggest that the poet adapts the basic premises of humanism to his own individual outlook on
the world
(D) illustrate a way the poet expresses the relationship of the individual to the
humanistic universe
(E) demonstrate that the poet is concerned with the well- being of all humans

4. According to the passage, the five writers object to the scientific method primarily
because they think it

(A) is not the best way to obtain an understanding of the relationship between the
individual and the cosmos
(B) is so specialized that it leads to an understanding of separate parts of the universe but not of the
relationships among those parts
(C) cannot provide an adequate explanation of intuition and imagination
(D) misleads people into believing they have an understanding of truth, when they do not
(E) prevents people from recognizing the symbolic nature of experience

in essence, symbolic. 80th these stresses (55) presume an organic relationship between the self and
the cosmos of which only intuition and imagination can properly take account

5. Which of the following statements would be compatible with the beliefs of the five
writers as described in the passage?
I. Democracy works as a form of government because every individual is unique.
II. Nature alone exists, and each person is nothing more than a shadow of that substance which is
the world.
III. The human mind is capable of discovering the meaning of life and understanding the order in
the universe.

(A) I only
(B) III only
(C) I and II only
(D) I and III only
(E) I, II, and III

6. It can be inferred that intuition is important to the five writers primarily because it
provides them with

(A) information useful for understanding abstract logic and the scientific method
(B) the discipline needed in the search for truth
(C) inspiration for their best writing
(D) clues to the interpretation of symbolic experience
(E) the means of resolving conflicts between the self and the world

7. The author discusses "the democratic ethic" in the highlighted text in order to

(A) explain the relationship between external experience and inner imagination
(B) support the notion that the self-contains two conflicting and irreconcilable factions
(C) illustrate the relationship between the self's desire to be individual and its desire to
merge with all other selves
(D) elaborate on the concept that the self constantly desires to realize its potential
(E) give an example of the idea that, in order to be happy, the self must reconcile its desires with
external reality

8. It can be inferred that the idea of "an organic relationship between the self and the
cosmos" in the highlighted text is necessary to the thinking of the five writers because
such a relationship

(A) enables them to assert the importance of the democratic ethic


(B) justifies their-concept of the freedom of the individual
(C) sustains their faith in the existence of a deity
(D) is the foundation of their humanistic view of existence
(E) is the basis for their claim that the writer is a seer

9. The passage is most relevant to which of the following areas of study?

(A) Aesthetics and logic


(B) History and literature
(C) Theology and sociology
(D) Anthropology and political science
(E) Linguistics and art

Temperament (n) : tính cách

Humanistic (adj) : nhân văn

Cosmos (n) : vũ trụ, thế giới

Deity (n) : thần linh

Universalize (v) : phổ cập

Imperceptibly (adv) : không thể nhận ra

Harmonious (adj) : hài hòa

Reconciliation (n) : hòa giải

Fraternity (n) : tình anh em


Intuition (n) : trực giác

Seer (n) : nhà tiên tri

Insight (n) : cái nhìn, hiểu biết sâu sắc


Nov 9 – 1

This history of responses to the work of the artist Sandro Botticelli (1444?-1510) suggests that
widespread appreciation by critics is a relatively recent phenomenon. Writing in 1550, (5) Vasari
expressed an unease with Botticelli's work, admitting that the artist fitted awkwardly into his
(Vasari's) evolutionary scheme of the history of art. Over the next two centuries, academic art
historians denigrated Botticelli in (10) favor of his fellow Florentine, Michelangelo. Even when
antiacademic art historians of the early nineteenth century rejected many of the standards of
evaluation espoused by their predecessors, Botticelli's work remained outside of ac(15)cepted taste,
pleasing neither amateur observers nor connoisseurs. (Many of his best paintings, however,
remained hidden away in obscure churches and private homes.) The primary reason for Botticelli's
unpopu-(20)larity is not difficult to understand: most observers, up until the mid-nineteenth century,
did not consider him to be noteworthy because his work, for the most part, did not seem to these
observers to exhibit the traditional characteris-(25)tics of fifteenth-century Florentine art. For
example, Botticelli rarely employed the technique of strict perspective and, unlike Michelangelo,
never used chiaroscuro. Another reason for Botticelli's unpopularity may have been that his at-(30)
titude toward the style of classical art was very different from that of his contemporaries.

Although he was thoroughly exposed to classical art, he showed little interest in borrowing from the
classical style. Indeed, it is paradoxical that (35) a painter of large-scale classical subjects adopted a
style that was only slightly similar to that of classical art. In any case, when viewers began to
examine more closely the relationship of Botticelli's work (40) to the tradition of fifteenth-century
Florentine art, his reputation began to grow. Analyses and assessments of Botticelli made between
1850 and 1870 by the artists of the Pre-Raphaelite movement, as well as by the writer Pater
(although (45) he, unfortunately, based his assessment on an incorrect analysis of Botticelli's
personality), inspired a new appreciation of Botticelli throughout the English-speaking world. Yet
Botticelli's work, especially the Sistine frescoes, did not (50) generate worldwide attention until it
was finally subjected to a comprehensive and scrupulous analysis by Horne in 1908. Horne rightly
demonstrated that the frescoes shared important features with paintings by other fifteenth-century
(55) Florentines- features such as a skillful representation of anatomical proportions, and of the
human figure in motion. However, Horne argued that Botticelli did not treat these qualities as ends
in themselves-rather, that he empha-(60)sized clear depiction of a story, a unique achievement and
one that made the traditional Florentine qualities less central. Because of Horne's emphasis on the
way a talented artist reflects a tradition yet moves beyond that tradi-(65)tion, an emphasis crucial to
any study of art, the twentieth century has come to appreciate Botticelli's achievements.

1. Which of the following would be the most appropriate title for the passage?

(A) Botticelli's Contribution to Florentine Art


(B) Botticelli and the Traditions of Classical Art
(C) Sandro Botticelli: From Denigration to Appreciation
(D) Botticelli and Michelangelo: A Study in Contrasts
(E) Standards of Taste: Botticelli's Critical Reputation up to the Nineteenth Century
2. It can be inferred that the author of the passage would be likely to find most
beneficial a study of an artist that

(A) avoided placing the artist in an evolutionary scheme of the history of art
(B) analyzed the artist's work in relation to the artist's personality
(C) analyzed the artist's relationship to the style and subject matter of classical art
(D) analyzed the artist's work in terms of both traditional characteristics and unique
achievement
(E) sanctioned and extended the evaluation of the artist's work made by the artist's contemporaries

3. The passage suggests that Vasari would most probably have been more enthusiastic
about BotticeIIi's work if that artist's work

(A) had not revealed Botticelli's inability to depict a story clearly


(B) had not evolved so straightforwardly from the Florentine art of the fourteenth century
(C) had not seemed to Vasari to be so similar to classical art
(D) could have been appreciated by amateur viewers as well as by connoisseurs
(E) could have been included more easily in Vasari's discussion of art history

Vasari's views are described in lines 4-8, which imply that Vasari was uneasy about Botticelli's work
because it did not Et Vasari's evolutionary scheme of the history of art. Thus, it can be inferred that,
had Botticelli's work been easier to include in Vasari's discussion of art history, Vasari would
probably have been more enthusiastic about Botticelli's work.

‘’Writing in 1550, (5) Vasari expressed an unease with Botticelli's work, admitting that the artist
fitted awkwardly into his (Vasari's) evolutionary scheme of the history of art’’
4. The author most likely mentions the fact that many of Botticelli's best paintings were
"hidden away in obscure churches and private homes"(highlighted text) in order to

(A) indicate the difficulty of trying to determine what an artist's best work is.
(B) persuade the reader that an artist's work should be available for general public viewing
(C) prove that academic art historians had succeeded in keeping Botticelli's work from general public
view
(0) call into question the assertion that antiacademic art historians disagreed with their predecessors
(E) suggest a reason why, for a period of time, Botticelli's work was not generally
appreciated

5. The passage suggests that most seventeenth- and eighteenth-century academic art
historians and most early-nineteenth-century antiacademic art historians would have
disagreed significantly about which of the following?

I. The artistic value of Botticelli's work


II. The criteria by which art should be judged
III. The features that characterized fifteenth-century Florentine art

(A) I only
(B) II only
(C) III only
(D) II and III only
(E) I, II, and III

I is incorrect because lines 8-16 indicate that most seventeenth-, eighteenth-, and early- nineteenth-
century art historians agreed on the artistic value of Botticelli's work - they all denigrated it. Choice
III is wrong because the lines that discuss the features that characterized fifteenth-century
Florentine art (lines 20-28) do not suggest that most seventeenth- and eighteenth-century academic
art historians would have disagreed with most early- nineteenth-century antiacademic art historians
about these features. Choice II is correct because lines 11-14 state that

Even when antiacademic art historians of the early nineteenth century rejected many of the
standards of evaluation espoused by their predecessors, Botticelli's work remained outside of
accepted taste, pleasing neither amateur observers nor connoisseurs.
6. According to the passage, which of the following is an accurate statement about
Botticelli's relation to classical art?

(A) Botticelli more often made use of classical subject matter than classical style.
(B) Botticelli's interest in perspective led him to study classical art.
(C) Botticelli's style does not share any similarities with the style of classical art.
(0) Because he saw little classical art, Botticelli did not exhibit much interest in imitating such art.
(E) Although Botticelli sometimes borrowed his subject matter from classical art, he did not create
large-scale paintings of these subjects.

Although he was thoroughly exposed to classical art, he showed little interest in borrowing from
the classical style. Indeed, it is paradoxical that (35) a painter of large-scale classical
subjects adopted a style that was only slightly similar to that of classical art.

7. According to the passage, Horne believed which of the following about the relation of
the Sistine frescoes to the tradition of fifteenth-century Florentine art?

(A) The frescoes do not exhibit characteristics of such art.


(B) The frescoes exhibit more characteristics of such art than do the paintings of Michel- Angelo.
(C) The frescoes exhibit some characteristics of such art, but these qualities are not the
dominant features of the frescoes.
(D) Some of the frescoes exhibit characteristics of such art, but most do not.
(E) More of the frescoes exhibit a skillful representation of anatomical proportions than a skillful
representation of the human figure in motion.

8. The passage suggests that, before Horne began to study Botticelli's work in 1908,
there had been

(A) little appreciation of Botticelli in the English-speaking world


(B) an overemphasis on Botticelli's transformation, in the Sistine frescoes, of the principles of
classical art
(C) no attempt to compare Botticelli's work to that of Michelangelo.
(D) no thorough investigation of Botticelli's Sistine frescoes
(E) little agreement among connoisseurs and amateurs about the merits of Botticelli's work

Unease (n) : sự khó chịu


Scheme (n) : kế hoạch

Denigrate (v) : chê bai

Connoisseur (n) : người sành sỏi

Obscure (adj) : bí mật

Scrupulous (adj) : chu đáo


Nov 10 – 1

Those examples of poetic justice that occur in medieval and Elizabethan literature, and that seems
so satisfying, have encouraged a whole school of twentieth-century scholars to "find" further
examples. In fact, these scholars have merely forced victimized characters into a moral framework
by which the injustices inflicted on them are, somehow or other, justified. Such scholars deny that
the sufferers in a tragedy are innocent; they blame the victims themselves for their tragic fates. Any
misdoing is enough to subject a character to critical whips. Thus, there are long essays about the
misdemeanors of Webster's Duchess of Malfi, who defied her brothers, and the behavior of
Shakespeare's Desdemona, who disobeyed her father.

Yet it should be remembered that the Renaissance writer Matteo Bandello strongly protests
the injustice of the severe penalties issued to women for acts of disobedience that men could, and
did, commit with virtual impunity. And Shakespeare, Chaucer, and Webster often enlist their readers
on the side of their tragic heroines by describing injustices so cruel that readers cannot but join in
protest. By portraying Griselda, in The Clerk's Tale, as a meek, gentle victim who does not criticize,
much less rebel against the persecutor, her husband Walter, Chaucer incites readers to espouse
Griselda's cause against Walter's oppression. Thus, efforts to supply historical and theological
rationalizations for Walter's persecutions tend to turn Chaucer's fable upside down, to deny its most
obvious effect on readers' sympathies. Similarly, to assert that Webster's Duchess deserved torture
and death because she chose to marry the man she loved and to bear their children is, in effect, to
join forces with her tyrannical brothers, and so to confound the operation of poetic justice, of which
readers should approve, with precisely those examples of social injustice that Webster does
everything in his power to make readers condemn. Indeed, Webster has his heroine so heroically
lead the resistance to tyranny that she may well inspire members of the audience to imaginatively
join forces with her against the cruelty and hypocritical morality of her brothers.

Thus Chaucer and Webster, in their different ways, attack injustice, argue on behalf of the victims,
and prosecute the persecutors. Their readers serve them as a court of appeal that remains free to
rule, as the evidence requires, and as common humanity requires, in favor of the innocent and
injured parties. For—to paraphrase the noted eighteenth-century scholar, Samuel Johnson—despite
all the refinements of subtlety and the dogmatism of learning, it is by the common sense and
compassion of readers who are uncorrupted by the prejudices of some opinionated scholars that the
characters and situations in medieval and Elizabethan literature, as in any other literature, can best
be judged.

1. According to the passage, some twentieth-century scholars have written at length


about

(A) Walter's persecution of his wife in Chaucer's The Clerk's Tale


(B) the Duchess of Malfi's love for her husband
(C) the tyrannical behavior of the Duchess of Malfi's brothers
(D) the actions taken by Shakespeare's Desdemona
(E) the injustices suffered by Chaucer's Griselda

2. The primary purpose of the passage is to

(A) describe the role of the tragic heroine in medieval and Elizabethan literature
(B) resolve a controversy over the meaning of "poetic justice" as it is discussed in certain medieval
and Elizabethan literary treatises
(C) present evidence to support the view that characters in medieval and Elizabethan tragedies are
to blame for their fates
(D) assert that it is impossible for twentieth- century readers to fully comprehend the characters and
situations in medieval and Elizabethan literary works
(E) argue that some twentieth-century scholars have misapplied the concept of "poetic
justice" in analyzing certain medieval and Elizabethan literary works

The author argues in the passage that a school of twentieth-century scholars has inappropriately
applied the concept of poetic justice to a number of medieval and Elizabethan literary figures,
including the Duchess of Malfi, Desdemona, and Griselda. Thus the first paragraph describes these
scholars as having "merely forced victimized characters" (line 6) into the framework of poetic
justice, the second paragraph presents specific examples of the misapplication of poetic justice, and
the third paragraph argues that it is readers' 'uncorrupted by the prejudices of some opinionated
scholars" (lines 60-61) who can best judge medieval and Elizabethan literature.

3. It can be inferred from the passage that the author considers Chaucer's Griselda to be

(A) an innocent victim


(B) a sympathetic judge
(C) an imprudent person
(D) a strong individual
(E) a rebellious daughter

the author considers Griselda to be "a meek, gentle victim" who does not even criticize her husband,
and thus is an innocent victim of his "oppression."

4. The author's tone in her discussion of the conclusions reached by the "school of
twentieth-century scholars" in the highlighted text is best described as

(A) plaintive
(B) philosophical
(C) disparaging
(D) apologetic
(E) enthusiastic

5. It can be inferred from the passage that the author believes that most people respond
to intended instances of poetic justice in medieval and Elizabethan literature with

(A) annoyance
(B) disapproval
(C) indifference
(D) amusement
(E) gratification

6. As described in the passage, the process by which some twentieth-century scholars


have reached their conclusions about the blameworthiness of victims in medieval and
Elizabethan literary works is most similar to which of the following?

(A) Derivation of logically sound conclusions from well-founded premises


(B) Accurate observation of data, inaccurate calculation of statistics, and drawing of incorrect
conclusions from the faulty statistics
(C) Establishment of a theory, application of the theory to ill-fitting data, and drawing of
unwarranted conclusions from the data
(D) Development of two schools of thought about a factual situation, debate between the two
schools, and rendering of a balanced judgment by an objective observer
(E) Consideration of a factual situation by a group, discussion of various possible explanatory
hypotheses, and agreement by consensus on the most plausible explanation

7. The author's paraphrase of a statement by Samuel Johnson (in the highlighted text)
serves which of the following functions in the passage?

(A) It furnishes a specific example.


(B) It articulates a general conclusion.
(C) It introduces a new topic.
(D) It provides a contrasting perspective.
(E) It clarifies an ambiguous assertion.

8. The author of the passage is primarily concerned with

(A) reconciling opposing viewpoints


(B) encouraging innovative approaches
(C) defending an accepted explanation
(D) advocating an alternative interpretation
(E) analyzing an unresolved question

Poetic justice (n) : công lý thi ca

Inflict on (v) : gây ra

Justify (v) : chứng minh là đúng

Whip (n) : roi da

Misdemeanor (n) : tội nhẹ, điều lăng nhục

Defy (v) : khinh thường, thách đấu

Impunity (n) : không bị trừng phạt

Enlist (v) : tranh thủ, chiêu mộ, đầu quân

Heroine (n) : nữ anh hùng

Meek (adj) : nhu mì, hiền lành

Rebel (v) : dấy loạn, nổi loạn

Incite (v) : xúi giục, khêu gợi, khuyến khích

Rationalization (n) : hợp lý hóa


Fable (n) : truyện ngụ ngôn

Refinement (n) : sàng lọc

Subtlety (n) : sự khôn khéo, tinh tế

Dogmatism (n) : giáo điều

Prejudice (n) : định kiến

Articulate (v) : khớp nối

Ill-fitting (adj) : không phù hợp

Unwarranted (adj) : không chính đáng

Gratification (n) : sự hài lòng, vừa ý

Amusement (n) : vui chơi, sự tiêu khiển


Nov 10 – 2

The antigen-antibody immunological reaction used to be regarded as typical of immunological


responses. Antibodies are proteins synthesized by specialized cells called plasma cells, which are (5)
formed by lymphocytes (cells from the lymph system) when an antigen, a substance foreign to the
organism’s body, comes in contact with lymphocytes. Two important manifestations of antigen
antibody immunity are lysis, the rapid (10) physical rupture of antigenic cells and the liberation of
their contents into the surrounding medium, and phagocytosis, a process in which antigenic particles
are engulfed by and very often digested by macrophages and polymorphs. (15) The process of lysis
is executed by a complex and unstable blood constituent known as complement, which will not work
unless it is activated by a specific antibody; the process of phagocytosis is greatly facilitated when
the par-(20)ticles to be engulfed are coated by a specific antibody directed against them. The
reluctance to—abandon this hypothesis, however well it explains specific processes, impeded new
research, and for many years anti-(25)gens and antibodies dominated the thoughts of
immunologists so completely that those immunologists overlooked certain difficulties. Perhaps the
primary difficulty with the antigen-antibody explanation is the informational problem of how (30) an
antigen is recognized and how a structure exactly complementary to it is then synthesized. When
molecular biologists discovered, moreover, that such information cannot flow from protein to
protein, but only from nucleic acid to (35) protein, the theory that an antigen itself provided the
mold that directed the synthesis of an antibody had to be seriously qualified. The attempts at
qualification and the information provided by research in molecular biology led (40) scientists to
realize that a second immunological reaction is mediated through the lymphocytes that are hostile to
and bring about the destruction of the antigen. This type of immunological response is called cell
mediated immunity. (45) Recent research in cell-mediated immunity has been concerned not only
with the development of new and better vaccines, but also with the problem of transplanting tissues
and organs from one organism to another, for although cir-(50)culating antibodies play a part in the
rejection of transplanted tissues, the primary role is played by cell-mediated reactions. During
cellmediated responses, receptor sites on specific lymphocytes and surface antigens on the foreign
(55) tissue cells form a complex that binds the lymphocytes to the tissue. Such lymphocytes do not
give rise to antibody-producing plasma cells but themselves bring about the death of the foreign-
tissue cells, probably by secreting a variety of (60) substances, some of which are toxic to the tissue
cells and some of which stimulate increased phagocytic activity by white blood cells of the
macrophage type. Cell-mediated immunity also accounts for the destruction of intracellular
parasites.

1. The author is primarily concerned with

(A) proving that immunological reactions do not involve antibodies


(B) establishing that most immunological reactions involve antigens
(C) criticizing scientists who will not change their theories regarding immunology
(D) analyzing the importance of cells in fighting disease
(E) explaining two different kinds of immunological reactions

2. The author argues that the antigen-antibody explanation of immunity “had to


seriously qualified” (line 37) because

(A) antibodies were found to activate unstable components in the blood


(B) antigens are not exactly complementary to antibodies
(C) lymphocytes have the ability to bind to the surface of antigens
(D) antibodies are synthesized from protein whereas antigens are made from nucleic acid
(E) antigens have no apparent mechanism to direct the formation of an antibody
3. The author most probably believes that the antigen-antibody theory of immunological
reaction.

(A) is wrong
(B) was accepted without evidence
(C) is unverifiable
(D) is a partial explanation
(E) has been a divisive issue among scientists

4. The author mentions all of the following as being involved in antigen-antibody


immunological reactions EXCEPT the

(A) synthesis of a protein


(B) activation of complement in the bloodstream
(C) destruction of antibodies
(D) entrapment of antigens by macrophages
(E) formation of a substance with a structure complementary to that of an antigen

5. The passage contains information that would answer which of the following questions
about cell-mediated immunological reactions?

I. Do lymphocytes form antibodies during cell-mediated immunological reactions?


II. Why are lymphocytes more hostile to antigens during cell-mediated immunological reactions than
are other cell groups?
III. Are cell-mediated reactions more pronounced after transplants than they are after parasites
have invaded the organism?

(A) I only
(B) I and II only
(C) I and III only
(D) II and III only
(E) I, II, and III

6. The passage suggests that scientists might not have developed the theory of cell-
mediated immunological reactions if

(A) proteins existed in specific group types


(B) proteins could have been shown to direct the synthesis of other proteins
(C) antigens were always destroyed by proteins
(D) antibodies were composed only of protein
(E) antibodies were the body’s primary means of resisting disease

7. According to the passage, antibody-antigen and cell-mediated immunological


reactions both involve which of the following processes?

I. The destruction of antigens


II. The creation of antibodies
III. The destruction of intracellular parasites
(A) I only
(B) II only
(C) III only
(D) I and II only
(E) II and III only

8. The author supports the theory of cell-mediated reactions primarily by

(A) pointing out a contradiction in the assumption leading to the antigen-antibody theory
(B) explaining how cell mediation accounts for phenomena that the antigen-antibody
theory cannot account for
(C) revealing new data that scientists arguing for the antigen-antibody theory have continued to
ignore
(D) showing that the antigen-antibody theory fails to account for the breakup of antigens
(E) demonstrating that cell mediation explains lysis and phagocytosis more fully than the antigen-
antibody theory does
Nov 11 – 2

Woodrow Wilson was referring to the liberal idea of the economic market when he said that the free
enterprise system is the most efficient economic system. Maximum freedom means maximum
productiveness; our “openness” is to be the measure of our stability. Fascination with this ideal has
made Americans defy the “Old World” categories of settled possessiveness versus unsettling
deprivation, the cupidity of retention versus the cupidity of seizure, a “status quo” defended or
attacked. The United States, it was believed, had no status quo ante. Our only “station” was the
turning of a stationary wheel, spinning faster and faster. We did not base our system on property
but opportunity—which meant we based it not on stability but on mobility. The more things
changed, that is, the more rapidly the wheel turned, the steadier we would be. The conventional
picture of class politics is composed of the Haves, who want a stability to keep what they have, and
the Have-Nots, who want a touch of instability and change in which to scramble for the things they
have not. But Americans imagined a condition in which speculators, self-makers, runners are always
using the new opportunities given by our land. These economic leaders (front-runners) would thus
be mainly agents of change. The nonstarters were considered the ones who wanted stability, a
strong referee to give them some position in the race, a regulative hand to calm manic speculation;
an authority that can call things to a halt, begin things again from compensatorily staggered
“starting lines.”

“Reform” in America has been sterile because it can imagine no change except through the
extension of this metaphor of a race, wider inclusion of competitors, “a piece of the action,” as it
were, for the disenfranchised. There is no attempt to call off the race. Since our only stability is
change, America seems not to honor the quiet work that achieves social interdependence and
stability. There is, in our legends, no heroism of the office clerk, no stable industrial work force of
the people who actually make the system work. There is no pride in being an employee (Wilson
asked for a return to the time when everyone was an employer). There has been no boasting about
our social workers—they are merely signs of the system’s failure, of opportunity denied or not taken,
of things to be eliminated. We have no pride in our growing interdependence, in the fact that our
system can serve others, that we are able to help those in need; empty boasts from the past make
us ashamed of our present achievements, make us try to forget or deny them, move away from
them. There is no honor but in the Wonderland race we must all run, all trying to win, none winning
in the end (for there is no end).

1. The primary purpose of the passage is to

(A) criticize the inflexibility of American economic mythology


(B) contrast “Old World” and “New World” economic ideologies
(C) challenge the integrity of traditional political leaders
(D) champion those Americans whom the author deems to be neglected
(E) suggest a substitute for the traditional metaphor of a race

2. According to the passage, “Old World” values were based on

(A) ability
(B) property
(C) family connections
(D) guild hierarchies
(E) education
We did not base our system on property but opportunity—which meant we based it not on stability
but on mobility.
3. In the context of the author’s discussion of regulating change, which of the following
could be most probably regarded as a “strong referee” (line 30) in the United States?

(A) A school principal


(B) A political theorist
(C) A federal court judge
(D) A social worker
(E) A government inspector

The nonstarters were considered the ones who wanted stability, a strong referee to give them
some position in the race, a regulative hand to calm manic speculation; an authority that can call
things to a halt, begin things again from compensatorily staggered “starting lines.”

4. The author sets off the word “Reform” (line 35) with quotation marks in order to

(A) emphasize its departure from the concept of settled possessiveness


(B) show his support for a systematic program of change
(C) underscore the flexibility and even amorphousness of United States society
(D) indicate that the term was one of Wilson’s favorites
(E) assert that reform in the United States has not been fundamental

5. It can be inferred from the passage that the author most probably thinks that giving
the disenfranchised “a piece of the action” (line 38) is

(A) a compassionate, if misdirected, legislative measure


(B) an example of Americans’ resistance to profound social change
(C) an innovative program for genuine social reform
(D) a monument to the efforts of industrial reformers
(E) a surprisingly “Old World” remedy for social ills

6. Which of the following metaphors could the author most appropriately use to
summarize his own assessment of the American economic system (lines 35-60)?

(A) A windmill
(B) A waterfall
(C) A treadmill
(D) A gyroscope
(E) A bellows
There is no honor but in the Wonderland race we must all run, all trying to win, none winning in the
end (for there is no end).

7. It can be inferred from the passage that Woodrow Wilson’s ideas about the economic
market

(A) encouraged those who “make the system work” (lines 45-46)
(B) perpetuated traditional legends about America
(C) revealed the prejudices of a man born wealthy
(D) foreshadowed the stock market crash of 1929
(E) began a tradition of presidential proclamations on economics
8. The passage contains information that would answer which of the following
questions?

I. What techniques have industrialists used to manipulate a free market?


II. In what ways are “New World” and “Old World” economic policies similar?
III. Has economic policy in the United States tended to reward independent action?

(A) I only
(B) II only
(C) III only
(D) I and II only
(E) II and III only

9. Which of the following best expresses the author’s main point?

(A) Americans’ pride in their jobs continues to give them stamina today.
(B) The absence of a status quo ante has undermined United States economic structure.
(C) The free enterprise system has been only a useless concept in the United States.
(D) The myth of the American free enterprise system is seriously flawed.
(E) Fascination with the ideal of “openness” has made Americans a progressive people.

Perpetuate (v) : làm cho lâu dài, kéo dài

Defy (v) : coi thường, bất chấp

Possessiveness (n) : tính sở hữu

Deprivation (n) : bị tước quyền, mất quyền lợi

Cupidity (n) : tánh tham lam

Retention (n) : sự giữ lại, tiếp tục kiểm soát

Seizure (n) : sự thâu tóm, chiếm lấy

Status quo (n) : hiện trạng, tình trạng

Stationary (adj) : không động đậy, đứng im

Steadier (adj) : vững vàng hơn

Scramble (v) : tranh giành, xô đẩy

Referee (n) : trọng tài

Regulative (adj) : quy định

Manic (adj) : hưng phấn

Halt (n) : tạm dừng lại

Stagger (v) : lảo đảo, loạng choạng

Sterile (adj) : khô khan


Disenfranchised (v) : bị tước quyền

Call off (v) : dừng lại, tạm dừng

Interdependence (n) : sự phụ thuộc lẫn nhau

Heroism (n) : chủ nghĩa anh hùng

Clerk (n) : nhân viên bán hàng

Boast (v) : khoe khoang

Bellow (n) : ống thổi


Nov 12 – 1

How many really suffer as a result of labor market problems? This is one of the most critical yet
contentious social policy questions. In many ways, our social statistics exaggerate the degree of
hardship. Unemployment does not have the same dire consequences today as it did in the 1930’s
when most of the unemployed were primary breadwinners, when income and earnings were usually
much closer to the margin of subsistence, and when there were no countervailing social programs
for those failing in the labor market. Increasing affluence, the rise of families with more than one
wage earner, the growing predominance of secondary earners among the unemployed, and
improved social welfare protection have unquestionably mitigated the consequences of joblessness.
Earnings and income data also overstate the dimensions of hardship. Among the millions with hourly
earnings at or below the minimum wage level, the overwhelming majority are from multiple-earner,
relatively affluent families. Most of those counted by the poverty statistics are elderly or
handicapped or have family responsibilities which keep them out of the labor force, so the poverty
statistics are by no means an accurate indicator of labor market pathologies.

Yet there are also many ways our social statistics underestimate the degree of labor-market-related
hardship. The unemployment counts exclude the millions of fully employed workers whose wages
are so low that their families remain in poverty. Low wages and repeated or prolonged
unemployment frequently interact to undermine the capacity for self-support. Since the number
experiencing joblessness at some time during the year is several times the number unemployed in
any month, those who suffer as a result of forced idleness can equal or exceed average annual
unemployment, even though only a minority of the jobless in any month really suffer. For every
person counted in the monthly unemployment tallies, there is another working part-time because of
the inability to find full-time work, or else outside the labor force but wanting a job. Finally, income
transfers in our country have always focused on the elderly, disabled, and dependent, neglecting the
needs of the working poor, so that the dramatic expansion of cash and in-kind transfers does not
necessarily mean that those failing in the labor market are adequately protected.

As a result of such contradictory evidence, it is uncertain whether those suffering seriously as a


result of labor market problems number in the hundreds of thousands or the tens of millions, and,
hence, whether high levels of joblessness can be tolerated or must be countered by job creation and
economic stimulus. There is only one area of agreement in this debate—that the existing poverty,
employment, and earnings statistics are inadequate for one their primary applications, measuring
the consequences of labor market problems.

1. Which of the following is the principal topic of the passage?

(A) What causes labor market pathologies that result in suffering


(B) Why income measures are imprecise in measuring degrees of poverty
(C) Which of the currently used statistical procedures are the best for estimating the incidence of
hardship that is due to unemployment
(D) Where the areas of agreement are among poverty, employment, and earnings figures
(E) How social statistics give an unclear picture of the degree of hardship caused by low
wages and insufficient employment opportunities

2. The author uses “labor market problems” in lines 1-2 to refer to which of the
following?
(A) The overall causes of poverty
(B) Deficiencies in the training of the work force
(C) Trade relationships among producers of goods
(D) Shortages of jobs providing adequate income
(E) Strikes and inadequate supplies of labor

3. The author contrasts the 1930’s with the present in order to show that

(A) more people were unemployed in the 1930’s


(B) unemployment now has less severe effects
(C) social programs are more needed now
(D) there now is a greater proportion of elderly and handicapped people among those in poverty
(E) poverty has increased since the 1930’s

4. Which of the following proposals best responds to the issues raised by the author?

(A) Innovative programs using multiple approaches should be set up to reduce the level of
unemployment.
(B) A compromise should be found between the positions of those who view joblessness as an evil
greater than economic control and those who hold the opposite view.
(C) New statistical indices should be developed to measure the degree to which
unemployment and inadequately paid employment cause suffering.
(D) Consideration should be given to the ways in which statistics can act as partial causes of the
phenomena that they purport to measure.
(E) The labor force should be restructured so that it corresponds to the range of job vacancies.

5. The author’s purpose in citing those who are repeatedly unemployed during a twelve-
month period is most probably to show that

(A) there are several factors that cause the payment of low wages to some members of the labor
force
(B) unemployment statistics can underestimate the hardship resulting from joblessness
(C) recurrent inadequacies in the labor market can exist and can cause hardships for individual
workers
(D) a majority of those who are jobless at any one time to not suffer severe hardship
(E) there are fewer individuals who are without jobs at some time during a year than would be
expected on the basis of monthly unemployment figures

6. The author states that the mitigating effect of social programs involving income
transfers on the income level of low-income people is often not felt by

(A) the employed poor


(B) dependent children in single-earner families
(C) workers who become disabled
(D) retired workers
(E) full-time workers who become unemployed
7. According to the passage, one factor that causes unemployment and earnings figures
to overpredict the amount of economic hardship is the

(A) recurrence of periods of unemployment for a group of low-wage workers


(B) possibility that earnings may be received from more than one job per worker
(C) fact that unemployment counts do not include those who work for low wages and remain poor
(D) establishment of a system of record-keeping that makes it possible to compile poverty statistics
(E) prevalence, among low-wage workers and the unemployed, of members of families
in which others are employed

Earnings and income data also overstate the dimensions of hardship. Among the millions
with hourly earnings at or below the minimum wage level, the overwhelming majority are from
multiple-earner, relatively affluent families. Most of those counted by the poverty statistics are
elderly or handicapped or have family responsibilities which keep them out of the labor
force, so the poverty statistics are by no means an accurate indicator of labor market pathologies.

8. The conclusion stated in lines 33-39 about the number of people who suffer as a
result of forced idleness depends primarily on the point that

(A) in times of high unemployment, there are some people who do not remain unemployed for long
(B) the capacity for self-support depends on receiving moderate-to-high wages
(C) those in forced idleness include, besides the unemployed, both underemployed part-time
workers and those not actively seeking work
(D) at different times during the year, different people are unemployed
(E) many of those who are affected by unemployment are dependents of unemployed workers

9. Which of the following, if true, is the best criticism of the author’s argument
concerning why poverty statistics cannot properly be used to show the effects of
problems in the labor market?

(A) A short-term increase in the number of those in poverty can indicate a shortage of
jobs because the basic number of those unable to accept employment remains
approximately constant.
(B) For those who are in poverty as a result of joblessness, there are social programs available that
provide a minimum standard of living.
(C) Poverty statistics do not consistently agree with earnings statistics, when each is taken as a
measure of hardship resulting from unemployment.
(D) The elderly and handicapped categories include many who previously were employed in the
labor market.
(E) Since the labor market is global in nature, poor workers in one country are competing with poor
workers in another with respect to the level of wages and the existence of jobs.

the author thinks that the current statistics cannot properly be used to indicate actual poverty
because they either over represent or under represent.
Criticism for it should be excatly opposite i.e., the measures can be used in some ways to predict
poverty.
Option A is one such answer which shows that the current poverty measures can still be used .

Contentious (adj) : gây tranh cãi


Hardship (n) : sự thiếu thốn, khó khăn, mệt mỏi, thách thức gay go

Dire (adj) : tàn khốc, khốc liệt, tai hại

Breadwinner (n) : trụ cột gia đình, gia trưởng

Subsistence (n) : sinh hoạt phí, sự tồn tại

Countervail (v) : đối trọng, làm cân bằng

Affluence (n) : sự sung túc, giàu có

Predominance (n) : chiếm ưu thế

Mitigate (v) : giảm thiểu

Overwhelming (adj) : áp đảo

Pathology (n) : bệnh lý

Exclude (v) : loại trừ

Prolonged (adj) : kéo dài

Tally (n) : bản đối chiếu

In-kind transfer (n) : chuyển bằng hiện vật

Tolerate (v) : tha thứ, bỏ qua


Nov 12 – 2

The fossil remains of the first flying vertebrates, the pterosaurs, have intrigued paleontologists for
more than two centuries. How such large creatures, which weighed in some cases as much as a
piloted hang-glider and had wingspans from 8 to 12 meters, solved the problems of powered flight,
and exactly what these creatures were—reptiles or birds—are among the questions scientists have
puzzled over.

Perhaps the least controversial assertion about the pterosaurs is that they were reptiles. Their skulls,
pelvises, and hind feet are reptilian. The anatomy of their wings suggests that they did not evolve
into the class of birds. In pterosaurs a greatly elongated fourth finger of each forelimb supported a
wing-like membrane. The other fingers were short and reptilian, with sharp claws. In birds the
second finger is the principal strut of the wing, which consists primarily of feathers. If the pterosaurs
walked on all fours, the three short fingers may have been employed for grasping. When a
pterosaur walked or remained stationary, the fourth finger, and with it the wing, could only turn
upward in an extended inverted V-shape along each side of the animal’s body.

The pterosaurs resembled both birds and bats in their overall structure and proportions. This is not
surprising because the design of any flying vertebrate is subject to aerodynamic constraints. Both
the pterosaurs and the birds have hollow bones, a feature that represents a savings in weight. In
the birds, however, these bones are reinforced more massively by internal struts.

Although scales typically cover reptiles, the pterosaurs probably had hairy coats. T. H. Huxley
reasoned that flying vertebrates must have been warm-blooded because flying implies a high rate of
metabolism, which in turn implies a high internal temperature. Huxley speculated that a coat of hair
would insulate against loss of body heat and might streamline the body to reduce drag in flight. The
recent discovery of a pterosaur specimen covered in long, dense, and relatively thick hairlike fossil
material was the first clear evidence that his reasoning was correct.

Efforts to explain how the pterosaurs became airborne have led to suggestions that they launched
themselves by jumping from cliffs, by dropping from trees, or even by rising into light winds from
the crests of waves. Each hypothesis has its difficulties. The first wrongly assumes that the
pterosaurs’ hind feet resembled a bat’s and could serve as hooks by which the animal could hang in
preparation for flight. The second hypothesis seems unlikely because large pterosaurs could not
have landed in trees without damaging their wings. The third calls for high waves to channel
updrafts. The wind that made such waves however, might have been too strong for the pterosaurs
to control their flight once airborne.

1. It can be inferred from the passage that scientists now generally agree that the

(A) enormous wingspan of the pterosaurs enabled them to fly great distances
(B) structure of the skeleton of the pterosaurs suggests a close evolutionary relationship to bats
(C) fossil remains of the pterosaurs reveal how they solved the problem of powered flight
(D) pterosaurs were reptiles
(E) pterosaurs walked on all fours

The pterosaurs resembled both birds and bats in their overall structure and proportions.
2. The author views the idea that the pterosaurs became airborne by rising into light
winds created by waves as

(A) revolutionary
(B) unlikely
(C) unassailable
(D) probable
(E) outdated

3. According to the passage, the skeleton of a pterosaur can be distinguished from that
of a bird by the

(A) size of its wingspan


(B) presence of hollow spaces in its bones
(C) anatomic origin of its wing strut
(D) presence of hooklike projections on its hind feet
(E) location of the shoulder joint joining the wing to its body

4. The ideas attributed to T. H. Huxley in the passage suggest that he would most likely
agree with which of the following statements?

(A) An animal’s brain size has little bearing on its ability to master complex behaviors.
(B) An animal’s appearance is often influenced by environmental requirements and
physical capabilities.
(C) Animals within a given family group are unlikely to change their appearance dramatically over a
period of time.
(D) The origin of flight in vertebrates was an accidental development rather than the outcome of
specialization or adaptation.
(E) The pterosaurs should be classified as birds, not reptiles.

5. It can be inferred from the passage that which of the following is characteristic of the
pterosaurs?

(A) They were unable to fold their wings when not in use.
(B) They hung upside down from branches as bats do before flight.
(C) They flew in order to capture prey.
(D) They were an early stage in the evolution of the birds.
(E) They lived primarily in a forest-like habitat.

6. Which of the following best describes the organization of the last paragraph of the
passage?

(A) New evidence is introduced to support a traditional point of view.


(B) Three explanations for a phenomenon are presented, and each is disputed by means
of specific information.
(C) Three hypotheses are outlined, and evidence supporting each is given.
(D) Recent discoveries are described, and their implications for future study are projected.
(E) A summary of the material in the preceding paragraphs is presented, and conclusions are drawn.
7. It can be inferred from the passage that some scientists believe that pterosaurs

(A) lived near large bodies of water


(B) had sharp teeth for tearing food
(C) were attacked and eaten by larger reptiles
(D) had longer tails than many birds
(E) consumed twice their weight daily to maintain their body temperature

Vertebrate (n) : động vật có xương sống

Intrigue (v) : khêu gợi

Paleontologist (n) : nhà cổ sinh vật học

Glider (n) : tàu lượn

Reptile (n) : bò sát

Pelvis (n) : xương chậu

Hind feet (n) : chân sau

Forelimb (n) : chân trước

Membrane (n) : màng, da mỏng

Strut (n) : thanh chống

Aerodynamic (adj) : khí động học

Constraint (n) : hạn chế

Hollow (adj) : lõm xuống

Metabolism (n) : sự trao đổi chất

Airborne (adj) : đang bay, trên không, đà cất cánh

Probable (adj) : có thể xảy ra

Unassailable (adj) : không thể công kích


Nov 13 – 1

Labeling Zora Neale Hurston "a writer of the Harlem Renaissance" is a characterization that may, at
first glance, obscure, rather than clarify, the particularities of her career. The Harlem Renaissance
(5) sance was a spirit more than a movement, and because a spirit is ephemeral, generalizations
about the Harlem Renaissance and its writers are either too hard or too easy. They have come easily
enough to a whole generation of critics, but their (10) pithy summaries seldom reflect the wide
divisions between Blacks and Whites, the Black intelligentsia and Black workers, Black writers and
their middle-class audience, that marked the era. When one studies in depth the phenomenon of
what was then (15) called the Negro Renaissance or the New Negro Renaissance, and what is now
called the Harlem Renaissance, one comes away with a bewildering complex of notions, statements,
affirmations, and manifestos. Although there is general agreement (20) that the Harlem Renaissance
is bounded by the 1918 armistice ending the First World War and the beginning of the Great
Depression in 1930, some historians have stretched the boundaries to before the war (1914) and
after Franklin Delano Roosevelt's second term (25) (1941). There has been a wide-spread tendency
to regard the Harlem Renaissance as a monolithic cultural movement, capable of reduction to one
orthodoxy or another or to a set of characteristic principles. This presumption (30) reflects the bias
in most American scholarship that postulates Black people as a united entity and then poses theories
ignoring individuation of thought and feeling. Sometimes, however, an individual career can be (35)
best assessed in the context of an age, and this is largely the case with the writer and
anthropologist Zora Neale Hurston. She spends exactly two paragraphs on the Renaissance in her
autobiography, and her other writing, public and private, offers (40) a very little discussion of what
the Harlem Renaissance meant to her. Yet her part in the Renaissance is well-documented in the
reminiscences of others, with unanimous agreement that she was one of the most memorable
personages of the period. As (45) Langston Hughes put it in The Big Sea, she "was certainly the
most amusing" of the Harlem Renaissance artists, "full of side-splitting anecdotes, humorous tales,
and tragicomic stories." Hughes's words should not imply that she was solely an (50) entertainer.
Although she was independent and scornful of literary movements, she shared in the historical and
cultural forces that made the Harlem Renaissance an identifiable moment in American intellectual
history, a part of a historical process (55) that, as most critics recognize, altered Black life in
America. She, in turn, responded to and helped to shape the aesthetic assumptions of that era.
Between 1919 and 1930, Black writers were published in greater numbers than in any single decade
(60) in American life prior to the 1960's. Hurston's awareness of this literary ferment certainly con-
tributed to her development as a writer.

1. The main purpose of the passage is to

(A) define the Harlem Renaissance as a distinct period in Afro-American culture and Ameri- can
culture as a whole
(B) demonstrate that the Harlem Renaissance included more varied points of view than scholars
generally assume
(C) dispute the accepted estimation of Zora Neale Hurston's contribution to Afro-American culture
(D) acknowledge the conflicts and contradictions that were a distinct part of the cultural life of the
Harlem Renaissance
(E) place Zora Neale Hurston within a historical and intellectual context

2. It can be inferred from the passage that the author would regard which of the
following as characteristic of a movement but not of a spirit?

(A) Individual statements of purpose


(B) Conflicts between rivals
(C) The cultivation of notoriety
(D) The development of new ideas
(E) An explicit ideology

3. The author suggests that which of the following is an overlooked yet significant
aspect of the Harlem Renaissance?

(A) The similarity of the works produced by Black writers during the Harlem Renaissance
(B) The perceptions of the central figures of the Harlem Renaissance as recorded in autobi-
ographies and letters
(C) The interdisciplinary nature of Hurston's work
(D) The nature of the relationship between Black artists and their audiences
(E) The effect of Roosevelt's policies on artists and writers

4. Which of the following facts, mentioned in the passage, supports the author's
statement that the Harlem Renaissance was "an identifiable moment in American
intellectual history" in the highlighted text?

(A) During this period, scholars who had previously been unaware of Black literary tradition began to
assess that tradition.
(B) Widespread social reform took place during this period.
(C) During this period, Black people acted upon common concerns for the first time in the twentieth
century.
(D) The number of published works by Black writers increased dramatically during this
period.
(E) The armistice ending the First World War began a period of increased prosperity for Americans.

5. According to the passage, Hurston's attitude toward literary movements in general


was

(A) bemused
(B) ambivalent
(C) indifferent
(D) disdainful
(E) belligerent

"Hughes's words should not imply that she was solely an entertainer. Although she was independent
and scornful of literary movements, she shared in the historical and cultural forces that made the
Harlem Renaissance an identifiable moment in American intellectual history"

6. The author implies that Hurston's account of her role in the Harlem Renaissance is

(A) embellished
(B) unenlightening
(C) comic
(D) sardonic
(E) sentimental
"She spends exactly two paragraphs on the Renaissance in her autobiography, and her other
writing, public and private, offers a very little discussion of what the Harlem Renaissance meant to
her"

7. It can be inferred from the passage that accounts of Hurston given by her
contemporaries emphasized which of the following?

(A) Her work as an anthropologist


(B) Her independence from political movements
(C) Her humor
(D) Her friendliness
(E) Her contributions to aesthetic theory

8. On which of the following grounds does the author dismiss previous critical work on
the Harlem Renaissance?

(A) It is based on a faulty assumption about the homogeneity of Black experience.


(B) It has failed to acknowledge the work of the most talented writers of the Harlem Renaissance.
(C) It has failed to explain the consequences of the Harlem Renaissance for Black life.
(D) It has taken a sociological and historical approach rather than an aesthetic one.
(E) It has concentrated on the personalities of the writers of the Harlem Renaissance rather than on
their work.

9. Which of the following statements is best supported by information in the passage?

(A) Black workers were not the primary audience for works produced by Black writers
during the Harlem Renaissance.
(B) The number of works produced by Black writers has continued to increase annually since 1919.
(C) Langston Hughes created an unintentionally misleading portrait of Zora Neale Hurston in The Big
Sea.
(D) Zora Neale Hurston is better known for her anthropological work than for her literary work.
(E) There was almost no market for works produced by Black writers during the Harlem
Renaissance.

They have come easily enough to a whole generation of critics, but their (10) pithy summaries
seldom reflect the wide divisions between Blacks and Whites, the Black intelligentsia and Black
workers, Black writers and their middle-class audience, that marked the era.

Obscure (adj) : tối nghĩa, khó hiểu

Ephemeral (adj) : không bền vững, mau tàn

Pithy (adj) : mạnh mẽ, đầy phí khách

Intelligentsia (n) : đội ngũ trí thức, người thông minh

Bewilder (v) : hoang mang

Manifesto (n) : tuyên ngôn


Armistice (n) : đình chiến, ngừng bắn

Postulate (v) : định đề, bổ nhiệm

Autobiography (n) : tự truyện

Unanimous (adj) : nhất trí, toàn thể

Reminiscence (n) : vật kỷ niệm

Amusing (adj) : vui, giải trí

Side-splitting (adj) : chuyện tức cười, vui cười

Anecdote (n) : giai thoại, chuyện vặt

Scornful (adj) : khinh thường

Ferment (n) : náo động, náo loạn

Embellish (v) : tôn tạo, tô điểm

Comic (adj) : hài hước

Sardonic (adj) : chế nhạo, chua chát

Sentimental (adj) : đa cảm

Unenlighten (v) : không được làm sáng tỏ, không có sự giác ngộ

Bemuse (v) : lưỡng lự, kinh ngạc

Ambivalent (adj) : cảm xúc lẫn lộn, ý tưởng trái ngược

Indifferent (adj) : vô tư, công bình, không thiên vị, lạnh lùng, trung lập

Belligerent (adj) : hiếu chiến, thù địch, hung hăng

Disdainful (adj) : coi thường, khinh bỉ


Nov 13 – 2

The conventional spark-ignition (Otto) automobile engine has inherent virtues that become more
apparent when alternative engines are considered. These virtues include a respectable efficiency
(especially under partial load), lightweight, ease of starting, acceptable emissions (with control
devices), and a negligible requirement for expensive fabrication materials (and hence a low
manufacturing cost). Nonetheless, concern about air pollution has focused attention on alternative
engines that are potentially more promising with respect to achieving minimal emission levels-the
compression-ignition (Diesel) engine, the steam or vapor-cycle (Rankine) engine, the gas turbine
(Brayton engine), and the Stirling engine.

The conventional automobile engine is indeed at a disadvantage with respect to exhaust emissions
because many pollutants are formed as a consequence of the intermittent combustion process, with
its rapid chilling of the combustion products. The engine has been substantially cleaned up but, with
one or two exceptions, this task has called for catalytic emission- control devices that tend to
degenerate rapidly. All the continuous-combustion engines emit far smaller quantities of pollutants
than the spark-ignition engine emits in the absence of emission-control devices; in fact,
experimental models of vapor-cycle engines, gas turbines, and Stirling engines have surpassed the
most stringent emission requirements yet established. Although the Diesel engine is an intermittent-
combustion engine, it has the advantage of operating with excess air, so that Its carbon monoxide
emission is negligible. Moreover, hydrocarbons are normally only a small constituent of Diesel
exhaust. However, because oxides of nitrogen are formed in hot flames and are retained as a result
of rapid chilling, the Diesel engine would not be able to meet stringent oxides-of-nitrogen standards.
There are also unsubstantiated suspicions that Diesel-engine exhaust particulates carry carcinogens.

If emission standards were to necessitate the replacement of the Otto engine, the Stirling engine
would be the most promising candidate. The part-load efficiency of some experimental Stirling
engines (40 percent) is the highest of any of the alternative engines. The engine has the further
advantage, shared by steam engines and gas turbines, of being able to operate on almost any fuel.
It is very quiet, is the easiest of all alternative engines to start, and operates satisfactorily in freezing
temperatures. Since the Stirling engine works on a gas cycle, it has the potential of operating at still
higher temperatures and pressures than it does at present, which would make it even more efficient
and lighter. Unfortunately, because present experimental models need a substantial amount of
metal capable of withstanding high temperatures, they would be expensive to manufacture, but
there is hope that alternative ceramic components can be developed.

In conclusion, at present the venerable spark-ignition engine remains largely unsurpassed except
with respect to emissions. But if an alternative engine must be developed, automobile makers would
be wise to invest in the Stirling.

1. According to the passage, the carcinogenicity of Diesel-engine exhaust particulates is

(A) an insoluble problem


(B) a known fact
(C) a common belief
(D) an unsupported hypothesis
(E) an unacceptable risk

2. The passage suggests that the major disadvantage of the spark-ignition engine would
be greatly reduced if which of the following were to occur?
(A) A starter that makes the engine much easier to start is invented.
(B) A method of removing oxides of nitrogen from the engine's exhaust emissions is devised.
(C) An emissions-control device that maintains its effectiveness is designed for the
engine.
(D) Materials that make the engine much less expensive to manufacture are developed.
(E) Manufacturing materials that make the engine much lighter in weight are developed

3. According to the passage, the Rankine, Brayton, and Stirling engines are similar in
which of the following respects?

I. They are continuous-combustion engines.


II. They can operate on almost any fuel.
III. They meet very strict antipollution standards.

(A) I only
(B) I and II only
(C) I and III only
(D) II and III only
(E) I, II, and III

If emission standards were to necessitate the replacement of the Otto engine, the Stirling engine
would be the most promising candidate. The part-load efficiency of some experimental Stirling
engines (40 percent) is the highest of any of the alternative engines. The engine has the further
advantage, shared by steam engines and gas turbines, of being able to operate on almost
any fuel.

4. It can be inferred from the passage that the major purpose of developing ceramic
parts for the Stirling engine would be to replace some current metal components with
parts that are

(A) less expensive


(B) lighter in weight
(C) easier to manufacture
(D) more capable of withstanding freezing temperatures
(E) more capable of withstanding high temperatures

5. The passage implies that which of the following is currently a disadvantage of at least
one of the alternative automobile engines?

(A) Rapid degeneration


(B) High manufacturing cost
(C) High cost of fuel
(D) Excessive carbon monoxide emissions
(E) Excessive hydrocarbon emissions

Unfortunately, because present experimental models need a substantial amount of metal capable of
withstanding high temperatures, they would be expensive to manufacture, but there is hope
that alternative ceramic components can be developed.
6. The author's attitude towards the potential of the Stirling engine can be best
described as

(A) ironic
(B) derogatory
(C) confused
(D) cautiously optimistic
(E) completely neutral

7. It can be inferred from the passage that which of the following events, if it were to
occur, would immediately cause the most difficulties for manufacturers of automobiles
that have Diesel engines?

(A) Emission requirements for carbon monoxide are made more rigid.
(B) Emission requirements for oxides of nitrogen are made very strict.
(C) A study is undertaken to determine whether Diesel-engine exhaust particulates are carcinogenic.
(D) Researchers find that catalytic emission control devices create a by-product that is harmful to
health.
(E) Researchers find that all continuous combustion engines have a serious, previously undiscovered
drawback when used as automobile engines.

8. Which of the following best summarizes the author's main point?

(A) Except in the area of oxides-of-nitrogen emissions, the Diesel engine is nonpolluting; of the
other alternative engines, the Stirling engine is the most promising.
(B) Except in the area of exhaust emissions, the conventional automobile engine is
currently better overall than alternative engines; of these, the Stirling engine most
warrants further development.
(C) Although the conventional automobile engine has many advantages, its failure to meet stringent
emission standards makes the development of the Stirling engine essential.
(D) Although concern about air pollution has focused attention on the emissions of alternative
automobile engines, it has not led to the full development of any of them.
(E) Once automobile makers become fully aware of the advantages offered by the spark- ignition
engine, they will realize that even the very efficient Stirling engine is not worth developing.

Ignition (n) : bộ phận đánh lửa

Spark (n/v) : tia lửa/ phát ra tia lửa

Negligible (adj) : không đáng kể, có thể bỏ qua

Fabrication (n) : sự cấu tạo, sự chế tạo

Compression (n) : sức nén

Intermittent (adj) : gián đoạn

Combustion (n) : đốt cháy, sự cháy

Catalytic (adj) : xúc tác

Degenerate (adj/v) : thoái hóa/ xuống cấp


Stringent (adj) : nghiêm khắc, nghiêm ngặt

Carcinogen (n) : chất gây ung thư

Necessitate (v) : bắt buộc, điều kiện thiết yếu


Nov 14 – 1

A review of the treatment of female characters in Chinese fiction reveals that Chinese social
attitudes have undergone dramatic changes. Prior to the twentieth century, women in novels were
stereotypes lacking any features that made them unique individuals and were also subject to
numerous restrictions imposed by the male-dominated culture. While authors of these novels often
sympathetically portrayed heroines who experienced social oppression, they never questioned the
position of women in Chinese culture. Not until the early twentieth century did Chinese fiction focus
on women's emancipation, and then the subject became the backdrop of most novels that
addressed the issue. After the Communist party established the People's Republic in the late 1940's,
attitudes changed again: the gaining of women's rights was treated as one of many ongoing social
revolutions, although from the beginning Communist party policy subordinated the women's struggle
to the class struggle.

In spite of the fact that the authors who dealt with women's issues prior to 1949 agreed in principle
that reforms had to be instituted, the outlook they depicted for reform was bleak. In their novels, a
pattern recurs: after an initial break with social conventions, women falter in their goals or tragically
end their lives, defeated by the overwhelming pressures of those conventions. If some writers
viewed the emancipation of women as an achievable end, most tended to regard it as related to
other seemingly unattainable social changes. Individualism alone would not lead to emancipation.
Taking his cue from Ibsen's play A Doll's House, in which the heroine, Nora, leaves home because
she resents her husband's treating her like a child, the writer Lu Hsün warned that Nora would need
money to support herself; she must have economic rights in order to survive.

In contrast to this view of women in fiction in the early part of the century, fiction after the late
1940's is not so pessimistic. The deeper problems of socially prescribed roles for wife and daughter,
for example, are not explored, but greater freedom for women is presented as the product of
collective action. Novels of this period focus primarily on two specific issues: voluntary marriage and
equal participation in work. After Mao Tse-tung's announcement of guidelines for a literature of
socialist realism, this emphasis on women's rights became more pronounced. Most women in fiction
after 1949 conform to the goals set for them by Communist party policy but still experience conflicts
within family and group relationships as a result of the double burden placed on them by their
domestic and job roles. Fiction of this period also depicts the problems of compensating women
adequately for their work and of giving them access to jobs previously performed by men. Although
these novels forcefully suggest that such reforms face much resistance, all clearly conclude that
eventually this resistance can be overcome. And, in fact, the past two decades have seen the
beginnings of some of these reforms in the lives of women in the People's Republic of China.

1. The passage is primarily concerned with discussing which of the following subjects?

(A) The impact on the Chinese novel of Mao Tsetung's guidelines for a literature of socialist realism
(8) The impact of Ibsen's plays on themes in twentieth-century Chinese literature
(C) Stylistic differences between pre-1900 and post-1900 Chinese fiction
(D) Changes in the role of women in Chinese society brought about by Communist party policy
(E) The characterization of women in twentieth-century Chinese literature
2. According to the passage, post-1949 Chinese fiction differs from earlier twentieth-
century Chinese fiction in that it

(A) lacks a coherent philosophical point of view


(B) views the possibility of social change with greater optimism
(C) is concerned primarily with entertaining rather than educating its readers
(D) ignores the problems facing groups other than women within Chinese society
(E) ignores the traditions and social conventions of Chinese culture

3. The author mentions Lu Hsiin's comments on A Doll's House in order to make the
point that

(A) individualism, as presented in the play, benefits only the individual, not the community in which
he or she lives
(8) Chinese writers looked to Western literature for evidence of pressures of social conventions
similar to those pressures in their own culture
(C) Chinese writers felt the need to focus on the practical problems presented by a
particular social environment as well as on personal needs
(D) the preoccupation in Western literature with women's emancipation blinded writers and thinkers
to the broader social issues involved
(E) the treatment of women's emancipation in Western literature has little relevance to Chinese
literature

4. The passage provides information for answering which of the following questions?

(A) How did the government of the People's Republic of China deal with resistance to their policy of
adequately compensating women for their work?
(B) What are some of the problems encountered by female characters in novels
describing life in the People's Republic of China after 1949 ?
(C) What were the goals set for Chinese women by Communist party policy?
(D) Do the stereotypes prevalent in Chinese fiction before the twentieth century still appear in
Chinese literature?
(E) What initiated the first changes in the depiction of women in Chinese literature?

5. According to the passage, the struggle of Chinese women for liberation is portrayed in
post-1949 Chinese literature as

(A) a struggle with roots in pre-twentieth century events


(B) a product of pre-1949 social reforms
(C) subordinate to the maintenance of traditional social patterns
(D) part of a much larger struggle for liberation
(E) hampered by unrealistic Communist party policy

After the Communist party established the People's Republic in the late 1940's, attitudes changed
again: the gaining of women's rights was treated as one of many ongoing social revolutions,
although from the beginning Communist party policy subordinated the women's struggle to the class
struggle.
6. Which of the following plots exemplifies what the author describes as the pessimism
of the Chinese literature written between 1900 and 1949?

(A) A woman trained as an economist takes a position in her field, only to relinquish it
because she realizes she cannot overcome her male colleagues' antagonism toward
professional women.
(B) A 60-year-old widow, who manages her village's nursery, refuses to quit her job to help her
daughter, who has just given birth to a child.
(C) A man in charge of a tea-curing plant selects a woman to fill his position when .he retires, but
she must spend a long time battling the resentment of the male workers.
(D) A woman whose illness forces her temporarily to resign her full-time job in a food coopera- tive
elects to work part-time in a bookstore.
(E) A daughter-in-law chooses not to stay home to care for her in-laws as is the custom, but takes a
job in a factory and plans to advance to a management position.

7. Which of the following statements would be most likely to begin the paragraph
immediately following the passage?
(A) The subject of women's emancipation, which had germinated years before, surfaced in public
debate in the period after 1919.
(B) The pessimistic view of the emancipation of women in the 1920's was dispelled when it was
seen that emancipation could be achieved by working with a series of limited and specific goals.
(C) A vivid reflection of the changes that occurred in Chinese fiction is found in Lo Pin-
chi's novel, Mother Wang (1953).
(D) In an interesting reversal of the problems produced by the generation gap, Ju Chihchiian's
novel, Wish Fulfilled (1959), depicts a son's difficulty in adjusting to his mother's newly acquired job
and independence.
(E) Recently written Chinese fiction, unlike earlier works, is didactic and not philosophical.

8. The passage suggests that the author would most probably agree with which of the
following statements about the relationship between Chinese novels written after 1949
and life in China during that period?
(A) The novels were slow to reflect changes in culture.
(B) The novels reminded citizens of their heritage and traditions, as well as pointed the way toward
future possibilities.
(C) The novels provided government-sanctioned role models for citizens.
(D) The authors presented a variety of experiences far wider than normally common to their
audience.
(E) The authors seemed to be indifferent to the problems created by women's familial and job
responsibilities.

"After Mao Tse-tung's announcement of guidelines for a literature of socialist realism, this emphasis
on women's rights became more pronounced. Most women in fiction after 1949 conform to the
goals set for them by Communist party policy"

Relinquish (v) : từ bỏ, bỏ việc

Vivid (adj) : sống động, rõ ràng

Sanction (v) : phê chuẩn, tán thành


Nov 15 – 1

I remember the astonishment I felt when I first read Shakespeare. I expected to receive a powerful
aesthetic pleasure, but having read, one after the other, works regarded as his
best: "King Lear," "Romeo and Juliet," "Hamlet" and "Macbeth," not only did I feel no delight, but I
also felt an irresistible repulsion and tedium, and doubted as to whether I was senseless in feeling
works regarded as the summit of perfection by the whole of the civilized world to be trivial and
positively bad, or whether the significance which this civilized world attributes to the works of
Shakespeare was itself senseless. My consternation was increased by the fact that I always keenly
felt the beauties of poetry in every form; then why should artistic works recognized by the whole
world as those of a genius not just fail to please me, but in fact be disagreeable to me?

For a long time I could not believe in myself, and during fifty years, in order to test myself, I several
times recommenced reading Shakespeare in every possible form, in Russian, in English, in German
and in Schlegel's translation, as I was advised. Several times I read the dramas and the comedies
and historical plays, and I invariably underwent the same feelings: repulsion, weariness, and
bewilderment. At the present time, before writing this preface, being desirous once more to test
myself, I have, as an old man of seventy-five, again read the whole of Shakespeare, including the
historical plays, the "Henrys," "Troilus and Cressida," the "Tempest," "Cymbeline," and I have felt,
with even greater force, the same feelings, this time, however, not of bewilderment, but of firm,
indubitable conviction that the unquestionable glory of a great genius which Shakespeare enjoys,
and which compels writers of our time to imitate him and readers and spectators to discover in him
non-existent merits,—thereby distorting their aesthetic and ethical understanding,—is a great evil, as
is every untruth.

Although I know that the majority of people so firmly believe in the greatness of Shakespeare that in
reading this judgment of mine they will not admit even to the possibility of its justice, and will not
give it the slightest attention, nevertheless I will endeavour, as well as I can, to show why I believe
that Shakespeare cannot be recognized either as a great genius, or even as an average author.

1) In context of the passage, what does the line - ‘artistic works recognized by the
whole world as those of a genius’ - refer to?

A. The poetry that was read and appreciated by the author


B. Those few works of Shakespeare that the author actually liked
C. The translations of Shakespeare‘s popular works
D. The writings of the author of the passage
E. The majority of Shakespeare‘s works

2) From the information in the passage, which of the following can most reasonably be
inferred about the author of the passage?

A. The author probably knew several languages


B. The author was most likely biased against Shakespeare
C. The author was scared of publicly criticising Shakespeare‘s works
D. The author believed that Shakespeare‘s works were actually evil
E. As the author read more of Shakespeare‘s works, his dislike of the same somewhat diminished
3) Each of the following can be inferred about Shakespeare from the passage EXCEPT:

A. There are some people who do not consider him a genius


B. His works are considered the best in the field by a lot of people
C. He is regarded as a great writer by a large number of people around the world
D. He did not limit himself to only one type of writing
E. He wrote in more than one language

4) What is the meaning of the word ‗consternation‘ in context of the passage?

A. Unhappiness
B. Sorrow
C. Disappointment
D. Confusion
E. Dejection

In earlier lines in the 1st para, the author states that he was not sure whether the people who consider
Shakespeare were wrong in their judgement or whether he was wrong in his. He further goes on to say
that this doubt was further aggravated by the fact that he was normally easily able to appreciate the
beauty in every type of writing, so he was confused as to why couldn‘t he find the same in Shakespeare‘s
works.

Astonishment (n) : sự ngạc nhiên

Delight (n) : vui sướng, hân hoan, khoái lạc

Irressistible (adj) : không thể cưỡng lại

Repulsion (n) : thù ghét, hiềm kỵ

Tedium (n) : sự nhàm chán

Trival (adj) : tầm thường

Consternation (n) : sự lo lắng, mất tinh thần

Weariness (n) : mệt mỏi

Bewilderment (n) : hoang mang, bối rối

Desirous (adj) : mong muốn, thèm muốn

Indubitable (adj) : bất khuất, không còn nghi ngờ

Conviction (n) : lòng tin chắc chăn

Endeavour (v) : nỗ lực


Nov 15 – 2

Evolutionary psychology takes as its starting point the controversial assertion that the anatomical
and physiological features of the human brain have arisen as a result of adaptations to the demands
of the environment over the millennia. However, from this reasonable point of departure, these
psychologists make unreasonable extrapolations. They claim that the behavior of contemporary man
(in almost all its aspects) is a reflection of features of the brain that acquired their present
characteristics during those earliest days of our species when early man struggled to survive and
multiply.

This unwarranted assumption leads, for example, to suggestions that modern sexual behavior is
dictated by realities of Pleistocene life. These suggestions have a ready audience, and the idea that
Stone Age man is alive in our genome and dictating aspects of our behavior has gained ground in
the popular imagination. The tabloids repeatedly run articles about discoveries relating to genes for
aggression, depression, repression, and anything for which we need a readymade excuse. Such
insistence on a genetic basis for behavior negates the cultural influences and the social realities that
separate us from our ancestors.

The difficulty with pseudo science of this nature is just this popular appeal. People are eager to
accept what is printed as incontrovertible, assuming quite without foundation, that anything printed
has bona fide antecedents. We would do well to remember that the phrenologists of the nineteenth
century held sway for a considerable time in the absence of any evidence that behavioral tendencies
could be deduced from the shape of the skull. The phrenologists are no more, but their genes would
seem to be thriving.

1. The authors primary purpose in the passage is to

A. argue for the superiority of a particular viewpoint


B. attack the popular press
C. ridicule a particular branch of science
D. highlight an apparently erroneous tendency in an area of social science
E. evaluate a particular theory of human behavior in all its ramifications

2. The author mentions phrenologists as

A. pseudo scientists who are the logical antecedents of evolutionary psychologists


B. a group with inherent appeal to the followers of evolutionary psychologists
C. a warning against blind acceptance of ideas
D. scientists with whom evolutionary psychologists share common assumptions
E. behavioral scientists who have spawned a variety of wrong ideas

3. The author apparently believes that the journalists writing for the tabloids

A. are more concerned with popular appeal than with authenticity


B. believe that human behavior has a genetic basis We have no knowledge of what the tabloids do
or do not believe
C. run the same articles over and over again
D. are victims of the human desire to excuse inexcusable behavior
E. are highly irresponsible in their efforts to pander to the public
Erroneous (adj) : sai lầm

Ramification (n) : sự chia nhánh, phân nhánh

Ridicule (v) : chê cười, nhạo báng

Anatomical (adj) : giải phẫu

Physiological (adj) : sinh lý

Extrapolation (n) : phép ngoại suy

Contemporary (adj) : cùng lúc, đồng thời

Dictate (v) : ra lệnh, bắt buộc

Genome (n) : bộ gen

Tabloid (n) : báo lá cải

Insistence (n) : sự nhấn mạnh

Incontrovertible (adj) : không thể chối cãi

Bona fide (adj) : thành thật, thiện ý

Antecedent (n) : tiền đề

Phrenologist (n) : bác sĩ nhi khoa


Nov 17 – 1

Between the eighth and eleventh centuries A. D., the Byzantine Empire staged an almost
unparalleled economic and cultural revival, a recovery that is all the more striking because it
followed a long period of severe internal decline. By the early eighth century, the empire had lost
roughly two-thirds of the territory it had possessed in the year 600, and its remaining area was
being raided by Arabs and Bulgarians, who at times threatened to take Constantinople and
extinguish the empire altogether. The wealth of the state and its subjects was greatly diminished,
and artistic and literary production had virtually ceased. By the early eleventh century, however, the
empire had regained almost half of its lost possessions, its new frontiers were secure, and its
influence extended far beyond its borders. The economy had recovered, the treasury was full, and
art and scholarship had advanced.

To consider the Byzantine military, cultural, and economic advances as differentiated aspects of a
single phenomenon is reasonable. After all, these three forms of progress have gone together in a
number of states and civilizations. Rome under Augustus and fifth-century Athens provide the most
obvious examples in antiquity. Moreover, an examination of the apparent sequential connections
among military, economic, and cultural forms of progress might help explain the dynamics of
historical change.
The common explanation of these apparent connections in the case of Byzantium would run like
this: when the empire had turned back enemy raids on its own territory and had begun to raid and
conquer enemy territory, Byzantine resources naturally expanded and more money became available
to patronize art and literature. Therefore, Byzantine military achievements led to economic
advances, which in turn led to cultural revival.

No doubt this hypothetical pattern did apply at times during the course of the recovery. Yet it is not
clear that military advances invariably came first, economic advances second, and intellectual
advances third. In the 860’s the Byzantine Empire began to recover from Arab incursions so that by
872 the military balance with the Abbasid Caliphate had been permanently altered in the empire’s
favor. The beginning of the empire’s economic revival, however, can be placed between 810 and
830. Finally, the Byzantine revival of learning appears to have begun even earlier. A number of
notable scholars and writers appeared by 788 and, by the last decade of the eighth century, a
cultural revival was in full bloom, a revival that lasted until the fall of Constantinople in 1453. Thus
the commonly expected order of military revival followed by economic and then by cultural recovery
was reversed in Byzantium. In fact, the revival of Byzantine learning may itself have influenced the
subsequent economic and military expansion

1. which of the following best states the central idea of the passage?

(a) the byzantine empire was a unique case in which the usual order of military and economic
revival preceding cultural revival was reversed.
(b) the economic, cultural, and military revival in the byzantine empire between the eighth and
eleventh centuries was similar in its order to the sequence of revivals in augustan rome and fifth-
century athens.
(c) after 810 byzantine economic recovery spurred a military and, later, cultural expansion that
lasted until 1453.
(d) the eighth-century revival of byzantine learning is an inexplicable phenomenon, and its economic
and military precursors have yet to be discovered.
(e) the revival of the byzantine empire between the eighth and eleventh centuries
shows cultural rebirth preceding economic and military revival, the reverse of the
commonly accepted order of progress.

2. the primary purpose of the second paragraph is which of the following?

(a) to establish the uniqueness of the byzantine revival


(b) to show that augustan rome and fifth-century athens are examples of cultural, economic, and
military expansion against which all subsequent cases must be measured
(c) to suggest that cultural, economic. and military advances have tended to be closely
interrelated in different societies.
(d) to argue that, while the revivals of augustan rome and fifth-century athens were similar, they
are unrelated to other historical examples
(e) to indicate that, wherever possible, historians should seek to make comparisons with the earliest
chronological examples of revival

3. it can be inferred from the passage that by the eleventh century the byzantine
military forces

(a) had reached their peak and begun to decline


(b) had eliminated the bulgarian army
(c) were comparable in size to the army of rome under augustus
(d) were strong enough to withstand the abbasid caliphate's military forces
(e) had achieved control of byzantine governmental structures

In the 860’s the Byzantine Empire began to recover from Arab incursions so that by 872 the military
balance with the Abbasid Caliphate had been permanently altered in the empire’s favor.

4. it can be inferred from the passage that the byzantine empire sustained significant
territorial losses

(a) in 600
(b) during the seventh century
(c) a century after the cultural achievements of the byzantine empire had been lost
(d) soon after the revival of byzantine learning
(e) in the century after 873

5. in the third paragraph, the author most probably provides an explanation of the
apparent connections among economic, military, and cultural development in order to

(a) suggest that the process of revival in byzantium accords with this model
(b) set up an order of events that is then shown to be not generally applicable to the
case of byzantium
(c) cast aspersions on traditional historical scholarship about byzantium
(d) suggest that byzantium represents a case for which no historical precedent exists
(e) argue that military conquest is the paramount element in the growth of empires

6. which of the following does the author mention as crucial evidence concerning the
manner in which the byzantine revival began?
(a) the byzantine military revival of the 860's led to economic and cultural advances.
(b) the byzantine cultural revival lasted until 1453.
(c) the byzantine economic recovery began in the 900's.
(d) the revival of byzantine learning began toward the end of the eighth century.
(e) by the early eleventh century the byzantine empire had regained much of its lost territory.

7. according to the author, "the common explanation" (line 28) of connections between
economic, military, and cultural development is

(a) revolutionary and too new to have been applied to the history of the byzantine empire
(b) reasonable, but an antiquated theory of the nature of progress
(c) not applicable to the byzantine revival as a whole, but does perhaps accurately
describe limited periods during the revival
(d) equally applicable to the byzantine case as a whole and to the history of military, economic, and
cultural advances in ancient greece and rome
(e) essentially not helpful, because military, economic, and cultural advances are part of a single
phenomenon

Unparalleled (adj) : vô song, không thể so sánh

Revival (n) : hồi sinh, sự phục hưng

Raid (v) : đột kích

Extinguish (v) : dập tắt, thủ tiêu

Frontier (n) : biên giới

Antiquity (n) : cổ xưa, đời xưa

Sequential (adj) : tuần tự, liền nhau, nối tiếp

Patronize (v) : bảo trợ, khuyến khích

Incursion (n) : sự xâm nhập, xâm lược, đột kích

Withstand (v) : chịu được, chống cự


Nov 18 – 1

Barbara Strozzi was a singer and composer of madrigals, arias, and cantatas published in Venice
between 1644 and 1664. Her use of these vocal forms places her directly within the cantata tradition
of the mid-seven-(5)teenth century, along with such major figures as Rossi, Carissimi, and Cesti.
With the notable exception of Francesca Caccini (1587 – c. 1640), she is the only known woman
among the many aria and cantata composers of seventeenth-century Italy, and is, presumably,
among the (10) very few women of the period to have pursued a career as a composer and to have
achieved some measure of public recognition. This historical distinction attracted attention to her
works early in the present century, even when the music (15) of most of her male contemporaries,
and,indeed, most women composers of any era, remained relatively ignored. But appreciation of her
style was limited by prevailing convention to an isolation of its supposedly feminine qualities: “great
spontaneity, exquisite grace, mar-(20)velously fine taste.” Such an appreciation now appears
irrelevant as well as polemical in its incompleteness because we are in a better position – with
regard to both historical knowledge and social awareness – to attempt a more precise evaluation of
a somewhat anomalous fig-(25)ure like Barbara Strozzi. Born in 1619 in Venice, she grew up in the
home of Giulio Strozzi, a renowned poet and leading figure among Venetian intellectuals. Barbara’s
presence in Giulio’s household guaranteed her and early and full (30) exposure to Venetian musical
and literary society. Indeed, she was able to enter a world that was, apparently, closed to other
members of her sex. Similarly, Francesca Caccini, the most prominent and successful Italian woman
composer of the period, was the daughter (35) of professional musicians and therefore exposed to
music for infancy. This parallel suggests that such an environment may have been essential for the
development of a female composer. But though Strozzi’s music certainly shares fully (40) the
aesthetic aim of her contemporaries, and of the baroque in general – to move the passions - her life
and her work distinguishes her form these contemporaries in various ways. Whereas other
composers sought (and found) a public forum for their effective expression in the the-(45)ater and
the Church, her world remained more private. Strozzi was a singer in Venice, surrounded by opera
librettists and impresarios at a time when opera was the main cultural interest of a large segment of
Venetian society, yet she apparently never sang in opera, nor did (50) she write an opera. She is not
a composer of dramatic works; her songs are addressed to a more intimate audience, expressing
less the feeling of fictive characters than her own: “These harmonic notes,” she writes, “are the
language of the soul, and instruments of the heart.”

1. According to the passage, Barbara Strozzi’s music attracted attention early in the
twentieth century because of

A) Its uniquely private character


B) Its influence on Francesca Caccini
C) The scarcity of seventeenth-century women composers
D) The conventionality of its forms
E) The historical importance of Giulio Strozzi.

With the notable exception of Francesca Caccini (1587 – c. 1640), she is the only known woman
among the many aria and cantata composers of seventeenth-century Italy, and is, presumably,
among the very few women of the period to have pursued a career as a composer and to have
achieved some measure of public recognition. This historical distinction attracted attention to her
works early in the present century, even when the music (15) of most of her male contemporaries,
and,indeed, most women composers of any era, remained relatively ignored.
2. The author’s use of word “supposedly” in the line 18 implies which of the following?

A) The author doubts the historical authenticity of the quotation that follows.
B) The author doubts the accuracy of the facts she is reporting.
C) The author disagrees with the judgment she is discussing.
D) The author does not believe that Stozzi’s music has the qualities cited in the quotation.
E) The author is not sure of the significance of the quotation.

"appreciation of her style was limited by prevailing convention to an isolation of its supposedly
feminine qualities: “great spontaneity, exquisite grace, marvelously fine taste.” Such an appreciation
now appears irrelevant as well as polemical in its incompleteness

In saying "supposedly feminine qualities", the author is not doubting that Strozzi's music
contains the listed qualities. She is doubting that those qualities are feminine. The word
"supposedly" shows that the author disagrees with the prevailing views of Strozzi's works because
they were limited by a gender-biased viewpoint. This is expressed in answer choice (C):

3. With which of the following statements would the author of the passage be most
likely to agree?

A) The music of the seventeenth-century Italy is less frequently performed today than it was 50
years ago.
B) Contemporary music historians no longer discuses the music of a given composer in terms of its
particular individual style.
C) The cantata tradition of seventeenth-century Italy is much better understood today than it ever
has been.
D) Late-twentieth-century music historians have more accurate historical information
than their early-twentieth-century counterparts.
E) Music historians of the early twentieth century were uninterested in the details of social life in
seventeenth-century Venetian musical circles.

But appreciation of her style was limited by prevailing convention to an isolation of its supposedly
feminine qualities: “great spontaneity, exquisite grace, marvelously fine taste.” Such an appreciation
now appears irrelevant as well as polemical in its incompleteness because we are in a better
position – with regard to both historical knowledge and social awareness – to attempt a
more precise evaluation of a somewhat anomalous figure like Barbara Strozzi.

4. The author of the passage bases her assertion that Strozzi is one of the very few
seventeenth-century Italian women composers (lines 7-12) on which of the following
assumptions?

A) Public recognition is an indispensable part of a career as a composer.


B) Strozzi and Caccini were influenced by the same composers.
C) The music of any woman composer whom her seventeenth-century contemporaries
regarded as noteworthy would be known to modern scholars.
D) The cantata tradition of the mid-seventeenth-century includes composers and performers of
madrigals and arias as well as cantatas.
E) More women pursued careers as composers in seventeenth-century Italy than is evident from
music published in the seventeenth-century.
5. The author of the passage implies that which of the following was most essential to
the success of both Francesca Caccini and Barbara Strozzi?

(A) Their presence in Venice at a time when vocal music flourished


(B) Their early firsthand exposure to music and musicians
(C) The lack of competition from other composers
(D) The popularity of madrigals and cantatas in seventeenth-century Italy
(E) The uniqueness of the musical forms they created

6. It can be inferred from the passage that all of the following made Barbara Strozzi "a
somewhat anomalous figure" (lines 24-25) EXCEPT:

(A) She was a woman composer during the seven teenth century.
(B) She was intimately involved in Venetian literary and musical society.
(C) She did not write dramatic works for voice.
(D) She did not perform operatic works.
(E) She wrote moving baroque vocal works. – to specific

But though Strozzi’s music certainly shares fully (40) the aesthetic aim of her contemporaries, and of
the baroque in general – to move the passions - her life and her work distinguishes her form
these contemporaries in various ways

7. The author of the passage implies that composers of Italian baroque music typically

(A) composed more operas than cantatas


(B) preferred sacred to secular musical forms
(C) aspired to reach large segments of the public with their works
(D) avoided emotional expression in their works
(E) wrote music for solo voices rather than for choral ensembles

8. The author of the passage quotes Barbara Strozzi in lines 53-54 most probably in
order to

A) Support the claim that strozzi’s works are self-revealing


B) Qualify the point concerning strozzi’s dramatic work
C) Illustrate Strozzi’s confident approach to her art
D) Show why Strozzi avoided opera as both composer and performer
E) Suggest that Strozzi’s viewpoint was typical of baroque musicians

9. The passage provides information to answer which of the following questions?

A) What was the exact family relationship between Giulio Strozzi and Barbara Strozzi?
B) What is the evidence that indicates that Barbara Strozzi never sang in an opera?
C) Were Barbara strozzi’s compositions know to her contemporaries?
D) Did Barbara Storzzi know Francesca Caccini?
E) What instruments provide the accompaniment for Barbara Strozzi’s vocal works?
The evidence for (C), on the other hand, is strong: Strozzi " achieved some measure of public
recognition" for her work, and therefore her "compositions [were] known to her contemporaries."
(C) is the right answer for question 9.

Presumably (adv) : có lẽ

Convention (n) : tập quán, tục lệ

Feminine (adj) : nữ tính

Spontaneity (n) : tự phát

Exquisite (adj) : tinh tế, dữ dội

Grace (n) : có duyên, duyên dáng, dễ thương, kiều diễm

Marvelously (adv) : tuyệt vời

Polemical (adj) : thuộc về tranh luận

Anomalous (adj) : dị thường

Intimate (adj) : thân mật

Fictive (adj) : hư cấu


Nov 18 – 2

The industrial revolution in nineteenth-century Europe brought about an explosion in productivity as


the quantity of manufactured goods increased while costs plummeted. However, the incomes of the
workers who produced the goods did not keep pace with the growing fortunes of the industrialists.
It was in this context that Karl Marx conceived of the economic and political theory that would
become known as communism. Marx argued that as productivity improved, existing social structures
inevitably become inefficient, stifling further growth. Thus, the increasing divide between the owners
and the workers would eventually necessitate a revolution. The twentieth century saw Marxist
revolutions spread throughout the globe. Country after country watched peasants rise up against
the ruling classes and impose governments based upon Marx’s ideas. The only problem was that
while Marx saw communism as emerging from contradictions in capitalist countries, all of these
emerging communist states had pre-industrial, feudal economies.

As the twentieth century drew to a close, these Marxist states either embraced capitalist principles,
as Russia and China did, or devolved into totalitarian kleptocracies, as Cuba and North Korea did.
Meanwhile, the industrialized countries in the West entered the twenty-first century with their
capitalist economies intact. What happened? As explanation, Neo-Marxists offer the novel, if
simplistic, concept of “false consciousness.” According to this theory, citizens of capitalistic states
are unable to accurately assess their chances of upward mobility, and thus accept economic inequity
as an unfortunate but “natural” transition phase between poverty and wealth. While this theory is
most likely to accurately describe the mindset of many, it remains, as yet, entirely within the realm
of conjecture.

1.Which of the following, if true, would most weaken the explanation provided by the
Neo-Marxists for why communist revolutions have not occurred in industrialized
countries?

A. Many citizens of capitalistic countries believe that they eventually will become wealthy regardless
of their current economic status.
B. Citizens of communist countries saw their standards of living decline following
communist revolutions in their countries.
C. There are more wealthy people in capitalistic countries than there are in countries with other
economic systems.
D. The standard of living in countries such as North Korea and Cuba was lower at the end of the
twentieth century than it was in capitalist countries.
E. Citizens of capitalistic countries tend to have higher rates of literacy than do those of communist
countries.

2.The passage suggests that the countries that actually had communist revolutions
differed from the countries that Marx predicted would have communist revolutions in
which of the following ways?

A. The countries that Marx predicted would have communist revolutions had larger economies than
did those that actually had communist revolutions.
B. The countries that had communist revolutions were industrialized whereas Marx predicted that
agricultural countries would have communist revolutions.
C. The countries that actually had communist revolutions tended to have economies
dominated by sectors different from those predicted by Marx.
D. The countries that actually had communist revolutions had more inefficient social structures than
did those predicted by Marx.
E. The countries that actually had communist revolutions did not suffer from the false consciousness
predicted by Marx.

3.The author of the passage would most likely describe the concept of false
consciousness as

A. compelling if impossible to substantiate


B. supported by history
C. inevitable but inefficient
D. creative but unproven
E. accurate but unfortunate

Compelling (adj) : thuyết phục

Unfortunate (adj) : không may, bất hạnh, vô phước

Conceive (v) : quan niệm

Totalitarian (adj) : chuyên chế

Upward mobility (n) : sự tiến lên

Conjecture (v/n) : phỏng đoán, ước lượng

Realm of (n) : lĩnh vực


Nov 19 – 1

James Joyce revolutionized the novel, the short story, and modern literature as we know it. He was
born in Dublin, the first of 10 children in a Catholic family. His father was a civil servant whose poor
financial judgment left the family impoverished for much of Joyce’s youth. Young James attended
Dublin’s fine Jesuit schools, which gave him a firm grounding in theology and classical languages—
subjects that appeared repeatedly in his later work. The story of his early life and his intellectual
rebellion against Catholicism and Irish nationalism are told in the largely autobiographical novel A
Portrait of the Artist as a Young Man.

In 1902, at the age of 20, Joyce left Dublin to spend the rest of his life in Paris, Trieste, Rome, and
Zurich, with only occasional visits back home. Despite this self-imposed exile, Dublin was the setting
for most of his writings. Dubliners (1914), Joyce’s most accessible work, is a collection of short
stories describing the paralyzing social mores of middle-class Catholic life. “The Dead,” the final
story in the collection, is frequently listed as one of the finest short stories ever written.

Joyce’s next book, Ulysses, took seven years to write; once he finished writing it, he almost couldn’t
find anyone to publish it. Upon the novel’s publication, both Ireland and the United States
immediately banned it as obscene. Despite these obstacles, Ulysses has come to be generally
recognized as the greatest twentieth-century novel written in English. The novel was revolutionary in
many ways. The structure was unique: Joyce recreated one full day in the life of his protagonist,
Leopold Bloom, and modeled the actions of the story on those of Ulysses in the Odyssey. In
recounting Bloom’s day, Joyce mentions everything that happens to Bloom—including thoughts,
bodily functions, and sexual acts—providing a level of physical actuality that had never before been
achieved in literature. To provide a psychological insight comparable to the physical detail, Joyce
employed a then-revolutionary technique called stream of consciousness,in which the protagonist’s
thoughts are laid bare to the reader.

From 1922 until 1939, Joyce worked on a vast, experimental novel that eventually became known as
Finnegan’s Wake. The novel, which recounts “the history of the world” through a family’s dreams,
employs its own “night language” of puns, foreign words, and literary allusions. It has no clear
chronology or plot, and it begins and ends on incomplete sentences that flow into each other. Many
of Joyce’s supporters thought he was wasting his time on the project, although the playwright
Samuel Beckett, who later won the Nobel Prize for Literature, helped Joyce compile the final text
when his eyesight was failing. Today, Finnegan’s Wake is viewed as Joyce’s most obscure and
possibly most brilliant work.

1. The author most likely mentions James Joyce’s childhood, family, and education to
serve what purpose?

A. To suggest that he had to write in order to make a living


B. To suggest that he became a writer because of his father’s influence
C. To provide the background and cultural context for his literary work
D. To provide evidence that his literary genius was present when he was a child
E. To explain his opposition to Catholicism and socialism in his later life
2. Who is the most likely intended audience for this passage?

A. Insurance professionals at a company seminar


B. University professors of English literature at a symposium on twentieth-century Irish playwrights
C. High school students in Ireland studying their nation’s traditional folklore
D. College students studying twentieth-century English literature
E. Elementary school students studying the Odyssey

3. Which of the following can be inferred about Joyce’s attitude toward Catholicism as
practiced in Ireland at the end of the nineteenth century?

A. He felt that it repressed intellectual freedom and individual expression.


B. He viewed it as the central component of the Irish national psyche.
C. He feared that it was impeding the Irish nationalist movement.
D. He felt that it forced him to leave Dublin for Paris, Trieste, Rome, and Zurich.
E. He believed that Dublin’s Jesuit schools provided the finest education in all of Ireland.

The passage mentions “his intellectual rebellion against Catholicism” and “the paralyzing social
mores of middle-class Catholic life”; A captures these sentiments better than any of the other
answers. There is insufficient support for B and C, D is incorrect because the passage never says
that Catholicism forced him to leave Ireland, or indeed that he was forced to leave (and why travel
to Paris, Trieste, and Rome to flee Catholicism?); and E is incorrect because it is too strong a
statement and it is not focused on the topic of the question.

Rebellion (n) : nổi loạn

Paralyze (v) : tê liệt

Catholicism (n) : công giáo, đạo thiên chúa

Self-imposed (adj) : tự áp đặt

Exile (n) : lưu vong

Protagonist (n) : nhân vật chính

Plot (n) : âm mưu

Compile (v) : biên dịch, biên soạn, sưu tập


Nov 19 – 2

There are a great many symbiotic relationships in the marine environment. A popular one, often
noted for the striking beauty of the juxtaposition, is that of the sea anemone and the clown fish.
The anemone has poison tentacles which—when they contact passing fish— paralyze the fish and
drag the prey in for a meal. The clownfish uses the anemone‘s tentacle "garden" as a safe haven
while attracting prey for the anemone to capture, for it alone is immune to the sting of the
anemone.

Another symbiotic relation that remains the subject of scientific puzzlement concerns the relationship
between Scleractinia, the coral type 10 whose colonization produces reefs, and their symbiotic
partners the zooxanthellae, the unicellular algae present in the corals‘ endodermic tissues. It is
known that each symbiont plays an integral part in the formation of a reef‘s protective limestone
foundation. The coral polyps secrete calceous exoskeletons which cement themselves into a 15
underlayer of rock, while the algae deposit still more calcium carbonate, which reacts with sea salt
to create an even tougher limestone layer.

It is also known that, due to the algal photosynthesis, the reef environment is highly oxygen-
saturated, while the similarly high amounts of carbon dioxide are carried off rapidly. All this accounts
for the amazing 20 renewability of coral reefs despite the endless erosion caused by wave activity.
However, the precise manner in which one symbiont stimulates the secretion of calcium carbonate
by the other remains unclear.

Scientists have also proposed various theories to explain the transformation of "fringing reefs,"
those connected above sea level to land masses, into "barrier reefs" that are separated from
shorelines by wide lagoons, and then into free-floating atolls. Though the theory postulated by
Charles Darwin is considered at least partially correct, some scientists today argue that the creation
of the reef forms has more to do with the rise of sea level that accompanied the end of the Ice Age.
However, recent drillings at Enewetak atoll have uncovered a large underlay of volcanic rock, which
suggests that Darwin‘s explanation may have been more valid after all.

Even the name given to the reefs is something of a misnomer. The Scleractinia themselves generally
comprise no more than 10 percent of the biota of the average reef community: zooxanthellae can
account for up to 90 percent of the reef mass, along with foraminifera, annelid worms, and assorted
mollusks. Moreover, reefs can flourish only in shallow, highly saline waters above 70°F., because the
algae require such circumstances; yet non-reef-building corals occur worldwide under various
environmental conditions, from the Arctic to the Mediterranean, home of the red coral prized for
jewellery. The most likely reason that the term "coral reefs" persists is that the brilliant variety of
coral shapes and colours makes aesthetic considerations more vivid than biological ones.
1. According to the author, some scientists consider the term "coral reef" a misnomer
because:

A. the beautiful shapes and colours of reefs are produced by the Scleractinia rather than the
zooxanthellae.
B. the coral portion of a reef has little to do with the reef‘s survival.
C. "non-reef-building" corals are found throughout the world.
D. the majority of a reef‘s substance comprises zooxanthellae, foraminifera, annelid
worms, and assorted molluscs while a small portion comprises the Scleractinia.
E. the reef does not have any coral whatsoever

2. Based on the passage, which of the following is probably an assumption of scientists


studying coral reefs?

A. The theories of reef evolution through glacial melting and through volcanic subsidence are
mutually exclusive.
B. The three main types of coral reefs did not develop independently of one another.
C. Zooxanthellae are always found in coral reefs.
D. Intense calcification single-handedly protects reefs from destruction by waves and other natural
causes.
E. Coral reefs are always blue in colour

3. The passage mentions the recent drillings at the Enewetak atoll. This reference serves
to:

A. stengthen the claims made by scientists today concerning reef transformation.


B. weaken the claims made by scientists today concerning reef transformation.
C. strengthen the claims made by Darwin concerning reef transformation.
D. weaken the claims made by Darwin concerning reef transformation.
E. has no impact on the claims made by Darwin concerning reef transformation.

Though the theory postulated by Charles Darwin is considered at least partially correct,
some scientists today argue that the creation of the reef forms has more to do with the rise of sea
level that accompanied the end of the Ice Age. However, recent drillings at Enewetak atoll have
uncovered a large underlay of volcanic rock, which suggests that Darwin‘s explanation may have
been more valid after all.

Symbiotic (adj) : cộng sinh

Juxtaposition (n) : vị trí kề nhau

Anemone (n) : hải quỳ

Tentacle (n) : xúc tu

Secretion (n) : sự bài tiết

Atoll (n) : đảo san hô

Misnomer (n) : hiểu sai, gọi sai tên


Coral (n) : san hô

Vivid (adj) : sống động

Reef (n) : rạn san hô

Exoskeletion (n) : khung xương

Endodermic (adj) : nội tiết


Nov 20 – 1

Caffeine, the stimulant in coffee, has been called “the most widely used psychoactive substance on
Earth.” Snyder, Daly, and Bruns have recently proposed that caffeine affects behavior by countering
the activity in the human brain of a naturally occurring chemical called adenosine. Adenosine
normally depresses neuron firing in many areas of the brain. It apparently does this by inhibiting the
release of neurotransmitters, chemicals that carry nerve impulses from one neuron to the next.

Like many other agents that affect neuron firing, adenosine must first bind to specific receptors on
neuronal membranes. There are at least two classes of these receptors, which have been
designated A1 and A2. Snyder et al. propose that caffeine, which is structurally similar to adenosine,
is able to bind to both types of receptors, which prevents adenosine from attaching there and allows
the neurons to fire more readily than they otherwise would.

For many years, caffeine’s effects have been attributed to its inhibition of the production of
phosphodiesterase, an enzyme that breaks down the chemical called cyclic AMP. A number of
neurotransmitters exert their effects by first increasing cyclic AMP concentrations in target neurons.
Therefore, prolonged periods at the elevated concentrations, as might be brought about by a
phosphodiesterase inhibitor, could lead to a greater amount of neuron firing and, consequently, to
behavioral stimulation. But Snyder et al. point out that the caffeine concentrations needed to inhibit
the production of phosphodiesterase in the brain are much higher than those that produce
stimulation. Moreover, other compounds that block phosphodiesterase’s activity are not stimulants.

To buttress their case that caffeine acts instead by preventing adenosine binding, Snyder et
al. compared the stimulatory effects of a series of caffeine derivatives with their ability to dislodge
adenosine from its receptors in the brains of mice. “In general,” they reported, “the ability of the
compounds to compete at the receptors correlates with their ability to stimulate locomotion in the
mouse; i.e., the higher their capacity to bind at the receptors, the higher their ability to stimulate
locomotion.”Theophylline, a close structural relative of caffeine and the major stimulant in tea, was
one of the most effective compounds in both regards.

There were some apparent exceptions to the general correlation observed between adenosine
receptor binding and stimulation. One of these was a compound called 3-isobutyl-1-methylxanthine
(IBMX), which bound very well but actually depressed mouse locomotion. Snyder et al. suggest that
this is not a major stumbling block to their hypothesis. The problem is that the compound has mixed
effects in the brain, a not unusual occurrence with psychoactive drugs. Even caffeine, which is
generally known only for its stimulatory effects, displays this property, depressing mouse locomotion
at very low concentrations and stimulating it at higher ones.

1. The primary purpose of the passage is to

(A) discuss a plan for investigation of a phenomenon that is not yet fully understood
(B) present two explanations of a phenomenon and reconcile the differences between them
(C) summarize two theories and suggest a third theory that overcomes the problems encountered in
the first two
(D) describe an alternative hypothesis and provide evidence and arguments that
support it
(E) challenge the validity of a theory by exposing the inconsistencies and contradictions in it
2. According to Snyder et al., caffeine differs from adenosine in that caffeine

(A) stimulates behavior in the mouse and in humans, whereas adenosine stimulates behavior in
humans only
(B) has mixed effects in the brain, whereas adenosine has only a stimulatory effect
(C) increases cyclic AMP concentrations in target neurons, whereas adenosine decreases such
concentrations
(D) permits release of neurotransmitters when it is bound to adenosine receptors,
whereas adenosine inhibits such release
(E) inhibits both neuron firing and the production of phosphodiesterase when there is a sufficient
concentration in the brain, whereas adenosine inhibits only neuron firing

3. In response to experimental results concerning IBMX, Snyder et al. contended that it


is not uncommon for psychoactive drugs to have

(A) mixed effects in the brain


(B) inhibitory effects on enzymes in the brain
(C) close structural relationships with caffeine
(D) depressive effects on mouse locomotion
(E) the ability to dislodge caffeine from receptors in the brain

4. According to Snyder et al., all of the following compounds can bind to specific
receptors in the brain EXCEPT

(A) IBMX
(B) caffeine
(C) adenosine
(D) theophylline
(E) phosphodiesterase

5. Snyder et al. suggest that caffeine’s ability to bind to A1 and A2 receptors can be at
least partially attributed to which of the following?

(A) The chemical relationship between caffeine and phosphodiesterase


(B) The structural relationship between caffeine and adenosine
(C) The structural similarity between caffeine and neurotransmitters
(D) The ability of caffeine to stimulate behavior
(E) The natural occurrence of caffeine and adenosine in the brain

6. The author quotes Snyder et al.[highlighted] in lines 43–48 most probably in order to

(A) reveal some of the assumptions underlying their theory


(B) summarize a major finding of their experiments
(C) point out that their experiments were limited to the mouse
(D) indicate that their experiments resulted only in general correlations
(E) refute the objections made by supporters of the older theory
7. Which of the following, if true, would most weaken the theory proposed by Snyder et
al?

(A) At very low concentrations in the human brain, both caffeine and theophylline tend to have
depressive rather than stimulatory effects on human behavior.
(B) The ability of caffeine derivatives at very low concentrations to dislodge adenosine from its
receptors in mouse brains correlates well with their ability to stimulate mouse locomotion at these
low concentrations.
(C) The concentration of cyclic AMP in target neurons in the human brain that leads to increased
neuron firing can be produced by several different phosphodiesterase inhibitors in addition to
caffeine.
(D) The concentration of caffeine required to dislodge adenosine from its receptors in
the human brain is much greater than the concentration that produces behavioral
stimulation in humans.
(E) The concentration of IBMX required to dislodge adenosine from its receptors in mouse brains is
much smaller than the concentration that stimulates locomotion in the mouse.

8. The passage suggests that Snyder et al believe that if the older theory concerning
caffeine’s effects were correct, which of the following would have to be the case?

I. All neurotransmitters would increase the short-term concentration of cyclic AMP in target neurons.
II. Substances other than caffeine that inhibit the production of phosphodiesterase would be
stimulants.
III. All concentration levels of caffeine that are high enough to produce stimulation would also
inhibit the production of phosphodiesterase.

(A) I only
(B) I and II only
(C) I and III only
(D) II and III only
(E) I, II, and III

9. The last paragraph of the passage performs which of the following functions?

(A) Describes a disconfirming experimental result and reports the explanation given by
Snyder et al in an attempt to reconcile this result with their theory.
(B) Specifies the basis for the correlation observed by Snyder et al and presents an explanation in
an attempt to make the correlation consistent with the operation of psychoactive drugs other than
caffeine.
(C) Elaborates the description of the correlation observed by Snyder et al and suggests an additional
explanation in an attempt to make the correlation consistent with the older theory.
(D) Reports inconsistent experimental data and describes the method Snyder et al will use to
reanalyze this data.
(E) Provides an example of the hypothesis proposed by Snyder et al and relates this example to
caffeine’s properties.

Stimulant (n) : chất kích thích


Psychoactive (adj) : tâm sinh lý

Counter (v) : chống lại

Inhibit (v) : ức chế, ngăn cản

Neurotransmitter (n) : dẫn truyền thần kinh

Receptor (n) : thụ thể

Designate (v) : chỉ định, gọi tên

Dislodge (v) : đánh bật

Locomotion (n) : sự vận động

Stumbling block (n) : tình huống khó khăn


Nov 21 – 1

Archaeology as a profession faces two major problems. First, it is the poorest of the poor. Only
paltry sums are available for excavating and even less is available for publishing the results and
preserving the sites once excavated. Yet archaeologists deal with priceless objects every day.
Second, there is the problem of illegal excavation, resulting in museum-quality pieces being sold to
the highest bidder.

I would like to make an outrageous suggestion that would at one stroke provide funds for
archaeology and reduce the amount of illegal digging. I would propose that scientific archeological
expeditions and governmental authorities sell excavated artifacts on the open market. Such sales
would provide substantial funds for the excavation and preservation of archaeological sites and the
publication of results. At the same time, they would break the illegal excavator’s grip on the market,
thereby decreasing the inducement to engage in illegal activities.

You might object that professionals excavate to acquire knowledge, not money. Moreover, ancient
artifacts are part of our global cultural heritage, which should be available for all to appreciate, not
sold to the highest bidder. I agree. Sell nothing that has unique artistic merit or scientific value. But,
you might reply everything that comes out of the ground has scientific value. Here we part
company. Theoretically, you may be correct in claiming that every artifact has potential scientific
value. Practically, you are wrong.

I refer to the thousands of pottery vessels and ancient lamps that are essentially duplicates of one
another. In one small excavation in Cyprus, archaeologists recently uncovered 2,000 virtually
indistinguishable small jugs in a single courtyard. Even precious royal seal impressions known
as l’melekh handles have been found in abundance—more than 4,000 examples so far.

The basements of museums are simply not large enough to store the artifacts that are likely to be
discovered in the future. There is not enough money even to catalogue the finds; as a result, they
cannot be found again and become as inaccessible as if they had never been discovered. Indeed,
with the help of a computer, sold artifacts could be more accessible than are the pieces stored in
bulging museum basements. Prior to sale, each could be photographed and the list of the
purchasers could be maintained on the computer. A purchaser could even be required to agree to
return the piece if it should become needed for scientific purposes.

It would be unrealistic to suggest that illegal digging would stop if artifacts were sold on the open
market. But the demand for the clandestine product would be substantially reduced. Who would
want an unmarked pot when another was available whose provenance was known, and that was
dated stratigraphically by the professional archaeologist who excavated it?

1. The primary purpose of the passage is to propose

(A) an alternative to museum display of artifacts


(B) a way to curb illegal digging while benefiting the archaeological profession
(C) a way to distinguish artifacts with scientific value from those that have no such value
(D) the governmental regulation of archaeological sites
(E) a new system for cataloguing duplicate artifacts
2. The author implies that all of the following statements about duplicate artifacts are
true EXCEPT:

(A) A market for such artifacts already exists.


(B) Such artifacts seldom have scientific value.
(C) There is likely to be a continuing supply of such artifacts.
(D) Museums are well supplied with examples of such artifacts.
(E) Such artifacts frequently exceed in quality those already catalogued in museum
collections.

3. Which of the following is mentioned in the passage as a disadvantage of storing


artifacts in museum basements?

(A) Museum officials rarely allow scholars access to such artifacts.


(B) Space that could be better used for display is taken up for storage.
(C) Artifacts discovered in one excavation often become separated from each other.
(D) Such artifacts are often damaged by variations in temperature and humidity.
(E) Such artifacts’ often remain uncatalogued and thus cannot be located once they are
put in storage.

4. The author mentions the excavation in Cyprus (lines 31-34) to emphasize which of
the following points?

(A) Ancient lamps and pottery vessels are less valuable, although more rare, than royal seal
impressions.
(B) Artifacts that are very similar to each other present cataloguing difficulties to archaeologists.
(C) Artifacts that are not uniquely valuable, and therefore could be sold, are available in
large quantities.
(D) Cyprus is the most important location for unearthing large quantities of salable artifacts.
(E) Illegal sales of duplicate artifacts are wide-spread, particularly on the island of Cyprus.

5. The author’s argument concerning the effect of the official sale of duplicate artifacts
on illegal excavation is based on which of the following assumptions?

(A) Prospective purchasers would prefer to buy authenticated artifacts.


(B) The price of illegally excavated artifacts would rise.
(C) Computers could be used to trace sold artifacts.
(D) Illegal excavators would be forced to sell only duplicate artifacts.
(E) Money gained from selling authenticated artifacts could be used to investigate and prosecute
illegal excavators.

6. The author anticipates which of the following initial objections to the adoption of his
proposal?

(A) Museum officials will become unwilling to store artifacts.


(B) An oversupply of salable artifacts will result and the demand for them will fall.
(C) Artifacts that would have been displayed in public places will be sold to private
collectors.
(D) Illegal excavators will have an even larger supply of artifacts for resale.
(E) Counterfeiting of artifacts will become more commonplace.

7. The author implies that which of the following would occur if duplicate artifacts were
sold on the open market?

I. Illegal excavation would eventually cease completely.


II. Cyprus would become the primary source of marketable duplicate artifacts.
III. Archaeologists would be able to publish the results of their excavations more frequently than
they currently do.

(A) I only
(B) III only
(C) I and II only
(D) II and III only
(E) I, II, and III

Curb (v) : ngăn lại

Catalogue (v) : chia ra từng loại, kê thứ tự, thống kê

Paltry (adj) : nhỏ, không đáng kể

Sum (n) : số tiền

Outrageous (adj) : thái quá, sốc


Nov 21 – 2

Federal efforts to aid minority businesses began in the 1960's when the small business
administration (SBA) began making federally guaranteed loans and government-sponsored
management and technical assistance available to minority business enterprises. While this program
enabled many minority entrepreneurs to form new businesses, the results were disappointing, since
managerial inexperience, unfavorable locations, and capital shortages led to high failure rates. Even
15 years after the program was implemented, minority business receipts were not quite two percent
of the national economy's total receipts.

Recently federal policymakers have adopted an approach intended to accelerate development of the
minority business sector by moving away from directly aiding small minority enterprises and toward
supporting larger, growth-oriented minority firms through intermediary companies. In this approach,
large corporations participate in the development of successful and stable minority businesses by
making use of government-sponsored venture capital. The capital is used by a participating
company to establish a minority enterprise small business investment company or MESBIC. The
MESBIC then provides capital and guidance to minority businesses that have potential to become
future suppliers or customers of the sponsoring company.

MESBIC's are the result of the belief that providing established firms with easier access to relevant
management techniques and more job-specific experience, as well as substantial amounts of capital,
gives those firms a greater opportunity to develop sound business foundations than does simply
making general management experience and small amounts of capital available. Further, since
potential markets for the minority businesses already exist through the sponsoring companies, the
minority businesses face considerably less risk in terms of location and market fluctuation. Following
early financial and operating problems, sponsoring corporations began to capitalize MESBIC's far
above the legal minimum of $500,000 in order to generate sufficient income and to sustain the
quality of management needed. MESBIC's are now emerging as increasingly important financing
sources for minority enterprises.

Ironically, MESBIC staffs, which usually consist of Hispanic and Black professionals, tend to approach
investments in minority firms more pragmatically than do many MESBIC directors, who are usually
senior managers from sponsoring corporations. The latter often still think mainly in terms of the
"social responsibility approach" and thus seem to prefer deals that are riskier and less attractive
than normal investment criteria would warrant. such differences in viewpoint have produced
uneasiness among many minority staff members, who feel that minority entrepreneurs and
businesses should be judged by established business considerations. these staff members believe
their point of view is closer to the original philosophy of MESBIC's and they are concerned that,
unless a more prudent course is followed, MESBIC directors may revert to policies likely to recreate
the disappointing results of the original SBA approach.
1. Which of the following best states the central idea of the passage?

(A) The use of MESBIC's for aiding minority entrepreneurs seems to have greater
potential for success than does the original SBA approach.
(B) There is a crucial difference in point of view between the staff and directors of some MESBIC's.
(C) After initial problems with management and marketing, minority businesses have begun to
expand at a steady rate.
(D) Minority entrepreneurs wishing to form new businesses now have several equally successful
federal programs on which to rely.
(E) For the first time since 1960, large corporations are making significant contributions to the
development of minority businesses.

2. According to the passage, the MESBIC approach differs from the SBA approach in that
MESBIC's

(A) seek federal contracts to provide markets for minority businesses


(B) encourage minority businesses to provide markets for other minority businesses
(C) attempt to maintain a specified rate of growth in the minority business sector
(D) rely on the participation of large corporations to finance minority businesses
(E) select minority businesses on the basis of their location

3. Which of the following does the author cite to support the conclusion that the results
of the SBA program were disappointing?

(A) The small number of new minority enterprises formed as a result of the program
(B) The small number of minority enterprises that took advantage of the management and technical
assistance offered under the program
(C) The small percentage of the nation's business receipts earned by minority
enterprises following the programs, implementation
(D) The small percentage of recipient minority enterprises that were able to repay federally
guaranteed loans made under the program
(E) The small number of minority enterprises that chose to participate in the program

4. Which of the following statements about the SBA program can be inferred from the
passage?

(A) The maximum term for loans made to recipient businesses was 15 years.
(B) Business loans were considered to be more useful to recipient businesses than was management
and technical assistance.
(C) The anticipated failure rate for recipient businesses was significantly lower than the
rate that actually resulted.
(D) Recipient businesses were encouraged to relocate to areas more favorable for business
development.
(E) The capitalization needs of recipient businesses were assessed and then provided for
adequately.

While this program enabled many minority entrepreneurs to form new businesses, the results
were disappointing, since managerial inexperience, unfavorable locations, and capital shortages
led to high failure rates. Even 15 years after the program was implemented, minority business
receipts were not quite two percent of the national economy's total receipts.

5. The author refers to the "financial and operating problems" (line 38) encountered by
MESBIC's primarily in order to

(A) broaden the scope of the discussion to include the legal considerations of funding MESBIC's
through sponsoring companies
(B) call attention to the fact that MESBIC's must receive adequate funding in order to
function effectively
(C) show that sponsoring companies were willing to invest only $500,000 of government-sponsored
venture capital in the original MESBIC's
(D) compare SBA and MESBIC limits on minimum funding
(E) refute suggestions that MESBIC's have been only marginally successful

6. The author's primary objective in the passage is to

(A) disprove the view that federal efforts to aid minority businesses have been ineffective
(B) explain how federal efforts to aid minority businesses have changed since the
1960's
(C) establish a direct link between the federal efforts to aid minority businesses made before the
1960's and those made in the 1980's
(D) analyze the basis for the belief that job-specific experience is more useful to minority businesses
than is general management experience
(E) argue that the "social responsibility approach" to aiding minority businesses is superior to any
other approach

7. Based on information in the passage, which of the following would be indicative of


the pragmatism of MESBIC staff members?

I. A reluctance to invest in minority businesses that show marginal expectations of return on the
investments
II. A desire to invest in minority businesses that produce goods and services likely to be of use to
the sponsoring company
III. A belief that the minority business sector is best served by investing primarily in newly
established businesses

(A) I only
(B) III only
(C) I and II only
(D) II and III only
(E) I, II and III

MESBIC's are the result of the belief that providing established firms with easier access to
relevant management techniques and more job-specific experience, as well as substantial amounts
of capital, gives those firms a greater opportunity to develop sound business foundations than does
simply making general management experience and small amounts of capital available
8. It can be inferred from the passage that the attitude of some MESBIC staff members
toward the investments preferred by some MESBIC directors can best be described as

(A) defensive
(B) resigned
(C) indifferent
(D) shocked
(E) disapproving

9. The passage provides information that would answer which of the following
questions?

(A) What was the average annual amount, in dollars, of minority business receipts before the SBA
strategy was implemented?
(B) What locations are considered to be unfavorable for minority businesses?
(C) What is the current success rate for minority businesses that are capitalized by MESBIC’s?
(D) How has the use of federal funding for minority businesses changed since the
1960’s?
(E) How do minority businesses apply to participate in a MESBIC program?

Reluctance (n) : sự miễn cưỡng

Prudent (adj) : cẩn thận, khôn ngoan, thận trọng

Revert (v) : trở lại

Capitalize (v) : cung cấp vốn

Uneasiness (n) : sự khó chịu


Nov 22 – 2

Nearly a century ago, biologists found that if they separated an invertebrate animal embryo into two
parts at an early stage of its life, it would survive and develop as two normal embryos. This led them
to believe that the cells in the early embryo are undetermined in the sense that each cell has the
potential to develop in a variety of different ways. Later biologists found that the situation was not
so simple. It matters in which plane the embryo is cut. If it is cut in a plane different from the one
used by the early investigators, it will not form two whole embryos.

A debate arose over what exactly was happening. Which embryo cells are determined, just when do
they become irreversibly committed to their fates, and what are the “morphogenetic determinants”
that tell a cell what to become? But the debate could not be resolved because no one was able to
ask the crucial questions in a form in which they could be pursued productively. Recent discoveries
in molecular biology, however, have opened up prospects for a resolution of the debate. Now
investigators think they know at least some of the molecules that act as morphogenetic
determinants in early development. They have been able to show that, in a sense, cell determination
begins even before an egg is fertilized.

Studying sea urchins, biologist Paul Gross found that an unfertilized egg contains substances that
function as morphogenetic determinants. They are located in the cytoplasm of the egg cell; i.e., in
that part of the cell’s protoplasm that lies outside of the nucleus. In the unfertilized egg, the
substances are inactive and are not distributed homogeneously. When the egg is fertilized, the
substances become active and, presumably, govern the behavior of the genes they interact with.
Since the substances are unevenly distributed in the egg, when the fertilized egg divides, the
resulting cells are different from the start and so can be qualitatively different in their own gene
activity.

The substances that Gross studied are maternal messenger RNA’s—products of certain of the
maternal genes. He and other biologists studying a wide variety of organisms have found that these
particular RNA’s direct, in large part, the synthesis of histones, a class of proteins that bind to DNA.
Once synthesized, the histones move into the cell nucleus, where section of DNA wrap around them
to form a structure that resembles beads, or knots, on a string. The beads are DNA segments
wrapped around the histones; the string is the intervening DNA. And it is the structure of these
beaded DNA strings that guide the fate of the cells in which they are located.

1. The passage is most probably directed at which kind of audience?

(A) State legislators deciding about funding levels for a state-funded biological laboratory
(B) Scientists specializing in molecular genetics
(C) Readers of an alumni newsletter published by the college that Paul Gross attended
(D) Marine biologists studying the processes that give rise to new species
(E) Undergraduate biology majors in a molecular biology course

2. It can be inferred from the passage that the morphogenetic determinants present in
the early embryo are

(A) located in the nucleus of the embryo cells


(B) evenly distributed unless the embryo is not developing normally
(C) inactive until the embryo cells become irreversibly committed to their final function
(D) identical to those that were already present in the unfertilized egg
(E) present in larger quantities than is necessary for the development of a single
individual

3. The main topic of the passage is

(A) the early development of embryos of lower marine organisms


(B) the main contribution of modern embryology to molecular biology
(C) the role of molecular biology in disproving older theories of embryonic development
(D) cell determination as an issue in the study of embryonic development
(E) scientific dogma as a factor in the recent debate over the value of molecular biology

4. According to the passage, when biologists believed that the cells in the early embryo
were undetermined, they made which of the following mistakes?

(A) They did not attempt to replicate the original experiment of separating an embryo into two
parts.
(B) They did not realize that there was a connection between the issue of cell determination and the
outcome of the separation experiment.
(C) They assumed that the results of experiments on embryos did not depend on the particular
animal species used for such experiments.
(D) They assumed that it was crucial to perform the separation experiment at an early stage in the
embryo's life.
(E) They assumed that different ways of separating an embryo into two parts would be
equivalent as far as the fate of the two parts was concerned.

5. It can be inferred from the passage that the initial production of histones after an egg
is fertilized takes place

(A) in the cytoplasm


(B) in the maternal genes
(C) throughout the protoplasm
(D) in the beaded portions of the DNA strings
(E) in certain sections of the cell nucleus

6. It can be inferred from the passage that which of the following is dependent on the
fertilization of an egg?

(A) Copying of maternal genes to produce maternal messenger RNA


(B) Synthesis of proteins called histones
(C) Division of a cell into its nucleus and the cytoplasm
(D) Determination of the egg cell's potential for division
(E) Generation of all of a cell's morphogenetic determinants

7. According to the passage, the morphogenetic determinants present in the unfertilized


egg cell are which of the following?

(A) Proteins bound to the nucleus


(B) Histones
(C) Maternal messenger RNA's
(D) Cytoplasm
(E) Nonbeaded intervening DNA

They are located in the cytoplasm of the egg cell; not are which

The substances that Gross studied are maternal messenger RNA’s—products of certain of the
maternal genes.

8. The passage suggests that which of the following plays a role in determining whether
an embryo separated into two parts will develop as two normal embryos?

I. The stage in the embryo's life at which the separation occurs


II. The instrument with which the separations is accomplished
III. The plane in which the cut is made that separates the embryo

(A) I only
(B) II only
(C) I and II only
(D) I and III only
(E) I, II, and III

9. Which of the following circumstances is most comparable to the impasse biologists


encountered in trying to resolve the debate about cell determination (as in the
highlighted portion)?

(A) The problems faced by a literary scholar who wishes to use original source materials that are
written in an unfamiliar foreign language
(B) The situation of a mathematician who in preparing a proof of a theorem for publication detects a
reasoning error in the proof
(C) The difficulties of a space engineer who has to design equipment to function in an environment
in which it cannot first be tested
(D) The predicament of a linguist trying to develop a theory of language acquisition
when knowledge of the structure of language itself is rudimentary at best
(E) The dilemma confronting a foundation when the funds available to it are sufficient to support
one of two equally deserving scientific projects but not both

But the debate could not be resolved because no one was able to ask the crucial
questions in a form in which they could be pursued productively

Invertebrate (adj) : động vật không xương sống

Embryo (n) : phôi thai

Plane (n) : mặt phẳng

Irreversibly (adv) : không thể đảo ngược

Morphogenetic (adj) : hình thái


Molecular (adj) : phân tử

Fertilize (v) : thụ tinh

Urchin (n) : nhím

Cytoplasm (n) : tế bào chất

Nucleus (n) : trung tâm

Protoplasm (n) : nguyên sinh chất

Predicament (n) : tình trạng khó khăn

Rudimentary (adj) : thô sơ


Nov 23 – 1

In the two decades between 1910 and 1930, over ten percent to the black population of the United
States left the South, where the preponderance of the black population had been located, and
migrated to northern states, with the largest number moving, it is claimed, between 1916 and 1918.
It has been frequently assumed, but not proved, that the majority of the migrants in what has come
to be called the Great Migration came from rural areas and were motivated by two concurrent
factors: the collapse of the cotton industry following the boll weevil infestation, which began in
1898, and increased demand in the north for labor following the cessation of European immigration
caused by the outbreak of the first world war in 1914. This assumption has led to the conclusion
that the migrants' subsequent lack of economic mobility in the north is tied to rural background, a
background that implies unfamiliarity with urban living and a lack of industrial skills.

But the question of who actually left the south has never been rigorously investigated. Although
numerous investigations document an exodus from rural southern areas to southern cities prior to
the Great Migration. No one has considered whether the same migrants then moved on to northern
cities. In 1910 over 600,000 black workers, or ten percent of the black work force, reported
themselves to be engaged in "manufacturing and mechanical pursuits," the federal census category
roughly encompassing the entire industrial sector. The Great Migration could easily have been made
up entirely of this group and their families. It is perhaps surprising to argue that an employed
population could be enticed to move, but an explanation lies in the labor conditions then prevalent
in the South.

About thirty-five percent of the urban black population in the South was engaged in skilled trades.
Some were from the old artisan class of slavery—blacksmiths, masons, carpenters—which had had a
monopoly of certain trades, but they were gradually being pushed out by competition,
mechanization, and obsolescence, the remaining sixty-five percent, more recently urbanized, worked
in newly developed industries—tobacco, lumber, coal and iron manufacture, and railroads. Wages in
the South, however, were low, and black workers were aware, through labor recruiters and the
black press, that they could earn more even as unskilled workers in the North than they could as
artisans in the South. After the boll weevil infestation, urban black workers faced competition from
the continuing influx of both black and white rural workers, who were driven to undercut the wages
formerly paid for industrial jobs. Thus, a move north would be seen as advantageous to a group that
was already urbanized and steadily employed, and the easy conclusion tying their subsequent
economic problems in the North to their rural background comes into question.

1. The author indicates explicitly that which of the following records has been a source
of information in her investigation?

(A) United States immigration service reports from 1914 to 1930


(B) Payrolls of southern manufacturing firms between 1910 and 1930
(C) The volume of cotton exports between 1898 and 1910
(D) The federal census of 1910
(E) Advertisements of labor recruiters appearing in southern newspapers after 1910
2. In the passage, the author anticipates which of the following as a possible objection
to her argument?

(A) It is uncertain how many people actually migrated during the great migration.
(B) The eventual economic status of the great migration migrants has not been adequately traced.
(C) It is not likely that people with steady jobs would have reason to move to another
area of the country.
(D) It is not true that the term "manufacturing and mechanical pursuits" actually encompasses the
entire industrial sector.
(E) Of the black workers living in southern cities, only those in a small number of trades were
threatened by obsolescence.

This weakens the argument. In the passage, it states that 35% of urban black population in the
South was engaged in skilled trades (Line 35). And the remaining 65% of this population worked in
newly developed industries (Line 40). Hence, this can imply that urban black population in the South
had jobs. As a result, if choice (C) is true, the great migration must come from rural areas (This
contradicts the author's argument and supported the accepted assumption).

3. According to the passage, which of the following is true of wages in southern cities in
1910?

(A) They were being pushed lower as a result of increased competition.


(B) They had begun t to rise so that southern industry could attract rural workers.
(C) They had increased for skilled workers but decreased for unskilled workers.
(D) They had increased in large southern cities but decreased in small southern cities.
(E) They had increased in newly developed industries but decreased in the older trades.

4. The author cites each of the following as possible influences in a black worker's
decision to migrate north in the great migration except

(A) Wage levels in northern cities


(B) Labor recruiters
(C) Competition from rural workers
(D) Voting rights in northern states
(E) The black press

5. It can be inferred from the passage that the "easy conclusion" mentioned in line 53 is
based on which of the following assumptions?

(A) People who migrate from rural areas to large cities usually do so for economic reasons.
(B) Most people who leave rural areas to take jobs in cities return to rural areas as soon as it is
financially possible for them to do so.
(C) People with rural backgrounds are less likely to succeed economically in cities than
are those with urban backgrounds.
(D) Most people who were once skilled workers are not willing to work as unskilled workers.
(E) People who migrate from their birthplaces to other regions of country seldom undertake a
second migration.
Thus, a move north would be seen as advantageous to a group that was already urbanized and
steadily employed, and the easy conclusion tying their subsequent economic problems in the North
to their rural background comes into question.

6. The primary purpose of the passage is to

(A) Support an alternative to an accepted methodology


(B) Present evidence that resolves a contradiction
(C) Introduce a recently discovered source of information
(D) Challenge a widely accepted explanation
(E) Argue that a discarded theory deserves new attention

7. According to information in the passage, which of the following is a correct sequence


of groups of workers, from highest paid to lowest paid, in the period between 1910 and
1930?

(a) artisans in the north; artisans in the south; unskilled workers in the north; unskilled workers in
the south
(b) artisans in the north and south; unskilled workers in the north; unskilled workers in the south
(c) artisans in the north; unskilled workers in the north; artisans in the south
(d) artisans in the north and south; unskilled urban workers in the north; unskilled rural workers in
the south
(e) artisans in the north and south, unskilled rural workers in the north and south; unskilled urban
workers in the north and south

8. the material in the passage would be most relevant to a long discussion of which of
the following topics?
(a) the reasons for the subsequent economic difficulties of those who participated in the
great migration
(b) the effect of migration on the regional economies of the united states following the first world
war
(c) the transition from a rural to an urban existence for those who migrated in the great migration
(d) the transformation of the agricultural south following the boll weevil infestation
(e) the disappearance of the artisan class in the united states as a consequence of mechanization in
the early twentieth century

Census (n) : điều tra dân số

Preponderance (n) : ưu thế

Boll weevil (n) : bọ cánh cứng

Infestation (n) : phá hoại

Blacksmith (n) : thợ rèn

Mason (n) : thợ xây

Carpenter (n) : thợ mộc

Influx (n) : dòng người, đám đông, sự chảy vào


Nov 23 - 2

In 1896 a Georgia couple suing for damages in the accidental death of their two year old was told
that since the child had made no real economic contribution to the family, there was no liability for
damages. In contrast, less than a century later, in 1979, the parents of a three-year-old sued in New
York for accidental-death damages and won an award of $750,000.

The transformation in social values implicit in juxtaposing these two incidents is the subject of
Viviana Zelizer’s excellent book, Pricing the Priceless Child. During the nineteenth century, she
argues, the concept of the “useful” child who contributed to the family economy gave way gradually
to the present-day notion of the “useless” child who, though producing no income for, and indeed
extremely costly to, its parents, is yet considered emotionally “priceless.” Well established among
segments of the middle and upper classes by the mid-1800’s, this new view of childhood spread
throughout society in the late-nineteenth and early-twentieth centuries as reformers introduced
child-labor regulations and compulsory education laws predicated in part on the assumption that a
child’s emotional value made child labor taboo.

For Zelizer the origins of this transformation were many and complex. The gradual erosion of
children’s productive value in a maturing industrial economy, the decline in birth and death rates,
especially in child mortality, and the development of the companionate family (a family in which
members were united by explicit bonds of love rather than duty) were all factors critical in changing
the assessment of children’s worth. Yet “expulsion of children from the ‘cash nexus,’ although clearly
shaped by profound changes in the economic, occupational, and family structures,” Zelizer
maintains, “was also part of a cultural process ‘of sacralization’ of children’s lives.” Protecting
children from the crass business world became enormously important for late-nineteenth-century
middle-class Americans, she suggests; this sacralization was a way of resisting what they perceived
as the relentless corruption of human values by the marketplace.

In stressing the cultural determinants of a child’s worth, Zelizer takes issue with practitioners of the
new “sociological economics,” who have analyzed such traditionally sociological topics as crime,
marriage, education, and health solely in terms of their economic determinants. Allowing only a
small role for cultural forces in the form of individual “preferences,” these sociologists tend to view
all human behaviors as directed primarily by the principle of maximizing economic gain. Zelizer is
highly critical of this approach, and emphasizes instead the opposite phenomenon: the power of
social values to transform price. As children became more valuable in emotional terms, she argues,
their “exchange” or “surrender” value on the market, that is, the conversion of their intangible worth
into cash terms, became much greater.

1. It can be inferred from the passage that accidental-death damage awards in America
during the nineteenth century tended to be based principally on the

(A) earnings of the person at time of death


(B) wealth of the party causing the death
(C) degree of culpability of the party causing the death
(D) amount of money that had been spent on the person killed
(E) amount of suffering endured by the family of the person killed
In 1896 a Georgia couple suing for damages in the accidental death of their two year old was told
that since the child had made no real economic contribution to the family, there was no
liability for damages

2. It can be inferred from the passage that in the early 1800’s children were generally
regarded by their families as individuals who

(A) needed enormous amounts of security and affection


(B) required constant supervision while working
(C) were important to the economic well-being of a family
(D) were unsuited to spending long hours in school
(E) were financial burdens assumed for the good of society

During the nineteenth century, she argues, the concept of the “useful” child who
contributed to the family economy gave way gradually to the present-day notion of the
“useless” child who, though producing no income for, and indeed extremely costly to, its parents, is
yet considered emotionally “priceless.”

3. Which of the following alternative explanations of the change in the cash value of
children would be most likely to be put forward by sociological economists as they are
described in the passage?

(A) The cash value of children rose during the nineteenth century because parents began to
increase their emotional investment in the upbringing of their children.
(B) The cash value of children rose during the nineteenth century because their
expected earnings over the course of a lifetime increased greatly.
(C) The cash value of children rose during the nineteenth century because the spread of
humanitarian ideals resulted in a wholesale reappraisal of the worth of an individual.
(D) The cash value of children rose during the nineteenth century because compulsory education
laws reduced the supply, and thus raised the costs, of available child labor.
(E) The cash value of children rose during the nineteenth century because of changes in the way
negligence law assessed damages in accidental death cases.

4. The primary purpose of the passage is to

(A) review the literature in a new academic sub-field


(B) present the central thesis of a recent book
(C) contrast two approaches to analyzing historical change
(D) refute a traditional explanation of a social phenomenon
(E) encourage further work on a neglected historical topic

The transformation in social values implicit in juxtaposing these two incidents is the subject of
Viviana Zelizer’s excellent book, Pricing the Priceless Child

5. It can be inferred from the passage that which of the following statements was true
of American families over the course of the nineteenth century?

(A) The average size of families grew considerably.


(B) The percentage of families involved in industrial work declined dramatically.
(C) Family members became more emotionally bonded to one another.
(D) Family members spent an increasing amount of time working with each other.
(E) Family members became more economically dependent on each other.

6. Zelizer refers to all of the following as important influences in changing the


assessment of children’s worth EXCEPT changes in

(A) the mortality rate


(B) the nature of industry
(C) the nature of the family
(D) attitudes toward reform movements
(E) attitudes toward the marketplace

7. Which of the following would be most consistent with the practices of sociological
economics as these practices are described in the passage?

(A) Arguing that most health-care professionals enter the field because they believe it to be the
most socially useful of any occupation
(B) Arguing that most college students choose majors that they believe will lead to the
most highly paid jobs available to them
(C) Arguing that most decisions about marriage and divorce are based on rational assessments of
the likelihood that each partner will remain committed to the relationship
(D) Analyzing changes in the number of people enrolled in colleges and universities as a function of
changes in the economic health of these institutions
(E) Analyzing changes in the ages at which people get married as a function of a change in the
average number of years that young people have lived away from their parents

Allowing only a small role for cultural forces in the form of individual “preferences,” these
sociologists tend to view all human behaviors as directed primarily by the principle of
maximizing economic gain

Implicit (v) : tiềm ẩn

Juxtapose (v) : để cạnh nhau

Give away (v) : nhường chỗ

Companionate (adj) : đồng hành tình yêu

Expulsion (n) : sự trục xuất

Nexus (n) : mối quan hệ

Sacralization (n) : phi tập trung hóa

Conversion (n) : chuyển đổi


Nov 24 – 1

Prior to 1975, union efforts to organize public-sector clerical workers, most of whom are women,
were somewhat limited. The factors favoring unionization drives seem to have been either the
presence of large numbers of workers, as in New York City, to make it worth the effort, or the
concentration of small numbers in one or two locations, such as a hospital, to make it relatively
easy. Receptivity to unionization on the workers’ part was also a consideration, but when there were
large numbers involved or the clerical workers were the only unorganized group in a jurisdiction, the
multi-occupational unions would often try to organize them regardless of the workers’ initial
receptivity. The strategic reasoning was based, first, on the concern that politicians and
administrators might play off unionized against non-unionized workers, and, second, on the
conviction that a fully unionized public work force meant power, both at the bargaining table and in
the legislature. In localities where clerical workers were few in number, were scattered in several
workplaces, and expressed no interest in being organized, unions more often than not ignored them
in the pre-1975 period.

But since the mid-1970’s, a different strategy has emerged. In 1977, 34 percent of government
clerical workers were represented by a labor organization, compared with 46 percent of government
professionals, 44 percent of government blue-collar workers, and 41 percent of government service
workers. Since then, however, the biggest increases in public-sector unionization have been among
clerical workers. Between 1977 and 1980, the number of unionized government workers in blue-
collar and service occupations increased only about 1.5 percent, while in the white-collar
occupations the increase was 20 percent and among clerical workers in particular, the increase was
22 percent.

What accounts for this upsurge in unionization among clerical workers? First, more women have
entered the work force in the past few years, and more of them plan to remain working until
retirement age. Consequently, they are probably more concerned than their predecessors were
about job security and economic benefits. Also, the women’s movement has succeeded in
legitimizing the economic and political activism of women on their own behalf, thereby producing a
more positive attitude toward unions. The absence of any comparable increase in unionization
among private-sector clerical workers, however, identifies the primary catalyst—the structural
change in the multi-occupational public-sector unions themselves. Over the past twenty years, the
occupational distribution in these unions has been steadily shifting from predominantly blue-collar to
predominantly white-collar. Because there are far more women in white-collar jobs, an increase in
the proportion of female members has accompanied the occupational shift and has altered union
policy-making in favor of organizing women and addressing women’s issues.

1. According to the passage, the public-sector workers who were most likely to belong
to unions in 1977 were

(A) professionals
(B) managers
(C) clerical workers
(D) service workers
(E) blue-collar workers
In 1977, 34 percent of government clerical workers were represented by a labor organization,
compared with 46 percent of government professionals, 44 percent of government blue-collar
workers, and 41 percent of government service workers.

2. The author cites union efforts to achieve a fully unionized work force (line 13-19)
[The strategic reasoning was based, first, on the concern that politicians and
administrators might play off unionized against non-unionized workers, and, second, on
the conviction that a fully unionized public work force meant power, both at the
bargaining table and in the legislature.] in order to account for why

(A) politicians might try to oppose public-sector union organizing


(B) public-sector unions have recently focused on organizing women
(C) early organizing efforts often focused on areas where there were large numbers of workers
(D) union efforts with regard to public-sector clerical workers increased dramatically after 1975
(E) unions sometimes tried to organize workers regardless of the workers’ initial
interest in unionization

Receptivity to unionization on the workers’ part was also a consideration, but when there were large
numbers involved or the clerical workers were the only unorganized group in a jurisdiction, the
multi-occupational unions would often try to organize them regardless of the workers’
initial receptivity.

3. The author’s claim that, since the mid-1970’s, a new strategy has emerged in the
unionization of public-sector clerical workers (line 23) [But since the mid-1970’s, a
different strategy has emerged.] would be strengthened if the author

(A) described more fully the attitudes of clerical workers toward labor unions
(B) compared the organizing strategies employed by private-sector unions with those of public-
sector unions
(C) explained why politicians and administrators sometimes oppose unionization of clerical workers
(D) indicated that the number of unionized public-sector clerical workers was increasing even before
the mid-1970’s
(E) showed that the factors that favored unionization drives among these workers prior
to 1975 have decreased in importance

4. According to the passage, in the period prior to 1975, each of the following
considerations helped determine whether a union would attempt to organize a certain
group of clerical workers EXCEPT

(A) the number of clerical workers in that group


(B) the number of women among the clerical workers in that group
(C) whether the clerical workers in that area were concentrated in one workplace or scattered over
several workplaces
(D) the degree to which the clerical workers in that group were interested in unionization
(E) whether all the other workers in the same jurisdiction as that group of clerical workers were
unionized
5. The author states that which of the following is a consequence of the women’s
movement of recent years?

(A) An increase in the number of women entering the work force


(B) A structural change in multi-occupational public-sector unions
(C) A more positive attitude on the part of women toward unions
(D) An increase in the proportion of clerical workers that are women
(E) An increase in the number of women in administrative positions

6. The main concern of the passage is to

(A) advocate particular strategies for future efforts to organize certain workers into labor unions
(B) explain differences in the unionized proportions of various groups of public-sector workers
(C) evaluate the effectiveness of certain kinds of labor unions that represent public-sector workers
(D) analyzed and explain an increase in unionization among a certain category of
workers
(E) describe and distinguish strategies appropriate to organizing different categories of workers

7. The author implies that if the increase in the number of women in the work force and
the impact of the women’s movement were the main causes of the rise in unionization
of public-sector clerical workers, then

(A) more women would hold administrative positions in unions


(B) more women who hold political offices would have positive attitudes toward labor unions
(C) there would be an equivalent rise in unionization of private-sector clerical workers
(D) unions would have shown more interest than they have in organizing women
(E) the increase in the number of unionized public-sector clerical workers would have been greater
than it has been

Also, the women’s movement has succeeded in legitimizing the economic and political activism of
women on their own behalf, thereby producing a more positive attitude toward unions. The
absence of any comparable increase in unionization among private-sector clerical
workers, however, identifies the primary catalyst—the structural change in the multi-
occupational public-sector unions themselves

8. The author suggests that it would be disadvantageous to a union if

(A) many workers in the locality were not unionized


(B) the union contributed to political campaigns
(C) the union included only public-sector workers
(D) the union included workers from several jurisdictions
(E) the union included members from only a few occupations

9. The author implies that, in comparison with working women today, women working
in the years prior to the mid-1970’s showed a greater tendency to

(A) prefer smaller workplaces


(B) express a positive attitude toward labor unions
(C) maximize job security and economic benefits
(D) side with administrators in labor disputes
(E) quit working prior of retirement age

Public-sector (n) : khu vực kinh tế kiểm soát bởi chính phủ

Clerical (adj) : văn thư, giấy tờ văn phòng

Unionization (n) : công đoàn

Receptivity (n) : sự thụ cảm, khả năng tiếp thu

Unorganized (adj) : không có tổ chức

Jurisdiction (n) : quyền lực pháp lý

Play-off (v) : đối đấu, phản đối

Scatter (v) : rải rác, tản mạn

Blue-collar (adj) : những công việc liên quan tới tay chân, thường là trong nhà máy

White-collar (adj) : những công việc liên quan tới văn phòng, giấy tờ, sổ sách dưới môi trường
chuyên nghiệp

Upsurge (n) : bùng nổ

Legitimize (v) : hợp pháp hóa

Catalyst (n) : chất xúc tác


Nov 25 – 1

The informal sector of the economy involves activities that, in both developed and underdeveloped
countries, are outside the arena of the normal, regulated economy and thus escape official
recordkeeping. These activities, which include such practices as off-the-books hiring and cash
payments, occur mainly in service industries like construction, hotels, and restaurants. Many
economists think that the informal sector is an insignificant supplement to the larger formal
economy. They base this belief on three assumptions that have been derived from theories of
industrial development. But empirical evidence suggests that these assumptions are not valid.

All three assumptions are, essentially, hypotheses about the character of the labor process at
different levels of development. The first is that informal activities are transitory, being a
consequence of the imperfect penetration of modern capitalism into less developed regions, and are
thus destined to disappear with the advance of industrialization. The second is that the principal
reason for the continuing existence of an informal sector is to keep a redundant segment of the
labor force alive through jobs invented to fit in the interstices of the modern economy. The third is
that the informal sector is primarily a feature of peripheral economies such as those of Latin
America, Africa, and most of Asia.

Data collected from both developed and underdeveloped countries, however, reveal that the
informal sector is neither disappearing with industrialization, limited in function to being an
employment alternative of last resort, nor exclusively or particularly a Third World phenomenon.
Informal sector employment rates in many countries have remained constant in the postwar era.
Moreover, if the informal economy were exclusively a refuge from destitution, two facts would
logically follow: First, average income levels among the informally employed would be significantly
lower than among workers in formal activities; second, those who found employment in formal
activities would never leave voluntarily in order to move into the informal economy. But neither of
these predictions is supported by data.

1). The primary purpose of this passage is to

A) compare the economies of developed and underdeveloped countries.


B) suggest that economists should develop a new framework for studying the global economy.
C) criticize the notion that the informal economy is primarily a Third World phenomenon.
D) dispute a widely held view of the relationship between the formal and informal
sectors of the world economy.
E) argue that workers in the informal economy are almost as well paid as those in the formal
economy.
2). From the passage, which of the following can be inferred about employment
opportunities in the informal economy?

A) Workers employed in the informal economy generally receive much lower wages than do workers
in the formal economy.
B) Some workers have purposely chosen to work in the informal economy rather than in
the formal economy.
C) The number of workers employed in the informal economy has remained stable over the last few
centuries.
D) Employment opportunities in the informal economy will decline as agriculture is displaced by
industry.
E) The informal economy has created no employment for workers who would otherwise be
unemployable.

We learn in the third paragraph that “informal sector employment rates in many
countries have remained constant in the postwar era,” but we have no support for the idea that such
employment has remained stable for “centuries.” (C) also confuses employment rates (which
are percentages) with the number employed

Informal sector employment rates in many countries have remained constant in the
postwar era.

3). The author states which of the following about activities in the informal economy?

A) They are an insignificant aspect of the global economy.


B) They provide income for workers who are unemployable in the formal economy.
C) They are a consequence of capitalist penetration of less developed economies.
D) They thrive mainly in underdeveloped countries in Asia, Africa, and Latin America.
E) They consist of such unregulated practices as off-the-books hiring and cash
payments.

Transitory (adj) : tạm thời, không bền

Interstice (n) : kẽ hở

Peripheral (adj) : ngoại vi

Exclusively (adv) : duy nhất

Destitution (n) : tình trạng thiếu thốn


Nov 25 – 2

Later Maya occupations of the Yucatan Peninsula site called Colha have undergone excavation since
1979. In 1993, researchers made the first systematic effort to document a pre-ceramic presence at
the tropical, forested location. Early Colha farmers inhabited the area in two phases. There are stone
tools in deeper soil layers dating from 2500 B.C. to 1700 B.C., based on radiocarbon age estimates
of accompanying charcoal bits. Comparable dates come from an adjacent swamp, where pollen
analysis documents forest clearance by 2500 B.C.

The pollen provides evidence for the existence of several cultivated crops soon thereafter, mainly
corn and manioc, a starchy plant. From about 1400 B.C. to 1000 B.C., Colha residents made foot-
shaped stone tools that were chipped and sharpened on one side. Preliminary scanning electron
microscope analysis of polish on these tools suggests that inhabitants used them to cut away
vegetation after controlled burning of trees, and, perhaps, also to dig.

An example of the same tool, known as a constricted uniface, also emerged last year at Pulltrouser
Swamp, a Maya site 20 miles northwest of Colha with a preliminary radiocarbon date of 1300 B.C. to
1000 B.C. for the artefact. Its unusual design led researchers to suspect that Colha might have
harboured an extremely early Maya population. Another sharpened stone point retrieved at
Pulltrouser Swamp dates to between 2500 B.C. and 2000 B.C. Several other sites in Belize have
yielded constricted unifaces, but archaeologists have been unsure of their ages and origins.

Techniques used to manufacture constricted unifaces show gradual refinement and modification in
stone tools of Colha residents living after 1000 B.C. Continuity in stone tool design and manufacture
suggests that pre-ceramic Maya inhabited Colha, rather than non-Maya peoples who migrated to the
area and later left or were incorporated into Maya villages. ―None of us had any reason to suppose
that Colha would produce a pre-ceramic Maya occupation,‖ remarks the director of excavations at
Cuello, a Maya site that dates to about 1000 B.C. ―This is a bit of archaeological serendipity.‖ This
is evidence of the earliest known Maya, who cleared and farmed land bordering swamps by 2,500
B.C. The earliest Central American farmers probably settled at the edges of
swampland that they had cleared and cultivated. Excavations of preceramic Colha so far have
focused on quarry and field areas. However, some pottery may still show up in early residential
structures.

1. The recent findings presented by the author in the passage provide new
insight into Mayan civilization because:

A. Mayans may have settled extensively throughout the Yucatan peninsula.


B. ceramic pottery may have been used by the Mayans.
C. Mayans may have settled in regions much earlier than previously thought.
D. stone tools were never used by the Mayans.
E. Mayans may actually be linked to Red Indians

2. In the context of the passage, the author quotes the use of the term
―archaeological serendipity‖ (line 33) to refer to:

A. the discovery of stone tools.


B. the unexpected findings that gave researchers a new understanding of ancient
settlements.
C. the method used by archaeologists to excavate ancient civilizations.
D. the Mayan‘s ability to work with their environment.
E. the possibility that Mayans may actually have used tools made of ceramics

―None of us had any reason to suppose that Colha would produce a pre-ceramic Maya occupation,‖
remarks the director of excavations at Cuello, a Maya site that dates to about 1000 B.C.

―This is a bit of archaeological serendipity.

3. According to the information presented by the author in the passage, analysis


of the stone tools retrieved from Colha led researchers to believe all of the
following EXCEPT:

A. a population of pre-ceramic Mayans existed who used and designed


stone tools.
B. Mayans had settlements prior to 1000 B.C.
C. non-Maya peoples inhabited the area before the Mayans migrated and
took over.
D. the tools underwent various stages of development.
E. tools used by the Mayans were not only restricted to ceramic material

Occupation (n) : sự chiếm đóng

Ceramic (n) : gốm sứ

Phrase (n) : giai đoạn

Adjacent (adj) : liền kề

Swamp (n) : đầm lầy

Pollen (n) : phấn hoa

Manioc (n) : sắn

Starchy (adj) : tinh bột

Chipped (v) : sứt mẻ

Preliminary (adj) : sơ bộ

Harbour (v) : cho ở, chứa chấp

Retrieve (v) : khôi phục, tìm lại được

Refinement (n) : sự tinh chế

Serendipity (n) : sự may mắn

Pottery (n) : đồ gốm, nghề làm gốm


Nov 26 – 1

Scholars of Native American history are profoundly interested in termination, the Truman-era policy
of ending Indian tribes’ special status with the federal government. In ending federal wardship, the
government sought to enhance individual and tribal “self-determination,” but, in fact, in most cases,
the policy’s effect was quite the opposite: Indian individuals and communities were dispossessed of
the land, rights, benefits, and aid that were their due and that were necessary for such self-
determination. The policy is considered to have had devastating effects on terminated tribes’
economies, autonomies, and cultures.

To understand the failure of termination as a whole, historians have had to look at tribes
individually, as each tribe’s relationship with the federal government was and is as nearly as unique
as each tribal culture itself. Most notable, perhaps, for the complexity of the issue are the cases of
California tribes. At the time, a significant number of Native Californians did not live on tribal land,
so for the promise of per capita distributions, many initially supported termination. In 1958, 44
California Indian groups were terminated, with tribal lands to be distributed to individual tribal
members, and general services, such as improved sanitation, water, education, and roads, to be
delivered by the federal government. However, in many communities, the promised improvements
never materialized. Moreover, in many cases, individual tribal members did not receive a land
allotment.

Perhaps worse than the unkept promises of material improvements was the loss of autonomy tribes
suffered with termination’s shifting laws and responsibilities. Once a tribe was terminated, and the
government-to-government relationship severed, tribes were regarded as a state’s responsibility. In
many cases, states were unwilling or unable to provide services for their new constituents.
Moreover, Public Law 280, which gave six states—including California—criminal jurisdiction in
previously sovereign Indian lands, was largely opposed by both states and Indian communities.
States were disinclined to enforce laws on a larger area without any additional funding, and Indian
communities were loath to have outside law enforcement without prior consent.

1. According to the passage, termination in California was

a. opposed by state lawmakers in California


b. originally supported by some California Indians without a tribal land base
c. one method for California Indians to access jobs in law enforcement
d. challenged by the state under Public Law 280
e. a source of federal income that the state could spend on increased local law enforcement

At the time, a significant number of Native Californians did not live on tribal land, so for
the promise of per capita distributions, many initially supported termination

2. Which of the following words best expresses the opinion of the author concerning the
welfare of Indians after termination?

a. self-determination
b. failure
c. unique
d. worse
e. disinclined
3. Which of the following best describes the relationship of the final paragraph to the
passage as a whole?

a. The main idea is reinforced by the inclusion of additional evidence.


b. The main idea is summarized and a transition to a new topic is given.
c. The main idea is restated in contrast to a contradictory opinion.
d. Exceptions to the main idea are critiqued and dismissed.
e. Local history is included to show that the main idea does not apply in every case.

4. Which of the following statements does the author imply about termination as it is
discussed in the second paragraph?

a. Its intent to create law enforcement jobs on California tribal lands would reduce crime there.
b. Its plan to improve roadways and water systems was stymied by state interference.
c. Its failure to implement infrastructure provisions worsened Native California
economies.
d. Its ultimate conditions were deemed satisfactory by the California Indians who did not live on
tribal lands.
e. Its plan to only allocate lands to those Indians who did not live on tribal land violated state law.

Termination (n) : sự đình chỉ, sự hủy bỏ, sự kết thúc

Wardship (n) : sự giám hộ

Devastating (adj) : tàn phá

Sanitation (n) : sự cải thiện vệ sinh

Materialize (v) : cụ thể hóa

Autonomy (n) : quyền tự trị

Constituent (n) : thành phần

Disincline (v) : không thích

Loath (adj) : không muốn, ghê tởm


Nov 26 – 2

When I talk about UFOs in my introductory astronomy classes, I always tell my students that I
absolutely believe in UFOs. After a brief pause for incredulous stares, I ask them to think about what
the term UFO actually means. I explain that I fully believe there are objects in the sky that the
average person may not be able to identify. This does not mean, however, that no one can identify
these objects. It only means that they could appear 'unidentified' to someone who is not familiar
with the sky or with the full range of sky phenomena that can surprise a novice. Indeed, upon more
careful investigation, many so-called UFOs turn out to be perfectly natural objects or processes in
the Earth's atmosphere or beyond. As the late Carl Sagan emphasized, 'Extraordinary hypotheses
require extraordinary proof.' Surely, the notion that some mysterious phenomenon you briefly
observed in the sky must be an interstellar spacecraft (and not a human craft, meteor, or a bright
planet) qualifies as such an 'extra-ordinary' hypothesis! Yet, amazingly, given the number of UFO
incidents believers report, not one UFO has left behind any proof - a piece of spacecraft material or
machinery (or even a sandwich wrapper) that laboratory analysis has shown to be of clearly
extraterrestrial origin. It's also remarkable how unlucky the UFO occupants are in their choice of
people to kidnap. Never do 'aliens' seem to snatch a person with a good knowledge of astronomy or
physics or someone with high-level government clearance. Time after time, their 'victims' turn out to
be homemakers, agricultural workers, or others whose relevant knowledge base seems to be limited
to reading UFO enthusiast literature. Even UFO sightings turn out to be reserved (for the most part)
for those who have not studied the sky in any serious way. Although the world's supply of
professional astronomers is not much larger than the population of Wasilla, Alaska, the world has
many tens of thousands of active amateur astronomers who spend a great deal of time observing
the sky. You would think that if UFOs really are alien spacecraft, a large majority of reported
sightings would come from this group. Yet, unsurprising to astronomers, you almost never get UFO
reports from experienced amateurs whose understanding of what they see in the sky is much more
sophisticated than that of the average person.

All of which does not mean that astronomers in general are pessimistic about the presence of
intelligent life on planets around other stars. Indeed, many observations over the last few decades
have increased the level of optimism in the astronomical community about the potential for life to
exist out there. Primary among these is the discovery of more than 300 planets around relatively
nearby stars, which certainly shows that planetary systems like our own may be far more common
than we dared to hope. We just don't think that intelligent aliens are necessarily visiting Earth. The
problem is that the stars are fantastically far away. If our Sun was the size of a basketball (instead
of 864,000 miles across), Earth would be a small apple seed about thirty yards away from the ball.
On that scale, the nearest star would be some 4,200 miles (7,000 km) away, and all the other stars
would be even farther! This is why astronomers are sceptical that aliens are coming here, briefly
picking up a random individual or two, and then going back home. It seems like an awfully small
reward for such an enormous travel investment.

1) The primary purpose of the author in the passage is to:

a. make distinctions between those who are mere lay people interested in the extraterrestrial and
qualified scientists and astronomers.
b. present us with a wealth of information to enable us to make our own judgements regarding
UFOs.
c. justify the presence of UFOs through examples of sightings and people's accounts.
d. debunk claims about UFO sightings by lay people, using reasoning and analysis to
back his assertion.
e. state that though he does not believe in the stories of UFO sightings, he believes in the existence
of aliens.

2) The author's statement in the first paragraph that he 'absolutely believes' in UFOs is:

a. purely sarcastic.
b. intended to ridicule those who believe in UFO sightings.
c. sarcastic, but also with a different meaning to it.
d. an honest and genuine assertion.
e. none of these.

Indeed, upon more careful investigation, many so-called UFOs turn out to be perfectly natural
objects or processes in the Earth's atmosphere or beyond.

"So called' can be interpreted as "...many unsupported UFO [sightings]..."

3) Which of these is a main argument of the author against the reported sightings of
UFOs?
a. UFO sightings are uncommon and seem to be experienced only by lay people, and not by
astronomers; hence they are unrealistic.
b. UFO sightings seem to happen only to amateurs with basic scientific knowledge.
c. UFO sightings are common only in certain areas and therefore cannot be described as a
widespread or universal phenomenon.
d. UFO sightings are little more than fantasy since there is little chance of intelligent beings existing
on any planet apart from Earth.
e. UFO sightings can be verified and legitimized only by scientists and astronomers, and
no such 'sightings' till date have been thus verified or legitimized.

4) Which of these is the author's precise outlook on the subject of UFOs and
extraterrestrial life?
a. He believes that the presence of intelligent life apart from human beings is possible,
but that aliens are visiting us in UFOs is merely an illusion.
b. He believes that extraterrestrial life exists, but it is not advanced enough to send 'UFO's' into
space.
c. He believes that extraterrestrial life exists, but UFOs are mere fantasy.
d. He advances the idea of extraterrestrial life while being non-committal as regards the existence of
UFOs.
e. He merely states that astronomers believe that extraterrestrial life can exist, without revealing his
own opinion, but debunks UFO theories.

We just don't think that intelligent aliens are necessarily visiting Earth. The problem is that the stars
are fantastically far away.

It seems like an awfully small reward for such an enormous travel investment.

Lay (adj) : không chuyên môn

Incredulous (adj) : hoài nghi


Nov 27 – 1

A 1973 Supreme Court decision and related Senate hearings focused congressional criticism on the
1966 Freedom of Information Act. Its unconditional exemption of any material stamped
“classified”—i.e., containing information considered relevant to national security—forced the Court to
uphold nondisclosure in EPA v. Mink. Justice Potter Stewart explained that the Act provided “no
means to question a decision to stamp a document ‘secret.’” Senate witnesses testified that the
wording of certain articles in the Act permitted bureaucrats to discourage requests for newsworthy
documents.

In response, a House committee drafted HR 12471, proposing several amendments to the Act. A
provision was reworded to ensure release of documents to any applicant providing a “reasonable
description”—exact titles and numbers were no longer to be mandatory. The courts were
empowered to review classified documents and rule on their status. The Senate companion bill, S
2543, included these provisions as well as others: standardization of search and copy fees, sanctions
against noncompliant federal employees, and a provision for nonexempt portions of a classified
document to be released.

The Justice and Defense departments objected to the changes as “costly, burdensome, and
inflexible.” They argued that the time limits imposed on response “might actually hamper access to
information.” The Pentagon asserted that judicial review of exemptions could pose a threat to
national security. President Ford, upon taking office in August 1974, concurred.

HR 12471 passed in March 1974; S 2543 was approved in May after the adoption of further
amendments to reduce the number of unconditional exemptions granted in 1966. The Hart
Amendment, for instance, mandated disclosure of law enforcement records, unless their release
would interfere with a trial or investigation, invade personal privacy, or disclose an informer’s
identity. This amendment provoked another presidential objection: millions of pages of FBI records
would be subject to public scrutiny, unless each individual section were proven exempt.

Before submitting the legislation to Ford, a joint conference of both houses amalgamated the two
versions of the bill, while making further changes to incorporate Ford’s criticisms. The administration
of disciplinary sanctions was transferred from the courts to the executive branch; provisions were
included to accord due weight to departmental expertise in the evaluation of “classified” exemptions.
The identity of confidential sources was in all cases to be protected. Ford nevertheless vetoed the
bill, but the veto was overridden by a two-thirds vote in both houses.

1). According to the passage, the Justice and Defense departments opposed the
proposed revision of the Freedom of Information Act on the grounds that it

A) was an attempt to block public access to information.


B) would violate national security agreements.
C) would pose administrative problems.
D) was an attempt to curtail their own departmental power.
E) would weaken the president’s authority.

The Justice and Defense departments objected to the changes as “costly, burdensome, and
inflexible.” They argued that the time limits imposed on response “might actually hamper access to
information.”
2). Which of the following statements, if true, supports the assertion that “judicial
review of exemptions could pose a threat to national security” (lines 20–21)?

A) Judges lack the expertise to evaluate the significance of military intelligence records.
B) Many of the documents that are presently stamped “classified” contain information that is
inaccurate or outdated.
C) It would be time-consuming and expensive for judges to review millions of pages of classified
records.
D) Some judges are likely to rule on exemptions in accordance with vested interests of political
action groups.
E) The practice of judicial review of exemptions will succeed only if it meets with presidential
approval.

3). Which of the following statements is in accordance with President Ford’s position on
disclosure of FBI records?

A) FBI records should be exempt from the provisions of the Freedom of Information Act.
B) FBI records should only be withheld from release if such release constitutes a threat to national
security.
C) It would be too expensive and time-consuming to identify exempt sections of FBI records.
D) Protection of the identity of confidential sources is more important than the protection of
personal privacy or investigative secrecy.
E) FBI records should not be reviewed section by section before being released to the public.

Hearings (n) : phiên điều trần

Senate (n) : thượng nghị viện

Congressional (adj) : đại hội, quốc hội

Unconditional (adj) : vô điều kiện

Exemption (n) : miễn trừ

Uphold (v) : duy trì, khuyến khích

Non-disclosure (v) : không tiết lộ

Bureaucrat (n) : quan chức

Sanction (n) : chế tài, hình phạt

Non-complaint (n) : không tuân thủ

Non-exempt (v) : không được miễn

The justice ans defense departments (n) : bộ tư pháp và quốc phòng

Burdensome (adj) : gánh nặng

Pentagon (n) : lầu năm góc

Judicial (adj) : tư pháp

Concur (v) : đồng tình


Mandate (v) : bắt buộc

Scrutiny (n) : xem xét kỹ lưỡng, giám sát

Amalgamate (v) : trộn lẫn

Veto (v) : phủ quyết


Nov 27 – 2

Through the glamour of the Colonial we are forced to acknowledge the classic charm shown in late
seventeenth and early eighteenth century window designs. Developed as they were by American
carpenters who were stimulated by remembrance of their early impressions of English architecture
received in the mother land, there is no precise or spiritless copy of English details; rather there is
expressed a vitality that has been brought out by earnest effort to reproduce the spirit desired.
Undoubtedly the lasting success of early American craftsmanship has been due to the perfect
treatment of proportions, as related one to the other. That these are not imitations is proved by an
occasional clumsiness which would be impossible, if they were exact copies of their more highly
refined English prototypes.

The grasp of the builder's mind is vividly revealed in the construction of these windows for, while
blunders are often made, successes are much more frequent. They are evolved from remembered
motives that have been unified and balanced, that they might accord with the exterior and be
knitted successfully into the interior trim. Some of these windows still grace seventeenth century
houses, and are found not only on old southern plantations, but all through New England, especially
along the sea coast. True products are they of Colonial craftsmanship, brought into existence by
skilled artisans who have performed their work so perfectly that today they are found unimpaired,
striking a dominant note in accord with the architectural feeling of the period.

There is no question that windows such as these lend character to any house, provided, of course,
that they coincide with the period. Doubtless the designing of modified Colonial houses is
responsible, in part, for the present-day revival of interest, not solely in windows of the Colonial
period, but also in that which immediately preceded and followed it.

1) Which of the following can be concluded about the colonial windows made by
American craftsmen?

A. Some of them were blatantly copied from the English windows


B. They may not have been of the same quality as the windows in England
C. They were free from all blemishes
D. They have been copied by craftsmen from other countries
E. They were primarily found in the Southern plantation areas

2) How does the author prove that the American windows are not a copy of the English
ones?

A. By pointing out that the American carpenters added their own vitality and expression to these
windows
B. By highlighting that the American windows have a slightly different design from the English ones
C. By providing examples of craftsmen who had never visited England and yet who have successfully
made these windows
D. By pointing out the occasional flaw in the design of these windows
E. By stating that the proportions of the American windows were different from the English ones

That these are not imitations is proved by an occasional clumsiness which would be impossible,
if they were exact copies of their more highly refined English prototypes.
3) Which of the following would the author of the passage NOT agree with?

A. The American colonial windows were not copies of the English ones
B. Colonial windows have always been popular in America
C. Colonial windows can make a house with any design look more graceful
D. American colonial windows were not as refined as the English ones
E. The American craftsmen used their memory to design these windows

That these are not imitations is proved by an occasional clumsiness which would be impossible, if
they were exact copies of their more highly refined English prototypes.

The author states in the last para that colonial windows can make any house with a colonial design
look good.

‘There is no question that windows such as these lend character to any house, provided, of course,
that they coincide with the period’
4) According to the information in the passage, which of these could be a characteristic
of American colonial windows?

A. Unmatched levels of refinement


B. Well-proportioned design
C. Use of high quality wood
D. Asymmetric design
E. Standing out in the design of a house

Undoubtedly the lasting success of early American craftsmanship has been due to the perfect
treatment of proportions, as related one to the other

Proportion (n) : tỷ lệ cân xứng

Glamour (n) : sự quyến rũ, say mê, lôi cuốn

Stimulate (v) : kích thích

Remembrance (n) : hồi tưởng

Spiritless (adj) : vô hồn

Vitality (n) : sức sống

Imitation (n) : bắt chước

Occasional (adj) : thỉnh thoảng

Clumsiness (n) : vụng về, xấu xí

Refine (v) : tinh chế

Grasp (n) : sự hiểu biết

Blunder (n) : sai lầm ngớ ngẩn


Interior trim (n) : trang trí nội thất

Accord with (v) : phù hợp với

Exterior (n) : ngoại thất

Grace (adj) : ân sủng, làm đẹp thêm

Plantation (n) : đồn điền

Craftmanship (n) : nghề thủ công

Unimpaired (adj) : không bị suy yếu hay tổn hại

Coincide (v) : phù hợp, trùng hợp


Nov 28 – 1

The United States National Park Service (NPS) is in the unenviable position of being charged with
two missions that are frequently at odds with one another. Created by an act of Congress in 1916,
the NPS is mandated to maintain the country’s national parks in “absolutely unimpaired” condition,
while somehow ensuring that these lands are available for “the use . . . and pleasure of the people.”
As the system of properties (known as units) managed by the NPS has grown over the years—a
system now encompassing seashores, battlefields, and parkways—so
has its popularity with the vacationing public. Unfortunately, the maintenance of the system has not
kept pace with the record number of visits, and many of the properties are in serious disrepair.

Several paths can be taken, perhaps simultaneously, to alleviate the deterioration of the properties
within the system. Adopting tougher requirements for admission could reduce the number of
additional units that the NPS manages. Congress has on occasion added properties without any
input from the NPS itself. It is debatable whether all of these properties, which may be of
importance to the constituents of individual representatives of Congress, pass the test of “national
significance.” Furthermore, some of the units now in the NPS (there are close to
400) receive few visitors, and there is no reason to think that this trend will reverse itself. These
units can be removed from the system, and their fates can be decided by local public and private
concerns. The liberated federal funds could then be rerouted to areas of greater need within the
system.

Another approach would be to attack the root causes of the deterioration. Sadly, a great deal of the
dilapidated condition of our national parks and park lands can be attributed not to overuse, but to
misuse. Visitors should be educated about responsible use of a site. Violators of these rules must be
held accountable and fined harshly. There are, of course, already guidelines and penalties set in
place, but studies strongly indicate that enforcement is lax.

1. Which of the following best describes the organization of the passage as a whole?

A) The author mentions a problem, and opposing solutions are then described.
B) The various factors that led to a problem are considered, and one factor is named the root cause.
C) A historical survey is made of an institution, followed by a discussion of the problem of
management of the institution.
D) A problem is described, and two possibly compatible methods for reducing the
problem are then outlined.
E) A description of a plan and the flaws of the plan are delineated.

Several paths can be taken, perhaps simultaneously, to alleviate the deterioration of the
properties within the system

2. The author is primarily concerned with

A) analyzing the various problems that beset the NPS


B) summarizing the causes of deterioration of NPS properties
C) criticizing the NPS’s maintenance of its properties
D) encouraging support for increased funding for the NPS
E) discussing possible solutions for NPS properties’ deterioration

Unenviable (adj) : không thể tin được


Disrepair (n) : hư hỏng

Alleviate (v) : giảm bớt

Deterioration (n) : xấu đi, giảm giá trị, hư hỏng

Dilapidated (adj) : đổ nát, hư hại, phung phí, phá hủy

Lax (adj) : lỏng lẻo


Nov 28 – 2

The Establishment Clause and the Free Exercise Clause of the First Amendment to the US
Constitution read, Congress shall make no law respecting an establishment of religion, or prohibiting
the free exercise thereof. Secularity of government and freedom of religious exercise form the basis
for the separation of church and state doctrine, the political and legal principle that posits that the
government is to remain neutral in matters of religion. The language of separation traces back to an
1802 letter by Thomas Jefferson to the Danbury Baptist Association, in which he referenced the First
Amendment as building a wall of separation between Church & State.

There is much debate over whether the framers of the Constitution truly intended government and
churchly organizations to be both separate and independent. Interpreting the Founding Fathers'
intentions has sparked much discussion of such contemporary issues as prayer in schools, school
vouchers, government funding of faith-based organizations, and religious displays on government
property. Accomodationists argue against separatism, claiming that the Founding Fathers based this
nation and the Constitution on Biblical values and beliefs, and want to see theology play an
important role in the governing and legislation of America. They cite the word Creator in the
Declaration of Independence -- "(all men) are endowed by their Creator with certain inalienable
Rights.... "as evidence that the framers of the Constitution meant the US to be ruled according to
divine law. Separationalists say that the Declaration of Independence, signed thirteen years before
the Constitution was ratified on March 4, 1789, is not the law of the land; therefore, the First
Amendment prohibits all intrusion between government and religion. They also claim that those who
ratified the US Constitution were enlightened Rationalists and evangelical Christians who, like the
Puritans who fled the Church of England's persecution and settled in Massachusetts Bay Colony in
1629, were concerned by entanglements between church and state.

The debate over whether the Constitution intended churchly institutions to be separate and
independent from government will continue. Yet it can be argued that according to the New
Testament (Matthew 22:21), when asked whether taxes should be paid to Caesar, Christ replied,
Render unto Caesar the things that are Caesar's, and unto God the things that are God's.

1. Which of the following, if true, would support the assertion that the settlers of
Massachusetts Bay Colony "were concerned by entanglements between church and
state"?

I. They established a body of government in the colony that barred members of the clergy from
holding public office.
II. They disagreed with the governing foundation in England in which the Anglican Church was
headed by the King.
III. Because of their persecution, the early American settlers wanted to ensure their own freedom of
religion in Massachusetts Bay.

Persecution could come from other factors such as other lay people, not from state

A. I only
B. II only
C. III only
D. I and II
E. I and III
2. With which of the following statements would the author of the passage most likely
agree?

A. Because separation of church and state is not historically justified, then correcting this
misconception has great implications.
B. The framers of America's political system did not intend for religious parties to hold office.
C. The interpretation of the American Founders' intentions regarding church and state is
an important issue for constitutional discourse.
D. is difficult to build sound constitutional doctrine on a misinterpretation of the Establishment and
Free Exercise Clauses.
E. The intentions of the Founding Fathers need to adapt and be reevaluated as societal norms
change over time.

3. Which of the following best describes how the quote in the last paragraph functions
within the passage?

A. A perspective is given that supports a viewpoint that was presented earlier.


B. An additional argument is raised that does not deserve further questioning.
C. A viewpoint is presented that disqualifies the theme of the passage.
D. An irrelevant outlook is given that intends to offer an alternative explanation for a debate.
E. A controversial view is outlined that aims to clarify the interpretation of a constitutional doctrine.

4. The primary purpose of the passage is to

A. question a constitutional principle, followed by a debate that weighs the pros and cons of that
principle.
B. present a fundamental policy rooted in American constitutionalism, as well as the
perspectives of those who interpret its meaning.
C. outline an argument that supports why the intentions of those who drafted the Constitution
should not be important.
D. evaluate the assumption of a constitutional debate.
E. discuss the historical context for a legal doctrine and propose explanations that raise concerns on
its relevance.

Secularity (n) : thế tục

Posit (v) : đặt ra, thỉnh cầu

Separatism (n) : người theo chủ nghĩa ly khai

Ratify (v) : phê chuẩn

Intrusion (n) : xâm nhập

Evangelical (adj) : truyền giáo

Entanglement (n) : vướng mắc

Testament (n) : thánh kinh


Nov 29 – 1

In 1964, Marshall McLuhan published his seminal work on media theory, Understanding Media. In
this book, McLuhan first articulated the idea that would come to be known as “the medium is the
message.” McLuhan coined the descriptions “hot” and “cool” to describe the level of interactivity
inherent to different forms of communication. To McLuhan, high definition media such as movies
were “hot” in that they engaged the sense of vision so completely that the audience need not fill in
any details on its own. In contrast, McLuhan deemed lower definition media, such as television and
comics, “cool” in that they offered less detail to the audience and thus required that the audience
participate more actively in determining meaning.

Critics have dismissed McLuhan’s work as being reductivist. Notably, Eco has claimed that within the
word “medium,” McLuhan has blurred the lines among the distinct concepts of channel, code, and
message. Likewise, Debray has dismissed McLuhan’s ideas as placing undue emphasis upon the
technology itself instead of analyzing how the technology is used. Criticisms aside, however,
McLuhan’s groundbreaking work has forever changed the landscape of popular cultural criticism and
has greatly enriched the vocabulary that scholars still use today.

1.The passage suggests that McLuhan would be most likely to agree with which of the
following statements?

a. Television is a “cooler” medium than comics are because television requires more participation
from the audience.
b. A lecture is “hotter” than a seminar is because a lecture requires less participation
from the audience.
c. As technology develops, media are becoming “cooler.”
d. Rather than being distinct ideas, channel, code, and message are merely different aspects of the
concept of “medium.”
e. Radio is a “hotter” medium than television is because radio requires more participation from the
audience.

Seminar  the audiences will need to listen more = television

Lecture  the audiences will need to think more, engage the sense of vision = movies

2.According to the passage, which of the following is the most accurate description of
Eco’s view of McLuhan’s work?

a. He dismisses it for placing undue emphasis upon the technology used to communicate rather than
on what is communicated.
b. He disagrees with its conclusion but acknowledges the contribution that it has made to the study
of popular culture.
c. He believes that it oversimplifies a concept by conflating disparate ideas.
d.He wholeheartedly rejects McLuhan’s assertion that “the medium is the message.”
e. He believes that it inappropriately elevates the concept of “message” over those of “channel” and
“code.”

3.The primary purpose of the passage is to


a. discuss criticisms of the ideas presented in McLuhan’s Understanding Media
b. dismiss the objections of two critics of McLuhan’s views on media theory
c. call for a renewed study of the theories that McLuhan presents in Understanding Media
d. present some of the contributions of and objections to a prominent scholar of media
theory
e. reconcile competing views of McLuhan’s claim that “the media is the message”

Seminar (n) : hội thảo

Articulate (v) : khớp nối

Interactivity (n) : tính tương tác

Groundbreaking (adj) : đột phá


Nov 29 – 2

The best-known platonic depiction of tyranny appears in Republic, where the tyrant is beastly,
subject to base and unnecessary appetites: power, vainglory, luxury, lust, and gluttony. To the
extent that passions control him—a decidedly male figure—the tyrant is a sort of slave, who
depends on both taxation to support him and his ―drink-mates…and…mistresses,‖ as well as
bodyguards to protect him from assassins and other ―worthless creatures‖ who proliferate under
tyrannical rule.

An argument recently propounded by the historian of philosophy Matteo Giovannini threatens to


unsettle this widely held view of the platonic tyrant as a brutish slave. According to Giovannini, the
traditional view, while sound as far as it goes, is incomplete in that it ignores insights into the
tyrannical character that are offered by Plato in the earlier and more obscure dialogue, Lysis.

If the ancient Greeks first inspired the ideological commitment to democracy that gripped Western
thought especially during and after the Enlightenment, the Greek philosophers contributed to this
development less by their embrace of the democratic principle than by their rejection of tyranny. In
Aristotle‘s schema, tyranny is the most perverse of six types of government; Plato designated five
types, with tyranny the least desirable, followed by democracy. Yet less clear than Plato‘s disregard
for the tyrannical character is his sense of its basic constitution.

Giovannini‘s account purports to complicate the one-dimensional view of tyranny associated with
Republic. But this account, while ingenious and provocative, is not beyond question. Most
significantly, Giovannini appears not to have anticipated an obvious objection to his research design.
While Lysis first appeared during Plato‘s formative period of aporetic dialogues in which the principal
interlocutors frequently pose questions but rarely provide lasting answers, Republic dates from a
later, more mature period in the development of Plato‘s thought, when conclusions are more
frequent and less concealed. If Plato intended the conception of tyranny that appears in Republic to
be somehow bound up in a paradox with the conception of tyranny in Lysis, he would presumably
have hinted as much. Absent such indications, the danger is heightened that Giovannini may have
invented, rather than discovered, subtle interconnections in Plato‘s thought.

According to Giovannini, Lysis forms a counterpoint to Republic by depicting a tyrant whose status
derives, not from his slavish dependency, but from his utter self-sufficiency; he is complete, or (to
use the language of the ancient philosophers) perfect. For such a figure, friendship—for many of the
Greek philosophers, the foundation of healthy political community—is ultimately impossible, because
―the one who is perfect does not depend on the many who are imperfect, but the many who are
imperfect depend on the one who is perfect.‖ In short, Giovannini argues, the tyranny found in Lysis
is the wake of a doomed union between the needy masses and the singular, complete one. Viewed
in the double light of Republic and Lysis, the platonic tyrant depicted by Giovannini is a paradoxical
figure: here a slave; there the epitome of wholeness.
1. The author makes a few different points throughout the passage. In paragraph 4, the
author is primarily concerned with:

A. providing a richer alternative to the one-dimensional view of tyranny furnished in Republic.


B. establishing a relationship between the content of platonic dialogues and the order in which they
first appeared.
C. dismissing Giovannini‘s findings on the grounds that they are more imagined than real.
D. supplying an overall assessment of Giovannini‘s argument about the platonic
conception of tyranny.
E. mildly criticising Giovannini‘s findings but also indirectly supporting them

2. According to the information put forth by the author in the passage, what does
Giovannini suggest about tyrannical regimes as depicted in Lysis?

A. They fulfil the brutish desires of the tyrant.


B. They are typically incompatible with the political community.
C. They result from a severe imbalance in the relationship between the ruler and the
ruled.
D. They promote strength and self-reliance among the general populace.
E. They are an anomaly in the otherwise benevolent regimes of other rulers

3. Suppose conclusive evidence emerged that, in order to shield his audience from
confusion, Plato on occasion intentionally avoided revealing complex or seemingly
contradictory conclusions in his dialogues. What relevance would this information have
to the passage?

A. It would weaken Giovannini‘s claim that the platonic tyrant is a paradoxical figure.
B. It would verify the author‘s assertion that Republic provides a reasonable but only partial
depiction of Plato‘s conception of the tyrannical character.
C. It would weaken the author‘s major criticism of Giovannini‘s research.
D. It would weaken the author‘s assessment of Giovannini‘s work as ingenious and provocative
E. It would have no impact on the claims made in the passage

Platonic (adj) : đương phương

Tyranny (n) : chuyên chế

Republic (n) : cộng hòa, đoàn thể

Beastly (adj) : dã thú, hung dữ

Apetite (n) : khao khát, thèm muốn

Vainglory (n) : hư vinh

Lust (adj) : ham muốn, dục vọng

Gluttony (adj) : háu ăn

Proliferate (v) : sinh sôi nảy nở

Mistress (n) : tình nhân


Propound (v) : đưa ra

Brutish (adj) : tàn bạo

Dialogue (n) : hội thoại, đàm thoại

Enlightenment (n) : giác ngộ

Schema (n) : lược đồ

Perverse (adj) : hư hỏng

Designate (v) : gọi tên, bổ nhiệm

Disgard (n) : xem thường

Ingenious (adj) : khéo léo

Provocative (adj) : trêu chọc, khiêu khích

Beyond (adj) : nằm ngoài

Aporetic (adj) : lời xin lỗi

Interlocutor (n) : người đối thoại

Conceal (adj) : che giấu, giữ kín

Hint (v) : ám chỉ

Subtle (adj) : tinh tế, khôn khéo

Utter (adj) : hoàn toàn

Doomed (adj) : cam chịu

Needy (n) : người túng thiếu

Epitome (n) : mẫu mực

Wholeness (n) : trọn vẹn

Regime (n) : chế độ

Benevolent (adj) : nhân từ

Anomaly (n) : dị thường


Nov 30 – 1

A monopoly exists when a specific person or enterprise is the only supplier of a particular commodity
(this contrasts with a monopsony which relates to a single entity's control of a market to purchase a
good or service, and with an oligopoly which consists of a few entities dominating an industry).
Monopolies are thus characterized by a lack of economic competition to produce the good or service
and a lack of viable substitute goods. The verb "monopolize" refers to the process by which a
company gains the ability to raise prices or exclude competitors. In economics, a monopoly is a
single seller. In law, a monopoly is a business entity that has significant market power, that is, the
power to charge high prices. Although monopolies may be big businesses, size is not a characteristic
of a monopoly. A small business may still have the power to raise prices in a small industry.

A monopoly is distinguished from a monopsony, in which there is only one buyer of a product or
service; a monopoly may also have monopsony control of a sector of a market. Likewise, a
monopoly should be distinguished from a cartel (a form of oligopoly), in which several providers act
together to coordinate services, prices or sale of goods. Monopolies, monopsonies and oligopolies
are all situations such that one or a few of the entities have market power and therefore interact
with their customers (monopoly), suppliers (monopsony) and the other companies (oligopoly) in
ways that leave market interactions distorted.

Monopolies can be established by a government, form naturally, or form by integration. In many


jurisdictions, competition laws restrict monopolies. Holding a dominant position or a monopoly of a
market is often not illegal in itself. However certain categories of behavior can be considered
abusive and therefore incur legal sanctions when a business is dominant. A government-granted
monopoly or legal monopoly, by contrast, is sanctioned by the state, often to provide an incentive to
invest in a risky venture or enrich a domestic interest group. Patents, copyright, and trademarks are
sometimes used as examples of government granted monopolies. The government may also reserve
the venture for itself, thus forming a government monopoly.

1- What is the primary purpose of the second paragraph in the passage:

A- To highlight the differences between monopolies and monopsonies and oligopolies.


B- To suggest that monopolies, monopsonies, and oligopolies are very similar.
C- To note some distinctions between and shared characteristics of monopolies and
several other similar economic constructs.
D- To help the reader understand when monopolies are illegal and when they are not.
E- To explain what a monopoly is and what distinguishes it from other businesses.
2- According to the passage, which of the following situations would most likely
constitute a monopsony, but NOT a monopoly (as defined in the legal sense)?

A- A large computer manufacturing company in this country has several major competitors, but only
uses one supplier for its chips and other hardware components.
B- The largest discount department chain in this country has the lowest retail prices in
the market, but makes a profit by controlling the suppliers and dictating the prices it
pays for the goods it sells.
C- The three largest grocery store chains control the market and collude to keep prices high, but
struggle to keep their purchase price for foodstuffs low.
D- The largest airline in the country merges with the second largest airline, allowing it to control a
majority of domestic routes and charge higher prices for its flights; simultaneously, it is able to
control the purchase prices of planes from the two major aircraft manufacturers.
E- The largest orange juice manufacturer in the country has a complete control on retail prices and
has driven up prices by 50% in the past two years; it has simultaneously controlled orange growers
and pushed down the prices it pays by over 25% in the same time period.

3- Which of the following can be properly inferred from the passage above?

A- Monopolies that develop through a patent or trademark are rarely illegal.


B- A monopoly could never also be considered a monopsony.
C- Most monopolies are considered illegal and incur government sanctions.
D- A business that is considered a monopoly in an economic sense might not be
considered one in a legal sense.
E- A monopoly is distinguished from an oligopoly (or cartel) by the degree of influence over the
market.

In economics, a monopoly is a single seller. In law, a monopoly is a business entity that has
significant market power, that is, the power to charge high prices.

 Definitions of monopoly in term of economic and in term of law are different ( not be
considered one )

Sense (n) : ý nghĩa

Monopsony (n) : thị trường chỉ có một người mua


Nov 30 – 2

The latest prominent principle of criminal sentencing is that of "selective incapacitation". Selective
incapacitation, like general incapacitation, involves sentencing with the goal of protecting the
community from the crimes that an offender would commit if he were on the street. It differs from
general incapacitation in its attempt to replace bluntness with selectivity.

Under a strategy of selective incapacitation, probation and short terms of incarceration are given to
convicted offenders who are identified as being less likely to commit frequent and serious crimes,
and longer terms of incarceration are given to those identified as more crime prone. Selective
incapacitation has the potential for bringing about a reduction in crime without an increase in prison
populations. This reduction could be substantial.

Reserving prison and jail space for the most criminally active offenders in some instances conflicts
not only with other norms of legal justice, but with norms of social justice as well. If we reserve the
sanction of incarceration only for the dangerous repeat offender, excluding the white collar offender
and certain other criminals who pose no serious threat of physical injury to others, we may end up
permitting harmful people from the middle class to evade a sanction that less privileged offenders
cannot.

One of the most pervasive criticisms of selective incapacitation is that it is based on the statistical
prediction of dangerousness; because such predictions are often erroneous, according to this point
of view, they should not be used by the court. This criticism is related to both the nature of the
errors and to the use of certain information for predicting a defendant‘s dangerousness. Let‘s first
consider the nature of errors in prediction.

Prediction usually results in some successes and in two kinds of errors: "false positives"- and "false
negatives". The problem of false positives in sentencing is costly primarily to incarcerated
defendants who are not really so dangerous, while false negative predictions impose costs primarily
on the victims of subsequent crimes committed by released defendants. In predicting whether a
defendant will recidivate, the problem of false positives is widely regarded as especially serious, for
many of the same reasons that it has been regarded in our society as better to release nine
offenders than to convict one innocent person.

A tempting alternative is to reject prediction altogether; obviously, if we do not predict, then no


errors of prediction are possible. A flaw in this logic is that, whether we like it or not-indeed, even if
we tried to forbid it-criminal justice decisions are now, and surely always will be, based on
predictions, and imperfect ones, at that. Attempts to discourage prediction in sentencing may in fact
produce the worst of both worlds: the deceit of predictive sentencing disguised as something more
tasteful, and inferior prediction as well. If we are to reserve at least some prison and jail space for
the most criminally active offenders, then the prediction of criminal activity is an inescapable task. Is
selective incapacitation truly an effective and appropriate proposal, an ―idea whose time has
come,or is it a proposal that carries with it a potential for injustice?

1. Suppose the number of dangerous criminals that would be imprisoned under selective
incapacitation but otherwise set free is greater than the number of harmless criminals
who would be set free under selective incapacitation but otherwise imprisoned. How
would this information be relevant to the passage?
A. It weakens the claim that the goal of selective incapacitation is to protect the community.
B. It strengthens the claim that there are more violent than non-violent criminals.
C. It weakens the claim that selective incapacitation would not increase prison
populations.
D. It strengthens the claim that white-collar criminals unfairly receive shorter sentences.
E. It is of no relevance to the passage

2. The author‘s statement that selective incapacitation may "end up permitting harmful
people from the middle class to evade a sanction that less privileged offenders cannot"
assumes that:

A. there are more offenders in the lower-class than in the middle-class.


B. the dangerous repeat offenders are lower-class and not middle-class.
C. harmful middle-class people can use their money to avoid prison.
D. lower-class offenders do not deserve to suffer incarceration.
E. the rich do not ever commit crimes

3. Based on the passage, which of the following would most likely be cited by an
opponent of statistical prediction as the reason that prediction should be abandoned?

A. The possibility of letting a dangerous criminal loose is too great.


B. The possibility of imprisoning a man who should be allowed to go free is too great.
C. The court makes more accurate decisions when statistics is employed.
D. Dangerousness has yet to be adequately defined as a legal concept.
E. Statistics is an inexact science

In predicting whether a defendant will recidivate, the problem of false positives is widely regarded
as especially serious, for many of the same reasons that it has been regarded in our society as
better to release nine offenders than to convict one innocent person.

Incapacitation (n) : mất khả năng

Bluntness (n) : sự cùn

Probation (n) : thời gian thử thách

Incarceration (n) : tống giam

Evade (v) : trốn tránh

Privileged (adj) : đặc quyền

Erroneous (adj) : sai

Offender (n) : người phạm tội

Recidivate (v) : tái phạm

Deceit (n) : lừa dối

Inescapable (adj) : không thể trốn


Dec 1 – 1

The anthropomorphic bias of those who would relegate marsupials to an inferior evolutionary status
is most apparent in their recourse to data on brain structure and behavior. Unlike humans and other
placentals, marsupials lack the corpus callosum, which facilitates inter-hemisphere transfer of data
acquired through the senses. Yet it cannot be inferred that marsupials are thus deprived of such
function. Didelphis Virginiana, one of the opossums, makes use of the anterior commissure, an
adaptation that is also found in reptiles and monotremes. Diprodontons, including kangaroos and
koalas, supplement the anterior commissure with the fasciculus aberrans. While the modes of
neocortical interconnection may be diverse, the work of Johnson, Heath, and Jones points to the
conclusion that, functionally speaking, the cortices and neocortices of both groups of mammals
exhibit parallel connections. Parker also notes "a similar range of brain size to body weight ratios
and of neocortical expansion."

Another stigma borne by marsupials is the consensus that they are less intelligent than placentals.
Yet Williams argues that, all else being equal, natural selection will favor instinctive over learned
behavior as being more biologically efficient and that it is the accidental death of the young that is
the prime selective pressure for the evolution of intelligence. Seen in this light, marsupials have a
competitive edge; their gestation period is brief and the young remain in the pouch for an extended
period exposed only to those dangers which also affect the mother. There they are directly exposed
to the mother's food supply and can observe her behavior at leisure. Placentals, on the other hand,
not only have a longer gestation period but, once their young are born, must often leave while
foraging. Such absences increase the risk of mortality and decrease the opportunity to learn. Thus,
among placentals, selection would favor the appearance of intelligence in the young and protective
behavior in the mother.

Marsupials are not known to exhibit maternal protective behavior. In fact, Serventy has reported
that frightened female kangaroos will drop their pouch-young as they flee, drawing a predator's
attention to the less able offspring while the adult escapes. This behavior, whether purposeful or
accidental, instantaneously relieves the female marsupial of the mechanical difficulties of pregnancy
with which her placental counterpart would be burdened, while marsupials can replace any lost
young quickly. Thus, in the absence of any need for close maternal supervision, sacrificing their
offspring in this manner may well have been favored in selection. Pointing to the absence of the
"virtue" of maternal protectiveness in marsupials is an instance of how mistaken are those theorists
who see similarities with humans as marks of evolutionary sophistication.

According to the passage, similarities between marsupials and placentals will most
likely be found in

A.brain function
B.brain anatomy
C.maternal behavior
D.the corpus callosum
E.selection for intelligence

'Yet it cannot be inferred that marsupials are thus deprived of such function.'
According to the passage, which of the following favor(s) the development of
intelligence as a trait of placental mammals?

I) The need to leave their young while foraging


II) The comparatively great risk of accidental death of the young
III) The opportunity for the young to observe the mother at leisure

A) I only
B) III only
C)I and II only
D) II and III only
E)I, II, and III

Seen in this light, marsupials have a competitive edge; their gestation period is brief and the young
remain in the pouch for an extended period exposed only to those dangers which also affect the
mother. There they are directly exposed to the mother's food supply and can observe her behavior
at leisure.

The author's attitude toward those who consider marsupials to occupy an inferior
evolutionary position would most probably be one of

A)criticism because they ignore evidence that marsupials are more intelligent than usually supposed
B)disagreement because current studies support the opposite view
C)disagreement because they apply human standards in an inappropriate context
D)agreement, but on the basis of marsupials' lack of maternal protective behavior rather than their
brain structure
E)neutrality, on the grounds that the concept of evolutionary inferiority has not been defined with
precision

All of the following authorities are cited for their work on marsupials EXCEPT

A) Johnson
B) Parker
C) Health
D) Williams
E) Serventy

Anthropomorphic (adj) : hình học

Relegate (v) : xuống hạng

Marsupial (n) : thú có túi

Inter-hemisphere (n) : liên bán cầu


Nov 1 – 2

At the end of the nineteenth century, a rising interest in Native American customs and an increasing
desire to understand Native American culture prompted ethnologists to begin recording the life
stories of Native American. Ethnologists had a distinct reason for wanting to hear the stories: they
were after linguistic or anthropological data that would supplement their own field observations, and
they believed that the personal stories, even of a single individual, could increase their
understanding of the cultures that they had been observing from without. In addition many
ethnologists at the turn of the century believed that Native American manners and customs were
rapidly disappearing, and that it was important to preserve for posterity as much information as
could be adequately recorded before the cultures disappeared forever.

There were, however, arguments against this method as a way of acquiring accurate and complete
information. Franz Boas, for example, described autobiographies as being “of limited value, and
useful chiefly for the study of the perversion of truth by memory,” while Paul Radin contended that
investigators rarely spent enough time with the tribes they were observing, and inevitably derived
results too tinged by the investigator’s own emotional tone to be reliable.

Even more importantly, as these life stories moved from the traditional oral mode to recorded
written form, much was inevitably lost. Editors often decided what elements were significant to the
field research on a given tribe. Native Americans recognized that the essence of their lives could not
be communicated in English and that events that they thought significant were often deemed
unimportant by their interviewers. Indeed, the very act of telling their stories could force Native
American narrators to distort their cultures, as taboos had to be broken to speak the names of dead
relatives crucial to their family stories.

Despite all of this, autobiography remains a useful tool for ethnological research: such personal
reminiscences and impressions, incomplete as they may be, are likely to throw more light on the
working of the mind and emotions than any amount of speculation from an ethnologist or
ethnological theorist from another culture.

1. Which of the following best describes the organization of the passage?

(A) The historical backgrounds of two currently used research methods are chronicled.
(B) The validity of the data collected by using two different research methods is compared.
(C) The usefulness of a research method is questioned and then a new method is proposed.
(D) The use of a research method is described and the limitations of the results obtained
are discussed.
(E) A research method is evaluated and the changes necessary for its adaptation to other subject
areas are discussed.

2. Which of the following is most similar to the actions of nineteenth-century


ethnologists in their editing of the life stories of Native Americans?

(A) A witness in a jury trial invokes the Fifth Amendment in order to avoid relating personally
incriminating evidence.
(B) A stockbroker refuses to divulge the source of her information on the possible future increase in
a stock’s value.
(C) A sports announcer describes the action in a team sport with which he is unfamiliar.
(D) A chef purposely excludes the special ingredient from the recipe of his prizewinning dessert.
(E) A politician fails to mention in a campaign speech the similarities in the positions held by her
opponent for political office and by herself.

3. According to the passage, collecting life stories can be a useful methodology because

(A) life stories provide deeper insights into a culture than the hypothesizing of
academics who are not members of that culture
(B) life stories can be collected easily and they are not subject to invalid interpretations
(C) ethnologists have a limited number of research methods from which to choose
(D) life stories make it easy to distinguish between the important and unimportant features of a
culture
(E) the collection of life stories does not require a culturally knowledgeable investigator

4. Information in the passage suggests that which of the following may be a possible
way to eliminate bias in the editing of life stories?

(A) Basing all inferences made about the culture on an ethnological theory
(B) Eliminating all of the emotion-laden information reported by the informant
(C) Translating the informant’s words into the researcher’s language
(D) Reducing the number of questions and carefully specifying the content of the questions that the
investigator can ask the informant
(E) Reporting all of the information that the informant provides regardless of the
investigator’s personal opinion about its intrinsic value

while Paul Radin contended that investigators rarely spent enough time with the tribes they were
observing, and inevitably derived results too tinged by the investigator’s own emotional tone to be
reliable.
Even more importantly, as these life stories moved from the traditional oral mode to recorded
written form, much was inevitably lost. Editors often decided what elements were significant
to the field research on a given tribe

5. The primary purpose of the passage as a whole is to

(A) question an explanation


(B) correct a misconception
(C) critique a methodology
(D) discredit an idea
(E) clarify an ambiguity

6. It can be inferred from the passage that a characteristic of the ethnological research
on Native Americans conducted during the nineteenth century was the use of which of
the following?

(A) Investigators familiar with the culture under study


(B) A language other than the informant’s for recording life stories
(C) Life stories as the ethnologist’s primary source of information
(D) Complete transcriptions of informants’ descriptions of tribal beliefs
(E) Stringent guidelines for the preservation of cultural data

7. The passage mentions which of the following as a factor that can affect the accuracy
of ethnologists’ transcriptions of life stories?

(A) The informants’ social standing within the culture


(B) The inclusiveness of the theory that provided the basis for the research
(C) The length of time the researchers spent in the culture under study
(D) The number of life stories collected by the researchers
(E) The verifiability of the information provided by the research informants

8. It can be inferred from the passage that the author would be most likely to agree
with which of the following statements about the usefulness of life stories as a source of
ethnographic information?

(A) They can be a source of information about how people in a culture view the world.
(B) They are most useful as a source of linguistic information.
(C) They require editing and interpretation before they can be useful.
(D) They are most useful as a source of information about ancestry.
(E) They provide incidental information rather than significant insights into a way of life.

Ethnologist (n) : nhà dân tộc học

Posterity (n) : hậu thế

Derive (v) : trích dẫn

Tinge (v) : nhuốm màu


Dec 2 – 1

One of the biggest critiques of the 2007-2008 financial crisis is the concept of “too big to fail.” The
"too big to fail" theory asserts that certain corporations, and particularly financial institutions, are so
large and interconnected that their failure would be disastrous to the entire economic system, and
therefore they must be supported by the government when they face potential failure. This is the
theory Congress and the president applied when bailing out the banking system at the end of 2008
with 700 billion dollars. What the government bought was 700 billion dollars of illiquid mortgage-
backed securities that the banks should not have lent to consumers in the first place, but were able
to because of a lack of regulatory oversight.

These mortgage-backed securities were created from pools of loans of which the homeowners did
not have the means to pay back the lender for the long-term asset. The primary reasons for
homeowners’ defaulting were fixed-rate loans converted to higher adjustable interest rates, loss of
employment in a recession, and underwater loans (a home purchase loan with a higher balance than
the free-market value of the home). Unfortunately, while the bailout from the United States federal
government may have saved the larger institutions, many of the loan borrowers found themselves
defaulting on their mortgages because of unforeseen circumstances.

Since the economic crisis, the Federal Reserve and other financial overseers have made it clear that
corporation size comes with costs. Now, it may no longer be in financial institutions’ interests to be
“too big to fail” as they have to carry huge amounts of capital against the risk of failure, which
makes enormous financial institutions cost ineffective. While massive banks have a competitive
advantage in the market, the regulatory rules instituted in 2015 heavily encourage large banking
institutions to break up. This is better for the borrowers, as they benefit from a competitive market
and a more regulated banking industry. This being said, borrowers must stay vigilant, as big banks
without enough capitalization may not completely disappear with the new regulations, and
regulations can always change with new legislation.

1. Which of the following is NOT brought by the author as an example of a supporter of


the concept that certain corporations were “too big to fail”?

a. The corporations themselves


b. Certain financial institutions
c. Congress
d. The American president
e. Mortgage consumers

2. The author of the passage suggests which of the following as the main issue that has
been resolved by the Federal Reserve and other financial overseers after the 2008
financial crisis?

a. The lack of regulatory oversight


b. The demand for mortgage-backed securities
c. Homeowners’ lack of financial means
d. The loss of employment
e. The occurrence of unforeseen circumstances
What the government bought was 700 billion dollars of illiquid mortgage-backed securities that the
banks should not have lent to consumers in the first place, but were able to because of a lack of
regulatory oversight.

A: A lack of regulatory oversight  leading to  B: 700 billion dollars of illiquid mortgage-backed


securities shouldn’t be lent to consumer

Must stop A to prevent the happen of B

3. Which of the following, if true, would provide the most support for the author’s views
on the influence of regulatory rules instituted in 2015?

a. Larger banks offer their customers lower loan rates.


b. Larger banks offer their customers higher deposit rates.
c. Regardless of their size, adjacent branches of rival banks offer identical loan rates.
d. Since the beginning of 2016, three major banks in the US have gone bankrupt.
e. Any change in the regulatory rules instituted in 2015 comes into effect only 3 years after it is
made.

Big banks carry huge amount of capitals  along with huge costs  cost ineffective  bankrupt

4. It can be inferred from the passage that which of the following will most probably
happen when homeowners with underwater loans sell their houses?

a. The profit on buying and selling the houses will be less than the market average.
b. The selling prices of the houses will be less than the price they paid for the houses.
c. Their mortgages on the houses will be larger than the profit on buying and selling the houses.
d. The selling prices of the houses will be less than the cost of the mortgages.
e. The losses from buying and selling the houses will be larger than the value of their mortgages.

5. All of the following appear in the passage as government regulatory actions, EXCEPT
for

a. Supporting financial institutions


b. Supervising the banking system
c. Legislating financial laws
d. Promoting market competition
e. Insuring the banking system

Insure (v) : bảo đảm, bảo hiểm

Disastrous (adj) : tai hại, thảm khốc

Bail (v) : bảo lãnh

Illiquid (adj) : kém thanh khoản

Vigilant (adj) : cảnh giác, cẩn thận


Dec 2 – 2

It comes as no surprise that there is little consensus among economists who study the prospects for
the new European Central Bank (ECB). Some are overwhelmingly enthusiastic about the ECB's
potential to eliminate exchange-rate currency fluctuations among its eleven member economies,
while others are convinced that cultural differences and inevitable squabbles over monetary policy
doom the central bank to failure.

The pessimists have a valid point. The ECB's primary goal, as specified by the Maastricht treaty, is to
bring about price stability within its area of sovereignty. This stability, however, is predicated on
reducing the current incongruity of prices throughout the region. Currently, though, there are too
many cultural and financial variables that prevent prices from coming into alignment anytime soon.

For a start, the prices of many products sold in Europe already vary from country to country by large
margins. The average cost of a pair of blue jeans is 34 percent higher in Germany than it is in Italy.
Some reasons for this can be remedied through legislation. lf Germany were to repeal its law
prohibiting supermarkets from stocking aspirin on their shelves. for example, aspirin prices might
come in line with the rest of the European as Union. But most price differentials derive from
seemingly immutable standards of living. Personal incomes in Spain and Portugal are well below the
European average, as they have been for decades. Thus, prices stay lower to accommodate weaker
buying power. And since Scandinavian countries show no sign of adopting the high demand for
coffee products that exists in France and Italy, coffee prices will remain much higher in southern
Europe than in the north.

Until now, Europe's different currencies have done well to divert consumers' attention from these
price differentials. But now that Europe has adopted its one currency. the euro, Europeans are more
likely to seek out the best price regardless of geography. This could mean depressed localized
economies of scale. Many countries will not recover from this situation in the short term.

Fortunately for the ECB‘s architects, there are two extraordinary financial models to study and
replicate: the German Bundesbank and America‘s Federal Reserve Board. Each country within the
European Union, regardless of its population or the size of its gross domestic product, will be
represented equally in the ECB's governing council, which will dictate the policies that the various
national central banks will implement. As long as the lines of communication remain open and
country representatives keep an open mind toward what may become a radical overhaul within
European commerce. the euro and the ECB have a fighting chance to survive.

1. The primary purpose of the passage is to

A. compare the ECB to other financial governing bodies


B. assert that the European countries that have not yet embraced the euro are destined to so do
before too long
C. provide support for the theory that Europe's transition to a central bank will have
many obstacles
D. enumerate several examples of the constantly changing global financial climate
E. prove that the ECB will never bring about the price stability on which its survival so clearly
depends
2. in the passage, the author is primarily concerned with which of the following?

A. comparing two different approaches to resolving a conflict


B. expressing dismay over a poorly reasoned conclusion
C. presenting information and offering different ways to interpret it
D. describing a problem that will not be solved easily
E. defending an established policy

3. Which of the following best describes the organization of the passage?

A. Two historical backgrounds are compared and contrasted.


B. A problem is identified. discussed, and resolved.
C. A theory is discussed. and evidence is introduced to condemn it.
D. A paradox is described. and one side is shown to be more dominant than the other.
E. A debate is introduced, and supporting evidence for both sides is presented.

4. Which of the following best describes the relation of the last paragraph to the
passage as a whole?

A. It advances an argument to be debated further.


B. it outlines a process to be reexamined,
C. It adds some positive points to an otherwise negative outlook.
D. It reinforces a theory that is not easily disproved.
E. lt poses a question that as yet has no answer.

5. The author‘s attitude toward the success of the ECB can best be described as

A. indifference
B. hostility
C. skepticism
D. amusement
E. admiration

6. The author most probably mentions the Bundesbank in order to

A. suggest an alternate composition of the ECB's governing council


B. compare its fiscal policies to those of the Federal Reserve Bank
C. provide an example of a financial institution to which the ECB will never measure up
D. indicate that there are other established fiscal entities for the ECB to emulate
E. urge the German government to ease its restrictions on supermarkets

7. The passage most clearly implies which of the following?

A. Countries whose economies are depressed by the price wars following the arrival of the euro will
eventually rebound and prosper.
B. The average German or Italian has more disposable income than does the average Portuguese or
Spaniard.
C. Were it not for the Maastricht treaty. the ECB would never have come about.
D. The arrival of the euro has not solved the problem of price disparity in Europe.
E. The governing council may have to alter its plan of equal representation to account for the
contributions of each country to Europe’s overall gross domestic product.

8. The passage suggests that the author believes which of the following?

A. Stabilizing the euro within global currency markets is the first step toward its legitimacy as a
viable saving vehicle.
B. The cultures of the various European countries will begin to grow together soon after the euro
completely supplants all the separate currencies that are now in use.
C. Multiple factors will prevent price stability from happening anytime soon, and it
probably won't happen at all.
D. The Bundesbank should assume all financial responsibilities for the European Union until the ECB
becomes fully operational.
E. The gross domestic product of a nation should determine the number of delegates that nation
sends to the ECB's governing council.

9. Which of the following, if true, would most clearly weaken the chances of the ECB's
survival?

A. Federal Reserve Board governors have no experience with having to reconcile various
languages and cultures into their financial policies.
B. The prices for most cosmetics in Belgium and The Netherlands are virtually identical.
C. Some of the legislation that is necessary for price reform will take years to put into effect.
D. As prices fall in Russia. which is not part of the European Union, commercial entities in Europe
will buy Russian raw materials in larger quantities.
E. The term of service for each member of the ECB's governing board is only three years.

Squabble (n) : sự cãi nhau, gây lộn

Incongruity (n) : không thống nhất

Council (n) : hội đồng

Dictate (v) : ra lệnh, bắt buộc

Overhaul (n) : đại tu, xem xét lại

Radical (adj) : triệt để


Dec 3 – 1

This report presents the results of a study conducted for the National Highway Traffic Safety
Administration (NHTSA) to assess the effects of two programs that were implemented to reduce the
incidence of aggressive driving. The programs were conducted by the Marion County Traffic Safety
Partnership (Indianapolis, Indiana), and The Tucson, Arizona, Police Department.

Samples of vehicle speed, collected unobtrusively in the special enforcement zones, and crash
incidence served as the primary measures of the programs' effect. The total number of crashes in
the Marion County special enforcement zones increased by 32 percent, compared to the same six-
month period one year earlier; the number of those crashes with primary collision factors (PCFs)
associated with aggressive driving increased by 41 percent. That is, the total number of crashes
increased, but the crashes with aggressive driving PCFs increased at a greater rate. The change in
proportion of crashes with the target PCFs provides a better measure than crash frequency because
it eliminates the effects of changes in traffic volume and other factors that might have contributed
to the overall increase in crash incidence.

In this regard, the Marion County zones experienced a six percent increase in the proportion of all
crashes with aggressive driving PCFs, despite the extensive publicity and special enforcement
efforts. The number of crashes in Tucson’s special enforcement zones increased by ten percent, but
the number of crashes with aggressive driving PCFs increased by less than one percent. More
important, the proportion of all crashes with target PCFs decreased by eight percent. That is, crash
incidence increased overall in Tucson’s zones, but the proportion of those crashes with aggressive
driving PCFs declined.

The original purpose of this report was to present, rather than compare, the two aggressive driving
programs. However, comparisons are inevitable and the substantially different results of the two
programs require an explanation. First, it is important to understand that it is impossible to control
all of the variables that could influence the outcome of a study when conducting large-scale quasi-
experiments, such as the programs described in this report. Highway maintenance projects and
large residential developments that increase traffic volumes on surface streets are examples of
unexpected and uncontrolled variables that can affect dependent measures in a field study involving
driver behavior. Every attempt was made to identify and control relevant variables within each
program, but the research was not originally designed to support systematic comparisons between
the two programs. Marion County, Indiana, and Tucson, Arizona, are different in many ways, and it
is possible that some of those differences could be responsible for the differential results reported
here.

1. What is the main idea of this passage?

(A) The causes and effects of road rage


(B) A study on two aggressive driving programs
(C) A comparison of Marion County, Indiana to Tucson, Arizona
(D) A new proposal for highway maintenance
(E) The recent increase in fatal traffic accidents
2. What is the primary purpose of this passage?

(A) To mediate between two traffic research syndicates


(B) To identify which factors have the most influence on PCFs
(C) To present a theory explaining how location affects collision frequency
(D) To defend the validity of a study on two aggressive driving programs
(E) To compare and discuss the results of a study of aggressive driving programs

3. Which of the following is an accurate summary of the passage?

(A) This passage examines the science of human behavior using the example of a study performed
in Marion County, Indiana about traffic accidents. It suggests that certain human behaviors can have
adverse effects on traffic conditions and even cause major traffic accidents. The results of the study
are discussed, and suggestions for reducing aggression are presented.
(B) This passage explores the relationship between location and aggressive driving. The drivers who
participated in a study in Indiana were more likely to get into traffic accidents because of certain
features of the area, such as residential developments and increased traffic volume. The passage
then introduces the finding that aggressive driving does not cause nearly as many traffic accidents
in Tucson, Arizona.
(C) The passage presents the results of two studies on aggressive driving behavior.
Although each study observed that the number of auto crashes increased, the
proportion of crashes caused by aggressive driving rose in one study and declined in the
other. Inherent differences in the locations of the two studies are discussed, and the
author suggests that these differences affect the studies' results.
(D) The passage attempts to defend the unpopular theory that aggressive driving is the leading
cause of accidents in America. Evidence linking aggressive driving to fatal car accidents is given, and
two strategies are proposed to counteract the effects of aggression: providing counseling for drivers
and creating "special enforcement zones" where police can stop aggressive drivers.
(E) The passage theorizes that PCFs (Primary Collision Factors) have a major effect on traffic
accidents, increasing their number by a large percentage each year in some places. One supporting
example of this phenomenon is then explored: the results of a study that links a variety of variables,
including location, traffic volume, and the creation of "special enforcement zones," to an increase in
the number of traffic accidents.

4. What attitude is expressed by the author of this passage?

(A) Mournfulness for the victims of traffic accidents


(B) Objective commitment to scientific accuracy
(C) Bewilderment at unexpected results
(D) Enthusiasm for increased law enforcement
(E) Disappointment in aggressive drivers

5. The phrase might have contributed at the end of the second paragraph contributes to
the tone of the passage in which of the following ways?

(A) It reinforces the timid, uncertain tone the author takes toward the research.
(B) It exemplifies the carefully analytical tone the author takes.
(C) It evokes the sarcastic tone present throughout the second paragraph.
(D) It marks a shift from certainty to uncertainty in the author's tone.
(E) It helps create suspense and foreboding in the passage.

Incidence (n) : tỷ lệ mắc

Unobtrusively (adv) : không phô trương


Dec 4 – 1

To meet the specific needs of a small, localized program like that of the PFF Fellows, an evaluation
should employ the case study design. A predominantly qualitative research method, the case study
is designed to examine a subject in its natural setting and, in our case, to provide an appreciative,
focused assessment of the individual concerns and idiosyncrasies of the program under
consideration. By virtue of engaging with the people involved with the program itself, the case study
can offer a respectful, expressive, sympathetic perspective on a program. Attending to the
community within a program as it does, the case study method of interviews, surveys, and other
anecdotal feedback instruments is ideal for a program of moderate scope and experiential nature.
Since its approach is qualitative, the case study provides descriptive data that lead to an increased
understanding of the program among all stakeholders, and can help facilitate discussion and
relationship-building during the consideration of results. A program of modest size and close
connection is accordingly a fine candidate for a case study evaluation.

The case study design was selected for two principal reasons: first, because the PFF Fellows
program is not being compared with any other group – this is a formative evaluation seeking
information that will improve the Fellows program, not a summative evaluation comparing its merit
to that of other similar and potentially implementable programs; second, because both the PFF
Fellows program’s own measures of success and the key questions guiding this evaluation (how well
prepared did faculty feel, how satisfied are they with their current positions and their career arcs,
etc.) are primarily qualitative themselves.

Other facets of the program contribute to making a case study design feasible. Since this
evaluation’s foremost concern with the data is that they bear directly on local aspects of the PFF
Fellows program, generalizability of results is not paramount. Also, since this evaluation is assessing
the program from the perspective of present and past participants, it is not an experiment;
pretreatment data are unnecessary when there is to be no treatment. The case study design
similarly favors programs where processes are to be scrutinized, and in this particular evaluation,
the approach, intervention, and guidance of the PFF Fellows program, not the value of the product,
are to be explored. Finally, the PFF Fellows program is young and still developing; consequently, an
evaluation that assesses which elements of the program have proven beneficial and which yet stand
to be improved would fit the current state of the program, introducing few pressures or outside
influences into the program itself and allowing for the naturalistic engagement so critical to a fruitful
qualitative evaluation.

1- Which of the following could most reasonably be inferred from this passage?

A- an evaluation of the PFF Fellows program would benefit from naturalistic


engagement
B- a summative evaluation would add to a complete assessment of the PFF Fellows program
C- a formative evaluation is not appropriate when evaluating a program of vast scope
D- a proper evaluation of the PFF Fellows program will require pretreatment data
E- evaluation is unlikely to yield meaningful results when applied to major sociocultural institutions
2- What is the primary purpose of this passage?

A- to recommend an approach
B- to criticize an approach
C- to define a method
D- to undermine a tradition
E- to identify a candidate for research

3- Which of the following best describes the function of the third paragraph in this
passage?

A- to voice a prominent concern


B- to answer a number of possible objections
C- to explain the results of a study
D- to illustrate a method already introduced
E- to rebut critics of the author

Qualitative (adj) : định tính, thuộc về tính chất

Facet (n) : khía cạnh

Pre-treatment (adj) : tiền xử lý

Scrutinize (v) : xem xét kỹ lưỡng

Engagement (n) : tham gia

Naturalistic (adj) : tự nhiên


Dec 4 – 2

While there is no guarantee that increased investment in achieving energy independence will solve
all of America’s unemployment problems, the results so far have demonstrated that it will definitely
help the cause. Under President Obama's watch, such increased investment has so far created 1.5
million jobs and has demonstrated the potential to create many more.

After a four-decade decline in oil production, the U.S. is now producing enough oil to serve more
than half of our oil needs. This has the potential to free us from our addiction to foreign-sourced
barrels, particularly if we utilize our dramatically larger and cheaper reserves of natural gas, which
now costs the equivalent of less than $15 per barrel, versus the $100-plus per barrel of oil we
import from the Middle East. The money saved by the increased use of natural gas has helped in
greater investments and has created more than 75,000 jobs domestically.

Moreover, the president’s policies have motivated companies to invest more in clean energy to the
extent that American companies make over 75% of all venture investments in clean technologies.
Overall, because of U.S. public and private investments in clean energy—including renewables,
efficiency, transportation, and infrastructure—the clean economy grew by 8.3% from 2008 to 2009,
even during the depths of the recession. Even though several technologies, such as solar power, are
still not as cost-competitive as imported oil, expanding these clean-energy investments is good
economics as they will help preserve and expand America's middle class, because energy
investments are a particularly effective method of "insourcing" manufacturing jobs, which in turn
spur jobs in invention, installation, and maintenance.

1.From the passage which of the following can be inferred about imported oil?

A. Most of the imported oil comes from the Middle East.


B. It is cheaper on a per barrel basis than natural gas.
C. It is not as cost competitive as solar energy.
D. Does not create the same number of jobs per unit of energy as solar power does.
E. Its unit price has increased over the past decade.

2. The author of the passage is primarily concerned with

A. Demonstrating that increased investment in achieving energy independence will


create more jobs.
B. Proving that the upside of investing in achieving energy independence is not limited to creating
jobs.
C. Advocating that America must invest in alternate renewable energy to attain Technology
independence.
D. Proving that America can tackle unemployment by lowering the cost of foreign energy.
E. Lobbying for a greater proportion of GDP to be spent on achieving energy independence.
Dec 5 – 1

Modern architecture has been criticized for emphasizing practical and technical issues at the expense
of aesthetic concerns. The high-rise buildings constructed throughout the industrialized world in the
1960s and 1970s provide ample evidence that cost-efficiency and utility have became the overriding
concerns of the modern architect. However, Otto Wagner’s seminal text on modern architecture,
first published in Germany in 1896, indicates that the failure of modern architecture cannot be
blamed on the ideals of its founders.

Wagner’s Modern Architecture called for a new style based on modern technologies and models of
construction. He insisted that there could be no return to traditional, preindustrial models; only by
accepting wholeheartedly the political and technological revolutions of the nineteenth century could
the architect establish the forms appropriate to a modern, urban society. “All modern creation,”
Wagner wrote, “must correspond to the new materials and demands of the present…must illustrate
our own better, democratic, self-confident, ideal nature,” and must incorporate the new “colossal
technical and scientific achievements” of the age. This would indeed seem to be the basis of a
purely materialist definition of architecture, a prototype for the simplistic form-follows-function
dogma that opponents have identified as the intellectual basis of modern architecture.

But the picture was more complex, for Wagner was always careful to distinguish between art and
engineering. Ultimately, he envisaged (to have a mental picture of especially in advance of
realization “envisages an entirely new system of education”) the architect developing the skills of the
engineer without losing the powers of aesthetic judgment that Wagner felt were unique to the artist.
“Since the engineer is seldom a born artist and the architect must learn as a rule to be an engineer,
architects will in time succeed in extending their influence into the realm occupied by the engineers,
so that legitimate aesthetic demands can be met in a satisfactory way.” In this symbiotic relationship
essential to Modernism, art was to exercise the controlling influence.

No other prospect was imaginable for Wagner, who was firmly rooted as a designer and, indeed, as
a teacher in the Classical tradition. The apparent inconsistency of a confessed Classicist advising
against the mechanical imitation of historical models and arguing for new forms appropriate to the
modern age created exactly the tension that made Wagner’s writings and buildings so interesting.
While he justified, for example, the choice of a circular ground plan for churches in terms of optimal
sight-lines and the technology of the gasometer, the true inspiration was derived from the
centralized churches of the Italian Renaissance. He acknowledged as rationalist that there was no
way back to the social and technological conditions that had produced the work of Michelangelo or
Fischer von Erlach, but he recognized his emotional attachment to the great works of the Italian
Renaissance and Austrian Baroque.
1. Which one of the following best expresses the main idea of the passage?

(A) Modern architecture has been criticized for emphasizing practical and technical issues and for
failing to focus on aesthetic concerns.
(B) Critics have failed to take into account the technological innovations and aesthetic features that
architects have incorporated into modern buildings.
(C) Wagner’s Modern Architecture provides architects with a chronicle of the origins of modern
architecture.
(D) Wagner’s Modern Architecture indicates that the founders of modern architecture
did not believe that practical issues should supersede the aesthetic concerns of the past.
(E) Wagner’s seminal text, Modern Architecture, provides the intellectual basis for the purely
materialistic definition of modern architecture.

2. According to the passage, Wagner asserts which one of the following about the roles
of architect and engineer?

(A) The architect should make decision about aesthetic issues and leave decision about technical
matters to the engineers.
(B) The engineer has often developed the powers of aesthetic judgment previously thought to be
unique to the architect.
(C) The judgment of the engineer should be as important as the judgment of the architect when
decisions are made about aesthetic issues.
(D) The technical judgment of the engineer should prevail over the aesthetic judgment of the
architect in the design of modern buildings.
(E) The architect should acquire the knowledge of technical matters typically held by the
engineer.

3. The passage suggests that Wagner would be LEAST likely to agree with which one of
the following statements about classical architecture and the modern architect?

(A) The modern architect should avoid the mechanical imitation of the models of the Italian
Renaissance and Austrian Baroque.
(B) The modern architect cannot design buildings appropriate to a modern, urban
society and still retain emotional attachments to the forms of the Italian Renaissance
and Austrian Baroque.
(C) The modern architect should possess knowledge of engineering as well as of the architecture of
the past.
(D) The modern architect should not base designs on the technological conditions that underlay the
design of the models of the Italian Renaissance and Austrian Baroque.
(E) The designs of modern architects should reflect political ideals different from those reflected in
the designs of classical architecture.
4. The passage suggests which one of the following about the quotations from Modern
Architecture cited in the second paragraph?

(A) They represent the part of Wagner’s work that has had the least influence on the architects who
designed the high-rise buildings of the 1960s and 1970s.
(B) They describe the part of Wagner’s work that is most often evoked by proponents of Wagner’s
ideas on art and technology.
(C) They do not adequately reflect the complexity of Wagner’s ideas on the use of
modern technology in architecture.
(D) They reflect Wagner’s active participation in the political revolutions of the nineteenth century.
(E) They provide an overview of Wagner’s ideas on the relationship between art and technology.

This would indeed seem to be the basis of a purely materialist definition of architecture, a
prototype for the simplistic form-follows-function dogma that opponents have identified as the
intellectual basis of modern architecture.

5. The author of the passage states which one of the following about the concerns of
modern architecture?

(A) Cost-efficiency, utility, and aesthetic demands are the primary concerns of the modern architect.
(B) Practical issues supersede aesthetic concerns in the design of many modern
buildings.
(C) Cost-efficiency is more important to the modern architects than are other practical concerns.
(D) The design of many new buildings suggests that modern architects are still inspired by
architectural forms of the past.
(E) Many modern architects use current technology to design modern buildings that are aesthetically
pleasing.

Modern architecture has been criticized for emphasizing practical and technical issues at the
expense of aesthetic concerns

6. The author mentions Wagner’s choice of a “circular ground plan for churches” (line
54) most likely in order to

(A) provide an example of the kinds of technological innovations Wagner introduced into modern
architecture
(B) provide an example of Wagner’s dismissal of historical forms from Italian Renaissance
(C) provide an example of a modern building where technological issues were much less significant
than aesthetic demands
(D) provide evidence of Wagner’s tendency to imitate Italian Renaissance and Austrian Baroque
models
(E) provide evidence of the tension between Wagner’s commitment to modern
technology and to the Classical tradition
7. The passage is primarily concerned with

(A) summarizing the history of a debate


(B) explaining a traditional argument
(C) describing and evaluating a recent approach
(D) justifying a recent criticism by presenting new evidence
(E) supporting an assertion by discussing an important work

Envisage (v) : suy xét


Dec 5 – 2
Sea turtles, reptilian amphibians that are classified into seven distinct sub-species, are quickly
beginning to rank among some of the most endangered species on the planet. These ancient
reptiles are vital to Earth’s ecosystem because they feed on botanical life that grows on sea beds
and thereby help maintain the natural habitat of several other marine life forms. Further, unhatched
sea turtle eggs provide nutrients for plants that grow beneath the surface of the sea, which in turn
strengthen sand dunes through their root systems and help prevent beaches from eroding.

Ironically, human attempts to preserve the marine environment in which sea turtles live are causing
the species’ numbers to dwindle even faster. In an effort to protect sea turtles that nest on the
beaches, in some popular fishing areas local authorities have placed restrictions on the amount of
fishing in order to protect the turtles from being caught in fishing nets and drowning. However, the
reduction in the amount of fishing in these areas has led to an increase in the numbers of the giant
cuttlefish, which feed on the same underwater plants as the sea turtles do. The subsequent
decrease in the turtles’ food source, particularly the sea grass that is part of the turtles’ staple diet,
has not only led to an alarming reduction in the number of sea turtles in these areas but also
affected the balance of the marine ecosystem in these regions.

1. The passage is primarily concerned with

A. questioning certain human activities that are causing damage to not only the local fishing
communities but also to a vital endangered species
B. outlining the reasons behind the importance of a certain species to the Earth’s ecosystem
C. establishing the relevance of a species and analyzing how a solution to protect it
might be defeating its very purpose
D. arguing that sometimes plans to rectify situations may actually cause more harm than good
E. explaining how some attempts to reduce fishing have not been successful

2. The author suggests which of the following about sea turtles?

A. An increase in the number of sea turtles will probably lead to a decrease in the number of giant
cuttlefish.
B. Restrictions in the amount of fishing in some areas have led to an increase in illegal poaching of
sea turtles.
C. In some areas, sea turtles would probably be less endangered if giant cuttlefish
populations were lower in number.
D. The plants that form part of the diet of sea turtles are found only on the seabed.
E. Sea turtles are more endangered than turtle species that live on land.

3. Which of the following CANNOT be inferred from the passage?

A. The habitat of some marine organisms is impacted by the existence of the sea turtles.
B. The staple diet of two marine organisms can be the same.
C. Even without feeding on plants, sea turtles can be useful to the environment they are born in.
D. Some attempts to help the endangered species of sea turtle have had effects that had not been
predicted.
E. The giant cuttlefish were being caught in greater numbers before certain restrictions on fishing
were placed in some areas.

These ancient reptiles are vital to Earth’s ecosystem because they feed on botanical life that grows
on sea beds and thereby help maintain the natural habitat of several other marine life forms.
The subsequent decrease in the turtles’ food source, particularly the sea grass that is part of the
turtles’ staple diet, has not only led to an alarming reduction in the number of sea turtles in these
areas but also affected the balance of the marine ecosystem in these regions.
In the passage we only know that the staple diet of sea turtle is sea grass, not
mentioned it is also giant cuttlefish’s staple diet.

4. The author refers to the giant cuttlefish in order to

A.explain how some species can thrive despite human activities


B. provide an example of a marine organism that forms part of the staple diet of another marine
organism
C. identify a species that is highly endangered
D. describe an event that has led to more than one adverse effect
E. argue in favor of a way in which a particular species can be protected
Staple (adj) : sản phẩm chính
Dec 6 – 1
Forget hostile aliens. According to a forthcoming book by noted astrophysicist Egbert Larson, the
intrepid humans who first attempt interstellar space travel will face far more daunting challenges
before they ever meet the Little Green Men. Larson begins with the problem of relativistic time
dilation. If you travel all the way to Alpha Centauri, you’d like to come back and tell your friends
about it, right? It’s not too likely to happen, though. If Mr. Einstein was right about relativity—and
we’re not going to say he wasn’t—then time slows down when you approach the speed of light. A
person traveling at any velocity near the speed of light will age only days for every week, month, or
even year that passes on earth. Relativity does not present a problem for interstellar space travel,
per se, but it does mean that interstellar civilizations or even just interstellar communications will
require a mind-boggling amount of calendar juggling.

Did we mention that you’d have to travel at near the speed of light? That’s because the distance
between stars is so vast that even if you could travel at the speed of light— which, Larson reminds
us, you can’t—it would take more than four years to reach our closest star neighbors, Alpha Proxima
and Alpha Centauri, and decades or centuries to reach the other stars in our “immediate
neighborhood.” And if you tried to accelerate directly to the speed of light like they do in the movies,
you’d be instantly splattered on the back of your theoretical spacecraft. Achieving anything close to
light speed will require sustained acceleration at a level that human bodies can withstand—say, a
crushing two gravities—for over a year. Better hope somebody brings some chips.

Speaking of chips, food is going to be a problem. Since it is economically, if not physically,


impossible to accelerate 200 years’ worth of food to nearly the speed of light, and since you’re not
likely to find any grocery stores along the way, someone will have to figure out how to make food in
space. Keeping a crew alive on the way turns out to be the trickiest part of all. Once you’ve got the
nearly impossible physics of space travel worked out, you still have to figure out the chemistry and
biology of keeping your air and water clean and keeping your crew fed and safe from radiation and
infection, and—did we mention the 200 years?—you’ll probably need several generations of crew
members to complete the trip. Ever been on a bus for more than 24 hours? It’s not a pretty picture.

We applaud Larson for his insightful writing and his scrupulous attention to scientific detail. For
those of you seeking a cold, hard look at the reality of interstellar space travel, this is a stellar read.
But be warned: Larson doesn’t let you down gently. For those of you sincerely hoping to beam up
with Scotty—and you know who you are—you might want to give this one a pass.

1. Which of the following would make the most appropriate title for this passage?

A. Going Boldly Where No One Has Gone Before: The Promise and Peril of Interstellar Space Travel
B. The Day the Earth Stood Still: Why Interstellar Space Travel Is Essential to Human Survival
C. The Wrath of Larson: Egbert Larson’s Quest to Build an Interstellar Spacecraft
D. Busted Flat in Beta Regulus: The Crushing Challenges of Interstellar Space Travel
E. Say It Isn’t So, Mr. Einstein: Egbert Larson’s Challenge to the Theory of Relativity
2. Based on the tone and content of the passage, it is most likely which of the following?

A. A book review in a journal intended for astrophysics professionals


B. A movie review in an entertainment industry publication
C. A book review in a science magazine aimed at a general audience
D. A book review in a newspaper
E. A transcript of a talk given at a science fiction convention dedicated to “The poetry of space”

3. The passage implies that all except which of the following could be threats to human
health during extended interstellar voyages?

A. Meteor impact
B. Radiation poisoning
C. Accelerating too fast
D. Starvation
E. Old age

4. According to the passage, which of the following will present the most difficult
challenge for humans attempting interstellar space travel?

A. Achieving velocities near the speed of light


B. Withstanding the acceleration necessary for traversing interstellar distances
C. Maintaining clean air and water on a journey that could last centuries
D. Accommodating the effects of relativistic time dilation
E. Enabling the humans on board to survive during the journey

Keeping a crew alive on the way turns out to be the trickiest part of all

Meteor (n) : thiên thạch


Dec 6 – 2

In regard to propaganda the early advocates of universal literacy and a free press envisaged only
two possibilities: the propaganda might be true, or it might be false. They did not foresee what in
fact has happened, above all in our Western capitalist democracies - the development of a vast
mass communications industry, concerned in the main neither with the true nor the false, but with
the unreal, the more or less totally irrelevant. In a word, they failed to take into account man's
almost infinite appetite for distractions. In the past most people never got a chance of fully
satisfying this appetite. They might long for distractions, but the distractions were not provided.
Christmas came but once a year, feasts were "solemn and rare," there were few readers and very
little to read, and the nearest approach to a neighborhood movie theater was the parish church,
where the performances, though infrequent, were somewhat monotonous. For conditions even
remotely comparable to those now prevailing we must return to imperial Rome, where the populace
was kept in good humor by frequent, gratuitous doses of many kinds of entertainment - from
poetical dramas to gladiatorial fights, from recitations of Virgil to all-out boxing, from concerts to
military reviews and public executions. But even in Rome there was nothing like the non-stop
distraction now provided by newspapers and magazines, by radio, television and the cinema. In
Brave New World non-stop distractions of the most fascinating nature (the feelies, orgy-porgy,
centrifugal bumblepuppy) are deliberately used as instruments of policy, for the purpose of
preventing people from paying too much attention to the realities of the social and political situation.
The other world of religion is different from the other world of entertainment; but they resemble one
another in being most decidedly "not of this world." Both are distractions and, if lived in too
continuously, both can become, in Marx's phrase, "the opium of the people" and so a threat to
freedom. Only the vigilant can maintain their liberties, and only those who are constantly
and intelligently on the spotcan hope to govern themselves effectively by democratic procedures. A
society, most of whose members spend a great part of their time, not on the spot, not here and now
and in the calculable future, but somewhere else, in the irrelevant other worlds of sport and soap
opera, of mythology and metaphysical fantasy, will find it hard to resist the encroachments of those
who would manipulate and control it. In their propaganda today's dictators rely for the most part on
repetition, suppression and rationalization - the repetition of catchwords which they wish to be
accepted as true, the suppression of facts which they wish to be ignored, the arousal and
rationalization of passions which may be used in the interests of the Party or the State. As the art
and science of manipulation come to be better understood, the dictators of the future will doubtless
learn to combine these techniques with the non-stop distractions which, in the West, are now
threatening to drown in a sea of irrelevance the rational propaganda essential to the maintenance of
individual liberty and the survival of democratic institutions.

Adapted from the introduction to Brave New World, A Huxley (1931)

1. The author would be most likely to agree that propaganda

A. can serve a vital function in democracy


B. is concerned mainly with the irrelevant
C. is now combined with entertainment
D. is universally recognized as a danger
E. needs constant vigilance to avoid

2. The “early advocates of universal literacy” (line 1) are mentioned as


A. advocates of propaganda
B. opponents of an idea that the author thinks is correct
C. proponents of an idea that the author wishes to counter
D. people who made wrong predictions about freedom of the press
E. social commentators unaware of man’s appetite for distractions

3. The author refers to “Brave New World” as a fictional example of a society in which

A. non-stop distractions are the main instrument of government policy


B. people are totally unaware of political realities
C. entertainment is used to keep people from full awareness of social realities
D. entertainment resembles religion in its effects on the masses
E. non-stop entertainment is provided as it was in Rome

4. By “intelligently on the spot” (line 37) the author apparently means

A. alert to the dangers of propaganda


B. in a particular society at a particular time
C. in a specific time and place
D. conscious of political and social realities
E. deeply aware of current trends

propaganda (n) : tuyên truyền

Universal literacy (n) : phổ cập kiến thức

Envisage (v) : bạo dạn trước sự nguy hiểm. suy tưởng, suy xét

Appetite (n) : khao khát

Distraction (n) : giải trí, tiêu khiển

Feast (n) : tiệc, ngày lễ

Solemn (adj) : trang nghiêm, theo nghi thức

Parish (adj) : giáo xứ

Monotonous (adj) : đơn điệu

Gratutious (adj) : cho không, không lấy tiền

Recitation (n) : trì tụng, sự đọc thuộc lòng

Deliberately (adv) : xét đoán kỹ lưỡng

Opium (n) : thuốc phiện

Vigilant (adj) : cảnh giác

Encroachment (n) : lấn chiếm, lấn quyền

Rationalization (n) : hợp lý hóa

Arousal (n) : kích thích


Dec 7 – 1

The Romans—for centuries the masters of war and politics across Europe, Northern Africa, and Asia
Minor—have often been criticized for producing few original thinkers outside the realm of politics.
This criticism, while in many ways true, is not without its problems. It was, after all, the conquest of
Greece that provided Rome with its greatest influx of educated subjects. Admittedly, two of the
great disasters in intellectual history—the murder of Archimedes and the burning of Alexandria’s
library—both occurred under Rome’s watch. Nevertheless, a city that was able to conquer so much
of the known world could not have been devoid of the creativity that characterizes so many other
ancient empires.

Engineering is one endeavor in which the Romans showed themselves capable. Their aqueducts
carried water hundreds of miles along the tops of vast arcades. Roman roads, built for the rapid
deployment of troops, crisscross Europe and still form the basis of numerous modern highways that
provide quick access between many major European and African cities. Indeed, a large number of
these cities owe their prominence to Rome’s economic and political influence.

Many of those major cities lie far beyond Rome’s original province, and Latin-derived languages are
spoken in most Southern European nations. Again a result of military influence, the popularity of
Latin and its offspring is difficult to underestimate. During the centuries of ignorance and violence
that followed Rome’s decline, the Latin language was the glue that held together the identity of an
entire continent. While seldom spoken today, it is still studied widely, if only so that such masters of
rhetoric as Cicero can be read in the original.

It is Cicero and his like who are perhaps the most overlooked legacy of Rome. While far from being
a democracy, Rome did leave behind useful political tools that serve the American republic today.
“Republic” itself is Latin for “the people’s business,” a notion cherished in democracies worldwide.
Senators owe their name to Rome’s class of elders; Representatives owe theirs to the Tribunes who
seized popular prerogatives from the Senatorial class. The veto was a Roman notion adopted by the
historically aware framers of the Constitution, who often assumed pen names from the lexicon of
Latin life. These accomplishments, as monumental as any highway or coliseum, remain prominent
features of the Western landscape.

1. The author describes "two of the great disasters in intellectual history" in order to

A. establish a point directly related to the main argument


B. show that certain historical claims are inaccurate
C. demonstrate the importance of certain historical data
D. disprove the claims made by others with a different view
E. concede the partial accuracy of an opposing view

2. According to the passage, ancient Roman roads

A. connected places of military importance in ancient Europe


B. are engineering marvels unequaled in modern times
C. are similar in some respects to modern highways
D. were products of democratic political institutions
E. caused the development of modern European cities
3. According to the passage, which of the following accurately describes the Latin
language?

I. It spread in part due to Rome's military power


II. It is reflected in some modern political concepts.
III. It is spoken today in some parts of Europe.
A. I only
B. II only
C. I and II only
D. I and III only
E. II and III only

During the centuries of ignorance and violence that followed Rome’s decline, the Latin language was
the glue that held together the identity of an entire continent.

While seldom spoken today, it is still studied widely, if only so that such masters of rhetoric as
Cicero can be read in the original.

4. It can be inferred from the passage that the framers of the Constitution

A. were familiar with certain aspects of Roman government


B. were similar to the Roman elders
C. embraced the veto as the hallmark of Roman democracy

While far from being a democracy, Rome did leave behind useful political tools that serve the
American republic today.
D. overlooked Cicero's contributions to the theory of democracy
E. formed a government based on worldwide democracy

Prerogative (n) : đặc quyền

Senatorial (adj) : thượng nghị sĩ

Veto (n) : phủ quyết

Lexicon (n) : từ vựng

̣ i
Monumental (adj) : hoành tráng, vỉ đa

Coliseum (n) : đấu trường


Dec 7 – 2

By 1784, Wolfgang Amadeus Mozart was internationally renowned as the composer of Marriage of
Figaro, and consequently received a commission from the Prague Opera House to compose another
opera. The resulting product was Don Giovanni, which tells the tale of a criminal and seducer who
nevertheless evokes sympathy from audiences, and whose behavior fluctuates from moral crisis to
hilarious escapade.

While Don Giovanni is widely considered to be Mozart's greatest achievement, eighteenth century
audiences in Vienna — Mozart's own city— were ambivalent at best. The opera mixed traditions of
moralism with those of comedy— a practice heretofore unknown among the composer's works—
creating a production that was not well liked by conservative Viennese audiences. Meanwhile,
however, Don Giovanni was performed to much fanfare throughout Europe.

1. The primary purpose of the passage is to

(A) relate the story of a somewhat likable antihero.


(B) discuss how a work of art has been met by diverging responses.
(C) give a history of the work of Mozart.
(D) make a case for the renown of Don Giovanni.
(E) emphasize the moral aspects of a musical work.

2. The author mentions the mixing of "traditions of moralism with those of comedy"
primarily in order to

(A) explain a work's cool reception among a particular group of people.


(B) remind the reader of the plot of Don Giovanni.
(C) highlight a practice common in contemporary opera.
(D) argue for an innovative approach to opera.
(E) undermine a previously presented assertion.

3. It can be inferred from the passage that which of the following is true about the
response of Viennese audiences to Don Giovanni?

(A) The audiences preferred purely moralistic works.


(B) The response was unequivocally positive.
(C) They did not know that the composer was attempting to mix musical styles.
(D) The play's moral themes were offensive to Viennese audiences.
(E) To say that the response was "mixed" would be a generous interpretation.

While Don Giovanni is widely considered to be Mozart's greatest achievement, eighteenth century
audiences in Vienna — Mozart's own city— were ambivalent at best

Ambivanlent (adj) : mâu thuẫn trong cảm xúc, tư tưởng / vừa yêu vừa ghét
Dec 8 – 1

In 1868, Hayward was hit with a major earthquake that shook the East Bay and was one of the
most destructive quakes in California history, with a magnitude of 6.8 to 7.0 on the Richter scale. As
this earthquake happened almost a century and a half ago, many people may have forgotten about
it. But to forget an earthquake, especially this one, would be a mistake. The residents of California’s
East Bay should note that the last five earthquakes on the Hayward Fault were each 140 years
apart.

The Hayward Fault is a type of earthquake fault called strike-slip. In a strike-slip fault, the
movement of blocks along a fault is horizontal. In these earthquake zones, the phenomenon of force
released from the fault has a repetitive pattern of occurrence, which provides geologists with an
approximate timeframe for estimating upcoming earthquakes. Thus, an earthquake in California can
be expected before long, especially after the 1989 Loma Prieta quake near the San Andreas Fault
zone, which registered as 6.9 on the Richter scale. This scale, developed in the 1930s, is a base-10
logarithmic scale that defines magnitude as the logarithm of the ratio of the amplitude of the seismic
waves to an arbitrary, minor amplitude. Today, because of earthquake retrofitting, most homes in
the Bay Area are designed to withstand small-to-medium earthquakes. The majority, however,
cannot withstand a quake of 8.0 magnitude on the Richter scale. The material of a home’s
foundation, i.e. whether it is built on solid stone, dirt or sand, is an important determining factor in
how much movement the structure experiences while the earth shakes. A solid stone foundation
experiences significantly less vibration than does sand, which acts like water during a quake.

The biggest changes between 1868 and today, in terms of earthquake preparedness, stem from
urbanization, which has made earthquakes more threatening. Buildings are no longer limited to
single-story houses and shops, but include skyscrapers. To assist residents in high-risk earthquake
zones like the Hayward Fault, California has invested in ShakeAlert, a system designed to deliver a
quick warning to people who may be in harm’s way due to collapsing infrastructure in the case of an
earthquake. However, the ShakeAlert system notifies users only a few seconds in advance of an
earthquake. Thus, the residents of California must make sure they are individually fully prepared for
the upcoming earthquake.

1. Which of the following titles best summarizes the contents of the passage?

a. Technology for Earthquake Strikes


b. The Richter Scale: No Alert is Better
c. The 1868 Earthquake in Hayward Fault: Why It Matters Today
d. California Earthquakes: How to Measure Them
e. The Evolution of Earthquake Assessment

2. Which of the following is true of the next Hayward Fault earthquake?

a. It is expected to be less devastating than the last Hayward Fault earthquake.


b. It should be expected any time soon.

"Thus, an earthquake in California can be expected before long, especially after the 1989 Loma
Prieta quake near the San Andreas Fault zone, which registered as 6.9 on the Richter scale."
c. It is the reason for the development of an early warning system called ShakeAlert.
d. It is forecast to happen in about 140 years from now.
the time period is not defined for "from now"
e. Its magnitude is forecast to be between 6.8 and 7.0 on the Richter scale.

3. The author implies that ALL the following statements about earthquakes are true,
EXCEPT:

a. A house built on sand would probably have more structural damage after a quake than a house
built on granite.
b. California residents living in major earthquake zones should be aware of past earthquakes
because they tend to be cyclical.
c. While buildings are now retrofitted for earthquakes, the risk of damage is higher nowadays
because of skyscrapers.
d. Had the Loma Prieta quake happened today, most houses in California would have survived it.
e. The magnitude of the quake determines the amount of damage, thus the Hayward
quake of 1868 and the Loma Prieta quake had the same destructive impact.

4. The author mentions the homes’ foundations mainly to

a. give a baseline structural integrity measurement for the Richter scale.


b. understand the different possible impacts earthquakes can have on homes, based on their
foundational materials.
c. show that the Richter scale does not alone determine the impact of quake damage.
d. demonstrate the vulnerability of houses in earthquake zones.
e. give a demonstration of the impact of an 8.0 earthquake on different foundational surfaces.

5. The author implies ShakeAlert is flawed because it does NOT provide

a. long-term detection against quakes.


b. an alert for residents against structural damage in their homes after a quake.
c. calculation of the strength and velocity of the P-waves as they arrive.
d. analysis of the infrastructure of the buildings.
e. any protection to the user of the device.
Dec 8 – 2

The media‘s particular understanding of the ways of influence and decision-making in government
colours the way they describe political reality. It also defines their responsibility in reporting that
reality; contemporary reporters are in many ways the grandchildren of the Progressive muckrakers.

Few aspects of American politics reinforce this Progressive world-view as effectively as the American
way of campaign finance. In assuming that public officials defer to contributors more easily than
they do to their party, their own values, or their voting constituency, one has the perfect dramatic
scenario for the triumph of wealthy special interests over the will of majorities and the public
interest.

Much has been made recently about campaign finance reform. Various politicians and voters' rights
groups have petitioned for a reworking of the campaign finance laws that govern how political
candidates can solicit and spent money on their races for office.

"Bias" is a word with many meanings. It suggests a single explanation—one of conscious, even wilful
preference—for a range of instances in which the message misinterprets or misconveys the reality.
The media have been attacked as biased in a partisan direction by both Democrats and Republicans,
and from both the left and the right. To be sure, media partisanship was apparent in earlier times,
when the partisan press was little more than a propagandist for the party it favoured.

But that overtly biased style seems to have given way in the 20 century to a media more concerned
with gaining audience than political proselytes, and an electronic media fearful of government
regulation if it strays into political controversy. Few objective observers of, for instance, the
reporting of campaign finance would argue that conventional biases are operating here. Rather one
has to look to more intrinsic and ingrained forms, to the structural biases of American newspapers
and the political assumptions of their reporters, editors, and headline-writers. Structural biases are
rooted in the very nature of journalism—in its professional norms, in marketplace imperatives, in the
demands of communicating information to an unsophisticated audience.

Stories need identifiable actors, understandable activity, and elements of conflict, threat or menace.
They cannot be long, and must avoid complexity—must focus on the horserace rather than on the
substance of a campaign; on controversy, personalities and negative statistics rather than on
concepts. These define the "good" story. Systematic bias and political assumption, finally, meet in
an analytical conundrum. A systematic bias dictates that newspapers print stories that will be read.
But does the press publish the story because readers have been conditioned by newspapers to
accept and believe such accounts, or does it publish the story because of its conviction that it
represents political truth? Is there really any difference? Ultimately, the Progressive view of reality
becomes a part of the imperatives of publishing a newspaper.

1. In the course of presenting his arguments, the author suggests that structural biases
in American journalism result primarily—but not necessarily exclusively—from:

A. problems intrinsic to the publishing and marketing of newspapers.


B. suppositions of journalists about the integrity of public officials.
C. reporters‘ cynicism about the public‘s level of intelligence.
D. growing competition among newspapers for a shrinking audience.
E. increasing influence of foreign nations

2. According to the passage, which of the following would indicate structural biases
inherent in journalists‘ work?

A. An article that adheres loyally to Progressivist dictates


B. An article that successfully masks its biased opinions
C. An article that is informed by political sophistication
D. An article that is entertaining and easy to comprehend
E. An article that criticises the current government

Structural biases are rooted in the very nature of journalism—in its professional norms,
in marketplace imperatives, in the demands of communicating information to an unsophisticated
audience.
3. Which of the following best describes the "analytical conundrum" referred to in the
sentence, "Systematic bias and political assumption, finally, meet in an analytical
conundrum," in the last paragraph?

A. Newspapers promote Progressive ideas in which they do not believe.


B. Since systematic biases and political assumptions have similar effects, it is difficult to differentiate
their roles in journalistic publishing decisions.
C. Systematic biases and political assumptions exert contradictory and conflicting pressures on
newspaper publishers.
D. Readers‘ preferences for dramatic news accounts reflecting Progressive ideas, rather
than journalists‘ objective understanding of the political system, determine what is
published.
E. the confusion over what types of articles to publish in newspapers

Comprehend (v) : hiểu rõ

Solicit (v) : gạ gẫm, kêu nài, nài xin

Partisan (n) : đảng phái

Propagandist (n) : tuyên truyền viên

Menace (n) : mối đe dọa

Conundrum (n) : câu hỏi học búa

Imperative (adj) : mệnh lênh, tính bắt buộc


Dec 9 – 1

Cargo cults are religious movements that have appeared in tribal societies following interaction with
technologically advanced cultures whose representatives have arrived bearing manufactured goods,
or "cargo." These cults were known in the late nineteenth century, but arose in earnest in the years
following World War II, as members of tribal societies came into contact with radios, televisions,
guns, airplanes, and other goods brought to New Guinea and other Micronesian and Melanesian
countries as part of the Allied war effort. Members of native societies, having little knowledge of
Western manufacturing, found soldiers' explanations of the cargo's provenance unconvincing and
drew the conclusion that the "cargo" had come about through spiritual means. Some concluded that
the cargo had been created by the deities and ancestors of the native people, and that the
foreigners had attracted the cargo to themselves through trickery, or through an error made by the
deities and ancestors. Cargo cults arose for the purpose of attracting material wealth back to
its "rightful owners"via religious rituals that sought to mimic the actions of the foreigners in order to
attract cargo. The most famous of the cargo cults still exists on the island of Tanna in Vanautu,
where cult members have constructed elaborate airstrips and control towers intended to attract
airplanes, and where an annual celebration features barefoot soldiers in re-created U.S. Army
uniforms conducting military exercises. Cult members wearing "headphones" made of wood can be
observed speaking into "radios" made of coconuts and straw, mimicking actions observed by Allied
troops during the war.

Members of cargo cults commit the fallacy of confusing a necessary condition with a sufficient one.
It is true, of course, that an airstrip and a control tower are necessary for executing a safe landing
of a military airplane; they are not, of course, sufficient to attract an airplane in the first place.
Communicating with a radio tower may bring new supplies; speaking gibberish into a straw replica
of a radio, of course, will not bring the desired cargo. Thus, the term "cargo cult" has arisen as an
idiom in English to describe those who mimic the superficial appearance of a procedure without
understanding the underlying purpose, meaning, or functioning of that procedure.

In the book Surely You're Joking, Mr. Feynman!, physicist Richard Feynman dedicates a chapter to
"cargo cult science," the product of researchers who create the appearance of real science— even
with the fastidiousness of those who create a full-size Jeep from bamboo and straw— but without an
understanding of the underlying workings of real science. Feynman gives the example of
experiments involving running rats through a maze. In 1937, a researcher named Young discovered
that rats who had run a maze previously were using some hidden cue on subsequent runs that
invalidated the results of those trials. Through meticulous experimentation, Young discovered that
the rats could use the sounds made by the maze's floorboards to memorize positions within the
maze; when the maze was put on a floor of sand, this cue was removed, and future experiments
could be conducted untainted by the "floorboard problem." However, Young's research made no
impact whatsoever; other scientists— cargo cult scientists— went on running rats through the maze
just as they had before, publishing their results and going about the motions of science without, as
Feynman argues, actually doing science.

1. The primary purpose of the passage is to

(A) offer a suggestion for improving the results of scientific experiments.


(B) suggest that two disparate groups of people share similar logical errors.
(C) explain that cargo cults mistakenly confuse preceding events with causal events.
(D) argue that it is important for scientists to take into account the research of other scientists in
their fields.
(E) imply that the cargo cult members' mistakes in logic could be remedied through the scientific
method.

2. The passage suggests that resident of Tanna concluded that they were the "rightful
owners" of the "cargo" because

(A) they found stories of the goods' actual origins to be incredible


(B) they believed all possessions were created by deities
(C) they believed they were owed a debt by their ancestors
(D) Allied forces had given them the cargo
(E) guns and airplanes were unknown to them prior to World War II

Members of native societies, having little knowledge of Western manufacturing, found soldiers'
explanations of the cargo's provenance unconvincing and drew the conclusion that the "cargo" had
come about through spiritual means.

3. According to the information provided by the passage, which of the following would
critics such as Feynman most likely describe as practitioners of "cargo cult science"?

(A) Scientists who create a sufficient cause for an event rather than a necessary one.
(B) Residents of Tanna who attempt a scientific experiment with only primitive equipment.
(C) Researchers who are unsuccessful in causing a desired phenomenon to occur.
(D) Westerners who believe in supernatural phenomena despite overwhelming evidence to the
contrary.
(E) Scientists who receive recognition for their work without disclosing possible flaws in
the design of their experiments.

physicist Richard Feynman dedicates a chapter to "cargo cult science," the product of researchers
who create the appearance of real science— even with the fastidiousness of those who create a full-
size Jeep from bamboo and straw— but without an understanding of the underlying
workings of real science.

4. According to the passage, the similarity between cargo cult members and
practitioners of "cargo cult science" can most appropriately be described in which of the
following ways?

(A) Both use inappropriate equipment in trying to cause a phenomenon.


(B) Both refuse to accept the principles of the scientific method.
(C) Both adhere to processes that lack scientific rigor.
(D) Both have no logical basis for their actions. – too extreme
(E) Both would benefit from enhanced scientific education.

Rigor (n) : nghiêm ngặt

Cult (n) : sùng bái, cúng tế

Provenance (n) : nguồn gốc

Deity (n) : thần linh


Trickery (n) : mánh khóe, gian lận, lừa gạt

Mimic (v) : bắt chước

Elaborate (adj) : kỹ lưỡng

Airstrip (n) : đường bay

Fallacy (n) : ngụy biện

Gibberish (n) : vô nghĩa, khó hiểu

Superficial (adj) : hời hợt, không sâu sắc

Fastidiousness (n) : khó tính

Meticulous (adj) : tỉ mỉ, cẩn thận, kỹ lưỡng


Dec 9 – 2

Companies have long relied on the creation of task specific groups to address such pressing issues
as decision making, problem solving, and perhaps most importantly, task completion. Group
oriented task completion has many advantages; a cohesive group with an appropriate organizational
context, sufficient material support, and a clearly delineated goal can perform tasks at a far higher
level of productivity than can any individual worker. In fact, according to Hackman’s normative
model of group effectiveness, a successful synergy of organizational context and group design
factors can lead to levels of productivity far exceeding even the most optimistic company forecasts.

Unfortunately, not all groups achieve this positive synergy. Many fail to meet even modest
productivity goals. One explanation for this lack of success could involve what Bruce Tuckman
describes as the “storming,” or conflict, stage of group formation.Tuckman contends that a
successful group must pass through a stage of interpersonal confrontation before it can enter the
cohesion phase. After cohesion is achieved, a group can move into the performance mode of
operation.

However, group cohesion alone is by no means a guarantee of high productivity. A survey of over
two hundred groups of factory workers found that some groups that scored extremely well on the
cohesion scale had low levels of productivity. Researchers attribute this finding to group norms--
groups that allow conflict issues to become the norm will generally perform at lower productivity
levels while groups that can remain focused on the task at hand can more readily achieve the sort of
productivity predicted by Hackman.

1.The primary purpose of the passage is to

A .suggest that a particular approach is no longer useful.


B. explain why a particular strategy might fail.
C.discuss how a certain practice is implemented.
D.enumerate the advantages of a business strategy.
E.reject a long-standing view of a practice.

2. According to the author, a group that can perform tasks at a far "higher level of
productivity" requires all of the following EXCEPT:

A.A level of group unity.


B. A specified goal.
C. A group structure appropriate to the task.
D. A clearly defined leadership position.
E. Adequate resources.

Synergy (n) : sức mạnh tồng thể

Delineate (v) : phác họa

Normative (adj) : quy chuẩn, có giá trị

Cohesion (n) : sự gắn kết

Confrontation (n) : sự đối đầu

Enumerate (v) : liệt kê


Dec 10 – 1

The efficacy of standard clinical trials in medicine has recently become the subject of contentious
debate between those practitioners who maintain that such trials, despite admitted shortcomings,
still represent the best means we have for learning about the effects of pharmaceutical drugs on the
human body, and those who maintain that the current system of collecting knowledge of such
effects is but one possibility and most likely not the most efficacious one.

Gimley and Lebsmith, in their recent work, fall into the latter camp and indeed go further by
challenging the idea that the standard medical trials can yield meaningful information on
pharmacogenetics, or how a drug interacts with the human body. Gimley and Lebsmith’s foremost
criticism is that the effect of a drug differs depending on a person’s physiology. To be sure, there
are cohorts, or groups, that react to a drug in a specific manner, but clinical trials are unequipped to
identify such groupings. The main reason is that clinical trials are allied to the notion that the larger
the number of subjects in a study, the greater the validity of a drug, should it show any promise.

Therefore, even if a drug can exercise a marked effect on a subset of subjects within a trial, this
information will be lost in the statistical noise. Another criticism of Gimley and Lebsmith concerns the
very idea of validity. Pharmaceutical companies will run hundreds of trials on hundreds of different
medications. Given the sheer number of trials, a few are likely to yield positive results, even if there
is no demonstrable effect. Gimley and Lebsmith cite the fact that most pharmaceuticals that have
exerted a positive effect in the first round of testing are likely to fail in the second round of testing.

Gimley and Lebsmith argue that a more effective approach is to identify groups who exhibit similar
genetic subtypes. This very approach is currently in use in groups possessing a particular molecular
subtype of breast cancer. Furthermore, these groups are not only trying one specific drug, but also a
combination of such drugs, subbing them in and out to measure the effects on a subject, a
procedure Gimley and Lebsmith endorse. Nonetheless, such an approach is often both time-
consuming and costly. However, given the constraints of current medical trials, trials that target
subtypes— even if they do not yield any significant advances— will encourage a culture of
experimentation on how clinical trials are conducted in the first place.

1.What is the primary purpose of this passage?

A. To evaluate and survey diverse opinions on the current state of a field


B. To point out an oversight in the work of two scholars
C. To discuss a field of inquiry and several ways in which that field can be improved
D. To describe a current debate and the analysis of specific researchers
E. To assess the effect the work of two scholars has had in their field
2.Which of the hypothetical trials best parallels a shortcoming in clinical trials as
described by Gimley and Lebsmith?

A. A subset of patients displays a slightly negative reaction to a drug, but that outcome is masked by
the majority of subjects’ responses, which are neither positive nor negative.
B. A drug exercises a strong negative effect on a subgroup but this effect is masked by
the majority of subjects’ responses, which are neither positive nor negative.
C. The majority of patients in a trial exhibit a negative effect to a drug, yet the fact that a few
subjects exhibited a positive effect diminishes the intensity of the negative effect.
D. A trial targeting a particular genetic subtype struggles to recruit enough subjects to establish that
an observed effect of a drug would be valid across a larger population.
E. Several subjects in a trial exhibited positive effects from a drug used during an initial phase of an
experiment, but once another drug was substituted for the drug used in the initial phase, those
patients failed to exhibit any positive effects.

Therefore, even if a drug can exercise a marked effect on a subset of subjects within a trial, this
information will be lost in the statistical noise.

Marked effect can be both negative or positive effects

3.It can be most reasonably inferred that those on the side of the debate endorsing
standard medical trials are likely to believe which of the following?

A. Those favoring standard clinical trials believe that such trials represent the only means scientists
have of studying pharmacogenetics.
B. The experimental methods espoused by Gimley and Lebsmith may ultimately come to displace
those currently in use.
C. The most effective method of gathering information concerning pharmacogenetics
will not be without flaws.
D. Clinical trials that focus on a specific genetic subtype will be better able to isolate the effects of a
specific drug.
E. Many drugs exhibiting a positive effect on subjects in the first round of testing will fail to exhibit
the same positive effect in subsequent tests.

Efficacy (n) : hiệu quả

Clinical trial (n) : thử nghiệm lâm sàng

Efficacious (adj) : hiệu quả, công hiệu

Subset (n) : tập hợp con

Marked (adj) : rõ rệt

Subtypes (n) : Phân nhóm


Dec 10 – 2

Recent research into antibiotic-resistant bacterial strains suggests the need for a reexamination of
the frequency with which doctors prescribe antibacterial therapy. One study demonstrated, for
example, that most minor bacterial infections will resolve without treatment within 5 to 14 days of
onset of symptoms; a course of antibiotics might reduce that time frame by only 1 to 2 days. A
second study indicated that the incidence of “superbugs,” which have resistance to a wide variety of
antibacterial agents, is increasing significantly and that these bugs are more likely to spread among
those who have been treated with antibiotics within the past 5 years. In particular, researchers have
become alarmed by NDM-1 (New Delhi metallo-beta-lactamase), which is not a single bacterial
species, but a multiple-antibiotic-resistant enzyme capable of infecting other strains of bacteria.

It is true that the proliferation of superbugs likely owes a great deal to the mistaken prescription of
antibacterial treatment for viral infections, against which such treatment is ineffective, and to the
routine addition of antibiotics to livestock feed in order to increase meat yields. Additionally, it is
possible that ongoing research into the means by which resistance spreads among bacterial
communities may lead to a new generation of antibiotics to which bacteria are unable to develop
resistance. Yet these factors do not change the need for individual physicians to be more
circumspect about drug therapy when treating cases of true bacterial infection.

1. The passage is primarily concerned with

(A) discussing research into the symptoms of bacterial infections


(B) explaining a change in the frequency with which antibacterial therapy is prescribed
(C) contrasting the views of doctors and medical researchers with respect to prescribing drugs
(D) questioning the routine prescription of antibiotics for bacterial infections
(E) contending that physicians need to be more careful about distinguishing between viral and
bacterial infections

2. The research cited in the first paragraph suggests which of the following about
antibacterial therapy?

(A) It frequently leads to infection with NDM-1.


(B) It is not generally used to treat minor bacterial infections.
(C) It may help to reduce the incidence of “superbugs” that are especially hard to treat.

No mentioned that superbugs are hard to treat


(D) Reducing the rate at which such therapy is used would cause fewer bacteria to develop
resistance to antibiotics.
(E) Its short-term benefits, if they exist, may not outweigh the potential harm to the
broader population.

The paragraph does mention that such therapy might reduce the illness by one to two days. This
could be a short-term benefit, but the author minimizes this benefit and goes on to discuss a much
worse drawback (the superbug). That all fits with the simple story and the first paragraph.

It’s sometimes possible to find the right answer even if you don’t yet know why some of the wrong
ones are wrong. On the real test, pick (E) and move on. When you’re studying, go back afterwards
to learn why answers (A), (C), and (D) are wrong.
Antibiotic-resistant becterial (n) : vi khuẩn kháng kháng sinh

Strain (n) : di truyền, huyết thống, khuynh hướng

Incidence (n) : tỷ lệ mắc

Proliferation (n) : sinh sôi nảy nở

Circumspect (adj) : cẩn thận, thận trọng


Dec 11 – 1

Electroconvulsive therapy (ECT) is a controversial psychiatric treatment involving the induction of a


seizure in a patient by passing electricity through the brain. While beneficial effects of electrically
Induced seizures are evident and predictable in most patients, a unified mechanism of action has
not yet been established and remains the subject of numerous investigations. ECT is extremely
effective against severe depression, some acute psychotic states, and mania, though, like many
medical procedures, It has its risks.

Since the inception of ECT in 1938, the public has held a strongly negative conception of the
procedure. Initially, doctors employed unmodified ECT. Patients were rendered Instantly
unconscious by the electrical current, but the strength of the muscle contractions from uncontrolled
motor seizures often led to compression fractures of the spine or damage to the teeth. In addition to
the effect this physical trauma had on public sentiment graphic examples of abuse documented In
books and movies, such as Ken Kesey's One Flew Over the Cuckoo's Nest, portrayed ECT as
punitive, cruel, overused, and violative of patients' legal rights.

Modem ECT is virtually unrecognizable from Its earlier days. The treatment Is modified by the
muscle relaxant succinylcholine, which renders muscle contractions practically nonexistent.
Additionally, patients are given a general anesthetic. Thus, the patient is asleep and fully unaware
during the procedure, and the only outward sign of a seizure may be the rhythmic movement of the
patient's hand or foot. ECT is generally used In severely depressed patients for whom
psychotherapy and medication prove ineffective. It may also be considered when there Is an
imminent risk of suicide, since antidepressants often take several weeks to work effectively. Exactly
how ECT exerts its effects is not known, but repeated applications affect several neurotransmitters
In the brain, Including serotonin, norepinephrine, and dopamine.

ECT has proven effective, but it Is not without controversy. Though decades old studies showing
brain cell death have been refuted in recent research, many patients do report loss of memory for
events that occurred in the days, weeks, or months surrounding the ECT. Some patients have also
reported that their short-term memories continue to be affected for months after ECT, though some
doctors argue that this memory malfunction may reflect the type of amnesia that sometimes results
from severe depression.

1. According to the passage, why has ECT been viewed negatively by the public?

(A) Though ECT is effective in many cases, the medical community is not certain exactly how it
works.
(B) Cultural depictions of ECT implied that the therapy infringed upon the lawful rights
of those receiving the treatment.
(C) Effective use of ECT requires exposure to concerning medications, such as muscle relaxants and
anesthesia.
(D) ECT does not benefit individuals with anxiety disorders.
(E) ECT cannot be performed without subsequent loss of memory in the patient.
2. Which of the following can be inferred about the way in which the modern form of
ECT works?

(A) Greater amounts of the neurotransmitters serotonin, norepinephrine, and dopamine seem to
reduce symptoms of depression.
(B) ECT cannot be used prior to attempting psychotherapy or medication.
(C) Succinylcholine completely immobilizes the patient's body.
(D) ECT generally works faster than antidepressants.
(E) One ECT treatment is often sufficient to reduce symptoms of depression significantly.

3. The author mentions amnesia as a possible side effect of severe depression in order
to
(A) acknowledge one of the possible negative side effects associated with ECT
(B) emphasize the seriousness of severe depression as a debilitating disease
(C) introduce a possible alternative cause for short-term memory loss reported by some
patients
(D) draw a connection between brain cell death and short-term memory loss
(E) refute claims that ECT is responsible for any form of amnesia in patients

4. Each of the following is cited in the passage as a current or historical criticism of


electroconvulsive therapy EXCEPT
(A) ECT may cause the death of brain cells and memory loss
(B) in certain cases, ECT was portrayed as a means to punish individuals
(C) ECT had the potential to be used in inappropriate situations
(D) early forms of ECT did not adequately protect patients from secondary harm brought on by the
treatment
(E) repeated applications of ECT affect several neurotransmitters in the brain

Electro-convulsive (adj) : co giật điện

Psychiatric (adj) : tâm thần

Seizure (n) : co giật

Induction (n) : cảm ứng

Unified (adj) : thống nhất

Contraction (n) : cơn co thắt

Relaxant (n) : sự giãn nở

Anesthetic (n) : thuốc mê

Rhythmic (adj) : nhịp nhàng

Imminent (adj) : sắp xảy ra

Antidepressant (n) : thuốc chống trầm cảm

Malfunction (n) : sự cố

Amnesia (n) : mất trí nhớ


Dec 12 – 1

Anyone who thinks that rabbits make cute and cuddly pets has never owned one, and has most
definitely never worked as a farmer or gardener. To people whose livelihood depends on
agribusiness, rabbits are nothing more than ravenous vermin that inflict millions of dollars in
damage to crops meant for both animal and human consumption. Until now, no one had undertaken
to quantify the annual cost to a farmer's output for which a single rabbit is accountable. Great
Britain’s Ministry of Agriculture, however, has shown itself to be up to the challenge. Gordon
McKillop, a biologist at the Central Science Laboratory in York, England. just finished a study that
monitored the appetites of rabbits let loose to graze on several crops. As a result, farmers can
gauge rabbit damage more effectively, allowing them to anticipate the mom they will lose and make
necessary compensation. During his three-year study.

McKillop released a set number of rabbits into several enclosed regions, each containing one type of
vegetation on which the rabbits subsisted. To keep numbers constant, each enclosure was
surrounded by fence that was entrenched ten feet into the ground, and all rabbits released in a
certain area were of the same sex. The rabbits did the least damage in the pens containing grass,
which many farmers cultivate as grazing land for their livestock. The average rabbit ate almost 300
pounds of grass in one year, which reduced the yield of one hectare (about two and a half acres) by
half a percent. This translates to more than $3 worth of damage per rabbit per year-a seemingly
nominal sum until one considers that most grasslands are home to as many as forty rabbits per
acre. The rabbits‘ taste for barley was about the same as that for grass in terms of percentage.but
the cost was calculated to be almost $7 per rabbit.

By far, the most endangered crop was wheat, which rabbits munch at a rate that depleted normal
yields by more than 1 percent of the maximum. Since wheat is also the most expensive on the open
market. McKillop‘s group calculated that one rabbit can eat almost $1 worth of the crop in one
month. This can mean financial ruin for wheat farmers in areas with abnormally high rabbit
populations. Farmers may now be able to attach a dollar value to the crops that rabbits feed on, but
they still lack the most important piece of information that Dr- McKiIlop's study did not reveal: how
to stop them. Shooting and trapping rabbits is too time-consuming and inefficient to keep up with
the approximate 2 percent increase in rabbit populations every year, and most rabbits have
developed resistance to viral diseases such as myxomatosis and viral hemorrhagic fever that have
been introduced to curb reproduction. Even the age-old remedy of releasing foxes on the property
has been blocked by chicken farmers, whose commodity, according to the Ministry. contributes a
almost 14 percent of Britain's gross domestic product.

1. Which of the following statements best sums up the purpose of McKillop’s


experiment?

A. He contrasted several methods for establishing more credible methods for controlling rabbit
populations.
B. He set out to express the damage inflicted by rabbits on farmers' crops in a more
tangible, monetary sense.
C. He endeavored to prove that rabbits are more destructive than most people perceive them to be.
D. He hoped to determine the crop for which rabbits showed the most ardent appetite.
E. He wanted to portray the rabbit in a less flattering manner.
2. The passage supplies information about each of the following EXCEPT

A. the population density of rabbits


B. the best way to prevent rabbits from decimating a certain crop
C. the duration of McKiIlop’s study
D. the rate at which rabbits normally reproduce
E. the amount of grass usually grown annually upon a hectare of land

3. Each of the following can be inferred from the passage EXCEPT

A. in the agricultural marketplace, barley is at least twice as expensive as grass


B. at one point. myxomatosis and viral hemorrhagic fever were more effective than they are now
C. the power wielded by a certain type of farmer is at least partly influenced by financial impact of
that farmer's product
D. the cost incurred by farmers to rid themselves of large rabbit populations far exceeds
the monetary damage done to the farmers' crops
E. rabbits are unable to tunnel through the ground at a depth that is greater than ten feet

and most rabbits have developed resistance to viral diseases such as myxomatosis and viral
hemorrhagic fever that have been introduced to curb reproduction.

 in the past , population of rabbit is curbed by the viral diseases and viral hemorrhagic fever

But now rabbits develop resistance

 the viral diseases and viral hemorrhagic fever used to be more effective in the past ( eliminate B)

4. Which of the following hypothetical situations best exemplifies a potential problem


that would most seriously undermine the merit of McKillop’s study?

A. Several rabbits develop a new strain of myxomatosis that renders each completely sterile.
B. Due to a decrease in supply, the price of barley suddenly doubles.
C. It is determined that younger, more energetic rabbits consume almost double the food that an
older rabbit does.
D. A rare drought inhibits plant growth in the enclosures for several months.
E. Soon after the experiment begins, a predatory animal finds its way into some of the
rabbit enclosures.

To keep numbers constant, each enclosure was surrounded by fence that was entrenched ten feet
into the ground, and all rabbits released in a certain area were of the same sex.

 ten feet fench is not effective to keep numbers of rabbit constant

5. in the last paragraph, the author is primarily concerned with

A. exposing a problem to which McKillop‘s study has failed to supply a solution


B. suggesting that rabbit farmers and chicken farmers are often at odds when it comes to
agricultural legislation
C. citing evidence that McKiIIop’s study is woefully incomplete – woefully is too extreme
D. comparing the various methods that farmers have used in order to keep rabbit populations under
control
E. establishing that foxes have an equal appetite for rabbits as they do for chickens.

Ravenous (adj) : rất đói, tham mồi

Vermin (n) : sâu bọ

Appetite (n) : sự thèm ăn

Gauge (v) : đánh giá, đo đạc, ước lượng

Subsist (v) : tồn tại

Enclosure (n) : sự nhốt, sự rào

Entrench (v) : đắp lũy

Barley (n) : lúa mạch

Myxomatosis (n) : bệnh nấm

Hemorrhagic (adj) : xuất huyết

Curb (v) : ngăn lại, kiềm chế


Dec 13 – 1

In her seminal work, The Continuum Concept, Jean Liedloff presents the controversial theory that
Western methods of child rearing create the very problem these methods purport to eliminate-
excessive dependency upon the caretaker. Liedloff contrasts her observations of modern American
society with those of the quuana tribe of the Amazon, with whom she lived on four separate
occasions spanning several years. Liedloff claims that all humans operate on a continuum and are
genetically predisposed to thrive under certain conditions. We also compensate for treatment which
does not coincide with these optimal conditions by adapting our behavior to stabilize our psyches.

A baby who is in constant bodily contact with its caregiver receives physical stimulation that allows
the child to feel secure and ”right." In contrast, a child who is alone most of the time and left to "cry
it out" does not receive the necessary physical contact. This creates a dependency in the child, who
soon develops the grasping need for attention, even negative attention. Liedloff contends that some
parents' fears that they will spoil their baby by holding it "too much” or by feeding it on demand are
unreasonable and even detrimental to the child's mental health and social development.

Adults who have been deprived of their continuum needs as children often seek to stabilize
themselves by engaging in self-defeating or destructive behavior, which replicates the treatment to
which they have become accustomed. Victims of emotionally barren parents may tend to seek mates
who are domineering and cold, making true intimacy virtually impossible. Others may feel a lingering
sense of guilt induced by a caregiver who never seemed to accept the child's existence. This guilt
makes it difficult for an adult to feel peaceful or happy unless he is in crisis either physically or
emotionally; the unresolved feelings of shame from childhood do not allow the adult to live a pain-
free life.

Liedloff's critics argue that this form of “attachment parenting" does not enforce the natural
hierarchy of parent over child in the family relationship. They stress that without a clear dominance
of parent over child and discipline requiring the child to bend to the will of the parents from the
moment of birth. the child does not develop a sense of right versus wrong.

Many of these critics espouse more scientific methods of raising children that rely on regularly
scheduled feedings in infancy and careful control of the child's environment as the child grows older.
Liedloff's response is that tight parental control over the child undermines its natural impulses
toward socially acceptable behavior. Without choice, the child will not gain experience in making
decisions and will be forced to look to an outside authority rather than use his own judgment.
Upsetting the balance of the continuum by co-opting the child's natural ability to move toward
independence and decision-making skills. she asserts, ultimately results in a lack of maturity and
self-reliance in the adult.

1. The primary purpose of the passage as a whole is to

A. describe a strange phenomenon i


B. clarify a vague notion
C. condemn an ill-informed Opinion
D. refute a grievous misconception
E. support an alternative theory
A possible trap. Paradox is strange, as are self-defeating child-rearing practices.
Careful, though. The word strange is not strong or specific enough, especially compared with (E).
Paradoxical parenting practices might be "strange," but the author's presentation of Liedloff's
scathing critique
goes well beyond description.

2. According to the passage, Liedloff believes that adults whose sense of continuum is
denied to them as children

A. are unable or unwilling to have children of their own


B. develop a strong sense of discipline and familial hierarchy
C. may suffer feelings of shame and inadequacy
D. seek mates whose strong sense of compassion can fill the void
E. are too self-absorbed and domineering to become adequate caregivers for their own children

Others may feel a lingering sense of guilt induced by a caregiver who never seemed to accept the
child's existence. This guilt makes it difficult for an adult to feel peaceful or happy unless he is in
crisis either physically or emotionally; the unresolved feelings of shame from childhood do not allow
the adult to live a pain-free life.

3. Which of the following best describes the function of the passage's fourth paragraph?

A. It provides the reader the opportunity to see how both Liedloff and her critics rebut
each other's opinions.
B. It resolves the argument between the two parenting schools by calling for further testing and
research.
C. It undermines the passage's overall conclusion by suggesting that Liedloff's theories may be
incomplete.
D. lt stresses the need for all parents to establish their familial authority over their children.
E. it suggests that Liedloff would be willing to modify her theories and acknowledge the importance
of more scientific methods.

4. According to the passage, which of the following qualifies as an adult behavior that
disturbs "the balance of the continuum"?

A. Prematurely encouraging interaction with other children who are slightly older
B. Imposing a strict sleeping regimen, regardless of the child's expressed desire to sleep
C. Resolving not to have children in order to stop the cycle of inadequate parenting
D. Providing the child with his own room as soon as is financially possible
E. Punishing a child who deliberately disobeys an order.

Liedloff's response is that tight parental control over the child undermines its natural impulses
toward socially acceptable behavior

Child rearing (n) : nuôi dưỡng trẻ

Continuum (n) : liên tục

Predispose (v) : khuynh hướng


Thrive (v) : phát triển mạnh

Coincide (v) : trùng hợp

Deprive (v) : tước đoạt

Barren (adj) : cằn cỗi

Intimacy (n) : sự thân mật

Lingering (adj) : kéo dài

Espouse (v) : ủng hộ, theo đuổi


Dec 14 – 1

Dementia is one of the most dreaded human conditions, both by those observing it and by those
experiencing it. It is defined as a persistent and progressive impairment of intellectual function, and
can affect memory, emotions, personality, language, judgment, and cognition. It is frequently
associated with other medical conditions, including Alzheimer’s disease, stroke, and spongiform
encephalopathies.

Encephalopathies are diseases of the brain. The spongiform encephalopathies are named for their
characteristic deterioration of central nervous system tissue, resulting in a sponge-like appearance of
the brain. They include kuru, Creutzfeldt-Jacob disease (CJD), Gerstmann-Straussler-Scheinker
(GSS) disease, and familial fatal insomnia (FFI). Unlike Alzheimer’s disease and strokes, the
spongiform encephalopathies can be transmissible. Transmission of dementia was originally thought
to be due to a virus; however, subsequent information suggests that the agent of transmission is
simply a protein, with no associated nucleic acid. These proteins were termed prions, and are
normal proteins found primarily in a sponge-like tissue. Disease is associated with abnormal versions
of prion proteins.

The normal function of prion proteins is unclear, but they are thought to protect critical brain cells
from dying. Mice that lack normal prions develop ataxia (coordination problems) that resembles the
ataxia of kuru and CJD patients. Prion-free mice develop symptoms at approximately 70 weeks (late
middle age for a mouse).

Since proteins are unable to self- replicate, it seemed unlikely that a transmissible disease could be
caused by protein alone. Contact between the normal and abnormal forms of protein can cause
conformational change from the normal to the flawed form, if the abnormal and normal proteins are
sufficiently similar. Encephalitis (inflammation of the brain) can present with similar initial symptoms
as the spongiform encephalopathies, most notably ataxia and mental disorientation. Encephalitis can
have several causes, some of which are viral infections.

1. All of the following individuals display symptoms of dementia EXCEPT someone who:

a. cannot do the most rudimentary arithmetic operations.


b. cannot distinguish between the smells of the distinct flowers.
c. displays abrupt change in character.
d. has lost the ability to speak.
e. has tremendous difficulty remembering the days of the week.

2. According to the passage, which one of the below is a medical condition that is the
least likely to be associated with dementia:

a.Alzheimer's disease
b. Familial fatal insomnia
c. Stroke
d. Meningitis
e. Spongiform encephalopathies
3. Spongiform encephalopathies are different from strokes because spongiform
encephalopathies:

a. are more difficult to cure.


b. have more varieties.
c. are less researched.
d. are associated with human brain.
e. are transmissible.

Transmissible (adj) : di truyền

Meningitis (n) : viêm màng não

Dementia (n) : sa sút trí tuệ

Dreaded (adj) : sợ hãi, đáng sợ

Impairment (n) : suy nhược

Deterioration (n) : xấu đi, xuống cấp, hư hỏng

Abnormal (adj) : khác thường

Conformational (adj) : hình dạng

Inflammation (n) : viêm


Dec 14 – 2

Increased atmospheric concentrations of carbon dioxide (CO2) in the atmosphere can have a
fertilizing effect on certain crops, provided that it is possible for farmers to provide adequate water
and fertilizer for the plants to grow more quickly. For this reason, many non-scientists have long
held that increases in global atmospheric carbon dioxide will also lead to increased agricultural
production, offsetting any shortages due to shifting water availability and increased temperature.
This in turn, these individuals argue, will lead to decreased global malnutrition and undernutrition as
the increased supply of food drives down its cost.

However, research on the nutritional content of crops grown at Free Air CO2
Enrichment - or FACE - sites suggests that while crops may grow more quickly in elevated
concentrations of carbon dioxide, increased growth comes at a price. Crops grown in elevated levels
of carbon dioxide have lower levels of protein and of micronutrients such as zinc. Since the world’s
population gets the majority of its protein and other micronutrients from plant-based sources, such
a shift could have dire consequences. Scientists estimate that by 2050 an additional 175 million
people could become zinc deficient, and an additional 120 million could become protein deficient as
the nutritional content of staple crops falls between 5 and 17 percent.

1. The author uses the word “non-scientists” in paragraph 1 to:

a) criticize those who make predictions without sufficient knowledge of a topic.


b) highlight the expertise of the researchers at Free Air CO2 Enrichment.
c) call into question the inferences ( suy luận ) discussed in the first paragraph.
d) demonstrate that the farmers mentioned in the previous sentence did not make the predictions
that follow.
e) imply that their predictions were made without any evidence.

2. The passage suggests that the nonscientists mentioned in paragraph 1 fail to consider
whether or not the growth rate increase mentioned in paragraph 1

a) will also lead to an increase in the amount of edible material that the plants produce.
b) is associated with any phenomena that would affect its ability to decrease global
malnutrition.
c) will require better distribution systems in order to mitigate zinc deficiency.
d) can offset the deleterious effects CO2 related warming will have on water availability.
e) will occur in actual crops to the same extent as it has occurred at FACE sites.

3. Which of the following can be inferred from the passage?

a) If carbon dioxide levels do not increase, there will not be an increase in the number of people
who are zinc deficient.
b) Increased carbon dioxide levels will not, despite what many non-scientists predict, cause crops to
grow more quickly.
c) It is possible for farmers to provide adequate water and fertilizer for crops to grow more quickly
in an increased-carbon environment.
d) There are currently some people who are considered to have a zinc deficiency.
e) Most micronutrients are only available via plant-based sources.
Scientists estimate that by 2050 an additional 175 million people could become zinc deficient, and
an additional 120 million could become protein deficient as the nutritional content of staple crops
falls between 5 and 17 percent.

An additional 175 million people could become zinz deficient  it can be inferred that some people
already suffer from zinz deficient

4. It can be inferred from the passage that increased atmospheric carbon dioxide

a) can decrease the nutritional content of plants.


b) increases the rate at which most plants grow.

Most is a red flag making statement extreme as the passage does not mention "most" plants
c) will lead to increased agricultural production.

"Will lead" is too strong and we cannot predict the future.


d) is the biggest threat to global nutrition that the world faces today.
e) will cause millions of people to become zinc deficient by 2050.

Again "will cause" is too strong

Malnutrition (n) : suy dinh dưỡng

Fertilizer (n) : phân bón

Fertilizing (adj) : màu mỡ, phì nhiêu

Dire (adj) : thảm khốc


Dec 15 – 1

Newton’s surprising success at developing the laws of motion, as well as the development and
refinement of other physical laws, led to
the idea of scientific determinism. The first expression of this principle was in the beginning of the
nineteenth century by Laplace, a French scientist. Laplace argued that if one knew the position and
velocity of all the particles in the universe at a given time, the laws of physics would be able to
predict the future state of the universe.

Scientific determinism held sway over a great many scientists until the early twentieth century, when
the quantum mechanics revolution occurred. Quantum mechanics introduced the world to the idea
of the uncertainty principle, which stated that it was impossible to accurately measure both the
position and the velocity of a particle at one time. Because Laplace’s omniscience could never occur,
even in theory, the principle of scientific determinism was thrown into doubt. However, quantum
mechanics does allow for a reduced form of scientific determinism. Even though physicists are
unable to know precisely where a particle is and what its velocity is, they can determine certain
probabilities about its position and velocity. These probabilities are called wave functions. By use of
a formula known as the Schrodinger equation, a scientist with the wave function of a particle at a
given time can calculate the particle’s future wave function. These calculations can give the particle’s
position or velocity, but not both. Thus, the physicist is in possession of exactly half of the
information needed to satisfy Laplace’s view of determinism. Unfortunately, under modern physics
theories, that is far as any researcher can go in predicting the future.

1. The passage suggests that if scientific determinism were true

A. scientists would, in theory, be able to predict the future


B. all the particles in the universe would have a measurable position and velocity
C. the theory of quantum mechanics would be false
D. Schrodinger’s equation could be used to calculate any particle’s position
E. the quantum mechanics revolution would not have occurred

2. According to the passage, wave functions

A. allow scientists to determine the position and velocity of a particle – to extreme


B. are determined by the Schrodinger equation
C. provide a range of possible locations and velocities for a particle
D. allow a scientist to calculate the future state of the universe
E. threw the proposition of scientific determinism into doubt

Even though physicists are unable to know precisely where a particle is and what its velocity is, they
can determine certain probabilities about its position and velocity.

3. Which of the following best describes the organization of the passage?

A. A paradox is introduced, competing explanations are offered, and a final resolution is reached.
B. Two opposing theories are introduced, critiqued, and reconciled.
C. An idea is introduced, its validity is questioned, and its application qualified.
D. A theory is introduced, its mathematical basis is examined, and it is rejected.
E. An argument is made, an objection to it is detailed, and the argument is revised.
4. Which of the following, if true, would most strengthen the author’s conclusion in the
passage’s final sentence?

A. Some physicists believe quantum mechanics will eventually be discarded in favor of a new theory.
B. Physicists still use Newton’s laws of motion to calculate the velocities and positions of planets and
stars.
C. Even if the position and velocity of a particle were known, predicting the future would be
impossible because there are too many other variables to calculate.
D. There is little to no chance that the modern theory of quantum mechanics will be
overturned by another theory.
E. No scientists are pursuing studies in the field of determinism.

Unfortunately, under modern physics theories, that is far as any researcher can go in predicting the
future.
The main subject of the author is about the dominance of modern physics theories (quantum
mechanics), not about the predicting future.

To strengthen the author’s conclusion, the answer must support that the quantum mechanics theory
is the most advanced theory scientists can ever develop.

Determinism (n) : thuyết định mạng

Omniscience (n) : toàn thức

Overturned (v) : luật ngược


Dec 15 – 2

Virginia Woolf made an original contribution to the form of the novel, but was also a distinguished
essayist, a critic for The Times Literary Supplement, and a central figure of the Bloomsbury group.
Dialogic in style and continually questioning what may be the reader‘s opinion (her rejection of an
authoritative voice links her to the tradition of Montaigne), her critical essays, when examined
carefully, reveal a thematic and technical complexity that rivals her novels.

Some of her most rigorous essays suggest that the personality of the author can be fixed if sufficient
evidence can be amassed and if its logical implications are followed. In ―The Novels of Turgenev,‖
Woolf pursues the problem of interpretation on the part of the reader by providing a detailed report
of her own response to Turgenev. She does this in order to make possible the question that leaps
the gap between reader and text. That question—―what principles guided Turgenev?‖—focuses on
the fictional strategies that must have been in operation in order to have produced Woolf's
experience. Thus Woolf accounts for this by reconstructing Turgenev‘s method. But she pushes
farther insofar as she asserts that the method must be a sign of a deeper informing power, the mind
of Turgenev itself. This distance can be traversed by interpretation, Woolf argues, because writers
like Turgenev achieve a level of personality beneath the surface distinctions among individuals. Her
greatest examples of this impersonal power in the English language are Jane Austen and
Shakespeare. According to Woolf, these authors write with a ―clarity of heart and spirit‖ that allows
their potential for genius to express itself ―whole an entire.‖ Unencumbered by impediments that
would be erected by such feelings on their part as fear, hatred, or dependency, we are allowed by
their art to make contact with what is most deeply personal, and therefore most widely human, in
them.

But one of the riches of Woolf‘s essays is that they critique this very same possibility of closing the
gap that exists between author and audience. This is evinced in Woolf‘s awareness of the
contemporary artist‘s self-consciousness: the enemy of human contact and knowing. There seem to
be so many barriers on the road to the deepest level of self that the journey there is impossible, but
it is this level of self through which the gap must be closed. In fact, Woolf asserts that the journey is
impossible for the modern writer. In ―How It Strikes a Contemporary,‖ Woolf contrasts writers of
the past—Chaucer is her most powerful example—who believed wholeheartedly in an atemporal
order verified by the entire culture, with modern writers who have lost this advantage. Woolf
suggests that, if, for writer and reader, no way to a shared, universal level of experience is available,
the very ground of the interpretive enterprise is removed.

1. Which of the following would most weaken Woolf‘s assertion that the distance
between reader and writer can be traversed by interpretation?

A. Contemporary writers are unable to construct a deep meaning for each reader
because they focus primarily on personal distinctions rather than similarities.
B. Every reader reacts differently to the same text and yet each constructs for himself/herself a
similar idea of the author‘s personality and presence.
C. Past writers were governed by a strong sense of individualism, which made it impossible for them
to appeal to human commonalities.
D. Authorial intent or perspective remains an abstract idea unless the writer is able to confirm or
deny the reader‘s interpretation.
E. Most readers are not learned enough to be able to understand the deeper meaning that is implied
by the author

This distance can be traversed by interpretation, Woolf argues, because writers like Turgenev
achieve a level of personality beneath the surface distinctions among individuals

2. According to the points elucidated by the author within the passage, all of the
following are characteristic of Woolf‘s essays EXCEPT that:

A. they focus primarily on examining whether or not a reader‘s experience of a text can reveal the
original authorial presence.
B. they are written in a more technically and thematically complex manner than are her
fictional works.
C. they betray Woolf‘s skepticism about the very idea she is attempting to demonstrate and justify.
D. they frequently utilize examples from other writers in order to illustrate and support her
conclusions.
E. they are as complex as her other works

P1 of passage : ..her critical essays, when examined carefully, reveal a thematic and technical
complexity that rivals her novels.
critical essays are more complex than novels , not fictional works
If it rivals that of the novels, it‘s not necessarily exceeding, as the answer choice suggests.
Fictional works

3. The passage implies that, in her essay ―The Novels of Turgenev,‖ Woolf assumes
that:

A. stable and defining qualities of an author‘s personality are discernible in his or her
fiction.
B. interpretation involves a compromise between the reader‘s perspective and the perspective of the
author.

Woolf is arguing that the reader‘s perspective can be used to discover the perspective of the author;
there‘s no discussion of compromise.

C. a reader‘s experience of a novel‘s text is determined by a standard set of fictional principles.


D. making contact with an author‘s mind requires the use of critical reasoning more than intuition.
E. an author‘s literary work must reflect the various facets of the author‘s personality

Some of her most rigorous essays suggest that the personality of the author can be fixed if sufficient
evidence can be amassed and if its logical implications are followed
 readers can use their own personal experiences to understand the personality and
motivations of the writer.

Looking for an assumption necessary to this argument turns up (A), which simply restates the point
that readers can figure out part of the author‘s personality. If (A) isn‘t true, the argument falls apart:

this is a sure sign of a critical assumption.


Discernible (adj) : có thể thấy rõ

Essayist (n) : người viết tiểu luận

Thematic (adj) : thuộc về chủ đề

Rigorous (adj) : nghiêm ngặt, chính xác

Amass (v) : tích lũy

Traverse (v) : đi qua, nghiên cứu kỹ lưỡng

Unencumbered (adj) : không bị cản trở

Impediment (n) : trở ngại


Dec 16 – 2

The law-and-literature movement claims to have introduced a valuable pedagogical innovation into
legal study: instructing students in techniques of literary analysis for the purpose of interpreting laws
and in the reciprocal use of legal analysis for the purpose of interpreting literary texts. The results,
according to advocates, are not only conceptual breakthroughs in both law and literature but also
more sensitive and humane lawyers. Whatever the truth of this last claim, there can be no doubt
that the movement is a success: law-and-literature is an accepted subject in law journals and in
leading law schools. Indeed, one indication of the movement’s strength is the fact that its most
distinguished critic, Richard A. Posner, paradoxically ends up expressing qualified support for the
movement in a recent study in which he systematically refutes the writings of its leading legal
scholars and cooperating literary critics.

Critiquing the movement’s assumption that lawyers can offer special insights into literature that
deals with legal matters, Posner points out that writers of literature use the law loosely to convey a
particular idea or as a metaphor for the workings of the society envisioned in their fiction. Legal
questions per se, about which a lawyer might instruct readers, are seldom at issue in literature. This
is why practitioners of law-and-literature end up discussing the law itself far less than one might
suppose. Movement leader James White, for example, in his discussion of arguments in the Iliad,
barely touches on law, and then so generally as to render himself vulnerable to Posner’s devastating
remark that “any argument can be analogized to a legal dispute.”

Similarly, the notion that literary criticism can be helpful in interpreting law is problematic. Posner
argues that literary criticism in general aims at exploring richness and variety of meaning in texts,
whereas legal interpretation aims at discovering a single meaning. A literary approach can thus only
confuse the task of interpreting the law, especially if one adopts current fashions like
deconstruction, which holds that all texts are inherently uninterpretable.

Nevertheless, Posner writes that law-and-literature is a field with “promise”. Why? Perhaps,
recognizing the success of a movement that, in the past, has singled him out for abuse, he is
attempting to appease his detractors, paying obeisance to the movements institutional success by
declaring that it “deserves a place in legal research” while leaving it to others to draw the conclusion
from his cogent analysis that it is an entirely factitious undertaking, deserving of no intellectual
respect whatsoever. As a result, his work stands both as a rebuttal of law-and-literature and as a
tribute to the power it has come to exercise in academic circles.

1. The primary purpose of the passage is to

(A) assess the law-and-literature movement by examining the position of one of its
most prominent critics
(B) assert that a mutually beneficial relationship exists between the study of law and the study of
literature
(C) provide examples of the law-and-literature movement in practice by discussing the work of its
proponents
(D) dismiss a prominent critics recent study of the law-and-literature movement
(E) describe the role played by literary scholars in providing a broader context for legal issues
2. Posner’s stated position with regard to the law-and-literature movement is most
analogous to which one of the following?

(A) a musician who is trained in the classics but frequently plays modern music while performing on
stage
(B) a partisan who transfers allegiance to a new political party that demonstrates more promise but
has fewer documented accomplishments
(C) a sports fan who wholeheartedly supports the team most likely to win rather than his or her
personal favorite
(D) an ideologue who remains committed to his or her own view of a subject in spite of compelling
evidence to the contrary
(E) a salesperson who describes the faults in a fashionable product while conceding that
it may have some value

As a result, his work stands both as a rebuttal of law-and-literature and as a tribute to the power it
has come to exercise in academic circles.

3. The passage suggests that Posner regards legal practitioners as using an approach to
interpreting law that

(A) eschews discovery of multiple meanings


(B) employs techniques like deconstruction
(C) interprets laws in light of varying community standards
(D) is informed by the positions of literary critics
(E) de-emphasizes the social relevance of the legal tradition

4. The Passage suggests that Posner might find legal training useful in the
interpretation of a literary text in which

(A) a legal dispute symbolizes the relationship between two characters


(B) an oppressive law is used to symbolize an oppressive culture
(C) one of the key issues involves the answer to a legal question
(D) a legal controversy is used to represent a moral conflict
(E) the working of the legal system suggests something about the political character of a society

5. The author uses the word “success” in line 11 to refer to the law-and-literature
movement’s

(A) positive effect on the sensitivity of lawyers


(B) widespread acceptance by law schools and law journals
(C) ability to offer fresh insights into literary texts
(D) ability to encourage innovative approaches in two disciplines
(E) response to recent criticism in law journals

6. According to the passage, Posner argues that legal analysis is not generally useful in
interpreting literature because

(A) use of the law in literature is generally of a quite different nature than use of the law
in legal practice
(B) law is rarely used to convey important ideas in literature
(C) lawyers do not have enough literary training to analyze literature competently
(D) legal interpretations of literature tend to focus on legal issues to the exclusion of other important
elements
(E) legal interpretations are only relevant to contemporary literature

Critiquing the movement’s assumption that lawyers can offer special insights into literature that
deals with legal matters, Posner points out that writers of literature use the law loosely to convey a
particular idea or as a metaphor for the workings of the society envisioned in their fiction. Legal
questions per se, about which a lawyer might instruct readers, are seldom at issue in literature.

7. According to Posner, the primary difficulty in using literary criticism to interpret law is
that

(A) the goals of the two disciplines are incompatible


(B) there are few advocates for the law-and-literature movement in the literary profession
(C) the task of interpreting law is too complex for the techniques of literary criticism
(D) the interpretation of law relies heavily on legal precedent
(E) legal scholars are reluctant to adopt the practice in the classroom

Pedagogical (adj) : sư phạm

Reciprocal (adj) : đối ứng

Humane (adj) : nhân đạo

Loosely (adv) : lỏng lẻo

Envision (v) : hình dung

Remark (n) : nhận xét, chỉ trích

Problematic (adj) : có vấn đề

Deconstruction (n) : giải cấu trúc

Uninterpretable (adj) : không thể giải thích

Single sb out for (n) : chỉ trích ai đó vì điều gì

Appease (v) : xoa dịu

Detractor (n) : gièm pha, phỉ báng

Obeisance (n) : vâng lời, kính trọng

Factitious (adj) : thực tế, nhân tạo

Rebuttal (n) : phản bác

Tribute (n) : cống vật

Academic circle (n) : giới học thuật


Dec 17 – 1

In eighteenth-century France and England, reformers rallied around egalitarian ideals, but few
reformers advocated higher education for women. Although the public decried women’s lack of
education, it did not encourage learning for its own sake for women. In spite of the general
prejudice against learned women, there was one place where women could exhibit their erudition:
the literary salon. Many writers have defined the woman’s role in the salon as that of an intelligent
hostess, but the salon had more than a social function for women. It was an informal university,
too, where women exchanged ideas with educated persons, read their own works and heard those
of others, and received and gave criticism.

In the 1750’s, when salons were firmly established in France, some English women, who called
themselves “Bluestocking,” followed the example of the salonnieres (French salon hostesses) and
formed their own salons. Most Bluestockings did not wish to mirror the salonnieres; they simply
desired to adapt a proven formula to their own purpose—the elevation of women’s status through
moral and intellectual training. Differences in social orientation and background can account perhaps
for differences in the nature of French and English salons. The French salon incorporated aristocratic
attitudes that exalted courtly pleasure and emphasized artistic accomplishments. The English
Bluestockings, originating from a more modest background, emphasized learning and work over
pleasure. Accustomed to the regimented life of court circles, salonnieres tended toward formality in
their salons. The English women, though somewhat puritanical, were more casual in their approach.

At first, the Bluestockings did imitate the salonnieres by including men in their circles. However, as
they gained cohesion, the Bluestockings came to regard themselves as a women’s group and to
possess a sense of female solidarity lacking in the salonnieres, who remained isolated from one
another by the primacy each held in her own salon. In an atmosphere of mutual support, the
Bluestockings went beyond the salon experience. They traveled, studied, worked, wrote for
publication, and by their activities challenged the stereotype of the passive woman. Although the
salonnieres were aware of sexual inequality, the narrow boundaries of their world kept their
intellectual pursuits within conventional limits. Many salonnieres, in fact, camouflaged their
nontraditional activities behind the role of hostess and deferred to men in public.

Though the Bluestockings were trailblazers when compared with the salonnieres, they were not
feminists. They were too traditional, too hemmed in by their generation to demand social and
political rights. Nonetheless, in their desire for education, their willingness to go beyond the confines
of the salon in pursuing their interests, and their championing of unity among women, the
Bluestockings began the process of questioning women’s role in society.

22.Which of the following could best be considered a twentieth-century counterpart of


an eighteenth century literary salon as it is described in the passage?

(A) A social sorority


(B) A community center
(C) A lecture course on art
(D) A humanities study group
(E) An association of moral reformers
It was an informal university, too, where women exchanged ideas with educated persons, read their
own works and heard those of others, and received and gave criticism.

24.Which of the following titles best describes the content of the passage?

(A) Eighteenth-Century Egalitarianism


(B) Feminists of the Eighteenth Century
(C) Eighteenth-Century Precursors of Feminism
(D) Intellectual Life in the Eighteenth Century
(E) Female Education Reform in the Eighteenth Century

the Bluestockings began the process of questioning women’s role in society.

Counterpart (n) : bản đối chiếu, bản sao

Sorority (n) : a society for female students in a university or college, typically for social
purposes.
Bluestocking (n) : phụ nữ trí thức

Rally (v) : chế giễu

Egalitarian (adj) : bình đẳng

Decry (v) : chê bai

Prejudice (n) : định kiến

Erudition (n) : sự thông thái, học rộng, uyên bác

Hostess (n) : bà chủ

Elevation (n) : phẩm giá

Cohesion (n) : sự gắn kết

Camouflage (v) : ngụy trang

Defer (v) : hoãn lại

Trailblazer (n) : tiên phong

Confine (n) : hạn chế


Dec 17 – 2

Relieving an improvished country of its debt would seem to almost unavoidably help the citizens of
that nation. Indeed, health care and education spending is now grater than debt service payments
in many countries(HIPC) that have been granted partial debt relief by the Heavily Indebted Poor
Countries Initiative launched by the International Monetray Debt Relief Initiative (MDRI). Several
factors, however, have kept these programs from becoming truly transformational.

Forr a nation to qualify for the HIPC, it must have a level of debt that cannot be managed through
traditional means. The removal of this enormous burden means that badly needed resources can go
to programs that aid needy citizens, just as is intended. However, the HIPC has strict rules that
sharply limit this spending. In some case, teachers are not hired and HIV/AIDS tests are not
administered because the sudden spending might lead to macroeconomic instability. This is a
paradox that must be confronted: poor nations need to spend money desperately but wisely.

A more daunting obstacle is tha lack of a private sector in many of the countries that are served by
the HIPC. Property rights may be virtually non-existent, Without individuals and businesses willing
and able to invest resources in their own country, progress can be glacial. Outside investors are
forced to deal not with business partners as such, but with government agencies ranging from the
inept to the murderously corrupt. There is no guarantee that funds meant for a hospital or school
will ever find their way to the intended destination, whether because of corruption or the sheer
difficulty of moving goods around in a place that is, as is so often the case in truly poor countries, at
war.

Debt relief remains an important tool in reducing the terrible suffering that affects so many people in
the underdeeveloped world. However, it is not enough to clean the slate and say ,"start anew".
Without the willingness on the part of the governing body to allow its citizens to take part in their
own development, and without the right balance of emergency spending and careful investment,
unmanagement debt will return, as evidenced by nations that have been borrowing money faster
than their debt can be relieved.

1. Which of the folloring titles best summarizes the contents of the passage?

A. Debt Relief as a Tool for increasing Private Investment in Impoverished Nations.


B. The difficulties of Implementing Debt Relief in Very Poor Countries
C. Obstacles to Channeling Aid to Needy Destinations in Vey Poor Countries.
D. the Inherent Flaws in Debt Relief Programs
E. HIPC: Well-meaning, but insufficient

Passage discuss mostly about HIPC, channeling aid is to general

2. Which of the following was not mentioned as a difficulty associated with debt relief?

A. long-term needs have to be addressed at the same time as more immediate one.
B. Nations that need debt relief are often in the midest of violent conflicts
C. Debt relief can swiftly be replaced by new debt.
D. Private investors in very poor countries are often corrupt.
E. The pace of development in poor nations can be extremely slow
Outside investors are forced to deal not with business partners as such, but with government
agencies ranging from the inept to the murderously corrupt.

The inept (n) : sự bất lực

Relieve (v) : cứu trợ

Impoverished (adj) : nghèo khó

Transformational (adj) : biến đổi

Glacial (adj) : băng hà


Dec 18 – 1

Introduced in 1978, video laserdiscs were technologically more advanced than video cassettes—they
offered better picture quality without degradation over time—yet video cassettes and recorders were
far more successful commercially, at least in part because relatively few movies were ever released
on laserdisc. As this example illustrates, superior technology is no guarantee of success in the home
audio and video market.

In home audio, vinyl records were the dominant format until the 1970s, when audio cassette tapes
were introduced. Cassette tapes offered no better sound quality than vinyl records (in fact, some
believed they offered lower quality), yet this format became widely successful for reasons having
little to do with technical advancements in sound quality. Cassettes were more portable than
records, and the ability to record from
records onto cassettes made the two formats complementary. In addition to buying prerecorded
records and tapes, consumers could now make copies of vinyl records and listen to them outside the
home. Thus, cassette tape sales grew even as vinyl remained a popular format.

The rise of audio compact discs (CDs) was quite different. Introduced in 1983, CDs clearly offered
higher sound quality than records or cassettes, yet they were not an immediate success. However,
CDs were persistently and aggressively marketed by the industry, and by the 1990s they had
become the most popular audio format. The dominant position of CDs was further cemented later in
the 1990s by the advent of new technology that allowed consumers to create their own CDs at
home—thus combining one of the best features of audio cassettes with the higher sound quality of
CDs.

In home video, after the failure of laserdiscs, video cassettes remained the dominant format until
the advent of digital video discs (DVDs). Introduced in 1997, DVDs quickly gained widespread
popularity. These discs were based on a technology similar to that of laserdiscs and offered several
clear advantages over video cassettes, including better picture quality and better search features.
Yet perhaps the real key to their rapid rise was the fact that manufacturers quickly made many titles
available on DVD. This combination of better technology and smart marketing helped the DVD avoid
the fate of the laserdisc.

1. The passage is primarily concerned with which of the following?

(A) Contrasting the success of DVDs with the failure of laserdiscs


(B) Describing the crucial role played by technology in the home audio and video market
(C) Questioning the wisdom of introducing new audio or video formats
(D) Illustrating that there is more than one path to success in the home audio and video
market
(E) Proving that good marketing is the only way to guarantee success in the home audio and video
market

2. The author of this passage would most likely agree that

(A) better technology alone will never cause the success of a new audio or video format.
(B) a combination of better technology and good marketing is the best way to ensure success when
introducing a new format.
(C) there is no perfectly reliable way to predict the success or failure of new audio and
video formats.
(D) companies in the home audio or video industry should invest less in product research and
development than in marketing.
(E) consumer behavior is inherently irrational.

3. According to the passage, which strategy is LEAST likely to produce a successful


media format?

(A) Aggressively marketing a brand new format


(B) Cautiously testing the market by releasing titles sparingly- not stated in the passage
(C) Relying on word-of-mouth advertising
(D) Creating a new format based on earlier technology
(E) Introducing a product that works with existing products

Sparingly (adv) : tiết kiệm


Dec 18 – 2

Legal cases can be termed “hard” cases if they raise issues that are highly controversial, issues
about which people with legal training disagree. The ongoing debate over the completeness of the
law usually concerns the extent to which such hard cases are legally determinate, or decidable
according to existing law.
H. L. A. Hart’s The Concept of Law is still the clearest and most persuasive statement of both the
standard theory of hard cases and the standard theory of law on which it rests. For Hart, the law
consists of legal rules formulated in general terms; these terms he calls “open textured,” which
means that they contain a “core” of settled meaning and a “penumbra” or “periphery” where their
meaning is not determinate. For example, suppose an ordinance prohibits the use of vehicles in a
park.

“Vehicle” has a core of meaning which includes cars and motorcycles. But, Hart claims, other
vehicles, such as bicycles, fall within the peripheral meaning of “vehicle,” so that the law does not
establish whether they are prohibited. There will always be cases not covered by the core meaning
of legal terms within existing laws; Hart considers these cases to be legally indeterminate. Since
courts cannot decide such cases on legal grounds, they must consider nonlegal (for example, moral
and political) grounds, and thereby exercise judicial discretion to make, rather than apply, law.

In Ronald Dworkin’s view the law is richer than Hart would grant; he denies that the law consists
solely of explicit rules. The law also includes principles that do not depend for their legal status on
any prior official recognition or enactment. Dworkin claims that many cases illustrate the existence
of legal principles that are different from legal rules and that Hart’s “model of rules” cannot
accommodate. For Dworkin, legal rules apply in an all-or-nothing fashion, whereas legal principles
do not: they provide the rationale for applying legal rules. Thus, because Dworkin thinks there is law
in addition to legal rules, he thinks that legal indeterminacy and the need for judicial discretion do
not follow from the existence of open texture in legal rules.

It would be a mistake, though, to dispute Hart’s theory of hard cases on this basis alone. If Hart’s
claim about the “open texture” of general terms is true, then we should expect to find legal
indeterminacies even if the law consists of principles in addition to rules. Legal principles, as well as
legal rules, contain general terms that have open texture. And it would be absurd to suppose that
wherever the meaning of a legal rule is unclear, there is a legal principle with a clear meaning.Most
interesting and controversial cases will occur in the penumbra of both rules and principles.

1. Which one of the following best expresses the main idea of the passage?

(A) The law will never be complete because new situations will always arise which will require new
laws to resolve them.
(B) The most difficult legal cases are those concerning controversial issues about which trained legal
minds have differing opinions.
(C) The concept of legal principles does not diminish the usefulness of the concept of
the open texture of general terms in deciding whether hard cases are legally
determinate.
(D) The concept of legal principles is a deleterious addition to the theory of law since any flaws
exhibited by legal rules could also be shared by legal principles.
(E) The inherent inconsistency of terms used in laws provides a continuing opportunity for judges to
exercise their discretion to correct defects and gaps in the law.

2. According to the passage, the term “legal principles” as used by Dworkin refers to

(A) a comprehensive code of ethics that governs the behavior of professionals in the legal system
(B) explicit analyses of the terms used in legal rules, indicating what meanings the terms do and do
not cover
(C) legal doctrines that underlie and guide the use of accepted legal rules
(D) legal rules that have not yet passed through the entire legislative procedure necessary for them
to become law
(E) the body of legal decisions regarding cases that required judicial discretion for their resolution

3. Which one of the following expresses a view that the author of the passage would
most probably hold concerning legal principles and legal rules?

(A) Legal rules are applied more often than legal principles when a case involves issues about which
legal professionals disagree.
(B) Both legal rules and legal principles areofficially recognized as valid parts of the law.
(C) Hart’s “model of rules” has been superseded by a “model of principles” that sheds light on legal
determinacy.
(D) Legal principles are just as likely as legal rules to have terms that have both core
and peripheral meanings.
(E) Legal principles eliminate the need for judicial discretion in resolving the problemsgenerated by
the open texture of legal rules.

4. It can be inferred that the author of the passage regards Hart’s theory of hard cases
and the theory of standard law as

(A) exhaustive
(B) worthy of respect
(C) interesting but impractical
(D) plausible but unwieldy
(E) hopelessly outmoded

5. In the passage, the author is primarily concerned with

(A) outlining the problems that might be faced by a legislature attempting to create a complete body
of law that would prevent judges from making rather than applying the law
(B) justifying the idea that “hard” cases will always exist in the practice of law, no matter what laws
are written or how they are applied
(C) presenting evidence to support Dworkin’s idea that legal rules apply in an all-or-nothing fashion,
whereas legal principles apply in more sophisticated ways
(D) critiquing the concept of the open texture of legal terms as a conceptual flaw in Hart’s otherwise
well-regarded book
(E) demonstrating that Dworkin’s concept of legal principles does not form the basis for
a successful attack on Hart’
Dec 19 – 2

Faced with the problems of insufficient evidence, of conflicting evidence, and of evidence relayed
through the flawed perceptual, retentive, and narrative abilities of witnesses, a jury is forced to draw
inferences in its attempt to ascertain the truth. By applying the same cognitive tools they have
developed and used over a lifetime, jurors engage in the inferential exercise that lawyers call fact-
finding. In certain decision-making contexts that are relevant to the trial of lawsuits, however, these
normally reliable cognitive tools may cause jurors to commit inferential errors that distort rather
than reveal the truth.

Although juries can make a variety of inferential errors, most of these mistakes in judgment involve
the drawing of an unwarranted conclusion from the evidence, that is, in reality, it does not prove.
For example, evidence that the defendant in a criminal prosecution has a prior conviction may
encourage jurors to presume the defendant’s guilt, because of their preconception that a person
previously convicted of a crime must be inclined toward repeated criminal behavior. That commonly
held belief is at least a partial distortion of reality; not all former convicts engage in repeated
criminal behavior. Also, a jury may give more probative weight than objective analysis would allow
to vivid photographic evidence depicting a shooting victim’s wounds, or may underestimate the
weight of defense testimony that is not delivered in a sufficiently forceful or persuasive manner.
Finally, complex or voluminous evidence might be so confusing to a jury that its members would
draw totally unwarranted conclusions or even ignore the evidence entirely.

Recent empirical research in cognitive psychology suggests that people tend to commit inferential
errors like these under certain predictable circumstances. By examining the available information,
the situation, and the type of decision being made, cognitive psychologists can describe the kinds of
inferential errors a person or a group is likely to make. These patterns of human decision-making
may provide the courts with a guide to evaluating the effect of evidence on the reliability of the
jury’s inferential processes in certain situations.

The fact that juries can commit inferential errors that jeopardize the accuracy of the fact-finding
process is not unknown to the courts. In fact, one of a presiding judge’s duties is to minimize jury
inferential error through explanation and clarification. Nonetheless, most judges now employ only a
limited and primitive concept of jury inferential error: limited because it fails to recognize the
potential for errors outside certain traditional situations, primitive because it ignores the research
and conclusions of psychologists in favor of notions about human cognition held by lawyers.

1 . Which of the following best explains the main idea of the passage?

(A) When making decisions in certain predictable situations, juries may commit
inferential errors that obscure rather than reveal the truth.
(B) The views of human cognition held by psychologists on the one hand and by the legal profession
are demonstrably dissimilar.
(C) When confronting powerful preconceptions, particularly shocking evidence, or complex
situations, jurors make errors in judgment.
(D) The problem of inferential error by juries is typical of the difficulties with cognitive processes
that people face in their everyday lives.
(E) Juries would probably make more reliable decisions if cognitive psychologists, rather than
judges, instructed them about the problems inherent in drawing unwarranted conclusions.
2 : “It can be inferred from the passage that the author would be most likely to agree
with which of the following generalizations about lawyers?”

(A) They have a less sophisticated understanding of human cognition than do


psychologists.
(B) They often present complex or voluminous information merely in order to confuse a jury.
(C) They are no better at making logical inferences from the testimony at a trial than are most
judges.
(D) They have worked to help judges minimize jury inferential error.
(E) They are unrealistic about the ability of jurors to ascertain the truth.

Clearly mentioned in the last sentence of the passage "judges are primitive because they give more
favour to lawyers understanding of inferential mistakes vs. that of psychologists - THIS IMPLIES
PSYCHOLOGISTS HAVE A BETTER UNDERSTANDING"

3. “The author would be most likely to agree with which one of the following
generalizations about a jury’s decision making process?”

(A) The more evidence a jury has, the more likely it is that the jury will reach a reliable verdict.
(B) Juries usually overestimate the value of visual evidence such as photographs.
(C) Jurors have preconceptions about the behavior of defendants that prevent them from making an
objective analysis of the evidence in a criminal trial.

Too extreme - if the word "MAY have or can have"were used then this could be a likely correct
answer.
(D) Most of the jurors who make inferential errors during a trial do so because they are
unaccustomed to having to make difficult decisions based on inferences.
(E) The manner in which evidence is presented to a jury may influence the jury either to
overestimate or to underestimate the value of that evidence.

Perceptual (adj) : tri giác

Retentive (adj) : nhớ lại

Inferential (adj) : suy luận

Incline (v) : khuynh hướng

Jeopardize (v) : gây nguy hiểm


Dec 20 – 1

Since the late 1970’s, in the face of a severe loss of market share in dozens of industries,
manufacturers in the United States have been trying to improve productivity—and therefore
enhance their international competitiveness—through cost-cutting programs. (Cost-cutting here is
defined as raising labor output while holding the amount of labor constant.) However, from 1978
through 1982, productivity—the value of goods manufactured divided by the amount of labor
input—did not improve; and while the results were better in the business upturn of the three years
following, they ran 25 percent lower than productivity improvements during earlier, post-1945
upturns. At the same time, it became clear that the harder manufactures worked to implement cost-
cutting, the more they lost their competitive edge.

With this paradox in mind, I recently visited 25 companies; it became clear to me that the cost-
cutting approach to increasing productivity is fundamentally flawed. Manufacturing regularly
observes a “40, 40, 20” rule. Roughly 40 percent of any manufacturing-based competitive advantage
derives from long-term changes in manufacturing structure (decisions about the number, size,
location, and capacity of facilities) and in approaches to materials. Another 40 percent comes from
major changes in equipment and process technology. The final 20 percent rests on implementing
conventional cost-cutting. This rule does not imply that cost-cutting should not be tried. The well-
known tools of this approach—including simplifying jobs and retraining employees to work smarter,
not harder—do produce results. But the tools quickly reach the limits of what they can contribute.

Another problem is that the cost-cutting approach hinders innovation and discourages creative
people. As Abernathy’s study of automobile manufacturers has shown, an industry can easily
become prisoner of its own investments in cost-cutting techniques, reducing its ability to develop
new products. And managers under pressure to maximize cost-cutting will resist innovation because
they know that more fundamental changes in processes or systems will wreak havoc with the results
on which they are measured. Production managers have always seen their job as one of minimizing
costs and maximizing output. This dimension of performance has until recently sufficed as a basis of
evaluation, but it has created a penny-pinching, mechanistic culture in most factories that has kept
away creative managers.

Every company I know that has freed itself from the paradox has done so, in part, by developing
and implementing a manufacturing strategy. Such a strategy focuses on the manufacturing structure
and on equipment and process technology. In one company a manufacturing strategy that allowed
different areas of the factory to specialize in different markets replaced the conventional cost-cutting
approach; within three years the company regained its competitive advantage. Together with such
strategies, successful companies are also encouraging managers to focus on a wider set of
objectives besides cutting costs. There is hope for manufacturing, but it clearly rests on a different
way of managing.

1. The author of the passage is primarily concerned with

(A) summarizing a thesis


(B) recommending a different approach
(C) comparing points of view
(D) making a series of predictions
(E) describing a number of paradoxes
2. It can be inferred from the passage that the manufacturers mentioned in line
2 expected that the measures they implemented would

(A) encourage innovation


(B) keep labor output constant
(C) increase their competitive advantage
(D) permit business upturns to be more easily predicted
(E) cause managers to focus on a wider set of objectives

3. The primary function of the first paragraph of the passage is to

(A) outline in brief the author’s argument


(B) anticipate challenges to the prescriptions that follow
(C) clarify some disputed definitions of economic terms
(D) summarize a number of long-accepted explanations
(E) present a historical context for the author’s observations

4. The author refers to Abernathy’s study (line 36) most probably in order to

(A) qualify an observation about one rule governing manufacturing


(B) address possible objections to a recommendation about improving manufacturing
competitiveness
(C) support an earlier assertion about one method of increasing productivity
(D) suggest the centrality in the United States economy of a particular manufacturing industry
(E) given an example of research that has questioned the wisdom of revising a manufacturing
strategy

5. The author’s attitude toward the culture in most factories is best described as

(A) cautious
(B) critical
(C) disinterested
(D) respectful
(E) adulatory

This dimension of performance has until recently sufficed as a basis of evaluation, but it has created
a penny-pinching, mechanistic culture in most factories that has kept away creative managers.-------
-> suggests author is critical of the culture prevalent in most factories. Is he cautioning anybody?
No. I think he is merely criticizing.

6. In the passage, the author includes all of the following EXCEPT

(A) personal observation


(B) a business principle
(C) a definition of productivity
(D) an example of a successful company
(E) an illustration of a process technology
7. The author suggests that implementing conventional cost-cutting as a way of
increasing manufacturing competitiveness is a strategy that is

(A) flawed and ruinous


(B) shortsighted and difficult to sustain
(C) popular and easily accomplished
(D) useful but inadequate
(E) misunderstood but promising

Wreak havoc (v) : tàn phá

Suffice (v) : đủ , trọn vẹn


Dec 20 – 2

Careful observers of wildlife along unspoiled freshwater shorelines in New England may encounter
the river otter (Lutra canadensis) or its close cousin, the mink (Mustela vison). Like other members
of the Mustelidae, or "weasel" family, these animals have elongated bodies with long tails and are
efficient hunters. On land, they commonly employ the 2-2 bounding gait characteristic of Mustelidae,
but as the only semi-aquatic members of the family in the northeast, they are also accomplished
swimmers. Both sport rich, dark fur insulating them from the wet and cold.

Despite such similarities, however, the otter and mink are readily distinguished by sight. The otter is
almost twice as long as the mink and may boast ten times the weight of the smaller animal. The
rudder-like tail of the otter is markedly heavy-set, tapering only gradually, while that of the mink is
gracile. Other, less obvious, differences exist. For example, the otter subsists largely on fish and
occasional small mammals, whereas the mink prefers rodents, especially the aquatic muskrat.
Similarly, although both animals give birth in the spring, the mink mates in midwinter; the otter
mates in spring immediately following the birth, taking advantage of a curious adaptation: delayed
implantation of the fertilized egg until the following year. Such variations reflect the fact that even
"cousins" traverse different evolutionary pathways.

1) The purpose of this passage is to

A) Encourage wildlife watchers to study the river otter and mink more carefully.
B) Compare and contrast two closely related animals.
C) Question whether the river otter and mink really belong in the same biological category.
D) Argue that the river otter and mink are fundamentally different.
E) Compare and contrast the Mustelidae to the "weasel" family.

2) The author mentions the "2-2 bounding gait" most probably in order to

A) Contrast aquatic animals with terrestrial animals.


B) Emphasize the energetic efficiency of Mustelidae locomotion.
C) Compare the 2-2 gait to the 1-2-1 loping gait of Mustelidae.
D) Describe a behavioral pattern common to the otter, the mink, and other members of
the weasel family.
the author wishes to show the similarity between otter mink and other weasels on land and
further show how the former two have evolved to aquatic conditions

E) specify the placement of Mustelidae forelegs and hind legs when bounding.

3) The passage suggests that Mustela vison

A) is a less important member of the Mustelidae than Lutra canadensis.


B) consumes smaller prey than Lutra canadensis.
C) may be disturbed by the impact of human activity on the environment.
D) has evolved delayed implantation of fertilized eggs.
E) is the most gracile of the Mustelidae.
For example, the otter subsists largely on fish and occasional small mammals, whereas the mink
prefers rodents, especially the aquatic muskrat.

 We can’t conclude that otter consumes bigger prey than mink

Elongate (v) : kéo dài

Gait (n) : dáng đi

Gracile (adj) : thon thả

Rodent (n) : loài gặm nhấm

Implantation (n) : cấy ghép

Fertilize (v) : thụ tinh


Dec 21 – 1

Marginal analysis is an important decision-making tool in the business world. Pricing decisions tend
to heavily involve analysis regarding marginal contributions to revenues and costs. In business, the
practice of setting the price of a product to equal the extra cost of producing an extra unit of output,
i.e. the marginal cost of producing the unit, is known as marginal-cost pricing. In the marginal
analysis of pricing decisions, if marginal revenue, the increase in revenue from the sale of an
additional unit of output, is greater than marginal cost at some level of output, marginal profit is
positive, and, therefore, a greater quantity should be produced. Alternatively, if marginal revenue is
less than the marginal cost, marginal profit is negative and a lesser quantity should be produced.
Accordingly, firms tend to use this analysis to increase their production until marginal revenue
equals marginal cost, and then charge a price which is determined by the demand curve. For
instance, businesses often set prices close to marginal cost during periods of poor sales. If, for
example, an item has a marginal cost of $1.00 and a normal selling price of $2.00, the firm selling
the item might wish to lower the price to $1.10 - if demand has waned. The business would choose
this approach because the incremental profit of 10 cents from the transaction is better than no sale
at all.

1. Which of the following best describes the primary purpose of the author?

A. To explain how companies change prices, using the market conditions as an indicator

this is discussed only in last line.


B. To discuss the use of a business tool in a particular context
C. To establish the supremacy of a price-setting tool in the business world
D. To advocate for the use of a business concept in determining prices
E. To discuss the various tools available to a company to alter the prices of its products during lean
periods

2. Which of the following is supported by the information given in the passage?

A. Marginal cost pricing is employed when even though the demand is on the rise, the sales are not.
B. The normal selling price of a product is not as affected by the demand of the product as the price
set close to the marginal cost of the product.
C. As companies realize economies of scale, the marginal cost of producing decreases with each
extra unit produced.
D. Companies are likely to shut down when they cannot even command a price that is identical to
the marginal cost of their product.
E. Setting the price close to the marginal cost is sometimes a question of relative
benefit.

3. Which of the following is stated in the passage?

A. The level of output produced is sometimes determined by taking in to account the


difference between marginal revenue and marginal cost
B. Marginal analysis is the most important tool through which the pricing of a product is decided.
C. The normal selling price of a product is usually close to the marginal cost of the product.
D. Profit from an additional unit of output decreases with every increase in production.
E. Marginal cost pricing is a technique used in the short run rather than in the long run.
Dec 21 – 2

When views can freely flourish in the marketplace of ideas, individuals are afforded the advantage of
deciding what notions and concepts to question, support or reject. On June 8, 1789, James Madison
introduced in the House of Representatives an amendment to the Constitution: “The people shall not
be deprived or abridged of their right to speak, to write, or to publish their sentiments; and the
freedom of the press, as one of the great bulwarks of liberty, shall be inviolable.” This commitment
to a free press is a principle Americans hold firmly, because they view it as a necessary ingredient
for a properly functioning political process and a critical component of a free society. Yet, since the
time of America’s founding, the politicized nature of the press has not fundamentally changed.

While conservatives and liberals alike claim that today’s mainstream media is biased, opinionated,
and devoid of objectivity and balanced analysis, American newspapers at the time of this nation’s
birth were all partisan, believing that their responsibility was not to report news, but to convey,
without apology, a particular political position. Perhaps the high point of partisan newspapers was in
New York during the 1920’s, when the city had over a dozen daily papers, each geared toward a
particular ethnic and political niche; people selected the paper that made the most sense of the
world to them. Despite the naysayers who warn that the lack of objectivity and fair-mindedness is
corrosive to society, partisan journalism can be good journalism. It produces plenty of excellent
reporting and analysis and is the norm in many nations. Two centuries ago, newspapers subsidized
by Andrew Jackson's Democrats and Henry Clay's Whigs were dependable supporters of their
parties. Today’s newspapers claim that they too are only giving their readership what it wants.

Legally, the Supreme Court has tried since 1919 to clarify how free the press is. Over time, older
laws that allowed publications to be punished for libel, obscenity, sedition, and publishing
inflammatory material have given way to more expansive rights to publish. The First Amendment
protections offered to journalists have evolved to a broader interpretation of freedom of the press.
During the 1960’s and 1970’s, journalists exposed the government’s mismanagement of the Vietnam
War, and their investigative reporting eventually brought about the resignation of President Nixon.
By the end of the twentieth century, the Constitution’s protections were broadly held to cover the
content of all papers, from the highly regarded New York Times to tabloids such as The National
Enquirer.

1 : According to the author, which of the following is true about partisan journalism
throughout American history?

a) It has had a limited impact on the political process.


b) Its lack of objectivity is detrimental.
c) It has played an important role in reliably informing individuals from diverse cultural backgrounds.
d) It is what all members of society want from their news sources.
e) It has essentially been the status quo since America’s founding.

2 Which of the following statements about American newspapers is supported by


information contained in the passage?

a) America’s newspapers in 1789 resembled those of today in form and content.


b) The character of the press has matured since the time of America’s founding.
c) In recent years, the press has become biased in regard to its political reporting.
d) Early American journalists did not necessarily provide a balanced analysis of events.
e) American newspapers of the past reported more reliably their political affiliations.

3 The author of the passage would disagree with which of the following statements?

a) The legal understanding of press freedoms has shifted over time.


b) Over time, the First Amendment protections offered to the press have become
absolute.
c) America’s legal evolution has given way to a more liberal understanding of press freedom.
d) First Amendment press rights today protect a broad section of the newspaper industry.
e) Freedoms offered to the press have helped shape events in history.

In the last paragraph, the author mentions how the press has gotten more leeway or flexiblity with
what it says/reports from the course and therefore has more opportunity to express itself. Choice B
uses the word ABSOLUTE, which is very strong and not what the author would agree with (author
would say the powers of the press have grown/become more influential but not absoulute)

4 All of the following are examples of limitations the courts have placed on freedom of
the press, EXCEPT

a) articles deemed maliciously defamatory of individuals


b) articles viewed as offensive to society’s views of decency
c) articles that comment negatively on a political affiliation
d) articles clearly dangerous to national security
e) articles that contain language intended to incite rebellion

Status quo (n) : hiện trạng

Deprive (v) : tước đoạt

Abridge (v) : giảm bớt

Sentiment (n) : ý kiến

Bulwark (n) : bức tường

Inviolable (adj) : bất khả xâm phạm

Devoid of (adj) : không có

Naysayers (n) : người phản đối

Subsidize (v) ; trợ cấp

Libel (v) : phỉ báng

Obscenity (n) : tục tĩu

Sedition (n) : nổi loạn, xúi giục

Inflammatory (adj) : khiêu khích

Tabloid (n) : báo lá cải


Dec 22 – 1

Nico Frijda writes that emotions are governed by a psychological principle called the “law of
apparent reality”: emotions are elicited only by events appraised as real, and the intensity of these
emotions corresponds to the degree to which these events are appraised as real. This observation
seems psychologically plausible, but emotional responses elicited by works of art raise
counterexamples.

Frijda’s law accounts for my panic if I am afraid of snakes and see an object I correctly appraise as
a rattlesnake, and also for my identical response if I see a coiled garden hose I mistakenly perceive
to be a snake. However, suppose I am watching a movie and see a snake gliding toward its victim.
Surely I might experience the same emotions of panic and distress, though I know the snake is not
real. These responses extend even to phenomena not conventionally accepted as real. A movie
about ghosts, for example, may be terrifying to all viewers, even those who firmly reject the
possibility of ghosts, but this is not because viewers are confusing cinematic depiction with reality.
Moreover, I can feel strong emotions in response to objects of art that are interpretations, rather
than representations, of reality: I am moved by Mozart’s Requiem, but I know that I am not at a
real funeral. However, if Frijda’s law is to explain all emotional reactions, there should be no
emotional response at all to aesthetic objects or events, because we know they are not real in the
way a living rattlesnake is real.

Most psychologists, perplexed by the feelings they acknowledge are aroused by aesthetic
experience, have claimed that these emotions are genuine, but different in kind from nonaesthetic
emotions. This, however, is a descriptive distinction rather than an empirical observation and
consequently lacks explanatory value. On the other hand, Gombrich argues that emotional
responses to art are ersatz; art triggers remembrances of previously experienced emotions. These
debates have prompted the psychologist Radford to argue that people do experience real
melancholy or joy in responding to art, but that these are irrational responses precisely because
people know they are reacting to illusory stimuli. Frijda’s law does not help us to untangle these
positions, since it simply implies that events we recognize as being represented rather than real
cannot elicit emotion in the first place

Frijda does suggest that a vivid imagination has “properties of reality”—implying, without
explanation, that we make aesthetic objects or events “real” in the act of experiencing them.
However, as Scruton argues, a necessary characteristic of the imaginative construction that can
occur in an emotional response to art is that the person knows he or she is pretending. This is what
distinguishes imagination from psychotic fantasy.

1. Which one of the following best states the central idea of the passage?

(A) The law of apparent reality fails to account satisfactorily for the emotional nature of belief.
(B) Theories of aesthetic response fail to account for how we distinguish unreasonable from
reasonable responses to art.
(C) The law of apparent reality fails to account satisfactorily for emotional responses to
art.
(D) Psychologists have been unable to determine what accounts for the changeable nature of
emotional responses to art.
(E) Psychologists have been unable to determine what differentiates aesthetic from nonaesthetic
emotional responses.

2. According to the passage, Frijda’s law asserts that emotional responses to events are

(A) unpredictable because emotional responses depend on how aware the person is of the reality of
an event
(B) weaker if the person cannot distinguish illusion from reality
(C) more or less intense depending on the degree to which the person perceives the
event to be real
(D) more intense if the person perceives an event to be frightening
(E) weaker if the person judges an event to be real but unthreatening

3. The author suggests that Frijda’s notion of the role of imagination in aesthetic
response is problematic because it

(A) ignore the unselfconsciousness that is characteristic of emotional responses to art


(B) ignores the distinction between genuine emotion and ersatz emotion
(C) ignores the fact that a person who is imagining knows that he or she is imagining
(D) makes irrelevant distinctions between vivid and weak imaginative capacities
(E) suggests, in reference to the observation of art, that there is no distinction between real and
illusory stimuli

However, as Scruton argues, a necessary characteristic of the imaginative construction that can
occur in an emotional response to art is that the person knows he or she is pretending. This is what
distinguishes imagination from psychotic fantasy.

4. The passage supports all of the following statements about the differences between
Gombrich and Radford EXCEPT:

(A) Radfod’s argument relies on a notion of irrationality in a way that Gomgbrich’s argument does
not.
(B) Gmbrich’s position is closer to the position of the majority of psychologists than is
Radford’s.
(C) Gombrich, unlike Radford, argues that we do not have true emotions in response to art.
(D) Gombrich’s argument rests on a notion of memory in a way that Radford’s argument does not.
(E) Radford’s argument, unlike Gombrich’s, is not focused on the artificial quality of emotional
responses to art.

5. Which one of the following best captures the progression of the author’s argument in
lines 9-31?

(A) The emotional responses to events ranging from the real to the depicted illustrate the
irrationality of emotional response.
(B) A series of events that range from the real to the depicted conveys the contrast between real
events and cinematic depiction.
(C) An intensification in emotional response to a series of events that range from the real to the
depicted illustrates Frijda’s law.
(D) A progression of events that range from the real to the depicted examines the precise nature of
panic in relation to feared object.
(E) The consistency of emotional responses to events that range from the real to the
depicted challenges Frijda’s law.

author shows how NF's law does not hold by giving example

6. Author’s assertions concerning movies about ghosts imply that all of the following
statements are false EXCEPT:

(A) Movies about ghosts are terrifying in proportion to viewers’ beliefs in the phenomenon of ghosts.
(B) Movies about imaginary phenomena like ghosts may be just as terrifying as movies
about phenomena like snake.
(C) Movies about ghosts and snakes are not terrifying because people know that what they viewing
is not real.
(D) Movies about ghosts are terrifying to viewers who previously rejected the possibility of ghosts
because movies permanently alter the viewers sense of reality.
(E) Movies about ghosts elicit a very different emotional response from viewers who do not believe
in ghosts than movies about snakes elicit from viewers who are frightened by snakes.

7. Which one of the following statements best exemplifies the position of Radford
concerning the nature of emotional response to art?

(A) A person watching a movie about guerrilla warfare irrationally believes that he or she is present
at the battle.
(B) A person watching a play about a kidnapping feels nothing because he or she rationally realizes
it is not a real event.
(C) A person gets particular enjoyment out of writing fictional narratives in which he or she figures
as a main character.
(D) A person irrationally bursts into tears while reading a novel about a destructive fire,
even while realizing that he or she is reading about a fictional event.
(E) A person who is afraid of snakes trips over a branch and irrationally panics.

Elicite (v) : gợi ra

Appraise (v) : thẩm định, đánh giá

Perplex (v) : bối rối

Arouse (v) : đánh thức

Genuine (adj) : xác thật

Melancholy (n) : u sầu, buồn rầu

Illusory (adj) : hão huyền

Untangle (v) : gỡ rối


Dec 22 – 2

Frailty of understanding is in itself no proper target for scorn and mockery. But the unintelligent
forfeit their claim to compassion when they begin to indulge in self-complacent airs, and to call
themselves sane critics, meaning that they are mechanics. And when, relying upon their numbers,
they pass from self-complacency to insolence, and reprove their betters for using the brains which
God has not denied them, they dry up the fount of pity.

If a hale man walks along the street upon two sound legs, he is not liable to be chased by crowds of
cripples vociferating ‗Go home and fetch your crutch.‘ If a reasoning man edits a classic rationally,
he is. What a critic is, and what advantage he has over those who are not critics, can easily be
shown by one example. Cicero‘s oration pro rege Deiotaro was edited between 1830 and 1840 by
Klotz, Soldan, and Benecke. The best MS then known was the Erfurtensis, and all three editors
pounced on this authority and clung to it, believing themselves safe. In 1841, Madvig, maintaining
reason against superstition in Cicero‘s text as I now maintain it in Juvenal‘s, impugned 17 readings
adopted from the Erfurtensis by these editors, and upheld the readings of inferior MSS. We now
possess MSS still better than the Erfurtensis, and in 12 of the 17 places they contradict it; they
confirm the inferior MSS and the superior critic.

But there are editors destitute of this discriminating faculty, so destitute that they cannot even
conceive it to exist; and these are entangled in a task for which nature has neglected to equip them.
What are they now to do? Set to and try to learn their trade? That is forbidden by sloth. Stand back
and leave room for their superiors? That is forbidden by vanity. They must have a rule, a machine to
do their thinking for them. If the rule is true, so much the better; if false, that cannot be helped: but
one thing is necessary, a rule. A hundred years ago it was their rule to count the MSS and trust the
majority. But this pillow was snatched from under them by the great critics of the 19 th century, and
the truth that MSS must be weighed, not counted, is now too widely known to be ignored.

The sluggard has lost his pillow, but he has kept his nature, and must needs find something else to
loll on; so he fabricates, to suit the change of season, his precious precept of following one MS
wherever possible. Engendered by infirmity and designed for comfort, no wonder if it misses the
truth at which it was never aimed. Its aim was purely humanitarian: to rescue incompetent editors
alike from the toil of editing and from the shame of acknowledging that they cannot edit.

1. The author‘s discussion takes the reader to the topic of the Erfurtensis MS in
paragraph two. The example of this manuscript is relevant to the claim that:

A. the Erfurtensis MS is not very reliable.


B. no single MS can be assumed to be always right.
C. Madvig was a lazy editor.
D. MSS must be weighed, not counted.
E. every MS is either right or wrong

2. According to the various arguments put forth by the author of the passage, which of
the following are true about the editing of classics?

I. It has not been undertaken in the case of Cicero.


II. It is sometimes undertaken by people who are unable to do it correctly.
III. There were important advances in the field during the 19th century.
A. I and II only
B. II and III only
C. I and III only
D. I, II and III
E. None of the above

3. Suppose that the author is present at a panel discussion on the topic of this passage
where all of the following statements are made by other panellists. According to the
arguments he has put forth in the text, the author is LEAST likely to agree with which of
the following statements?

A. It should not be assumed that the majority of the MSS of a classical text are correct.
B. Madvig was a better editor than Klotz, Soldan, or Benecke.
C. It is a mistake to think that one MS of a particular text is better than another.
D. There is no simple rule for editing that eliminates the need for critical discrimination.
E. The Erfurtensis was one of the best known MS in the period 1830-1840 Yes - clearly mentioned in
para 2

4. In spite of what may or may not appear in the first paragraphs of the passage, the
bulk of the passage is devoted to showing:

A. that incompetent editors have developed methods for avoiding the difficulties of
responsible editing.
B. that the Erfurtensis MS is no longer considered the best MS of Cicero‘s pro rege Deiotaro.
C. that it was discovered in the nineteenth century that MSS must be weighed, not merely counted.
D. that Cicero was editing more often during the 1830s than during any other decade.
E. that responsible editing has become an extinct concept

Frailty (n) : lỗi, sai lầm

Scorn (n) : khinh bỉ

Mockery (n) : nhạo báng

Indulge (v) : lạm dụng

Self-complacent (adj) : tự mãn

Sane (adj) : lành mạnh

Insolence (n) : xấc lược

Vociferate (v) : la lối om sòm

Cripple (n) : nguời tàn tật

Crutch (n) : nạng

Manuscript (n) : bản thảo

Pounce (v) : vồ lấy

Clung to (v) : bám vào


Superstition (n) : mê tín

Impugned (v) : công kích, phản đối

Uphold (v) : duy trì

Destitute of (adj) : vận mệnh, không có

Entangle in (v) : vướng vào

Sloth (n) : sự lười biếng

Vanity (n) : phù phiếm

Snatch (v) : giật lấy

Sluggard (n) : người làm biếng

Fabricate (v) : ngụy tạo

Engender (v) : gây ra

Infirmity (n) : bệnh tật


Dec 23 – 1

The profession of engineering—which, by the way, is merely the adapting of discoveries in science
and art to the uses of mankind is a peculiarly isolated one. But very little is known about it among
those outside of the profession. Laymen know something about law, a little about medicine, quite a
lot—nowadays—about metaphysics. But laymen know nothing about engineering. Indeed, a source
of common amusement among engineers is the peculiar fact that the average layman cannot
differentiate between the man who runs a locomotive and the man who designs a locomotive. In
ordinary parlance both are called engineers. Yet there is a difference between them—a difference as
between day and night.

For one merely operates the results of the creative genius of the other. This almost universal
ignorance as to what constitutes an engineer serves to show to what broad extent the profession of
engineering is isolated. Yet it is a wonderful profession. I say this with due regard for all other
professions. For one cannot but ponder the fact that, if engineers started the greatest war—the
world war—the world has ever known—and engineers as a body freely admit that if they did not
start it they at least made it possible—they also stopped it, thereby proving themselves possessed of
a power greater than that of any other class of professional men—diplomats and lawyers and
divinities not excepted.

That engineering is a force fraught with stupendous possibilities, therefore, nobody can very well
deny. That it is a force generally exercised for good— despite the world war I myself, as an
engineer, can truly testify. With some fifteen years spent on the creative end of the work—the
drafting and designing end—I have yet to see, with but two or three rare exceptions, the genius of
engineers turned into any but noble channels. Thus, engineering is not only a wonderful profession,
with the activities of its followers of utmost importance, but also it is a profession the individual work
of whose pioneers, from Watt to Westinghouse and from Eiffel to Edison, has been epoch-making.

1. What is the primary purpose of the author in writing the passage?

(A) To describe the contributions of engineering to improving the lives of mankind.


(B) To analyse why the layman knows more about other professions than he does about
engineering.
(C) To evaluate the good and bad results made possible by the engineering profession.
(D) To explain how engineering is different from other professions.
(E) To discuss a unique aspect of the engineering profession and highlight the
importance of engineering.

2. Which of the following assertions is supported by the information in the passage?

(A) A layman probably knows more about law than he does about engineering.
(B) Engineering has been put to as many noble uses as evil ones.
(C) A layman has considerable knowledge of subjects such as law and medicine.
(D) Engineers played no role in starting the world war.
(E) Had it not been for the efforts of engineers, the world war would have not ended.
3. In the passage, why does the author mention the man who runs a locomotive?

(A) To show how engineering has different possible uses


(B) To explain how his expertise is different from that of the engineer who designed the locomotive
(C) To highlight the differing nature of expertise required by different types of engineers
(D) To emphasise his point that the layman knows almost nothing about engineering
(E) To describe how certain classes of engineers are unaware of the achievements of other classes
of engineers.

Peculiarly (adv) : đặc biệt, dị thường

Laymen (n) : giáo dân

Metaphysic (n) : siêu hình học

Locomotive (n) : đầu máy

Parlance (n) : cách nói

Ponder (v) : suy ngẫm, cân nhắc

Fraught with (adj) : đầy đủ với

Stupendous (adj) : tuyệt vời, kỳ diệu

Noble (adj) : cao quý

Utmost (adj) : hết sức

Epoch-making (adj) : kỷ nguyên mới


Dec 23 – 2

Gamification (the trend of applying video game mechanics such as points and achievements to non-
gaming work processes such as innovation, training, and employee performance) is increasingly
being used by organizations, with many realizing significantly improved performance and increased
profitability as a result. Michael Wu sees significant potential for increased corporate performance
through gamification of processes, but expresses a concern that gamification could also result in
decreased employee performance over time. Gamified processes can create a short-term increase in
performance and productivity as employees seek to earn points and badges, but eventually the
employees, like even the most avid gamers, would grow bored with playing and move on.
Furthermore, a danger inherent in the gamification of processes is the so-called “moral hazard” of
game play--that people will become focused on the extrinsic rewards and mechanics of the game
play rather than on the intrinsic rewards to be gained by participating in the activity. As a result, a
corporation might become significantly more efficient and profitable as a result of gamifying its
internal processes, while at the same time its employees become gradually less motivated by the
intrinsic rewards of their work as well as developing a stronger expectation for external rewards that
increase over time.

Wu argues that in order for companies to successfully employ gamification, they must develop a
strategy to make it sustainable over the long term, such as tying extrinsic rewards to specific
intrinsic benefits that have long-term value to the individual, or using the gamification process only
long enough to discover each employee''s intrinsic motivations and develop new reward systems
that address each employee individually. Focusing narrowly on gamification, while offering a
significant opportunity for businesses to increase efficiency and profitability over the short term, may
ironically result in decreased employee performance and corporate earnings over the long term.

1) The primary purpose of the passage is to

A) suggest a potential alternative to a business strategy that may result in a lack of employee
motivation
B) demonstrate that a specific business strategy will not be successful over the long run
C) explain why intrinsic motivations are more important to employee performance than extrinsic
rewards are
D) describe the dangerous moral implications of a particular business strategy
E) explain concerns about the potential effects of implementing a specific business
strategy

2) The passage implies that which of the following is a potential consequence of a


company's implementation of gamification processes?

A) A company that does not see improvements in efficiency or increases in profitability as a result of
gamification may have little reason to continue implementing such processes.
B) Employees with little or no previous exposure to game-style processes could find the gamified
process to be confusing, leading to less employee motivation and reduced morale.
C) In order to continue to see results from a gamified process over a longer term, the
company may need to adjust the gamification process by implementing increases in the
extrinsic rewards provided to employees.
D) The increased profits generated by the company as a result of the implementation of the
gamification process might be reinvested in other, non-gamified areas of the company, with no
guarantee that gamification processes will result in additional future performance improvements.
E) The company will become significantly more efficient and profitable over the long term, while its
employees gradually lose motivation.

3) According to the passage, which of the following is a possible effect of gamifying a


company process?

A) Giving employees the opportunity to treat work like a game and earn points and badges would
lead to improved company morale.
B) Company employees who are avid gamers would earn more rewards than those who are not,
because they are familiar with the process of earning points and badges.
C) The employees would expect increasing intrinsic rewards the longer they participate in the
system, and would gradually lose motivation if such rewards were not periodically increased.
D) After a time, employees may lose interest in competing in the system to earn
rewards, and choose to focus instead on other aspects of work.
E) The improvements in productivity and profitability may cause the company to implement
gamification processes in a more widespread manner throughout the company.

That's supported by this part in the first paragraph: "eventually the employees, like even the most
avid gamers, would grow bored with playing and move on". That implies that eventually the
employees will lose interest in playing the game, and move on to focus on other aspects of their
work.

Avid (adj) : khao khát

Gamification (n) : trò chơi điện tử ứng dụng hóa

Extrinsic (adj) : bên ngoài


Dec 24 – 1

When catastrophe strikes, analysts typically blame some combination of powerful mechanisms. An
earthquake is traced to an immense instability along a fault line; a stock market crash is blamed on
the destabilizing effect of computer trading. These explanations may well be correct. But systems as
large and complicated as the Earth’s crust or the stock market can break down not only under the
force of a mighty blow but also at the drop of a pin. In a large interactive system, a minor event can
start a chain reaction that leads to a catastrophe.

Traditionally, investigators have analyzed large interactive systems in the same way they analyze
small orderly systems, mainly because the methods developed for small systems have proved so
successful. They believed they could predict the behavior of a large interactive system by studying
its elements separately and by analyzing its component mechanisms individually. For lack of a better
theory, they assumed that in large interactive systems the response to a disturbance is proportional
to that disturbance.

During the past few decades, however, it has become increasingly apparent that many large
complicated systems do not yield to traditional analysis. Consequently, theorists have proposed a
“theory of self-organized criticality”: many large interactive systems evolve naturally to a critical
state in which a minor event starts a chain reaction that can affect any number of elements in the
system. Although such systems produce more minor events than catastrophes, the mechanism that
leads to minor events is the same one that leads to major events.

A deceptively simple system serves as a paradigm for self-organized criticality: a pile of sand. As
sand is poured one grain at a time onto a flat disk, the grains at first stay close to the position
where they land. Soon they rest on top of one another, creating a pile that has a gentle slope. Now
and then, when the slope becomes too steep, the grains slide down, causing a small avalanche. The
system reaches its critical state when the amount of sand added is balanced, on average, by the
amount falling off the edge of the disk.

Now when a grain of sand is added, it can start an avalanche of any size, including a “catastrophic”
event. Most of the time the grain will fall so that no avalanche occurs. By studying a specific area of
the pile, one can even predict whether avalanches will occur there in the near future. To such a local
observer, however, large avalanches would remain unpredictable because they are a consequence
of the total history of the entire pile. No matter what the local dynamics are, catastrophic avalanches
would persist at a relative frequency that cannot be altered: Criticality is a global property of the
sandpile.

1. The passage provides support for all of the following generalizations about large
interactive systems EXCEPT:

(A) They can evolve to a critical state.


(B) They do not always yield to traditional analysis.
(C) They make it impossible for observers to make any predictions about them.
(D) They are subject to the effects of chain reactions.
(E) They are subject to more minor events than major events.
2. According to the passage, the criticality of a sandpile is determined by the

(A) size of the grains of sand added to the sandpile


(B) number of grains of sand the sandpile contains
(C) rate at which sand is added to the sandpile
(D) shape of the surface on which the sandpile rests
(E) balance between the amount of sand added to and the amount lost from the
sandpile

3. It can be inferred from the passage that the theory employed by the investigators
mentioned in the second paragraph would lead one to predict that which one of the
following would result from the addition of a grain of sand to a sandpile?

(A) The grain of sand would never cause anything more than a minor disturbance.
(B) The grain of sand would usually cause a minor disturbance, but would occasionally cause a small
avalanche.
(C) The grain of sand would usually cause either minor disturbance or a small avalanche, but would
occasionally cause a catastrophic event.
(D) The grain of sand would usually cause a catastrophic event, but would occasionally cause only a
small avalanche or an event more minor disturbance.
(E) The grain of sand would invariably cause a catastrophic event.

4. Which one of the following best describes the organization of the passage?

(A) A traditional procedure is described and its application to common situations is endorsed: its
shortcomings in certain rare but critical circumstances are then revealed.
(B) A common misconception is elaborated and its consequences are described a detailed example
of one of these consequences is then given.
(C) A general principle is stated and supported by several examples; an exception to the rule is then
considered and its importance evaluated.
(D) A number of seemingly unrelated events are categorized: the underlying processes that connect
them are then detailed.
(E) A traditional method of analysis is discussed and the reasons for its adoption are
explained; an alternative is then described and clarified by means of an example.

5. Which one of the following is most analogous to the method of analysis employed by
the investigators mentioned in the second paragraph?

(A) A pollster gathers a sample of voter preferences and on the basis of this information makes a
prediction about the outcome of an election.
(B) A historian examines the surviving documents detailing the history of a movement and from
these documents reconstructs a chronology of the events that initiated the movement.
(C) A meteorologist measures the rainfall over a certain period of the year and from this data
calculates the total annual rainfall for the region.
(D) A biologist observes the behavior of one species of insect and from these observations
generalizes about the behavior of insects as a class.
(E) An engineer analyzes the stability of each structural element of a bridge and from
these analyses draws a conclusion about the structural soundness of the bridge.
6. In the passage, the author is primarily concerned with

(A) arguing against the abandonment of a traditional approach


(B) describing the evolution of a radical theory
(C) reconciling conflicting points of view
(D) illustrating the superiority of a new theoretical approach
(E) advocating the reconsideration of an unfashionable explanation.

Catastrophe (n) : thảm kịch, thảm họa

Fault line (n) : đường nứt

Pin (n) : cái chốt

Paradigm (n) : kiểu mẫu

Deceptively simple (adj) : đơn giản

Pile (n) : một chồng, đóng cọc

Avalanche (n) : tuyết lở


Dec 25 – 1

In general, the impossible must be justified by reference to artistic requirements, or to the higher
reality, or to received opinion. With respect to the requirements of art, a probable impossibility is to
be preferred to a thing improbable and yet possible. And by extrapolation, it goes without saying
that the improbable impossibility makes for a not too enticing option while the probable possibility
will not even be discussed here as it, by its very nature, tends towards the mundane.

The poet being an imitator, like a painter or any other artist, must of necessity imitate one of three
objects – things as they were or are, in the past – be it ancient or near – and in the present – as he
observes those things around him or those things that are observed by others contemporary to him;
things as they are said or thought to be, be they products of philosophical discourse, a study in
divinity, or the mythos of a people; or things as they ought to be as often expressed in laments for
the state of affairs in a society. The vehicle of expression is language – either current terms or, it
may be, rare words or metaphors. There are also many modifications of language which we concede
to the poets. Add to this that the standard of correctness is not the same in poetry and politics, any
more than in poetry and any other art.

Within the art of poetry itself there are two kinds of faults – those which touch its essence, and
those which are of the cause of a lack of advertence. If a poet has chosen to imitate something, but
has fallen short through want of capacity, the error is inherent in the poetry. But if the failure is due
to a wrong choice – if he has represented a horse as throwing out both his off legs at once, or
introduces technical inaccuracies in medicine, for example, or in any other art – the error is not
essential to the poetry. These are the points of view from which we should consider and answer the
objections raised by the critics.

As to matters which concern the poet‘s own art. If he describes the impossible, he is guilty of an
error; but the error may be justified, if the end of the art be thereby attained – if, that is, the effect
of this or any other part of the poem is thus rendered more striking. If, however, the end might
have been as well, or better, attained without violating the special rules of the poetic art, the error is
not justified, for every kind of error should, if possible, be avoided. Again, does the error touch the
essentials of the poetic art, or some accident of it? For example, not to know that a hind has no
horns is a less serious matter than to paint it inartistically.

Further, if it be objected that the description is not true to fact, the poet may perhaps reply – ―But
the objects are as they ought to be‖: just as Sophocles said that he drew men as they ought to be;
Euripides, as they are. In this way the objection may be met. If, however, the representation is of
neither kind, the poet may answer – ―This is how men say the thing is.‖ This applies to tales about
the gods. It may well be that these stories are not higher than fact nor yet true to fact. But anyhow,
―this is what is said.‖ Again, a description may be no better than the fact.
1. Assuming that the poet‘s artistic goals are achieved, the passage implies that which
of the following would NOT be an example of a justifiable error?

A. Describing a lioness as a hunter in a metaphor for the behaviour of predatory government


officials
B. Using awkward language to create an analogy between a ruler‘s hand as a symbol of
authority and a city‘s capitol as a symbol of power
C. Creating anachronistic errors by mentioning inappropriate historical or contemporary events
D. Representing human characters as improbably courageous or strong
E. Comparing soldiers with ancient Greek warriors who could not be killed

The errors (in poems) are justifiable if "rendered more striking" and factual errors are justified. (as
mentioned in para 4).

We need an answer choice that doesn't meet both the aforementioned points. Option B (barring
other options) - mentions an example of an inherent error (awkward language) not creating the
striking effect.

2. The author brings up the ancient Greek poets Sophocles and Euripides to make a
point within the passage. According to the information cited in the passage, they differ
from each other in that:

A. Euripides‘ characters provide ideal models of human behaviour.


B. Sophocles portrays people as common public opinion supposed them to be.

HOLD - what does "supposed them to be" imply? He did intend to show humas as they ought to be
C. the characters in Sophocles‘ work are meant to inspire improved human behaviour
and actions.

Hold!! - If the characters are shown as they ought to be surely they would inspire CORRECT
D. humans are unfavourably described by Euripides in order to show detrimental behaviour to avoid.
E. one of them makes a much greater use of metaphors than the other

Further, if it be objected that the description is not true to fact, the poet may perhaps reply – ―But
the objects are as they ought to be‖: just as Sophocles said that he drew men as they ought
to be; Euripides, as they are.

3. The author‘s argument that the poet is ―an imitator, like a painter or any other
artist‖ suggests that the author would be most likely to agree with which of the
following statements?

A. Different types of creative or aesthetic talent have different means of representation.


B. Creating text and chiselling marble are similar forms of representation.
C. The visual arts are superior to the rhetorical arts.
D. The forms of imitation found in poetry are inefficient.
E. Painting is easier than writing poems

Improbably (adv) : không thể

Capitol (n) : thủ đô


Extrapolation (n) : phép ngoại suy

Mundane (adj) : thuộc về trần thế, trần tục

Discourse (n) : đàm luận, bài luận

Lament (n) : than thở

State of affair (n) : tình trạng

Advertence (n) : quảng cáo

Striking (adj) : nổi bật

Hind (n) : hươu cái

Inartistically (adv) : thiếu nghệ thuật

Potrebbero piacerti anche